You are on page 1of 274

Professional Stage Financial Accounting March 2008

PROFESSIONAL STAGE (APPLICATION) - FINANCIAL ACCOUNTING


OT EXAMINERS COMMENTS
The following table summarises how well candidates answered each syllabus content area.

How well* candidates answered each syllabus area

Syllabus area Number of questions Well answered Poorly answered

LO1 3 3 0

LO2 9 9 0

LO3 3 2 1

Total 15 14 1

*If 50% or more of the candidates gave the correct answer, then the question was classified as well answered.

Item 1

This item tested simple elimination of inter-company trading, the resulting unrealised profit figure in inventory at
the period end and goods in transit. This is an area of the syllabus that is tested frequently but is consistent with
candidates performance in the written test questions in the examination in this area.

The Institute of Chartered Accountants in England and Wales 2008, Page 1 of 15


Professional Stage Financial Accounting March 2008

MARK PLAN AND EXAMINERS COMMENTARY

The mark plan set out below was that used to mark these questions. Markers are encouraged to use discretion
and to award partial marks where a point was either not explained fully or made by implication. More marks are
available than could be awarded for each requirement, where indicated. This allows credit to be given for a
variety of valid points, which are made by candidates.

Question 1

General comments: this is a trial balance question with the requirement to prepare a balance sheet and
income statement. Adjustments are required for the disposal of an item of plant, the recognition of a
provision and a related asset and the revaluation of property, as well as other minor adjustments.

(a)

Ginger Ltd - Balance Sheet as at 31 December 2007



ASSETS
Non-current assets
Property, plant and equipment
((4,135,000 -1,696,750) + 4,835,000 + 7,766,850
(857,000 363,400)) (W3)

Current assets
Inventories (168,000 8,500) 159,500
Trade receivables 516,500
Other receivables (200,000 + 800,000 (W5)) 1,000,000
Cash and cash equivalents 22,600

1,698,600

Total assets 9,465,450

EQUITY AND LIABILITIES


Capital and reserves
Ordinary share capital 6,000,000
Revaluation reserve (470,000 6,750(W2)) 463,250
Retained earnings (W6) 700,900

Equity 7,164,150

Non-current liabilities
Bank loan 700,000

Current liabilities
Trade and other payables (461,300 + 140,000) 601,300
Provisions (W5) 1,000,000
1,601,300

Total equity and liabilities 9,465,450

The Institute of Chartered Accountants in England and Wales 2008, Page 2 of 15


Professional Stage Financial Accounting March 2008

Ginger Ltd Income Statement for year ended 31 December 2007



Revenue 6,800,000
Cost of sales (W4) (3,286,900)

Gross profit 3,513,100

Administrative expenses (W4) (1,656,750)


Other operating costs
(450,000 15,000 (W1)+ 200,000 (W5)) (635,000)
Other income 31,800
1,253,150

Finance cost (680,000)


Profit before tax 573,150
Income tax expense (140,000)

Profit for the period 433,150

Note: Marks will be awarded if items are included in a different line item in the income statement
provided that the heading used is appropriate.

W1 Disposal of piece of machinery

Machinery cost 200,000

Accumulated depreciation 200,000 x 20% x 3 years = 120,000

Carrying amount at disposal 200,000 - 120,000 = 80,000


Proceeds 95,000
Less: CA (80,000)
Profit on disposal 15,000

W2 Revaluation reserve

Property Land Total



Retail units 400,000 400,000 800,000
Accumulated depreciation (200,000) -
Carrying amount 200,000 400,000

Valuation at 1 Jan 2007 535,000 535,000


Revaluation reserve 335,000 135,000 470,000

Depreciation on cost 400,000 x 5% years = 20,000 pa

Depreciation on revalued amount 535,000 x 5% = 26,750 pa

Transfer between reserves for additional depreciation 26,750 20,000 = 6,750

The Institute of Chartered Accountants in England and Wales 2008, Page 3 of 15


Professional Stage Financial Accounting March 2008

W3 Property, plant and equipment

Freehold land and buildings

Property Land

Cost (4,000,000 400,000 (W2)) 3,600,000
(4,700,000 400,000 (W2)) 4,300,000
Valuation 535,000 535,000

At 31 December 2007 4,135,000 4,835,000

Accumulated depreciation 1,690,000


Less revalued depreciation (W2) (200,000)
Depreciation in year (5%) 206,750

At 31 December 2007 1,696,750

Plant and equipment

Cost 962,000
Add back disposal proceeds 95,000
Less disposal cost (200,000)

At 31 December 2007 857,000

Accumulated depreciation 312,000


Less disposal (W1) (120,000)
Depreciation in year (20%) 171,400

At 31 December 2007 363,400

W4 Expenses

Cost of sales Administrative


expenses
Trial balance 1,450,000
Purchases 2,600,000
Opening inventory 675,000
Depreciation (W3)
Land & buildings 206,750
Plant & machinery 171,400
Write down (9,000 500) 8,500
Closing inventory (168,000)

3,286,900 1,656,750

W5 Provision

Legal claim 1,000,000 most likely outcome


Counter claim 1,000,000 x 80% = 800,000 receivable

Income statement effect: 1,000,000 - 800,000 = 200,000

The Institute of Chartered Accountants in England and Wales 2008, Page 4 of 15


Professional Stage Financial Accounting March 2008

W6 Retained earnings


Trial balance 261,000
Transfer from revaluation reserve (W2) 6,750

267,750
Add: Profit for period 433,150
700,900

Almost all candidates produced a well laid out income statement and balance sheet in appropriate
formats, although some candidates lost marks by not adding across numbers in brackets or transferring
numbers from workings. Presentation marks were lost by failing to complete the totals on the statements.

Most candidates were able to take items from the trial balance and insert them in the correct place in the
formats, with most candidates using an efficient matrix style costs working.

Marks were awarded where presentation differed to the marking guide but resulted in a reasonable
alternative. For example, trade receivables were often combined with other receivables and the 450,000
of operating costs was included in administrative expenses.

Candidates generally dealt correctly with the inventory adjustments and completed the double entry for
the taxation figure.

The effect of the legal claim caused candidates a number of problems, such as, calculating the closing
provision on a weighted average basis, as opposed to at its most likely outcome, describing the provision
as a contingent liability, but still including it in the balance sheet and failing to recognise that the income
statement should show the net effect of the provision and the counter claim of 200,000 (1,000,000 less
800,000).

Other common errors included charging depreciation on the land balance as well as the buildings,
calculating the revaluation uplift based on cost as opposed to on carrying amounts and forgetting to
include any revaluation uplift in the revaluation reserve on the balance sheet.

Total possible marks 24


Maximum full marks 22

The Institute of Chartered Accountants in England and Wales 2008, Page 5 of 15


Professional Stage Financial Accounting March 2008

Question 2

General comments: this question focuses on the differences between cash accounting and the accrual basis of
accounting. The question develops by considering these concepts in relation to a number of revenue
transactions, with extracts from the financial statements required. The question is completed by looking at the
differences between a cash flow statement under IFRS and UK GAAP.

(a)(i)

Cash and accruals accounting

Cash accounting only considers the cash impact of a transaction. For example, sales are recorded in the
period when the buyer pays for the goods. This may not be the same period in which the buyer takes
delivery of the goods if delayed payment terms are provided.

The accrual basis of accounting instead records transactions in the period in which they occur, not when
the related cash flow arises. Sales are instead recorded when the risks and rewards of ownership pass
from the seller to the buyer, not when the buyer pays for the goods or services.

(a)(ii)

Revenue is recorded when there is an increase in economic benefits during the period. However, revenue
can only be recognised when an entity is sufficiently certain that it will be paid for the goods or services and
that payment is for a known amount.

The accrual basis of accounting is followed with revenue being recognised in the period in which the
associated work is undertaken rather than when cash is received.

In making the assessment of the timing of the economic benefits an entity should consider when the
following conditions have been met:

The entity has transferred the significant risks and rewards of ownership of the goods to the buyer.
The seller no longer has management involvement or effective control over the goods.
The amount of revenue and costs can be measured reliably.
It is probable that the economic benefits associated with the transaction will flow to the entity.

When an entity has met all the above conditions it recognises the revenue even though payment may still
be outstanding.

The answers to this part of the question were disappointing with a significant number of candidates failing to
appreciate the difference between cash and accrual accounting.

The majority of candidates were able to describe the basic features of cash and accruals accounting. A
significant minority were able to back their answers up by providing a well thought out example.

In (a)(ii) most candidates gained the marks for when revenue can be recognised per IAS 18 but few were
able to link this to the Framework principles for the recognition of income. This was mainly due to
candidates copying straight from the open book text without relating the information to the question
requirements.

Total possible marks 6


Maximum full marks 6

The Institute of Chartered Accountants in England and Wales 2008, Page 6 of 15


Professional Stage Financial Accounting March 2008

(b) Extracts from the financial statements for the year ended 31 December 2007

(1) Sale of products on delayed payment terms

(i) Cash accounting

Nothing will be recorded under the cash accounting basis as no cash has been received at the year
end.

(ii) Accrual basis

Extract from income statement



Revenue 14,100

Investment income (900 x 6/12) 450

Extract from balance sheet



Trade and other receivables (14,100 + 450) 14,550

(2) Gardening services

(i) Cash accounting

Extract from income statement



Revenue 3,000

Extract from balance sheet



Cash 3,000

(ii) Accrual basis

Extract from income statement



Revenue (3,000 x 2/3) 2,000

Extract from balance sheet



Current liabilities
Deferred income (3,000 2,000) 1,000

(3) Garden design

(i) Cash accounting

Extract from income statement

At 31 December 2007, although some work has been undertaken on the garden design, the customer
has not paid anything. Therefore no revenue would be recognised under cash accounting.

Cost of sales 4,000

The Institute of Chartered Accountants in England and Wales 2008, Page 7 of 15


Professional Stage Financial Accounting March 2008

(ii) Accrual basis

Extract from income statement



Revenue (11,000 x 50%) 5,500

Cost of sales ((4,000 + 4,000) x 50%) (4,000)

Extract from balance sheet



Trade and other receivables 5,500

This part of the question asked for the preparation of extracts from the financial statements in relation
to three sales arrangements under both cash accounting and IAS 18. It was often impossible to
distinguish between which part of the answer was supposed to be cash accounting and which was
IAS 18.

Candidates obviously struggled with the concept of both cash and accrual accounting when related to
a scenario. Answers were poor in this area and often incomplete.

Common errors included failing to complete the double entry with the chosen revenue figure, to show
the appropriate receivable figure (under accrual accounting), including deferred income as an asset as
opposed to a liability, including a full years worth of interest of 900 rather than 6 months and
recognising all of the revenue of 11,000 rather than only half which represented how much work had
been completed.

Total possible marks 9


Maximum full marks 8

(c) Marks
Cash flow statement differences between IFRS and UK GAAP

Under UK GAAP a cash flow statement is based on the movement in cash rather than
cash and cash equivalents. Cash equivalents are instead treated as part of management
of liquid resources under UK GAAP.

Under UK GAAP a cash flow statement has eight headings rather than the three used
under IFRS.

UK GAAP includes an exemption for the preparation of a cash flow statement for small
entities and also in certain situations where you have a group of companies and
consolidated financial statements have been prepared which include a group cash flow
statement. No such exemptions are included under IFRS.

Total possible marks 3


Maximum full marks 3
The identification of the differences between UK GAAP and IFRS in relation to the cash flow
statements was answered well. The majority of candidates were able to identify that there were a
different number of headings under the two accounting regimes although some candidates wasted
time by writing out the headings in details, which was not required. The majority of candidates also
stated that UK GAAP uses pure cash rather than cash and cash equivalents.

Total possible marks 19


Maximum full marks 17

The Institute of Chartered Accountants in England and Wales 2008, Page 8 of 15


Professional Stage Financial Accounting March 2008

Question 3

General comments: this question considers three specific areas of accounting. The first is the acquisition of a
subsidiary, with fair value adjustments required, specifically in relation to intangible assets. The second area
considers the accounting required for an associate and the effect of an impairment and the final area is the
development of a non-current asset. The question is completed by the preparation of a statement of changes in
equity.

(a)

(i) Goodwill Hop Ltd

Fair value of net assets:


Net assets (FV)

Per question (500+18+250) 768,000
Staff retraining costs (15,000)
Start up costs (85,000)
Contingent liability (75,000)

Adjusted FV 593,000

Goodwill calculation

Fair value of consideration 400,000
Share of net assets acquired:
75% x 593,000 444,750
Discount on acquisition 44,750

(ii) Associate

Year end carrying amount


Cost 200,000
Share of post-acq change in net assets
25% x (300,000) (75,000)

125,000
Impairment in year (bal fig) (45,000)

Recoverable amount 80,000

Investment in associate at 31 Dec 2007 is 80,000

(iii) Property, plant and equipment

Land cost (1,200,000 + 20,000) 1,220,000

Building cost (1,600,000 72,000 + 56,000) 1,584,000

Depreciation: (1,584,000 / 30 x (6/12)) 26,400

The Institute of Chartered Accountants in England and Wales 2008, Page 9 of 15


Professional Stage Financial Accounting March 2008

Head Office

Cost
At 1 January 2007 1,220,000
Additions 1,584,000

At 31 December 2007 2,804,000

Depreciation
At 1 January 2007 -
Charge for the year 26,400

At 31 December 2007 26,400

Carrying amount
At 31 December 2006 1,220,000
At 31 December 2007 2,777,600

Total possible marks 10


Maximum full marks 10

The Institute of Chartered Accountants in England and Wales 2008, Page 10 of 15


Professional Stage Financial Accounting March 2008

(b)

Consolidated statement of changes in equity

Attributable to equity holders of


Lovage plc
Share
Minority
Share premiu Retained Total
interest
capital m earnings Total
Net profit for the
year (W2&W3) - - 208,600 208,600 33,750 242,350
Added on
acquisition of 148,250 148,250
subsidiary (W3) - - - -
Dividend declared
(W4) - - (63,000) (63,000) (63,000)
Share issue (W1) 160,000 240,000 - 400,000 - 400,000

1,615,00 2,515,00 - 2,515,00


Brought forward 900,000 - 0 0 0
3,060,60
Carried forward 1,060,000 240,000 1,760,600 0 182,000 3,242,600

Workings

W1 Share issue

160,000 shares issued:



Share capital (160,000 x 1) 160,000
Share premium (160,000 x 1.50) 240,000
400,000
W2 Net profit

Net profit for period:



Draft profit 425,000
Discount on acquisition (from a(i)) 44,750
Share of associates loss (from a(ii)) (75,000)
Impairment associate (from a(ii)) (45,000)
Overheads (72,000 56,000) (from a(iii)) (16,000)
Depreciation (from a(iii)) (26,400)
Relocation costs (200,000)
Hop Ltds profit share (75% x 135,000 (W)) 101,250

Profit for period 208,600

(W) Hop Ltds post-acquisition profit


Per question 150,000
Computer software amortisation (15,000)

135,000

Computer software 90,000 / 3 x 6/12months = 15,000

The Institute of Chartered Accountants in England and Wales 2008, Page 11 of 15


Professional Stage Financial Accounting March 2008

W3 Minority interest

Minority interest profit in year 135,000 x 25% = 33,750


Minority share of b/fwd reserves 593,000 (part a(i)) x 25% = 148,250

W4 Dividend

Dividend 7p x 900,000 = 63,000

The majority of candidates prepared the necessary proforma for the consolidated statement of changes in
equity but were often then unable to populate it. A common omission from the proforma was the total
column immediately before the minority interest column.
An average candidate correctly inserted the brought forward figures, calculated the share issue split
between share capital and premium and inserted a dividend figure, although this was often an incorrect
figure. The profit for the period was usually inserted, although unadjusted, and the minority interest
figures, including that added on acquisition.
Common errors included not making any adjustments to the parents draft profit for the year, if
adjustments to the parents draft profit for the year were made these were often shown on the face of
the SOCIE rather than in a working, failing to adjust the subsidiarys post-acquisition profits for the
amortisation of the computer software and failing to add the groups share of the subsidiarys post-
acquisition profits to the parents draft profit for the year.

Total possible marks 12


Maximum full marks 12

The Institute of Chartered Accountants in England and Wales 2008, Page 12 of 15


Professional Stage Financial Accounting March 2008

Question 4

General comments: this is a consolidation question. A consolidated income statement is required along with
the movement on retained earnings. The parent company has an investment in a subsidiary and an
associate. Adjustments are required for FV changes from the acquired subsidiary, a write down of inventory
and inter-company trading between the associate and parent company.

(a)

Hyssop plc
Consolidated income statement for the year ended 31 December 2007
000
Revenue (W2) 38,000
Cost of sales (W2) (24,075)

Gross profit 13,925


Operating expenses (W2) (7,300)

Profit from operations 6,625


Investment income (W2) 530
Share of profits of associate (W7) 960

Profit before tax 8,115


Income tax expense (W2) (2,400)

Profit for the period 5,715

Attributable to:
Equity holders of Hyssop plc (Bal) 5,540
Minority interest (W6) 175
5,715

(b) Consolidated movement on retained earnings

Retained
earnings
000
Balance at 31 Dec 2006 1,242
Profit for the period 5,540
Dividends (1,200)

Balance at 31 Dec 2007 5,582

Workings (All figures in 000)

W1 Group Structure
Hyssop
30% 80%

Arnica Sorrel

The Institute of Chartered Accountants in England and Wales 2008, Page 13 of 15


Professional Stage Financial Accounting March 2008

W2 Consolidation schedule
Hyssop Sorrel Adjustments Total
Revenue 28,500 9,500 38,000
Cost of sales
Per question (16,800) (7,200) (24,075)
Impairment inventory (W4) (50)
Fair value adj (W3) (25)
Operating expenses
Per question (6,460) (820) (7,300)
Impairment loss (20)
Investment income 1,150 20 *(640) 530
Tax (1,800) (600) (2,400)
PAT 875

* Elimination of intragroup dividend (80% x 200) + (30% x 1,600)

W3 Fair value adjustment



Fair value 700,000
Carrying amount 525,000
Difference 175,000

Additional annual depreciation 175,000 / 7 years = 25,000

W4 Inventory adjustment

Net realisable value - 2.50 50p = 2.00


Carrying amount - 2.20

Inventory write down (2.20 - 2.00) x 250,000 = 50,000

W5 Unrealised profit - associate

250,000 125%
(200,000) 100%
50,000 25%

Hs share - 50,000 x 30% = 15,000

W6 Minority interest

Sorrel Ltd (20% x 875 (W2)) = 175

W7 Associate

Profit for the year 3,250
Less: Intragroup trading (50)
3,200
Hyssops share x 30% 960
Less: impairment for year -
Share of associates profit 960

The Institute of Chartered Accountants in England and Wales 2008, Page 14 of 15


Professional Stage Financial Accounting March 2008

W8 Consolidated retained earnings brought forward

Hyssop plc per question 1,070


Sorrel Ltd (80% x (1,580 1,350 (2 x 25))) 144
Impairment - Sorrel (50)
Arnica ((30% x (750 240)) 75(imp)) 78
1,242

W9 Consolidated retained earnings carried forward

Hyssop plc per question 4,460


Inventory write off (50)
Sorrel Ltd (80% x (2,280 1,350 (3 x 25))) 684
Impairment (50 + 20) (70)
Arnica ((30% x (2,400 240 - 50)) 75(imp)) 558
5,582

Note this working is for tutorial purposes and is not required to obtain full marks.

A common failing on this question was time wasting by candidates who prepared standard balance
sheet workings which were simply not required.
In part a) almost all candidates produced an income statement in the correct format, with a split between
the group and the minority interest. However, many could not deal with the more complex adjustments
such as the trading between the parent entity and its associate.
Candidates answers were strong in preparing the inventory adjustment and the basic consolidation
schedule.
Common errors in answers included adjusting for the goodwill impairment against the subsidiarys
column in the consolidation schedule rather than against the parents column, treating the mark-up on
the inter-company sales as a margin and adjusting for the whole unrealised profit rather than only the
associates share.
Other common errors included adjusting for three years worth of depreciation, i.e the accumulated
depreciation, instead of just one year, and then including the adjustment in the parents column instead
of the subsidiarys column and not eliminating the subsidiarys and associates dividends.
Part b) required the movement on retained earnings to be shown. This illustrated that many candidates
did not appreciate the link between the income statement and the balance sheet. Candidates often
wasted time by including a column for the minority interest which was not required.
A common mistake was to calculate the brought forward figures using the carried forward figures and to
include the groups share of the subsidiarys and / or associates dividends in the dividends shown as a
reduction to retained earnings.

Total possible marks 20


Maximum full marks 19

The Institute of Chartered Accountants in England and Wales 2008, Page 15 of 15


Financial Accounting Professional Stage (New Syllabus)- June 2008

PROFESSIONAL STAGE FINANCIAL ACCOUNTING OT EXAMINERS COMMENTS

The performance of candidates in the June 2008 objective test section of the Professional Stage
Financial Accounting paper was good. Candidates performed well across all syllabus areas, although
slightly less well at this session on the preparation of consolidated financial statements than on the
other two syllabus areas.

Care should always be taken to ensure that the principles underlying any particular item are
understood rather than the answer learned from previous experience. In particular, candidates should
ensure that they read all items very carefully.

The following table summarises how well* candidates answered each syllabus content area.

Syllabus area Number of questions Well answered Poorly answered

LO1 4 4 0

LO2 5 4 1

LO3 6 4 2

Total 15 12 3

*If 50% or more of the candidates gave the correct answer, then the question was classified as well
answered.

Both of the items which were answered poorly on the preparation of consolidated financial statements
(LO3) tested the preparation of consolidated cash flow statements.

Item 1

This item tested what amount would be shown in respect of the acquisition of an associate during the
year in a consolidated cash flow statement. Most candidates incorrectly concluded that the amount
shown would be the net of the cash paid for the associate and the cash and cash equivalents held by
the associate at the date of acquisition. It may be that candidates misread the question as asking
about the impact of the acquisition of a subsidiary during the year, not an associate.

Item 2

This item tested what amount would be shown in respect of dividends paid to minority interest in a
consolidated cash flow statement. The information given included opening and closing balances on
the minority interest account, the profit attributable to the minority interest for the year and details of
a new subsidiary acquired during the year. The most commonly selected incorrect answer indicated
that candidates ignored the impact of the subsidiary acquired during the year on the minority interest
account.

The Institute of Chartered Accountants in England and Wales 2008 Page 1 of 17


Financial Accounting Professional Stage (New Syllabus)- June 2008

MARK PLAN AND EXAMINERS COMMENTARY

The marking plan set out below was that used to mark this question. Markers were encouraged to use discretion
and to award partial marks where a point was either not explained fully or made by implication. More marks
were available than could be awarded for each requirement. This allowed credit to be given for a variety of valid
points which were made by candidates.

Question 1 Total marks 27

General comments
Part (a) tested the preparation of financial statements (in this case an income statement and balance
sheet) from a trial balance plus a number of adjustments. Adjustments included write-downs to trade
receivables and inventories, a lease of land and buildings, and movements on non-current assets,
including an asset held for sale. Part (b) tested differences between the financial statements prepared
under IFRS in Part (a) and those which would have been prepared under UK GAAP, including marks for
recognising that the lease of land and buildings would be treated differently under the two bases.

Thirsk Ltd

(a) Income statement for the year ended 31 March 2008


Revenue (1,403,000 5,000) 1,398,000
Cost of sales (W1) (680,900)
Gross profit 717,100
Distribution costs (W1) (286,100)
Administrative expenses (W1) (323,100)
Profit from operations 107,900
Finance cost (W3) (3,000)
Profit before tax 104,900
Income tax expense (26,500)
Profit for the period 78,400

The Institute of Chartered Accountants in England and Wales 2008 Page 2 of 17


Financial Accounting Professional Stage (New Syllabus)- June 2008

Balance sheet as at 31 March 2008


Assets
Non-current assets
Property, plant and equipment (W2) 1,299,300

Current assets
Inventories (W1) 529,000
Trade and other receivables (419,200 30,000 5,000) 384,200
Cash and cash equivalents 15,900
929,100
Non-current asset held for sale (W6) 9,500
938,600
Total assets 2,237,900

Equity and liabilities


Capital and reserves
Ordinary share capital 500,000
Preference share capital (irredeemable) 100,000
Retained earnings (W4) 1,052,000
Equity 1,652,000
Non-current liabilities

Finance lease liabilities (W3) 195,500

Current liabilities
Trade and other payables 348,900
Taxation 26,500
Dividends payable (W4) 5,000
Finance lease liabilities (W3) 10,000
390,400
Total equity and liabilities 2,237,900

The Institute of Chartered Accountants in England and Wales 2008 Page 3 of 17


Financial Accounting Professional Stage (New Syllabus)- June 2008

Workings

(1) Allocation of expenses


Cost of sales Administrative Distribution
expenses costs

Per Q 675,400 316,600 286,100
Movement on bad debt provision (36,000 (6,000)
30,000)
Opening inventories 415,000
Adj re lease (20,000 x ) (10,000)
Closing inventories (525,000 + 4,000) (529,000)
Depreciation/impairment charges (W5) 119,500 22,500
680,900 323,100 286,100

(2) Property, plant and equipment


Land and Plant and Total
buildings equipment

Cost b/f 1,300,000 545,000
Leased building (W3) 212,500 -
Machine held for sale (31 December 2007) - (50,000)
Acc dep b/f (280,000) (326,700)
Eliminated on machine held for sale (W6) - 40,500
Depreciation/impairment charges for year (22,500) (119,500)
(W5)
1,210,000 89,300 1,299,300

(3) Lease of buildings

SOD = (24 x 25)/2 = 300



Total payments (20,000 x x 25) 250,000
Fair value (425,000 x ) (212,500)
Finance charge 37,500

Year ended 31 B/f Payment Capital Interest C/f


March

2008 212,500 (10,000) 202,500 (24/300 x 205,500
37,500) 3,000
2009 205,500 (10,000) 195,500 (23/300 x 198,375
37,500) 2,875

The Institute of Chartered Accountants in England and Wales 2008 Page 4 of 17


Financial Accounting Professional Stage (New Syllabus)- June 2008

(4) Retained earnings


At 1 April 2007 978,600


Preference dividend (100,000 x 5%) (5,000)
Profit for the period 78,400
At 30 September 2007 1,052,000

(5) Depreciation and impairment charges



Existing buildings ((1,300,000 600,000) 50) 14,000
Leased building (212,500 25) 8,500
22,500

Plant held throughout year ((545,000 50,000) x 20%) 99,000


On plant held for sale (13,000 + 7,500) (W6) 20,500
119,500

(6) Plant held for sale



Cost 50,000
Acc dep to 31 March 2007 (50,000 x 20% x 2) (20,000)
Dep for year (50,000 x 20% x 9/12) (7,500)
Carrying amount at classification as held for sale 22,500
Fair value less costs to sell (10,000 500) (9,500)
Impairment 13,000

The Institute of Chartered Accountants in England and Wales 2008 Page 5 of 17


Financial Accounting Professional Stage (New Syllabus)- June 2008

As in previous sittings, candidates were clearly well-prepared for this type of question. Almost all
candidates produced a well-laid out income statement and balance sheet in appropriate formats, although
some lost presentation marks by not adding across numbers in brackets or transferring numbers from
workings. Others lost presentation marks by failing to complete the sub-totals and/or totals on their
statements or by having incomplete or abbreviated narrative or no heading. Candidates should remember
that this type of question requires financial statements to be in a form suitable for publication.

Although many workings, in particular the cost matrix, were clearly laid out some candidates workings
were disorganised, untidy and therefore hard to follow, making it difficult to establish candidates
approaches where they had not calculated the correct figure.

Most candidates were able to deal with the more straightforward adjustments such as the increase in the
bad debt provision, the inventory write-down, the income tax charge/liability and the basic annual
depreciation charges. Most correctly classified the irredeemable preference shares as equity but not all
treated the dividends as such, instead including them as a finance cost.

Errors in dealing with the other adjustments included the following:


Failing to split the lease into a finance lease for the building and an operating lease for the land.
Depreciating the leased buildings over 50 years (the useful life for other buildings) instead of 25
years (the lease term) or not depreciating them at all.
Arriving at the incorrect sum of the digits figure.
Although candidates often correctly calculated the impairment on the plant held for sale few
followed this through fully by charging it to the income statement, including it in the
depreciation/impairment charge for the year in the property, plant and equipment working and
taking it out of that working when adjusting for the asset held for sale.
Failing to disclose the asset held for sale correctly on the balance sheet (within current assets after
a separate sub-total for all other current assets).
Treating the debit cash balance as an overdraft.
Showing the impairment amount as a liability rather than as a deduction from receivables.
Failing to properly account for the returned goods by reducing the revenue figure but not making
the corresponding entry to receivables. However, most candidates correctly adjusted closing
inventory for the cost of these goods.

Total possible marks 25


Maximum full marks 23

(b) Differences between IFRS and UK GAAP

Under UK GAAP, where formats are laid down by the Companies Act, the terminology in the balance
sheet would differ. For example, non-current assets would be referred to as fixed assets, trade and other
receivables would become trade and other debtors.

Under UK GAAP, the balance sheet would also usually be prepared on a net assets basis whereby the
top half of the balance sheet would show all assets and liabilities and the bottom half would show just
equity (capital and reserves) although other formats are in fact permissible.

Under UK GAAP, the income statement would be referred to as a profit and loss account and a
sub-total for operating profit would be required (FRS 3). Although this is not prohibited by IAS 1 it is
not explicitly required.

Specifically with regard to Thirsk Ltd, a difference between an IFRS and a UK GAAP balance sheet
might arise with regard to the treatment of the lease of land and buildings. Under IFRS (IAS 17) there
is a specific requirement to split leases of land and buildings at inception into a separate lease of
land and a separate lease of buildings. These should be classified appropriately, although, usually,
the lease of land will be an operating lease, and the lease of buildings, a finance lease.

Under UK GAAP (SSAP 21) the lease of land and buildings is considered as one lease and is usually
treated as an operating lease. Under UK GAAP therefore the whole of the 20,000 would have been
correctly taken to administrative expenses as operating lease rentals.

The Institute of Chartered Accountants in England and Wales 2008 Page 6 of 17


Financial Accounting Professional Stage (New Syllabus)- June 2008

Answers to Part (b) were mixed. Almost all candidates were able to state that the terminology would differ
and to give an example of this, but many answers did not go beyond this. Common failings included the
following:

Stating that the UKs version of the income statement is called a profit and loss as opposed to a
profit and loss account.
Stating that a UK balance sheet is prepared on a net assets basis but being unable to explain
this.
Giving differences between a UK cash flow statement and a cash flow statement prepared in
accordance with IAS 7 when the question was clearly restricted to differences between IAS 1 and
UK GAAP.
Giving differences between IFRS and UK GAAP in respect of the preparation of consolidated
financial statements when the question was clearly restricted to single entity financial statements.
A number of candidates referred to the UK having a statement of total recognised gains and
losses instead of a statement of changes in equity (although many referred to this as a STRGL
with no indication that they knew what this stood for) but very few mentioned the reconciliation of
movements in shareholders funds.
Failing to spot that the treatment of the lease of land and buildings would differ under UK GAAP.
Of those who did spot this, a number got the treatment the wrong way round.

Total possible marks 5


Maximum full marks 4

The Institute of Chartered Accountants in England and Wales 2008 Page 7 of 17


Financial Accounting Professional Stage (New Syllabus)- June 2008

Question 2 Total marks 16

General comments
This question tested the preparation of a single company cash flow statement and supporting note.
Missing figures to be calculated included interest paid, tax paid, dividends paid, property, plant and
equipment acquired and proceeds from the issue of share capital. A bonus issue of shares and a
revaluation of property, plant and equipment during the year also featured.

Wetherby plc

Cash flow statement for the year ended 31 March 2008



Cash flows from operating activities
Cash generated from operations (Note) 6,341,500
Interest paid (W1) (30,600)
Income tax paid (W2) (1,789,000)
Net cash from operating activities 4,521,900
Cash flows from investing activities
Purchase of property, plant and equipment (W3) (7,052,100)
Proceeds from sales of property, plant and equipment 1,556,500
(1,356,000 + 200,500)
Net cash used in investing activities (5,495,600)
Cash flows from financing activities
Proceeds from issue of ordinary share capital (500,000 2,150,000
(W4) + 1,650,000 (W5))
Proceeds from issue of borrowings 50,000
Dividends paid (W7) (898,200)
Net cash from financing activities 1,301,800
Net increase in cash and cash equivalents 328,100
Cash and cash equivalents at beginning of period 352,500
Cash and cash equivalents at end of period 680,600

Note: Reconciliation of profit before tax to cash generated from operations



Profit before tax 3,355,500
Finance cost 31,600
Depreciation charge 3,560,000
Profit on disposal of property, plant and equipment (200,500)
Increase in inventories (1,567,800 1,479,600) (88,200)
Increase in trade and other receivables (540,000 356,000) (184,000)
Decrease in trade and other payables ((1,678,500 4,000) (1,546,600 (132,900)
5,000))
Cash generated from operations 6,341,500

The Institute of Chartered Accountants in England and Wales 2008 Page 8 of 17


Financial Accounting Professional Stage (New Syllabus)- June 2008

Workings

(1) Interest paid


Cash () 30,600 B/d 4,000
C/d 5,000 IS 31,600
35,600 35,600

(2) Tax paid


Cash () 1,789,000 B/d 1,670,000
C/d 1,450,000 IS 1,569,000
3,239,000 3,239,000

(3) PPE


B/d 15,299,900 Disposal 1,356,000
Revaluation reserve (1,560,000 + 1,830,000 Depreciation 3,560,000
270,000)
Additions () 7,052,100 C/d 19,266,000
24,182,000 24,182,000

(4) Share capital


B/d 5,000,000
Share premium (bonus 500,000
issue)
C/d 6,000,000 Cash () 500,000
6,000,000 6,000,000

(5) Share premium


Share capital (bonus issue) (W4) 500,000 B/d 600,000
C/d 1,750,000 Cash () 1,650,000
2,250,000 2,250,000

Note: The bonus issue could be taken out of retained earnings since the reserve to be used is not
specified by the question.

(6) Retained earnings


Dividends in SCE () 748,200 B/d 7,689,500
Revaluation reserve 270,000
C/d 8,997,800 IS 1,786,500
9,746,000 9,746,000

The Institute of Chartered Accountants in England and Wales 2008 Page 9 of 17


Financial Accounting Professional Stage (New Syllabus)- June 2008

(7) Dividends paid


Cash () 898,200 B/d 400,000
C/d 250,000 SCE (W6) 748,200
1,148,200 1,148,200
This was the first time this topic had been set and candidates were clearly very well prepared for it. Many
scored high marks on the reconciliation note, and on the figures for tax paid, interest paid, proceeds from
the issue of borrowings, proceeds from the sale of property, plant and equipment and the opening and
closing figures for cash and cash equivalents. Presentation was generally good. Most candidates
produced workings in the form of T accounts and very few made the mistake of putting opening and
closing balances on the wrong side of those T accounts. Generally, candidates showed a good grasp of
basic double entry principles, which underpin the preparation of a cash flow statement, whether single
company or consolidated.

Where errors were made they included the following:


Failing to adjust the trade payables figure for the opening and closing accrued interest.
Adding the profit on disposal of property, plant and equipment in the reconciliation note instead of
deducting it.
Treating the 50,000 proceeds from the issue of borrowings as a cash outflow rather than as a
cash inflow.
Failing to account for the bonus issue (or putting through the credit entry but no debit entry
particularly common where candidates combined the share capital and share premium accounts).
Accounting for the reserve transfer in the retained earnings T account but omitting the other side
of the entry from the property, plant and equipment T account (or via a revaluation reserve T
account).
Failing to adjust dividends for the year as calculated in a retained earnings T account for opening
and closing dividends payable in order to arrive at dividends paid.

Total possible marks 16


Maximum full marks 16

The Institute of Chartered Accountants in England and Wales 2008 Page 10 of 17


Financial Accounting Professional Stage (New Syllabus)- June 2008

Question 3 Total marks 11

General comments
Part (a) required the preparation of a statement of changes in equity for a single company, featuring an
adjustment for interest on redeemable preference shares, a revaluation during the year and a share issue.
In Part (b) the resultant profit figure had to be combined with figures for a subsidiary acquired during the
year and an associate to generate the consolidated profit figure, split between the group and the minority.
In arriving at this figure there was also an adjustment to be made for the unrealised profit on the sale of an
item of plant within the group.

Doncaster plc
(a) Statement of changes in equity for the year ended 31 March 2008

Ordinary Share Revaluation Retained


share premium reserve earnings
capital

Recognised directly in equity
Revaluation of non-current 409,500
assets (650,000 (456,000
215,500))
Transfer between reserves (22,200) 22,200
(67,800 45,600)
Profit for the period (526,700 476,700
(1,000,000 x 5%))
Total recognised income and 387,300 498,900
expense for the period
Issue of share capital 250,000 300,000
250,000 300,000 387,300 498,900
Balance brought forward 1,000,000 2,365,500
Balance carried forward 1,250,000 300,000 387,300 2,864,400

This part of the question was very well-answered. Statements of changes in equity were well laid out with,
as instructed, no total column, although very few candidates drew a sub-total of total recognised income
and expense for the period. Where errors were made they included the following:
Failing to adjust the profit for the year by the finance cost on the redeemable preference shares.
Some showed this as dividends in the statement, others showed it separately as a finance cost,
when it would not appear separately in the statement.
Calculating the revaluation gain arising in the year incorrectly, based on the cost of the asset
being revalued as opposed to its carrying amount.
Making an incorrect adjustment for the transfer in respect of the additional depreciation on the
revalued asset, when the relevant figures were given in the question.

Total possible marks 5


Maximum full marks 5

The Institute of Chartered Accountants in England and Wales 2008 Page 11 of 17


Financial Accounting Professional Stage (New Syllabus)- June 2008

(b) Profit attributable for the year ended 31 March 2008

Doncaster plc Redcar Ltd



Net profit per own accounts (a) (457,000 x 6/12) 476,700 228,500
Share of associates profits (103,400 x 45%) 46,530
Non-current asset PURP (40,000 (30,000 x 4/5)) (16,000)
Additional depreciation on non-current asset (((40,000 2,000
4) (30,000 5)) x )
Impairments (50,000 + 10,000) (60,000)
463,230 214,500
Profit attributable to the minority interest (214,500 x (53,625)
25%)
Profit attributable to the equity holders of Doncaster plc 463,230 160,875
= 624,105

Candidates continue to struggle with extracts type questions. Layouts were disorganised and difficult to
follow which at times made it difficult to award credit.

In spite of the fact that the status of the investments (as subsidiary or associate) and the percentage
shareholdings were given in the question, the vast majority of candidates wasted time drawing up a group
structure diagram for which no marks were available. This indicates candidates over-reliance on learnt
techniques.

Common errors included the following:


Failing to adjust the profit of the subsidiary to allow for the fact that it was acquired half way through
the year.
Taking minority interest at the wrong point (for example, correctly calculating the subsidiarys profit
for the year as half of the figure for the whole year but then failing to apply the minority percentage
to the net figure).
Although a number of candidates correctly calculated the net adjustment in respect of the plant sold
within the group, far less correctly made this adjustment against the subsidiary. Of those who did,
less still then calculated the minority interest (see above) on this net figure.
When calculating the additional depreciation on the plant sold within the group a number of
candidates failed to base this adjustment on a half years charge (as the subsidiary was acquired
half way through the year and the transfer made on this date).
A disappointing number of candidates omitted to add the parents share of the associates profit
into the profit attributable to the equity shareholders of the parent. Others omitted to add in the
parents own profit figure.
A significant minority of candidates calculated a minority interest for the associate as well as for the
subsidiary.
Reducing the subsidiarys and the associates profits by the given impairments instead of reducing
the parents profits. Where candidates did this they often dealt correctly with similar impairments in
question four, illustrating yet again that candidates cope less well with topics tested in a less
familiar way.

Total possible marks 6


Maximum full marks 6

The Institute of Chartered Accountants in England and Wales 2008 Page 12 of 17


Financial Accounting Professional Stage (New Syllabus)- June 2008

Question 4 Total marks 26

General comments
Part (a) was a consolidated balance sheet question, featuring one subsidiary and one associate.
Adjustments were typical of this type of question and included a fair value adjustment on acquisition, intra-
group balances and transactions and impairment write-downs. Part (b) tested an understanding of the
concepts underlying the preparation of consolidated financial statements: namely the single entity concept
and control versus ownership.

York plc

(a) Consolidated balance sheet as at 31 March 2008


Assets
Non-current assets
Property, plant and equipment (3,963,900 + 1,686,900) 5,650,800
Intangibles (W3) 192,000
Investments in associates (W7) 261,920
6,104,720
Current assets
Inventories (860,000 + 650,000 35,000 (W6) 1,463,000
30,000 x 40% (W6))
Trade and other receivables (730,000 + 540,000 1,060,000
210,000 (W6))
Cash and cash equivalents (29,600 + 15,500) 45,100
2,568,100
Total assets 8,672,820

Equity and liabilities


Capital and reserves
Ordinary share capital 2,000,000
Share premium account 1,000,000
Retained earnings (W5) 3,424,060
Attributable to the equity holders of York plc 6,424,060
Minority interest (W4) 414,160
Equity 6,838,220
Current liabilities
Trade and other payables (878,000 + 546,600 1,214,600
210,000 (W6))
Taxation (380,000 + 240,000) 620,000
1,834,600
Total equity and liabilities 8,672,820

The Institute of Chartered Accountants in England and Wales 2008 Page 13 of 17


Financial Accounting Professional Stage (New Syllabus)- June 2008

Workings

(1) Group structure

200
York plc = 40%
500

800
= 80%
1,000

Ripon Ltd
Beverley Ltd

(2) Net assets Ripon Ltd

Balance Acquisition Post acq


sheet date

Share capital 1,000,000 1,000,000 -
Share premium 500,000 500,000 -
Retained earnings
Per Q 625,800 (215,000)
PURP (W6) (35,000) -
Amortisation adj intangible 30,000 - 835,800
FV adj intangible (50,000) (50,000) -
2,070,800 1,235,000 835,800

(3) Goodwill Ripon Ltd


Cost of investment ((1,000,000 x 1.20) + 400,000) 1,600,000
Less Share of FV of net assets acquired (1,235,000 (W2) x 80%) (988,000)
612,000
Impairments to date (400,000 + 20,000) (420,000)
192,000

The Institute of Chartered Accountants in England and Wales 2008 Page 14 of 17


Financial Accounting Professional Stage (New Syllabus)- June 2008

(4) Minority interest Ripon Ltd



Share of net assets (2,070,800 (W2) x 20%) 414,160

(5) Retained earnings

York plc 3,175,500


Ripon Ltd (835,800 (W2) x 80%) 668,640
Beverley Ltd ((210,800 56,000 30,000 (W6)) x 40%)) 49,920
Less Impairments to date (420,000 (W3) + 50,000) (470,000)
3,424,060

(6) PURP

Ripon Ltd Beverley Ltd


%

SP (210,000/180,000 x ) 150 105,000 90,000


Cost (140,000/120,000 x ) (100) (70,000) (60,000)
GP 50 35,000 30,000

(7) Investments in associates Beverley Ltd



Cost (200,000 x 1.25) 250,000
Add: Share of post acquisition increase in net assets ((210,800 56,000)) x 61,920
40%))
Less: Impairment to date (50,000)
261,920
Note: Candidates who correctly calculated a discount on acquisition of the
associate of 12,400 and dealt with it appropriately were also given credit.

The Institute of Chartered Accountants in England and Wales 2008 Page 15 of 17


Financial Accounting Professional Stage (New Syllabus)- June 2008

Candidates were clearly very well prepared for this question and generally scored highly. Almost all
candidates demonstrated a sound technique and most dealt easily with the required adjustments. Errors
included the following:

Failing to adjust both receivables and payables for the invoice value of the sale of goods from the
subsidiary to the parent, with a number of candidates making the adjustment at cost.
Calculating unrealised profit based on the full invoice value, as opposed to only half of that value,
when the question clearly stated that only half of the goods remained in year-end inventory.
Taking the cost figures given in the question for the intra-group sale as being the selling price of
the goods and hence calculating an incorrect provision for unrealised profit.
Treating the pre-acquisition loss of the subsidiary as a pre-acquisition profit.
Failing to include the share premium account correctly in the net assets table for the subsidiary.
Not adjusting for the accumulated impairments in the group retained earnings working, instead
adjusting only for the impairments which had arisen during the current year.
Pleasingly, many candidates correctly adjusted for the group share of the provision for unrealised
profit arising on goods sold by the associate to the parent against retained earnings and inventory,
but many also made an adjustment against the carrying amount of the associate. Others
calculated an initial post-acquisition profit figure for the associate less a 40% share of the
provision for unrealised profit but then adjusted that total by 40%, consequently scaling down the
provision for unrealised profit twice.
Only a minority of candidates correctly dealt with the goodwill in the subsidiarys own books.
Others took 20,000 out of the subsidiarys net assets at both acquisition and at the balance sheet
date (instead of 50,000 out at acquisition and 20,000 at the balance sheet date) and some were
clearly confused between this goodwill and that arising on consolidation.
When calculating the cost of investment in the subsidiary a significant number of candidates failed
to allow for an issue price of 1.20 per share, as given in the question, and instead assumed that
the shares were issued at par.

A number of candidates failed to provide workings for assets and liabilities on the face of the consolidated
balance sheet. Where these balance sheet figures were incorrect no partial marks could then be awarded.
Candidates must show their workings in all cases so that partial credit can be given.

A number of candidates also failed to complete the consolidated balance sheet, but rather abbreviated line
items and/or included partial workings, which were not totalled, although this had improved since a similar
question was last set. As the question required the preparation of a consolidated balance sheet,
candidates are expected to complete all additions and present a complete balance sheet. Very few
candidates gained the presentation marks which were available for clearly disclosing the minority interest
as a separate component of equity.

Total possible marks 21


Maximum full marks 21

The Institute of Chartered Accountants in England and Wales 2008 Page 16 of 17


Financial Accounting Professional Stage (New Syllabus)- June 2008

(b) Concepts underlying preparation of consolidated financial statements

Group accounts are prepared on the basis that the group is a single entity (single entity concept). This
reflects the substance of the group arrangement.

For example, in the consolidation of the York plc group, all assets and liabilities are added together, as if the
group were a single entity (so, for example, trade receivables of 730,000 and 540,000 are added).
However, the single entity concept also means that any intra-group transactions and balances need to be
eliminated, as otherwise items would be double counted in the context of the group as a single entity.

Hence, because Ripon Ltd has sold goods for 210,000 to York plc, that amount needs to be
subtracted from York plcs cost of sales and from Ripon plcs revenue as if the group were a single
entity that transaction would not have occurred. That adjustment cannot be seen in the context of the
preparation of a consolidated balance sheet, though it would be seen in the preparation of a
consolidated income statement.

In the context of the consolidated balance sheet any related intra-group balances need to be
eliminated. This amount is included in York plcs trade payables and Ripon Ltds trade receivables as
this amount is unpaid at the year end. It needs to be eliminated from both.

Any profit made between parent and subsidiary companies also needs to be eliminated where that profit has
not yet been realised outside the group. So, for the 210,000 intra-group sale, because half of these goods
have not yet been sold outside the group, inventory needs to be reduced by the profit on half that amount,
otherwise inventory will be overstated from the point of view of the group as a whole. The adjustment
effectively brings inventory back down to what it would have been stated at if the intra-group sale has never
taken place.

The other principle underlying the preparation of consolidated financial statements is the distinction between
control and ownership. Control is reflected by including all of the subsidiarys assets, liabilities, income and
expenses in the consolidated financial statements, even where the parent does not own 100% of that
subsidiary. So, for York plc, 100% of Ripon Ltds inventories of 650,000 are added in even though, in
effect, York plc only owns 80% of those inventories.

Ownership is then reflected by showing that part of the subsidiarys net assets and results included in the
consolidation, which is not owned by the parent, as a minority interest. York plcs consolidated balance
sheet shows a minority interest of 414,160, representing that part of Ripon Ltd not owned by York plc.

Where an investor (York plc) does not have control but does have significant influence over an investee
(Beverley Ltd), line-by-line consolidation is not appropriate, because York plc cannot determine Beverley
Ltds assets and liabilities. But because York plc has this influence, it should be accountable for the total
investment in Beverley Ltd, ie cost plus share of post-acquisition retained earnings (the latter are added to
group earnings).

As with previous papers, the quality of written answers was disappointing. In common with the other written
part of this paper, some candidates made no attempt at this part.

Whilst most candidates were able to pick up marks for referring to the single entity concept and substance
over form few got beyond this. Almost no candidates scored 5 or even 4 marks on this part of the question,
in spite of the number of marks available. Many wasted time discussing the techniques used for
consolidating financial statements and/or the factors which might indicate control or significant influence. A
number thought the question was about the qualitative characteristics of financial information and based
their answer around those or discussed the advantages of consolidated financial statements. Although
some of these approaches enabled candidates to pick up the odd extra mark scores were generally low.
Very few candidates related their answer to York plc as specified in the requirement.
Total possible marks 8
Maximum full marks 5

The Institute of Chartered Accountants in England and Wales 2008 Page 17 of 17


Professional Stage Financial Accounting September 2008

PROFESSIONAL STAGE FINANCIAL ACCOUNTING OT EXAMINERS COMMENTS

The performance of candidates in the September 2008 objective test questions section for the
Application Stage Financial Accounting paper was good. Candidates performed well across all
syllabus areas.

However, care should always be taken to ensure that the principles underlying any particular item
are understood rather than the answer learned from previous experience. In particular, candidates
should ensure that they read all items very carefully.

The following table summarises how well* candidates answered each syllabus content area.

Syllabus area Number of questions Well answered Poorly answered

LO1 4 3 1

LO2 5 3 2

LO3 6 5 1

Total 15 11 4

*If 50% or more of the candidates gave the correct answer, then the question was classified as
well answered.

The poorly answered questions covered a range of different syllabus areas. Comments on two
items of particular note are:

Item 1

This item tested the impact that a restoration obligation has on the asset and provisions. There
appeared to be a lack of knowledge in this area with candidates either not realising that the
provision is added to the cost of the asset or believing that a provision is built up over time.

Item 2

This item required a calculation of the minority interest dividend in relation to a consolidated cash
flow statement. A subsidiary, with a minority interest, had been acquired during the period;
however candidates failed to adjust for this in their workings.

The Institute of Chartered Accountants in England and Wales 2008 Page 1 of 14


Professional Stage Financial Accounting September 2008

MARK PLAN AND EXAMINERS COMMENTARY

The mark plan set out below was that used to mark these questions. Markers are encouraged to use
discretion and to award partial marks where a point was either not explained fully or made by implication.
More marks are available than could be awarded for each requirement, where indicated. This allows credit to
be given for a variety of valid points, which are made by candidates.

Question 1 - Total marks 19

General comments: This wqs a typical question testing the preparation of an income statement and
statement of changes in equity from a trial balance plus a number of adjustments. Adjustments included a
held for sale asset, the correction of an error, a reorganisation and a share issue.

Agate Ltd Income Statement for year ended 30 June 2008



Revenue (W1) 1,907,500
Cost of sales (W2) (688,250)

Gross profit 1,219,250

Administrative expenses (W2) (1,036,750)


Distribution costs (W2) (145,000)
Net operating costs (6,700 30,400) (23,700)

13,800

Finance costs (19,300 + (30,000 x 5%)) (20,800)


Profit before tax (7,000)
Taxation (5,000 + 2,000) (7,000)

Net loss for the period (14,000)

Note: Marks will be awarded if items are included in a different line item in the income statement
provided that the heading used is appropriate.

Statement of changes in equity for the year ended 30 June 2008

Ordinary Share Retained Reval. Total


share premium earnings reserve
capital
Loss for the period - - (14,000) - (14,000)
Final dividend ((31,000 + - - (7,420) - (7,420)
75,000) x 7p))
Issue of shares 75,000 97,500 - - 172,500
(75,000 x 1 / 1.30)
Revaluation of non-
current assets - - - 171,000 171,000
(900,000 729,000)

75,000 97,500 (21,420) 171,000 322,080


Balance b/fwd
As previously stated 31,000 15,000 22,500 - 68,500
Correction of error - - (7,350) - (7,350)

Balance c/fwd 106,000 112,500 (6,270) 171,000 383,230

The Institute of Chartered Accountants in England and Wales 2008 Page 2 of 14


Professional Stage Financial Accounting September 2008

W1 Revenue

Trial balance 1,925,000


Advertising adjustment
(30,000 x 14/24 months) (17,500)

1,907,500

W2 Allocation of expenses

Cost of sales Administrative Distribution


expenses costs

Per question: 230,000 145,000


Purchases 367,000
Manufacturing costs 291,300
Opening inventory 17,000
Impairment (W4) 7,500
Depreciation (W3) 24,950
Less: closing inventory (19,500)
Relocation expenses (1,500,000 700,000) 800,000
Patent amortisation (45,000 / 5yrs x 9/12) 6,750

688,250 1,036,750 145,000

W3 Plant and equipment

Depreciation charge for the year 28,700


Less: IFRS 5 classified as held for sale (3,750)
(75,000 x 10% x 6/12)

As at 30 June 2008 24,950

W4 Held for sale asset

Carrying amount 45,000

Fair value 39,000


Less: costs (1,500)
37,500

Impairment (45,000 37,500) 7,500

Consistent with previous sittings candidates were well-prepared for this type of question and generally
answered it well. This question was slightly different in its requirements compared with previous papers
as the preparation of a balance sheet was replaced by the statement of changes in equity. As a result of
this change candidates found this more challenging compared with previous sittings.

Almost all candidates produced a well laid out income statement. The format of the statement of
changes in equity was slightly more varied and often had no total column. A common omission was the
date for one or both of the statements. A significant minority of candidates prepared a balance sheet as
they clearly had not read the requirement. Candidates should be reminded that question requirements
may vary from paper to paper.

Most candidates were able to take items from the trial balance and insert them in the correct place in the
formats, with most candidates using an efficient matrix style costs working. Marks were awarded where
presentation differed to the marking guide but resulted in a reasonable alternative. For example, the
patent amortisation being included in costs of sales rather than administrative expenses.

The Institute of Chartered Accountants in England and Wales 2008 Page 3 of 14


Professional Stage Financial Accounting September 2008

Candidates generally dealt with the share issue and the dividend correctly in the statement of changes
in equity, although the dividend payment was occasionally also incorrectly shown in the income
statement.

Common errors included a wrong adjustment to revenue, not appreciating that the amortisation of the
patent should have been for less than a year, failing to deal with the prior period error, missing that the
coupon on the redeemable preference shares should have been included as part of finance costs and
incorrectly calculating the relocation expenses.

Total possible marks 19


Maximum full marks 19

The Institute of Chartered Accountants in England and Wales 2008 Page 4 of 14


Professional Stage Financial Accounting September 2008

Question 2 Total marks 28

General comments: This question represented a combination of preparing extracts from the financial
statements (in this case from both the income statement and balance sheet) together with a written
conceptual part (b) and part (c) on the differences between IFRS and UK GAAP.

(a)
Extract from income statement for year ended 30 June 2008


Operating expenses to include:
Depreciation (W7) 91,200
Management expenses 72,000
Loss on disposal (W3) 4,500

Finance charge (W6) 3,500

Extract from balance sheet as at 30 June 2008


Non-current assets
Property, plant and equipment (W7) 2,123,300

Non-current liabilities
Finance lease liabilities (W6) 27,500

Current liabilities
Finance lease liabilities (W6) 4,000

W1 New building

Architects fees 23,000
Legal costs 7,000
Project management fees 30,000
Building costs 375,000

435,000
Less: lift cost (15,000)

As at 1 May 2008 420,000

W2 Plant and machinery


Cost Dep. Acc. Dep.
charge

B/fwd 160,000 48,000
Less: scrapped machine (18,000) X 15% x 5 yrs (13,500)
Additions - leased plant (W6) 60,000

202,000 X 15% 30,300

Lift (W1) 15,000 / 10yrs x 2/12 250

Scrapped machine (W3) 18,000 x 15% x 6/12 1,350


Dep. charge in year 31,900 31,900
C/fwd 217,000 66,400

The Institute of Chartered Accountants in England and Wales 2008 Page 5 of 14


Professional Stage Financial Accounting September 2008

Carrying amount at 30 June 2008 (217,000 66,400) = 150,600

W3 Scrapped machine

Cost 18,000
Accumulated depreciation (W2) (13,500)

Carrying amount at disposal / loss on disposal 4,500

W4 Fixtures and fittings


Cost Dep. Acc. dep
charge

B/fwd 75,000 36,000
Less: accumulated depreciation (36,000)

Carrying amount 39,000 X 25% 9,750

B/fwd - cost 75,000


Additions:
1 Oct 2007 20,000 X 25% x 9/12 3,750

13,500 13,500

C/fwd 95,000 49,500

Carrying amount at 30 June 2008 (95,000 49,500) = 45,500

W5 Land and buildings


Cost Dep. charge Accumulated
depreciation

Land
Cost 650,000

Buildings
B/fwd 1,075,000 172,000

Depreciation (1,075,000 x 4%) 43,000


Additions (dep 420,000 x 4% x 2/12) 420,000 2,800

Dep. charge in year 45,800 45,800


C/fwd 2,145,000 217,800

Carrying amount at 30 June 2008 (2,145,000 217,800) = 1,927,200

W6 Leased asset

Deposit 25,000
Instalments (7,000 x 7) 49,000

74,000
Fair value of asset (60,000)

Finance charges 14,000

The Institute of Chartered Accountants in England and Wales 2008 Page 6 of 14


Professional Stage Financial Accounting September 2008

SOTD = (7 x 8) = 28
2
B/fwd (60,000 25,000 = 35,000)

Year end B/fwd Interest Payment C/fwd



30 June 2008 35,000 3,500 (7,000) 31,500
30 June 2009 31,500 3,000 (7,000) 27,500

31,500
> 1yr < 1yr

27,500 4,000 (31,500 27,500)

Tutorial note: Fixed asset carrying amounts & depreciation charge

Depreciation Carrying amounts


charge

Plant & machinery 31,900 150,600
Fixtures & fittings 13,500 45,500
Land & buildings 45,800 1,927,200
91,200 2,123,300

The answers to this question were quite varied. Most candidates made a good attempt at part a) and
there had clearly been an improvement in candidates ability since December 2007 when a similar
question had been set. There was a clear improvement in the calculation of individual balances,
however, poor layouts often meant that it was hard to see what exactly had been included in a total and
what hadnt. Often candidates seemed to prepare what looked like a random set of workings with no
linkage. Workings must be referenced clearly.

Most candidates coped well with the calculations in relation to the leased asset, the elements of the new
building, cost calculations and depreciation on the assets held for a complete year. Assets that had
been held for less than a full year often saw an incorrect calculation of the number of months that the
asset should have been depreciated for.

The majority of candidates scored well in the lease calculation, although there were a number of
common errors, such as, showing the current lease liability as 7,000 rather than deducting the interest
from this amount. Another common mistake was to include the lease payment of 7,000 in the income
statement along with the finance charge. These errors show that there is clearly a lack in understanding
for accounting for a finance lease. Candidates appear to have simply learnt how to construct the leasing
table.

A minority of candidates produced a detailed PPE note along with additional notes in relation to the
leased asset even though these were not required by the question and therefore wasted time that could
have otherwise been spent productively.

Other common errors included correctly removing the 72,000 management cost from the cost of the
building but then failing to charge this amount in the income statement, including 60,000 as the brought
forward figure in the leasing table, an incorrect calculation of the sum-of-the-digits and failing to notice
that the fixtures and fittings should have been depreciated on a reducing balance basis.

Total possible marks 21


Maximum full marks 20

The Institute of Chartered Accountants in England and Wales 2008 Page 7 of 14


Professional Stage Financial Accounting September 2008

(b) Elements of financial statements


Assets/liabilities
A non-current asset acquired under a finance lease meets the definition of an asset, even though the
asset is not legally owned by them, as it is:
Controlled by the lessee, as they have physical possession of the asset
Results from a past event, the lease was signed at a particular date
Gives rise to future economic benefits, the lessee uses the asset to generate revenue for the
company.
The lease payments are a liability as the company has an obligation arising from a past event to transfer
economic benefits. The economic benefits that the lessee is obliged to transfer are the lease payments.
Generally, the lessee will have some kind of legal obligation to make the lease payments.
Recognition
The asset should be recognised if:
It is probable that future economic benefits will flow to the company; and
Those benefits can be measured reliably.
Conversely, the liability should be recognised if:
It is probable that future economic benefits will be made by the company; and
Those benefits can be measured reliably.
The inflows and outflows will be probable as a lease contract agreement has been signed and the benefits
can be reliably measured as the lease contract will set out the present value of the minimum lease
payments.

The answers to part b) were often disappointing with a significant number of candidates explaining the
accounting treatment for a finance lease with no reference to the IASB Framework, this approach gained
no marks. Candidates often copied text straight from the open book text without explaining further how it
related to a finance lease. Many candidates recognised that the signing of the lease was the past event
which created the lease obligation but few managed to explain how economic benefits would be obtained.

Total possible marks 7


Maximum full marks 5

(c) Key differences between IFRS and UK GAAP


IAS 17 lists a number of factors that would indicate that the risks and rewards of ownership have been
transferred to the lessee in order for the lease to be classified as a finance lease. Such as, the lease term
is for a major part of the assets life and the lessee is responsible for repairs and maintenance.
However, under UK GAAP, SSAP 21, there is a rebuttable presumption that if, at the inception of the
lease, the present value of the minimum lease payments is at least 90% of the assets fair value then
there is a finance lease.
IAS 17 specifically requires a lease which covers both land and building to be split at inception into two
leases, one for the land and one for the buildings. A lease for land will normally be an operating lease
since land will normally have an indefinite life.
Under UK GAAP there is no requirement to split a lease which covers both land and buildings. Such a
lease will therefore normally be recognised as an operating lease.

The Institute of Chartered Accountants in England and Wales 2008 Page 8 of 14


Professional Stage Financial Accounting September 2008

The answers to part c) were mixed. Most candidates wrote something regarding the minimum lease
payment being 90% of the fair value of the asset under a finance lease under UK GAAP. This was
commonly referred to as the 90% rule without further explanation as to what exactly this was. A typical
answer included some explanation about land and buildings and whether they should be separated or not
under IFRS and UK GAAP. However, a number of candidates were clearly confused about which regime
separates leases of land and buildings and which does not. The majority of candidates also stated that
land would always be an operating lease, with only the very best candidates appreciating that while this
might normally be the case it was possible for land to be held under a finance lease.

Total possible marks 4


Maximum full marks 3

The Institute of Chartered Accountants in England and Wales 2008 Page 9 of 14


Professional Stage Financial Accounting September 2008

Question 3 Total marks 16

General comments: This question was split into two distinct parts. Part (a) asked for the preparation of a
consolidated income statement and related information following the acquisition of a subsidiary and
associate. Part (b) asked for calculations of inventory.

(a)(i)

Consolidated income statement for the year ended 30 June 2008

000

Revenue 40,000
Cost of sales (35,975)

Gross profit 4,025


Operating costs (3,010)

Share of profit of associate (W2) 456

Profit before tax 1,471


Taxation (375)
Profit after tax 1,096

Attributable to:
Equity holders of Spinal plc (1,096 85) 1,011
Minority interest (W4) 85
1,096

a)(ii) Goodwill calculation:


Consideration 850,000
Less: share of net assets acquired (681,000)
(908,000 x 75%)

Goodwill 169,000

W1 Consolidation schedule

Spinal plc Carnelian Adj Consolidated


Ltd
000 000 000 000
(x 3/12)

Revenue 36,340 3,800 (140) 40,000


Cost of sales (33,920) (2,175) 140 (35,975)
PURP (W3) (20)

Gross profit 1,605 4,025


Operating costs (1,980) (1,090) (3,010)
Management fee 60

Investment income 60 (60) -


Profit before tax 515 1,015
Tax (200) (175) (375)
Profit after tax 340

The Institute of Chartered Accountants in England and Wales 2008 Page 10 of 14


Professional Stage Financial Accounting September 2008

W2 Associate profit


Share of profit after tax (1,670,000 x 30%) 501,000
Less: Current year impairment (45,000)

456,000

W3 PURP

(140,000 x (0.40 / 1.40)) x = 20,000

W4 - Minority interest

Profits attributable to the minority interest in year (340 (W1) x 25%) = 85,000

Almost all candidates made a good attempt at part a), with many obtaining full marks on this part of the
question. The most common errors were to not apportion the income statement figures for the subsidiary
acquired during the year and to miss the impairment of the associate.

A significant number of candidates included drawing a group structure even though the percentages were
given in the question. In such circumstances no marks are allocated for such a diagram. Other common
errors included failing to account for the intra-group sales or adjusting it in the wrong place and producing
balance sheet extracts, including attempting to calculate a balance sheet figure for the associate, when
these were not required.

Total possible marks 9


Maximum full marks 9

(b)

Raw materials

Raw metal in closing inventory = (1,050 x 50 tonnes) + (950 x 10 tonnes) = 62,000

Work-in-progress & finished goods

Total output 5,000 + (80% x 500) = 5,400 units

Cost per
unit

Raw material 195,000
Direct labour 253,200

448,200 448,200 / 5,400 83.00

Admin costs 107,000


Factory overheads 155,800

262,800 262,800 / 6,000 43.80

126.80

WIP Inventory = 500 x 80% x 126.80 = 50,720

Finished goods inventory = 350 x 126.80 = 44,380

The Institute of Chartered Accountants in England and Wales 2008 Page 11 of 14


Professional Stage Financial Accounting September 2008

Attempts at part b) were generally disappointing. This was the most commonly missed part of the paper
overall, with candidates simply not attempting it. There were also a significant number of candidates who
stopped their calculations at raw materials. This highlighted that a worrying number of candidates simply
do not have a broad level of knowledge across the syllabus. This part of the question was straight
forward in nature and required simple application of inventory accounting.

The most common errors included:


not including administrative costs, but including unplanned maintenance costs, in the calculation
of a cost per unit;
not basing one or both of administrative costs and factory overheads on budgeted production as
opposed to actual production;
basing actual production on the complete number of units produced rather than adding the 500
units that were only 80% complete;
valuing finished goods at NRV even when this was higher than the calculated cost per unit; and
ignoring that the work in progress was only 80% complete.

Total possible marks 7


Maximum full marks 7

The Institute of Chartered Accountants in England and Wales 2008 Page 12 of 14


Professional Stage Financial Accounting September 2008

Question 4 Total marks 17

General comments: This question was a consolidated cash flow statement question with the disposal of a
subsidiary.

(a)

Consolidated cash flow statement for the year ended 30 June 2008


Note: Cash flow from operating activities
Cash generated from operations (note) 161,610
Interest paid (14,400)
Tax paid (W2) (57,060)
90,150
Cash flow from investing activities
Purchase of property, plant and equipment (200,000)
Proceeds from disposal of subsidiary (W3) 66,000
(134,000)
Cash flows from financing activities
Proceeds from issue of ordinary share capital (W4) 27,510
Borrowings advanced 25,000
Dividends paid (7,560)
44,950

Net increase in cash and cash equivalents 1,100


Cash and cash equivalents at beginning of period 400
Cash and cash equivalents at end of the period 1,500

Reconciliation of profit before tax to cash generated from operations


Profit before tax (118,320 + 15,800) 134,120
Finance charge 14,400
Depreciation charge (W1) 67,100
Amortisation charge (95,600 86,000) 9,600
Increase in inventories (107,730 97,200 + 13,000) (23,530)
Increase in trade and other receivables (56,340 53,250 + 9,500) (12,590)
Decrease in trade and other payables (303,000 342,190 + 11,700) (27,490)

Cash generated from operations 161,610


Workings

(1) Property, plant and equipment

B/fwd 948,800 Depreciation (bal fig) 67,100


Additions 200,000 Disposal of sub 52,000
C/fwd 1,029,700

1,148,800 1,148,800

(2) Income tax

Cash (bal fig) 57,060 B/fwd 31,800


C/fwd 30,700 IS 51,460
Disposal of sub 4,500

87,760 87,760

The Institute of Chartered Accountants in England and Wales 2008 Page 13 of 14


Professional Stage Financial Accounting September 2008

W3 Disposal of subsidiary


Cash proceeds on disposal 68,000
Less: cash (2,000)

Cash flow on disposal 66,000

W4 Share issue


Share capital (129,000 125,000) 4,000
Share premium (372,210 348,700 ) 23,510

27,510

There was a wide range of marks obtained on this question showing that some candidates were clearly
well prepared for the inclusion of such a question, whilst others were not. However, weaker candidates
were still able to pick up marks for items which would appear in a single company cash flow statement.

There were a notable number of arithmetic errors in the answers to this question. Figures in the final cash
flow statement were sometimes incorrect, but figures in the T-account workings were correct, with an error
being made in calculating the balancing figure. A number of candidates also seemed to be confused as to
whether items were an inflow (ie a positive figure) or an outflow (ie a negative figure and therefore shown
in brackets).

The most common error seen on this question was in relation to the group aspect of the question and
included not adjusting for the disposal in the reconciliation, both in the profit before tax figure and the
movements on working capital. Instead, candidates often added back the profit on disposal of the
subsidiary when it was not included in the profit figure in the first instance.

Other common errors included subtracting the disposal figures from the working capital calculations rather
than adding the figures, not adjusting for the disposal in T-account workings, such as depreciation, setting
out a retained earnings T-account to find dividends paid, when this figure was provided in the question,
showing the dividends paid as an inflow rather than an outflow and showing the movement in borrowings
as an outflow when it was an inflow.

Total possible marks 17


Maximum full marks 17

The Institute of Chartered Accountants in England and Wales 2008 Page 14 of 14


Financial Accounting Professional Stage December 2008

PROFESSIONAL STAGE FINANCIAL ACCOUNTING OT EXAMINERS COMMENTS

The performance of candidates in the December 2008 objective test questions section for the Professional
Stage Financial Accounting paper was good. Candidates performed well across LOs 1 and 2 but less well,
on this occasion, on LO3.

However, care should always be taken to ensure that the principles underlying any particular item are
understood rather than the answer learned from previous experience. In particular, candidates should ensure
that they read all items very carefully.

The following table summarises how well* candidates answered each syllabus content area.

Syllabus area Number of questions Well answered Poorly answered

LO1 4 4 0

LO2 6 4 1

LO3 5 2 3

Total 15 11 4

*If 50% or more of the candidates gave the correct answer, then the question was classified as well
answered.

Three out of the four poorly answered questions were on LO3 (the preparation of consolidated financial
statements) although the worst answered question was that from LO2 (the preparation of single company
financial statements). Comments on the LO2 item and one other of particular note are:

Item 1

This item tested which of four material events should be accounted for as a prior period error in accordance
with IAS 8. Although most candidates recognised that misstated opening inventory and misstated opening
receivables (due respectively to a computational error and fraud) should be corrected in this way many also
thought that the HMRC challenging items in the last periods tax return (which led to an additional liability)
should also be dealt with as a prior period adjustment. The latter, as the normal revision of an accounting
estimate, should be dealt with in the income statement for the period.

Item 2

This item required a calculation of the amount to be shown in a consolidated cash flow statement in respect
of the acquisition of a subsidiary. The subsidiary had a net overdraft and therefore this needed to be added
to the cash paid for the subsidiary. Many candidates deducted this net figure instead of adding it.

The Institute of Chartered Accountants in England and Wales Page 1 of 16


Financial Accounting Professional Stage December 2008

PROFESSIONAL STAGE FINANCIAL ACCOUNTING

MARK PLAN AND EXAMINERS COMMENTARY

The marking plan set out below was that used to mark this question. Markers were encouraged to use
discretion and to award partial marks where a point was either not explained fully or made by implication.
More marks were available than could be awarded for each requirement. This allowed credit to be given for a
variety of valid points which were made by candidates.

Question 1

Total marks 24

General comments
This is a typical question testing the preparation of full financial statements (in this case an income
statement and balance sheet) from a trial balance plus a number of adjustments. The requirement
included a specific instruction to analyse expenses by their nature, as opposed to the more usual analysis
by function. Adjustments included a write-down to inventories based on an adjusting post balance sheet
event, adjusting revenue to reflect the provisions of IAS 18, adjusting the warranty provision and providing
for an interim dividend.

Grasmere Ltd

Income statement for the year ended 30 September 2008


Revenue (W2) 5,545,700
Changes in inventories of finished goods and work in progress (W1) 6,600
Raw materials and consumables used (W1) (1,157,900)
Employee benefits expense (2,570,000)
Depreciation and amortisation expense (W3) (87,700)
Other expenses (W1) (568,000)
Profit from operations 1,168,700
Finance cost (1,500 + 10,000 (W1)) (11,500)
Profit before tax 1,157,200
Income tax expense (450,000)
Profit for the period 707,200

Balance sheet as at 30 September 2008


Assets
Non-current assets
Property, plant and equipment (W3) 2,230,800

Current assets
Inventories (W1) 31,600
Trade and other receivables (265,500 + 15,000 (15,000 + 21,500) 244,000
Cash and cash equivalents 570
276,170
Total assets 2,506,970

The Institute of Chartered Accountants in England and Wales Page 2 of 16


Financial Accounting Professional Stage December 2008


Equity and liabilities
Capital and reserves
Ordinary share capital 100,000
Revaluation reserve (W5) 616,000
Retained earnings (W4) 803,370
1,519,370
Non-current liabilities
Preference share capital (redeemable) 200,000
Deferred income (105,000 (W2) x 9/21) 45,000
245,000
Current liabilities
Trade and other payables (146,700 + 17,400) 164,100
Deferred income (105,000 (W2) x 12/21) 60,000
Taxation 450,000
Provisions (W6) 26,000
Dividends payable (W4) 40,000
Borrowings 2,500
742,600
Total equity and liabilities 2,506,970

Workings

(1) Allocation of expenses


Changes in Raw Other
inventories materials and expenses
consumables

Per Q 25,000 1,140,500 567,500
Prepayments and accruals 17,400 (15,000)
Bad debts 21,500
Closing inventories (32,000 (20 x 20)) (31,600)
Adj re redeemable pref dividend (200,000 x 5%) (10,000)
Increase in warranty provision (W6) 4,000
(6,600) 1,157,900 568,000

(2) Revenue

Per Q 5,650,700
Less After-sales support relating to future years (100,000 x 120% x 21/24) (105,000)
5,545,700

The Institute of Chartered Accountants in England and Wales Page 3 of 16


Financial Accounting Professional Stage December 2008

(3) PPE

Property valuation 2,000,000
Plant cost 676,000
acc dep b/f (357,500)
Depreciation for year property (1,056,000 44) 24,000
plant ((676,000 357,500) x 20%) 63,700
(87,700)
2,230,800

(4) Retained earnings



At 30 September 2007 132,170
Ordinary dividend (100,000 50p x 20p) (40,000)
Transfer from revaluation reserve (W5) 4,000
Profit for the period 707,200
At 30 September 2008 803,370

(5) Revaluation reserve



Valuation 2,000,000
Carrying amount at 1 October 2007 (1,500,000 120,000) (1,380,000)
620,000
Transfer to retained earnings
Depreciation charge based on revalued amount (W3) 24,000
Depreciation charge based on HC (1,000,000 50) 20,000
(4,000)
616,000

(6) Warranty provision



At 1 October 2007 22,000
Income statement charge 4,000
At 30 September 2008 (1,300,000 x 2%) 26,000

The Institute of Chartered Accountants in England and Wales Page 4 of 16


Financial Accounting Professional Stage December 2008

As in previous sittings candidates were well-prepared for this type of question and generally answered it
well. The majority of candidates produced a well laid out income statement and balance sheet in
appropriate formats, although some lost presentation marks by not adding across numbers in brackets or
transferring numbers from workings. Others lost presentation marks by failing to complete the sub-totals
and/or totals on their statements or by having incomplete or abbreviated narrative or no heading.
Candidates should remember that this type of question requires financial statements to be in a form
suitable for publication.

A significant minority of candidates either ignored or were unable to deal with the requirement to analyse
expenses by their nature in the income statement and lost presentation marks as a result. Candidates
must ensure they are familiar with both income statement formats and should be aware that they are
clearly illustrated in the open book text.

Although many workings, in particular the cost matrix, were clearly laid out some candidates workings
were disorganised, untidy and therefore hard to follow, making it difficult to establish candidates
approaches where they had not calculated the correct figure.

Most candidates were able to accurately process given figures but struggled with even some of the more
straightforward adjustments to the income statement such as the increase in the bad debt allowance and
the inventory write-down. Most dealt accurately with the income tax charge/liability, the annual depreciation
charges and the revaluations, with many candidates arriving at the correct figure for property, plant and
equipment in the balance sheet and some at the correct figure for the revaluation reserve. Most correctly
classified the redeemable preference shares as debt but not all treated the dividends as a finance cost.

Other common errors included the following:


Failing to take the (given) increase in the bad debt allowance to the income statement and instead
treating this figure as if it were the closing allowance.
Incorrectly calculating the write-down to inventory and/or calculating that there was an overall
decrease in inventory over the period instead of an increase.
Although many candidates correctly calculated the closing warranty provision, many were unable
to correctly process the movement on this provision (or failed to spot that an opening provision
was given) and a significant minority of candidates adjusted revenue by this figure.
The adjustment to revenue for the after-sales support was correctly calculated by only a few
candidates, although a good number made a reasonable attempt at this adjustment. However, the
corresponding deferred income (balance sheet) adjustment was only attempted by a few
candidates, and was generally incorrect when an attempt was made.
Failing to process the transfer between the revaluation reserve and retained earnings even though
a number of candidates correctly calculated this adjustment. The adjustment was often made to
the revaluation reserve or retained earnings, but not to both.
Incorrectly calculating the ordinary dividend although most did remember to adjust retained
earnings by the calculated figure.

Total possible marks 26.5


Maximum full marks 24

The Institute of Chartered Accountants in England and Wales Page 5 of 16


Financial Accounting Professional Stage December 2008

Question 2

Total marks 19

General comments
This question tested the preparation of a single company cash flow statement and supporting note. Missing
figures to be calculated included interest received, tax paid, dividends paid, payment of finance lease liabilities,
property, plant and equipment acquired and proceeds from the issue of share capital. A bonus issue of shares
and a revaluation of property, plant and equipment during the year also featured.

Coniston plc

Cash flow statement for the year ended 30 September 2008



Cash flows from operating activities
Cash generated from operations (Note) 991,930
Interest paid (121,000)
Income tax paid (W2) (226,000)
Net cash from operating activities 644,930
Cash flows from investing activities
Purchase of property, plant and equipment (W3) (1,151,400)
Proceeds from sales of property, plant and equipment (576,700 551,100
25,600)
Interest received (W1) 23,000
Net cash used in investing activities (577,300)
Cash flows from financing activities
Proceeds from issue of ordinary share capital (170,000 (W4) + 200,000 370,000
(W5))
Payment of finance lease liabilities (190,300 121,000) (W7) (69,300)
Dividends paid (270,000)
Net cash from financing activities 30,700
Net increase in cash and cash equivalents 98,330
Cash and cash equivalents at beginning of period 3,450
Cash and cash equivalents at end of period 101,780

Note: Reconciliation of profit before tax to cash generated from operations



Profit before tax 633,300
Investment income (24,500)
Finance costs 121,000
Depreciation charge 665,600
Loss on disposal of property, plant and equipment 25,600
Increase in inventories (1,680,220 1,188,400) (491,820)
Decrease in trade and other receivables ((556,700 6,000) (543,600 7,500)) 14,600
Increase in trade and other payables (444,100 430,950) 13,150
Increase in warranty provision (420,000 385,000) 35,000
Cash generated from operations 991,930

The Institute of Chartered Accountants in England and Wales Page 6 of 16


Financial Accounting Professional Stage December 2008

Workings

(1) Interest received


B/d 6,000 Cash () 23,000
IS 24,500 C/d 7,500
30,500 30,500

(2) Tax paid


Cash () 226,000 B/d 246,000
C/d 275,000 IS 255,000
501,000 501,000

(3) PPE


B/d 3,299,400 Disposal 576,700
Revaluation reserve (W6) 133,000 Depreciation 665,600
Finance leases 225,000
Additions () 1,151,400 C/d 3,566,500
4,808,800 4,808,800
(4) Share capital


B/d 1,000,000
Bonus issue 40,000
C/d 1,210,000 Cash () 170,000
1,210,000 1,210,000

(5) Share premium


B/d 540,000
C/d 740,000 Cash () 200,000
740,000 740,000

(6) Revaluation reserve


SCE 56,000 B/d 435,000
C/d 512,000 PPE () 133,000
568,000 568,000

(7) Finance lease liabilities


Cash () 190,300 B/d (500,000 + 67,800) 567,800
Interest 121,000
C/d (600,000 + 123,500) 723,500 PPE 225,000
913,800 913,800

The Institute of Chartered Accountants in England and Wales Page 7 of 16


Financial Accounting Professional Stage December 2008

This was the second time this topic had been set and some candidates were clearly very well prepared for it.
Others did not have a sufficient grasp of the double-entry techniques which underpin the preparation of such a
statement to score as well as might have been expected on a question of this type. However, the majority of candidates
scored high marks on the reconciliation note, and on the figures for tax paid, proceeds from the sale of
property, plant and equipment and the opening and closing figures for cash and cash equivalents.

Presentation was generally good and most candidates produced workings in the form of T accounts. However,
a significant number made the mistake of putting opening and closing balances on the wrong side of those T
accounts. There were also a minority of candidates that produced no workings for the cash flow statement.
This is a risky approach to take as if figures are calculated incorrectly it is not possible to award any partial
marks.

Candidates generally made a good attempt at the property, plant and equipment T account, with the figures
given in the question for depreciation and for the disposal both being correctly used. However, the finance
lease adjustment was only included by a minority of candidates. An adjustment for the revaluation during the
year was generally made, although the most common figure used was 77,000 being the difference between
the opening and closing figures on the revaluation reserve, ie not adjusted for the transfer made during the
year shown in the question in the statement of changes in equity.

Candidates were clearly confused by interest received and the amounts for the finance lease liabilities, and
these amounts were often included in one T account. A calculation of proceeds from the issue of shares was
attempted by almost all candidates although it was fairly common to see the bonus issue either missed or
included in the wrong T account.

Other common errors included the following:


In the reconciliation note, failing to adjust for the increase to the warranty provision, finance costs and
investment income or making the adjustment(s) in the wrong direction.
Failing to adjust the trade receivables figure for the opening and closing accrued interest.
Attempting to calculate dividends paid in the year by constructing a retained earnings T account, even
though the T account was effectively given in the question as an extract from the statement of
changes in equity, and included the dividend figure (this was very common).
In the interest received T account, putting the amounts for opening and closing interest accrued on
the wrong side of the T account or posting these to an interest paid T account.
Omission of the figure for interest paid in the cash flow statement a figure that should simply have
been taken from the income statement in the question.

Total possible marks 19.5


Maximum full marks 19

The Institute of Chartered Accountants in England and Wales Page 8 of 16


Financial Accounting Professional Stage December 2008

Question 3

Total marks 14

General comments
This question mixed two discrete topics and contained a conceptual Part (c). Part (a) required extracts
from a consolidated cash flow statement and consolidated balance sheet in respect of an associate
acquired at the start of the year. Part (b) required the calculation of income statement figures in respect of
a machine classified as held for sale during the year. The conceptual Part (c) tested an understanding of
the four measurement bases from the IASBs Framework.

Thirlmere plc
(a) Investment in associate Wastwater Ltd

Consolidated cash flow statement for the year ended 30 September 2008 (extracts)


Cash flows from investing activities
Purchase of associate, Wastwater Ltd (52,000)

Consolidated balance sheet as at 30 September 2008 (extracts)

Non-current assets
Investments in associates (W) 1,325,100

Working

Investment in associate


Cost ((1,000,000 x 1.20) + 52,000) 1,252,000
Share of post acquisition change in net assets
Share of post acquisition profits ((210,000 x 40%) 84,000
Share of additional depreciation based on fair value (((250,000 (900)
160,000) 40) x 40%)
83,100
Less Impairment losses to date (10,000)
1,325,100

This part of the question was very poorly answered, with a large number of candidates failing to recognise
that the acquired entity was an associate, not a subsidiary. Even those who recognised it as an associate
seemed unsure as to how to calculate the investment in associate figure for the consolidated balance
sheet with many using the old UK GAAP method (which, although it could give the correct answer was a
more complex calculation). Some attempted some sort of hybrid calculation falling between the latter
method and that used in the learning materials.

Even those who were able to calculate the carrying amount for the consolidated balance sheet failed to
show this amount as an extract as specified in the requirement (producing only a working) and lost marks
because of this. Very few candidates identified the need to adjust the post acquisition retained earnings of
the associate for the extra depreciation arising from the fair value adjustment.

Few candidates knew that the acquisition of an associate is shown in the consolidated cash flow
statement as the amount of cash paid to acquire the shares, not the amount of cash paid less cash and
cash equivalents acquired, as for a subsidiary although most knew that this would be presented under
investing activities.

The Institute of Chartered Accountants in England and Wales Page 9 of 16


Financial Accounting Professional Stage December 2008

Those candidates who decided that the acquired entity was a subsidiary wasted significant time by
attempting to do unnecessary consolidations/disclosure notes.

Few candidates knew that the acquisition of an associate is shown in the consolidated cash flow
statement as the amount of cash paid to acquire the shares, not the amount of cash paid less cash and
cash equivalents acquired, as for a subsidiary although most knew that this would be presented under
investing activities.

Those candidates who decided that the acquired entity was a subsidiary wasted significant time by
attempting to do unnecessary consolidations/disclosure notes.

Total possible marks 7


Maximum full marks 5

(b) Asset held for sale

Income statement charges for the year ended 30 September 2008



Depreciation (150,000 x 20% x 10/12) 25,000


Cost on 1 April 2005 150,000
Depreciation to 30 September 2007 (150,000 x 20% x 2.5) (75,000)
Depreciation for year ended 30 September 2008 (25,000)
Carrying amount at 1 August 2008 50,000
Fair value less costs to sell (45,000 1,000) (44,000)
Impairment loss on classification as held for sale 6,000

Answers to this part were better than those to Part (a) but were still disappointing.

The majority of candidates miscalculated both the total number of months the asset had been owned and
the number of months in the current year (in order to calculate the current year depreciation charge).
Although most candidates recognised that an impairment loss on classification as held for sale would be
taken to the income statement for the year relatively few also showed the depreciation expense for the
year.

In contrast to Part (a) where extracts were not produced when they were required, in this part where
extracts were not required, only the calculations (although candidates did need to identify in their answer
the two distinct charges being taken to the income statement for the year) many candidates also produced
extracts.
Total possible marks 5
Maximum full marks 4

(c) The four measurement bases

Historical cost

Assets are recorded at the amount of cash or cash equivalents paid or the fair value of the consideration
given to acquire them at the time of their acquisition.

At historical cost, the machine was recorded at its total price of 150,000.

Current cost

Assets are carried at the amount of cash or cash equivalents that would have to be paid if the same or an
equivalent asset was acquired currently.

The Institute of Chartered Accountants in England and Wales Page 10 of 16


Financial Accounting Professional Stage December 2008

If the machine had not been held for sale in the current period and was to be measured at its current cost
it would have been restated in the balance sheet to 175,000 x 1 years/5 years = 52,500
representing an aged version of the 175,000 current cost.

Realisable (settlement) value

Assets are measured at the amount of cash or cash equivalents that could currently be obtained by selling
an asset in an orderly disposal.

This is effectively the measurement basis that has been adopted for the machine held for sale at the year
end ie measured at 44,000 (or 45,000).

Present value

Assets are measured at the current estimate of the present discounted value of the future cash flows in
the normal course of business.

Under this basis, the machine, if retained, would be measured at 40,000, being the present value of
future cash flows generated within the business.
Many candidates either knew this material or extracted it from their open book text. However, some then
lost marks because they failed to explain each of the four bases, as required, with reference to the figures
in the question. Others wasted time setting out how the measurement bases relate to liabilities although
the question clearly asked the concepts to be explained by reference to the machine in the question (an
asset).

Other candidates scored poorly as instead of the four measurement bases they wrote about things such
as qualitative characteristics or concepts such as accruals/going concern a worrying indication that they
are not as familiar as they should be with the content of the IASB Framework (and their open book text).

Total possible marks 5.5


Maximum full marks 5

The Institute of Chartered Accountants in England and Wales Page 11 of 16


Financial Accounting Professional Stage December 2008

Question 4

Total marks 23

General comments
Part (a) was a consolidated income statement question, featuring two subsidiaries (one fully disposed of
within the year) and one associate. Adjustments were typical of this type of question and included a fair
value adjustment on acquisition with subsequent impact on the annual consolidated income statement,
intra-group trading and unrealised profits and impairment write-downs. The minority interest column from
the consolidated statement of changes in equity was also required. Part (b) tested the differences between
IFRS and UK GAAP in respect of the preparation of group financial statements, with the treatment of
goodwill arising on acquisition specifically excluded.

Windermere plc

(a) Consolidated income statement for the year ended 30 September 2008
000
Revenue (W2) 59,480
Cost of sales (W2) (41,490)
Gross profit 17,990
Operating expenses (W2) (9,680)
Profit from operations 8,310
Share of profit of associates ((1,150 x 30%) 10) 335
Profit before tax 8,645
Income tax expense (W2) (3,300)
Profit for the period from continuing operations 5,345
Profit for the period from discontinued operations (1,240 120 (W4)) 1,120
Profit for the period 6,465

Attributable to
Equity holders of Windermere plc () 5,381
Minority interest (W5) 1,084
6,465

Consolidated statement of changes in equity for the year ended 30 September 2008 (extracts)
Minority
interest
000
Profit for the year 1,084
Eliminated on disposal of subsidiary (W7) (1,220)
(136)
Balance at 30 September 2007 (W6) 3,352
Balance at 30 September 2008 () 3,216

The Institute of Chartered Accountants in England and Wales Page 12 of 16


Financial Accounting Professional Stage December 2008

Workings

(1) Group structure

Windermere plc

0.6
= 30%
2
3.2 2.1
= 80% = 60%
4 for 6/12 3.5 Buttermere Ltd

Rydal Ltd
Derwent Ltd

(2) Consolidation schedule

Windermere Derwent Ltd Adj (W3) Consol


plc
000 000 000 000
Revenue 38,700 21,500 (720) 59,480
Cost of sales per Q (26,400) (15,750) 720
PURP (W3) (60) (41,490)
Op expenses per Q (7,450) (2,200)
GW impairment (30) (9,680)
Tax (1,900) (1,400) (3,300)
2,090

(3) Intra-group sale and PURP

% 000
SP 120 720
Cost (100) (600)
GP 20 120
X 60

The Institute of Chartered Accountants in England and Wales Page 13 of 16


Financial Accounting Professional Stage December 2008

(4) Group loss on disposal of Rydal Ltd


000 000
Sale proceeds 7,500
Less: Share of net assets at disposal
Net assets at 30 September 2008 7,340
Less: Profit since 1 April 2008 (2,480 x 6/12) (1,240)
6,100
x 80% (4,880)
Less: Carrying amount of goodwill
Cost of investment (3,200 x 2.30) 7,360
Less: Share of net assets at acquisition ((4,000 (4,520)
+ 1,650) x 80%)
Less: Impairments to date (100)
(2,740)
(120)

(5) Minority interest in year


000
Rydal Ltd (20% x 1,240 (W4)) 248
Derwent Ltd (40% x 2,090 (W2) 836
1,084

(6) Minority interest brought forward


000
Rydal Ltd (20% x (7,340 2,480)) 972
Derwent Ltd (40% x (8,100 2,150)) 2,380
3,352

(7) Minority interest eliminated on disposal of Rydal Ltd 000

B/f (W6) 972


Current year 248
1,220
(8) Minority interest carried forward (for proof only)
000
Derwent Ltd (40% x (8,100 2,150 + 2,090 (W2)) 3,216

The majority of candidates showed that they could construct a consolidated income statement and
demonstrated that they understood the underlying principles. However, candidates were less clear on the
movement on the minority interest account as demonstrated by the minority interest column from the
consolidated statement of changes in equity although some of the more able candidates scored full marks
on this part demonstrating that they understood the link between the minority interest figure in the
consolidated income statement and that in the consolidated balance sheet.

The most worrying aspect of the workings to the consolidated income statement was the number of
candidates who included a column for the subsidiary disposed of during the year in their consolidation
schedule (Working 2). This was not necessary, as, per the learning materials, a separate calculation should
have been performed to add together the group profit on the disposal of the subsidiary and the subsidiarys
profit for the period. This should then have been presented on the face of the consolidated income statement
as the profit for the period from discontinued operations. Those candidates who included a column for the
subsidiary disposed of might have been going on to present discontinued operations line-by-line on the face
of the consolidated income statement but none of them did this. Indeed many candidates seemed to be
very confused about how to deal with the IFRS 5 disclosures in respect of discontinued operations. Even
those who did arrive at a correct figure for discontinued operations often failed to take out the minority interest
on this figure at the bottom of the income statement.

The Institute of Chartered Accountants in England and Wales Page 14 of 16


Financial Accounting Professional Stage December 2008

Another common error was to make no attempt to calculate a group profit on the disposal of the subsidiary
but to take the parent company profit as given in the question and add this to the subsidiarys profit for the
period to arrive at a figure for profit for the period from discontinued operations. However, the more able
candidates did calculate the correct figure and its minority interest.

Other common errors included the following:


Incorrectly calculating the subsidiarys net assets at disposal and/or acquisition in the calculation of
group profit on the disposal.
Writing off the impairment on the associate against the parent company instead of against the group
share of the associates profits.
Reducing the associates profits by the impairment before taking the group share of those profits.
Writing off the impairment on the subsidiary against the subsidiary itself instead of against the parent
company (or as a consolidation adjustment).
Taking the figure of 600,000 in respect of the intra-group sale as the selling price, when it was
clearly stated as being the cost.
Adjusting for the provision for unrealised profits against the parent company instead of against the
subsidiary (which was the selling company).
Including a line for dividends in the consolidated statement of changes in equity when the question
stated that there were no dividends declared or paid during the period and/or failing to include a line
in respect of the disposal of the subsidiary.
When calculating the minority interest brought forward, failing to exclude the profit for the year from
the figure for year-end retained earnings given in the question.

A worrying minority of candidates completely omitted any figures in respect of the associate and/or made no
attempt at the minority interest reconciliation.

Total possible marks 19.5


Maximum full marks 18

The Institute of Chartered Accountants in England and Wales Page 15 of 16


Financial Accounting Professional Stage December 2008

(b) Differences between IFRS and UK GAAP re preparation of group financial statements

Merger accounting is required by UK GAAP (FRS 6) where certain criteria are met. IFRS 3 requires all
business combinations to be accounted for using the purchase method.

UK GAAP gives specific guidance on fair value measurement in FRS 7. IFRS 3 provides less detailed
guidance.

Under FRS 7only separable intangible assets are required to be measured at fair value. Under IFRS 3
more intangibles can be recognised as intangible assets recognised under a business combination include
separable assets and those arising from contractual or legal rights (regardless of whether those rights are
transferable or separable).

UK GAAP (FRS 2) includes an exclusion of a subsidiary from consolidation on the grounds of severe long-
term restrictions. No such exemption exists under IAS 27 (although control may be lost as a result of the
restriction such that the entity will no longer be classified as a subsidiary).

UK GAAP (FRS 2) requires the minority interest to be presented separately from shareholders funds. IAS
27 requires it to be shown as a separate component of equity.

Under UK GAAP no consideration of the existence of potential voting rights is required in the assessment
of control. IAS 27 requires the existence of potential voting rights to be considered.

In a consolidated cash flow statement presentation under UK GAAP (FRS 1) is different to that under IAS
7, as follows.

Item FRS 1 classification IAS 7 classification


Dividends from associates Disclosed as a separate caption Included under investing cash flows
Dividends paid to minority Included under return on Included under financing cash flows
interest investments and servicing of
finance
Acquisitions and disposals of Disclosed as a separate caption Included under investing cash flows
subsidiaries/associates

Under UK GAAP (FRS 9) the group share of an associates operating profit and the group share of the
associates interest and tax are brought into the consolidated income statement separately. IAS 1
suggests a single line presentation.

UK GAAP (FRS 9) requires the parent company to recognise its share of an associates net liabilities. IAS
28 only requires this where there is a legal or constructive obligation to make good those losses.
Although the requirement asked for candidates to set out the differences between IFRS and UK GAAP in
respect of the preparation of group financial statements, the majority of candidates set out differences
which applied to the preparation of single company financial statements. As a result, many candidates
scored few marks on this part of the question which should have been an opportunity to score a good
number of marks.

Those who did address group differences generally scored highly, although a few included differences in
respect of the treatment of goodwill, which were specifically excluded from the requirement. Others listed
some very worrying incorrect differences such as the fact that associates are not consolidated in the UK
and that in the UK no fair value adjustments are made.

This is an important area of the syllabus and candidates should ensure that they know these differences
(which are clearly set out in both the learning materials and at the back of the syllabus document) and can
apply them to given scenarios.

Total possible marks 10.5


Maximum full marks 5

The Institute of Chartered Accountants in England and Wales Page 16 of 16


Professional Stage Financial Accounting September 2009

MARK PLAN AND EXAMINERS COMMENTARY

The mark plan set out below was that used to mark these questions. Markers are encouraged to use
discretion and to award partial marks where a point was either not explained fully or made by implication. More
marks are available than could be awarded for each requirement, where indicated. This allows credit to be
given for a variety of valid points, which are made by candidates.

Question 1 Total marks: 22

General comments
This is a trial balance question with the preparation of a statement of financial position and a statement of
changes in equity required. Adjustments are required for depreciation, the recognition of a provision and
related asset, development expenditure, an operating lease as well as other minor adjustments.

(a)
Adeje Ltd Statement of financial position as at 30 June 2009

ASSETS
Non-current assets
Property, plant and equipment (W2) 2,175,050
Intangible assets (120,000 40,000 25,000) 55,000

2,230,050
Current assets
Inventories (42,000 + 2,500) 44,500
Trade & other receivables (32,000 + 10,000) 42,000

86,500

Total assets 2,316,550

EQUITY AND LIABILITIES


Equity
Ordinary share capital 1,050,000
Share premium account 200,000
6% Irredeemable preference shares 60,000
Retained earnings 874,050

2,184,050

Current liabilities
Trade and other payables (58,000 + 250 (W3)) 58,250
Bank overdraft 18,250
Taxation 41,000
Provisions 15,000
132,500

Total equity and liabilities 2,316,550

The Institute of Chartered Accountants in England and Wales 2009 Page 1 of 15


Professional Stage Financial Accounting September 2009

Statement of changes in equity for the year ended 30 June 2009

Ordinary Share Irredeem. Retained Total


share premium pref earnings
capital shares
Total comprehensive (84,900) (84,900)
loss for the period (W3)
Interim ordinary (21,250) (21,250)
dividend
Issue of shares 60,000 60,000
Irredeemable dividend (1,800) (1,800)
(60,000 x 6% x 6/12)

60,000 (107,950) (47,950)


Balance b/fwd 1,050,000 200,000 982,000 2,232,000

Balance c/fwd 1,050,000 200,000 60,000 874,050 2,184,050

W1 Depreciation

Depreciation on buildings (2,500,000 700,000) x 3% = 54,000 pa

Depreciation on plant & equipment (351,000 1,000 (lease) 97,000) x 15% = 37,950 pa

W2 Property, plant and equipment


Land & Plant & Total
buildings equipment

Cost b/fwd 2,500,000 351,000
Adjustment (1,000)
350,000

Acc. deprec. b/fwd 486,000 97,000


Charge for year (W1) 54,000 37,950
Acc. deprec. b/fwd 540,000 134,950

Carrying amount 1,960,000 215,050 2,175,050

W3 Operating lease

Deposit 1,000
4 instalments of 1,000 4,000
Total lease payments 5,000

Straight line over two years = 5,000 / 2 years = 2,500

6 months charge in the period = 2,500 x 6/12 months = 1,250

Accrual: 1,250 1,000 (paid) = 250

The Institute of Chartered Accountants in England and Wales 2009 Page 2 of 15


Professional Stage Financial Accounting September 2009

W4 Total comprehensive profit/loss for the period


Trial balance profit before tax 115,000
Adjustments:
Irredeemable preference dividend 1,800
Provision (15,000)
Reimbursement 10,000
Inventory 2,500
Research & development exp (40,000 + 25,000) (65,000)
Depreciation charge (54,000 + 37,950) (91,950)
Lease charge (W2) (1,250)
(158,900)
Taxation (41,000)

Loss for the period (84,900)

Candidates performance on this question was good and fairly consistent with previous sittings although the
preparation of the statement of changes in equity rather than the income statement continues to cause
weaker candidates problems. Almost all candidates attempted this question highlighting just how
comfortable candidates are with this style of question, which continues to be fundamental to the Financial
Accounting syllabus. A good majority of candidates attempted this question first.

Almost all candidates produced a well laid out statement of financial position, however the preparation of the
statement of changes in equity was often a little haphazard. Candidates seem extremely comfortable when
they are asked to prepare the statement of financial position and income statement, but less so when the
statement of changes in equity is asked for. Candidates should be reminded that it is likely that any two of
these three statements could be, and frequently are, asked for.

Candidates generally spent time totalling each line item in their statement of financial position, although sub-
totals were often missing. Candidates should continue to be reminded that presentation marks are available
in this type of question as the requirement asked for statements that are suitable for publication. Workings
were generally clearer than in the past with most candidates producing well laid out workings for the property,
plant and equipment calculations instead of adding or subtracting figures on the face of the statement which
has been a criticism in the past.

Most candidates were able to take items from the trial balance and insert them in the correct place in the
statement of financial position. Marks were awarded where presentation differed to the marking guide but
resulted in a reasonable alternative.

A number of candidates produced a working for retained earnings carried forward, seeming not to realise that
this simply duplicated the information from one column of their statement of changes in equity which itself
acted as a working for the figure in the statement of financial position. A good number of candidates missed
the column for preference shares in the statement of changes in equity and consequently incorrectly showed
the share issue as ordinary shares.

Most candidates coped well with the goods omitted from the inventory count, the calculation of the
appropriate provision, the annual depreciation charges, the issue of irredeemable preference shares and the
tax charge for the year. Pleasingly, only a minority of candidates misclassified the irredeemable preference
shares as debt, rather than equity.

Strong candidates coped well with adjusting the profit before tax figure in the question for each adjustment
made. Weaker candidates made the adjustments to statement of financial position figures but failed to make
the corresponding adjustment to profit in order to arrive at the correct figure for total comprehensive income
in the statement of changes in equity.

The Institute of Chartered Accountants in England and Wales 2009 Page 3 of 15


Professional Stage Financial Accounting September 2009

The worse attempted adjustment was that of the lease, with only a minority of candidates arriving at the
correct adjustment to profit and the correct closing accrual. Common errors were to treat the operating lease
as a finance lease, in spite of the fact that the question specified that this was an operating lease. Some
candidates simply treated the 1,000 as an expense, others treated half of it as a prepayment, while others
set up the future payments as lease liabilities. A number of candidates failed to deal with this adjustment at
all.

Other common errors included netting off the 10,000 expected refund against the 15,000 provision and
thereby showing only a net 5,000 provision in the statement of financial position, failing to reduce the
amount capitalised for research and development expenditure by one of the two amounts that were not
allowable under IAS 38, although candidates generally completed their double entry adjustment correctly,
calculating the preference dividend based on a whole year rather than six months and reducing trade
receivables by 10,000 in respect of the returned goods.

Total possible marks 22


Maximum full marks 22

The Institute of Chartered Accountants in England and Wales 2009 Page 4 of 15


Professional Stage Financial Accounting September 2009

Question 2 Total marks: 19

General comments
This question requires the preparation of a statement of cash flows along with the note reconciling profit
before tax to cash generated from operation using the indirect method. A number of adjustments are
required, including the disposal of an item of plant with a deferred receipt, two share issues, one for cash and
one as part of an acquisition and deferred credit terms on the acquisition of new equipment.

Caleta plc

Statement cash flows for the year ended 30 June 2009



Cash flows from operating activities
Cash generated from operations (Note) 546,680
Interest paid (W1) (165,200)
Income tax paid (W2) (20,780)
Net cash from operating activities 360,700
Cash flows from investing activities
Purchase of property, plant and equipment (50,000)
Purchase of intangible assets (100,000)
Proceeds from sales of property, plant and equipment
(35,000 11,000) / 2 12,000
Net cash used in investing activities (138,000)
Cash flows from financing activities
Proceeds from issue of ordinary share capital
(80,000 (W5) + 10,000 (W6)) 90,000
Repayment of loan (600,000 350,000) (250,000)
Dividends paid (W7) (34,000)
Net cash from financing activities (194,000)
Net increase in cash and cash equivalents 28,700
Cash and cash equivalents at beginning of period 59,300
Cash and cash equivalents at end of period 88,000

Note: Reconciliation of profit before tax to cash generated from operations



Profit before tax 133,380
Finance costs 164,000
Depreciation charge (W3) 36,600
Amortisation charge (W4) 281,700
Loss on disposal of property, plant and equipment 11,000
Increase in inventories (123,100 106,000) (17,100)
Increase in trade and other receivables ((229,800 12,000) 216,500) (1,300)
Decrease in trade and other payables (61,600)
((334,800 6,300 39,000) (358,600 7,500))
Cash generated from operations 546,680

The Institute of Chartered Accountants in England and Wales 2009 Page 5 of 15


Professional Stage Financial Accounting September 2009

Workings

(1) Interest paid


Cash () 165,200 B/d 7,500
C/d 6,300 IS 164,000
171,500 171,500

(2) Tax paid


Cash () 20,780 B/d 22,000
C/d 32,000 IS 30,780
52,780 52,780

(3) PPE


B/d 366,500 Disposal 35,000
Depreciation () 36,600
Additions 89,000 C/d 383,900
455,500 455,500

(4) Intangibles


B/d 1,245,000 Amortisation () 281,700
Additions 200,000 C/d 1,163,300
1,445,000 1,445,000

(5) Share capital


B/d 550,000
Non-cash issue 50,000
C/d 680,000 Cash () 80,000
680,000 680,000

(6) Share premium


B/d 110,000
Non-cash issue 50,000
C/d 170,000 Cash () 10,000
170,000 170,000

(7) Retained earnings


Dividends in SCE () 34,000 B/d 352,700
C/d 421,300 IS 102,600
455,300 455,300

The Institute of Chartered Accountants in England and Wales 2009 Page 6 of 15


Professional Stage Financial Accounting September 2009

Candidates are clearly very comfortable with the preparation of a statement of cash flows. This question was
generally completed early on showing that candidates continue to favour this topic.

Presentation was generally good and most candidates produced workings in T-accounts. A significant
minority of candidates produced T-accounts with the brought forward and carried forward figures the wrong
way round and got in-flows and out-flows back to front on the face of the statement of cash flows.

The vast majority of candidates arrived at the correct figures for interest paid, tax paid, purchase of property,
plant and equipment and intangibles, repayment of loan and dividends paid in the statement of cash flows
itself. In the reconciliation most arrived at correct figures (or used their incorrectly calculated adjustments
correctly) for all but the changes in working capital, where there were a number of more unusual adjustments
to be made.

Where errors were made they included adjusting the closing interest accrual by the 1,700, not taking the
issue of shares for non-cash consideration into account when calculating the cash proceeds from issue of
ordinary share capital, not adjusting the movement on trade payables for the 39,000 due in respect of
property, plant and equipment and adjusting the property, plant and equipment T-account by the accumulated
depreciation on the asset sold instead of by the carrying amount.

Total possible marks 19


Maximum full marks 19

The Institute of Chartered Accountants in England and Wales 2009 Page 7 of 15


Professional Stage Financial Accounting September 2009

Question 3 Total marks: 18

General comments
This is a mixed topic question covering both revenue recognition and the treatment of an associate acquired
during the period in the consolidated statement of financial position. The question also contains a separate
element on the different users of financial statements and their information needs.

(a)

Income statement for year ended 30 June 2009



Revenue
Fixed price contract (120,000 x 50%) 60,000
Interest-free credit 23,500
Commission sales (1,300,000 x 15%) 195,000
High technology fittings
(85,000 (4,000 x 2 yrs)) 77,000
Maintenance income (4,000 x 3/12) 1,000

Cost of sales (45,000 + 35,000) x 50% 40,000

Finance income (25,000 23,500) x 6/12 months 750

Statement of financial position as at 30 June 2009



Trade and other receivables
Fixed price contract (60,000 40,000) 20,000
Interest-free credit (23,500 + 750) 24,250

Non-current liabilities
Deferred income (4,000 x (21 12)/12 months) 3,000

Current liabilities
Deferred income 4,000

(It was not necessary to split the deferred income between non-current and current liabilities to gain the
marks.)

Candidates seem to struggle with extract style questions whether they are single or mixed topic.
Candidates are not able to simply follow a process with this style of question and as a result they appear to
forget their basic accounting knowledge. This question was often left to the end suggesting that candidates
are not comfortable with this style of question.

Answers to this question were mixed with the question proving to be a good discriminator between strong
and weak candidates.

In Part (a) although the majority of candidates arrived at some correct figures, the weaker candidates failed
to properly complete their double entry, so whilst they might arrive at, say, a correct figure for revenue they
failed to give the correct figure for receivables, or deferred income. A significant number of candidates
wasted time providing extracts from the financial statements when all that was required was clearly labelled
calculations. A significant number of candidates also made, what appeared to be, correct calculations but
there were no clues as to what these figures represented, for example, revenue, cost of sales, receivables
etc and therefore lost marks.

Comments on specific parts of Part (a) are as follows:


Fixed price contract: Most candidates correctly arrived at revenue of 60,000 but then calculated cost of
sales as being 45,000, being the cost incurred to date (instead of adding the costs incurred to date and
estimated future costs and then taking 50% of the total as reflecting the stage of completion). Few
calculated a closing receivables figure and others described this as deferred income. A minority used
the costs basis to calculate cost of sales when a completion basis was specified in the question.
Interest free credit: Many arrived correctly at revenue of 23,500 but gave receivables as 25,000 (or

The Institute of Chartered Accountants in England and Wales 2009 Page 8 of 15


Professional Stage Financial Accounting September 2009

vice versa). A number of candidates calculated finance income based on one year instead of six months
and others described this figure as finance cost, as opposed to finance income. Of those who calculated
these two income figures few then gave the corresponding total receivable.
Commission sales: the revenue figure was correctly calculated by the majority of candidates.
High technology fittings: attempts at calculating a revenue figure varied with most candidates reducing
the 85,000 fee by only one years worth of maintenance charges, rather than two. Only the better
candidates arrived at the correct figure for revenue and the corresponding figure for deferred income.

Total possible marks 7


Maximum full marks 6

(b)

Extract from consolidated statement of financial position


ASSETS
Non-current assets
Investment in associate (W1) 129,100

Current assets
Dividend from Alcala Ltd (W2) 14,000

(W1) Associate
Year end carrying amount
Cost 175,000
Share of post-acq change in net assets
35% x (576,000 650,000) (25,900)
149,100
Impairment in year (20,000)
Investment in associate 129,100

(W2) Dividend
(400,000 / 0.50) x 5p x 35% = 14,000

In Part (b), which did require extracts, a significant number of candidates provided calculations only.
Candidates must learn to distinguish between requirements asking for extracts (where marks will be given
for presentation and own figure marks from workings) and those requiring calculations only.

There was a split between candidates who attempted to calculate a figure for dividend receivable from the
associate and those who ignored this totally. Many arrived at an incorrect figure, either missing the fact that
these were 50p shares, or that a 35% share was needed. Once calculated, the most common mistake was
for this figure to be used to adjust the retained earnings of the associate, with few candidates realising that
retained earnings given in the question would already have been reduced by this figure. Disappointingly, it
was very rare to see a consolidated statement of financial position extract for the dividend receivable.

Most candidates made a reasonable attempt at calculating the carrying amount for the investment in the
associate and correctly deducted the impairment of 20,000. Common errors included failing to recognise
that the associate had made a post-acquisition loss, as opposed to a post-acquisition profit, consequently
adding 25,900 to the cost of the associate rather than deducting it.

A worrying minority of candidates treated the associate as a subsidiary and attempted to calculate figures
such as goodwill. Even though the (given) 35% shareholding should in itself have indicated an associate, as
opposed to a subsidiary, the question also clearly stated that the investment should be treated as an
associate.

Total possible marks 5


Maximum full marks 5

The Institute of Chartered Accountants in England and Wales 2009 Page 9 of 15


Professional Stage Financial Accounting September 2009

(c)(i)
Investors Investors need to decide whether to buy or sell their shares. They analyse information from the
financial statements, such as the companys past dividend policy, the level of profits that the company is
making and how the company uses its resources.

Employees financial statement information is needed to assess their employers stability and profitability
and their ongoing ability to offer further remuneration and career progression. Does the company have plans
to expand in the future, is competition fierce or does the company have a niche market? Does the company
have share plans for its employees?

Lenders - financial statement information is needed to assess whether the company will have sufficient funds
available to pay the finance costs and repayments when they fall due. Working capital and cash flow will be
of particular interest.

Suppliers - financial statement information is needed to assess the companys ability to pay their debts when
they fall due. Working capital and cash flow will be of particular interest.

Customers - financial statement information is needed to assess a companys ability to continue trading,
providing continuity of supply for customers. What are the companys plans for the future, is it planning to
expand, diversify or reorganise its business?

Government agencies - financial statement information is needed to assess the allocation of resources and
therefore the activities of the company. Various different financial and non-financial information will be used
by the Government in its collection of national statistics, such as number of employees, level of revenue,
geographical locations etc.

The general public - financial statement information is needed to assess trends and recent developments in
the companys activities and future trading potential. Expansion plans for the future will be of particular
interest as they may provide additional local employment opportunities and are likely to encourage other
businesses into the local area. The financial performance of the company may provide some insight into
whether the company is likely to remain in the local area.

[Note: information presented on the first two user groups only will be marked.]

Total possible marks 5


Maximum full marks 4

(c)(ii)
The financial statements have a number of limitations as set out below:
Financial statements are prepared to a specific date, the information, when published is therefore
historic and backward looking. Although, historic information is useful in assessing how a company
has been performing it is limited in the amount of information that it can provide about a companys
future performance.
Financial statements are prepared in a standardised manner with much of the information
aggregated. While this means that it is easier to compare information between companies because it
is presented in a similar manner it also means that the content of standardised and aggregated
information may be difficult to identify.
Financial statements only contain a limited amount of narrative information about the business which
can provide valuable insight into the companys future, for example, how it is operating, what the
companys plans are for the future, the risks facing the company, such as the number of competitors
in the market and how the company is managed.

(Markers were encouraged to use their judgement and award marks where candidates had provided
alternative limitations.)

Total possible marks 5


Maximum full marks 3

The Institute of Chartered Accountants in England and Wales 2009 Page 10 of 15


Professional Stage Financial Accounting September 2009

Answers to Part (c) continued the trend of candidates struggling with the written part of questions although
there were some good answers from some of the stronger candidates. However, it was pleasing that almost
all candidates attempted this part of the question, which was an improvement on previous sittings where
weaker candidates often miss out the written elements.

Common misconceptions included identifying management/directors and/or auditors as users of the


financial statements, stating that banks looking to lend to companies will rely on historic financial
statements (as opposed to requiring cash flow forecasts), that financial statements include no narrative
information and that immaterial errors matter to users (indicating a lack of understanding of the concept of
materiality).

Some candidates wasted time discussing the qualitative characteristics of financial information, cost versus
benefit, materiality in general and the inherent limitations of an audit.

Total possible marks 22


Maximum full marks 18

The Institute of Chartered Accountants in England and Wales 2009 Page 11 of 15


Professional Stage Financial Accounting September 2009

Question 4 Total marks: 21

General comments
This is a consolidation question. A consolidated income statement is required along with an extract from the
consolidated statement of financial position showing equity. A subsidiary has been acquired during the
period and the consolidation includes an associate. Adjustments are required for differences between the
fair value and carrying amount of the subsidiary acquired and inter-company trading has taken place
between the parent and both a subsidiary and the associate companies.

(a)

Galletas plc
Consolidated income statement for the year ended 30 June 2009

Revenue (W2) 2,291,300
Cost of sales (W2) (1,238,125)

Gross profit 1,053,175


Operating expenses (W2) (263,980)

Profit from operations 789,195


Share of profits of associate (W6) 51,383

Profit before tax 840,578


Income tax expense (W2) (240,685)

Profit for the period 599,893

Attributable to:
Equity holders of Galletas plc (Bal) 517,579
Non-controlling interest (W5) 82,314
599,893

Consolidated statement of financial position (extract)


Share capital 4,000,000
Retained earnings (W7) 1,879,116

5,879,116
Non-controlling interest (W8) 2,070,600

Total equity 7,949,716

The Institute of Chartered Accountants in England and Wales 2009 Page 12 of 15


Professional Stage Financial Accounting September 2009

Workings (All figures in 000)

W1 Group Structure
Galletas
900 / 3,000 = 30%
Arico
2.1 / 3.5 =
60%
85%
1 Apr 09
(3/12 months)
Vilaflor
Masca
W2 Consolidation schedule
Galletas Vilaflor Masca Adjustments Total
3/12
Revenue 1,410,500 870,300 160,000 (149,500) 2,291,300
Cost of sales
Per question (850,000) (470,300) (54,875) 149,500 (1,238,125)
PURP (W4) (9,750)
PURP (W4) (2,700)
Operating expenses
Per question (103,200) (136,000) (23,780) (263,980)
Fair value adj (dep) (W3) (1,000)
Tax (137,100) (79,200) (24,385) (240,685)
PAT 184,800 55,960

W3 Fair value adjustment


Additional fair value 320,000

Buildings 320,000 x 50% = 160,000

Additional depreciation charge in year 160,000 / 40 years x 3/12 months = 1,000

W4 Unrealised profit
Arico Vilaflor

207,000 149,500 115%


180,000 130,000 100%
27,000 19,500 15%

Vilaflor - 19,500 x = 9,750

Arico - 27,000 x 1/3 = 9,000


Galletas share of Arico PURP - 9,000 x 30% = 2,700

W5 Non-controlling interest
Vilaflor Ltd (40% x 184,800 (W2)) = 73,920
Masca Ltd (15% x 55,960 (W2)) = 8,394

Non-controlling interest = 73,920 + 8,394 = 82,314

The Institute of Chartered Accountants in England and Wales 2009 Page 13 of 15


Professional Stage Financial Accounting September 2009

W6 Associate

Profit for the year 204,610

Galletas share x 30% 61,383


Less: impairment for year (10,000)
Share of associates profit 51,383

W7 Consolidated retained earnings



Galletas plc c/fwd 1,560,000
Less: PURP with Vilaflor (W4) (9,750)
Less: PURP with Arico (W4) (2,700)
Vilaflor Ltd (60% x (580 195)) 231,000
Masca Ltd (85% x 55,960) (W2) 47,566
Arico Ltd ((30% x (340 130)) 10(imp)) 53,000

1,879,116

W8 Non-controlling interest - SFP



Vilaflor Ltd (4,080,000 x 40%) 1,632,000
Masca Ltd
Net assets per question 2,605,000
Fair value adjustment (increase) 320,000
Less: extra depreciation on FV adj (1,000)

2,924,000
NCI 2,924,000 x 15% 438,600

2,070,600

The average mark on this question was pleasing with candidates performance much improved since the
last time a consolidated income statement was set. Almost all candidates made a good attempt at the
consolidated income statement itself although attempts at the extracts from the consolidated statement of
financial position varied, showing once again that weaker candidates struggle when required to move away
from a pure learnt technique style of requirement.

The consolidated income statement showed a well-honed exam technique, with candidates clearly
understanding the principles of consolidation, adding together the results for the subsidiaries line by line
and only including Mascas results for the three months since acquisition. Presentation was generally good
with the majority of candidates showing the two attributable to figures although very few struck a sub-total
for operating profit before adding the share of the associates profits.

Several common errors were in respect of the associate, including failing to take 30% of the PURP in
respect of sales to the associate, setting the PURP in respect of sales to the associate against the share of
the associates profits instead of against the parents profits, adjusting sales and cost of sales for sales
between the associate and the parent and setting the impairment in respect of the associate against the
parents profits instead of against the parents share of the associates profits.

Other common errors included calculating the additional depreciation on the fair value adjustment based on
a whole year instead of just three months, setting the additional depreciation against the parent instead of
against the subsidiary and classifying the additional depreciation as a cost of sale instead of as an
operating expense (as it related to the companys head office).

Errors on the consolidated statement of financial position extract were more common, with a few candidates
making little or no attempt at this. A small minority of candidates produced a consolidated statement of

The Institute of Chartered Accountants in England and Wales 2009 Page 14 of 15


Professional Stage Financial Accounting September 2009

changes in equity instead of an extract from the consolidated statement of financial position. Many
candidates produced lengthy (and largely unnecessary) net assets workings in the way they would have
practised for a question requiring a complete consolidated statement of financial position.

Errors on this part included when calculating consolidated retained earnings, producing a new working to
arrive at post acquisition retained earnings for the subsidiary acquired during the year when the figure had
already been calculated in the consolidation schedule, omitting to reduce consolidated retained earnings by
the PURPs and when calculating non-controlling interest, failing to adjust the net assets of the subsidiary
acquired during the year by the fair value adjustment and additional depreciation thereon. Even if those
adjustments had been made in a separate net assets working, many candidates failed to use the figure
from that working and took the non-controlling interest share of just the net assets at the year end.

Total possible marks 21


Maximum full marks 21

The Institute of Chartered Accountants in England and Wales 2009 Page 15 of 15


Financial Accounting Professional Stage December 2009

PROFESSIONAL STAGE FINANCIAL ACCOUNTING OT EXAMINERS COMMENTS

The performance of candidates in the December 2009 objective test questions section for the Professional
Stage Financial Accounting paper was good. Candidates performed well across all syllabus areas.

When practising OT items, care should always be taken to ensure that the principles underlying any particular
item are understood rather than the answer learned from previous experience. In particular, candidates should
ensure that they read all items very carefully.

The following table summarises how well* candidates answered each syllabus content area.

Syllabus area Number of questions Well answered Poorly answered

LO1 4 4 0

LO2 3 3 0

LO3 8 7 1

Total 15 14 1

*If 50% or more of the candidates gave the correct answer, then the question was classified as well answered.

The only poorly answered question was on LO3 (the preparation of consolidated financial statements).
Comments on this item are:

Item 1

This item tested what amount would be shown in respect of dividends paid to the non-controlling interest in a
consolidated statement of cash flows. The information given included opening and closing balances on the non-
controlling interest account, the profit attributable to the non-controlling interest for the year and details of a
new subsidiary acquired during the year. The most commonly selected incorrect answer indicated that
candidates ignored the impact of the subsidiary acquired during the year on the non-controlling interest account.

The Institute of Chartered Accountants in England and Wales 2010 Page 1 of 19


Financial Accounting Professional Stage December 2009

MARK PLAN AND EXAMINERS COMMENTARY

The marking plan set out below was that used to mark this question. Markers were encouraged to use discretion
and to award partial marks where a point was either not explained fully or made by implication. More marks
were available than could be awarded for most requirements. This allowed credit to be given for a variety of
valid points which were made by candidates.

Question 1 Total Marks: 19

General comments
This question tested the preparation of single entity financial statements (in this case an income statement
and a statement of financial position) from a trial balance plus a number of adjustments. Adjustments
included prepayments and accruals, the correction of a suspense account, an adjustment for inventories
held by third parties, two provisions and an asset held for sale.

Moreton Ltd

Income statement for the year ended 30 September 2009


Revenue (2,885,500 30,000) 2,855,500
Cost of sales (W1) (1,879,900)
Gross profit 975,600
Distribution costs (W1) (309,600)
Administrative expenses (W1) (637,400)
Profit from operations 28,600
Finance cost (200,000 x 5%) (10,000)
Profit before tax 18,600
Income tax expense (4,000)
Profit for the period 14,600

Statement of financial position as at 30 September 2009

Assets
Current assets
Inventories (W1) 176,000
Trade and other receivables (978,400 + 56,000 1,004,400
30,000)
Other receivables (W2) 45,000
Cash and cash equivalents 820
1,226,220
Non-current asset held for sale (W4) 21,000

Total assets 1,247,220

The Institute of Chartered Accountants in England and Wales 2010 Page 2 of 19


Financial Accounting Professional Stage December 2009

Equity and liabilities


Equity
Ordinary share capital 100,000
Share premium account 20,000
Retained earnings (W3) 193,500
313,500
Non-current liabilities
Borrowings 200,000
Current liabilities
Trade and other payables (578,620 + 75,000 + (10,000 656,120
7,500))
Taxation 4,000
Borrowings 13,600
Provisions (W2) 60,000
733,720
Total equity and liabilities 1,247,220

Tutorial note
Equal credit was given if candidates assumed that the 7,500 interest paid in the trial balance related to
the overdraft and therefore accrued the whole 10,000 interest on the loan.

Marks were also awarded if the impairment of 9,000 and/or the provision expense of 15,000 were shown
separately on the face of the income statement (instead of within cost of sales and cost of sales or
administrative expenses respectively) on the grounds that either of these amounts would have a significant
impact on the profit for the period.

Workings

(1) Allocation of expenses


Cost of sales Administrative Distribution
expenses costs

Per Q 1,345,600 456,700 234,600
Rent (70:30) 552,300 236,700
Opening inventories 134,000
Prepayments and accruals (56,000) 75,000
Closing inventories (156,000 + 20,000) (176,000)
Movement on provision (W2) 15,000
Movement on provision (W2) (2,000)
Amortisation/impairment charges (2,000 + 11,000
9,000) (W4)
1,879,900 637,400 309,600

(2) Provision for legal claims



At 1 October 2008 27,000
Settled at (25,000)
Released to IS 2,000

New provision at 30 September 2009 (most likely outcome) 60,000


Counter claim @ 75% (45,000)
Charge to IS 15,000

(3) Retained earnings



At 1 October 2008 178,900
Profit for the period 14,600
At 30 September 2009 193,500

The Institute of Chartered Accountants in England and Wales 2010 Page 3 of 19


Financial Accounting Professional Stage December 2009

(4) Patent held for sale



Cost 40,000
Accumulated amortisation to 30 September 2008 (8,000)
Amortisation to date of classification as held for sale ((40,000 10) x 6/12) (2,000)
Carrying amount at classification as held for sale 30,000
Fair value less costs to sell (22,000 1,000) (21,000)
Impairment 9,000

As in previous sittings, candidates were clearly very well-prepared for this type of question. Almost all
candidates produced a well-laid out income statement and statement of financial position with all narrative
and sub-totals completed. Although some candidates lost presentation marks for the statement of financial
position by not adding across numbers in brackets or failing to complete sub-totals and/or totals on their
statements or by having incomplete or abbreviated narrative, presentation for this statement was much
improved from previous sittings. Others lost presentation marks for failing to include a sub-total for profit
from operations on their income statement. As ever, candidates should remember that this type of question
requires financial statements to be in a form suitable for publication.

Although many workings, in particular the cost matrix and impairment working, were clearly laid out, a few
candidates workings were disorganised, untidy and therefore hard to follow, making it difficult to establish
candidates approaches where they had not calculated the correct figure. It is particularly difficult to follow
workings which use little or no narrative, or costs workings done on the face of the income statement.

Most candidates were able to deal with the more straightforward adjustments such as the prepayments
and accruals, adjusting the closing inventories for goods held by a customer on sale or return, the
settlement of the opening provision and the creation of a new provision at the year end and the income tax
charge/liability. However, a number of candidates failed to complete the double entry on their adjustments,
for example:
adding prepayments to expenses (a debit) and showing them as a current asset (another debit) on
the statement of financial position
correctly adjusting trade and other receivables by 30,000 for the goods held by a customer on
sale or return but failing to make the corresponding adjustment to revenue
correctly recognising that a provision for 60,000 should be set up and charging the 60,000 to the
income statement, but failing to recognise the provision itself within current liabilities
recognising an amount for accrued finance costs in the income statement, but failing to recognise
the same amount on the statement of financial position.

The majority of candidates arrived at the correct figure for the impairment on the asset classified as held
for sale although a few calculated accumulated amortisation incorrectly. However, not all candidates who
arrived at an impairment loss then charged this loss to the income statement. Others took the impairment
loss to the income statement but failed to take the amortisation charge for the year to the income
statement or classified one under cost of sales, the other as an administrative expense.

The most disappointing area in candidates answers related to the two provisions, with a number of
candidates clearly confused about the appropriate double entry. Errors included:
Not releasing the 2,000 over-provision brought forward to the income statement (with a number
of candidates instead adding it to the closing provision).
Calculating the closing provision on a weighted average basis rather than taking the most likely
outcome (as this was a single obligation).
Not recognising the contingent asset even though it was virtually certain to be recovered.
Netting the provision and the contingent asset off on the face of the statement of financial position
instead of presenting them separately.
Recognising the provision and contingent asset on the statement of financial position, but ignoring
the impact on the income statement.

The Institute of Chartered Accountants in England and Wales 2010 Page 4 of 19


Financial Accounting Professional Stage December 2009

Errors in dealing with the other adjustments included the following:


Failing to disclose the asset held for sale correctly on the statement of financial position (within
current assets after a separate sub-total for all other current assets).
Offsetting the cash in hand against the overdraft, sometimes even showing a net positive cash
balance.
Failing to make the correct (or any) adjustment for the goods held by a customer on sale or return,
with a number of candidates deducting these goods from closing inventories rather than adding
them.
Adding prepayments to expenses and deducting accruals or adjusting for both in the same
direction.
Reflecting the income tax charge for the year in the income statement but not showing the
corresponding liability.

Total possible marks 21


Maximum full marks 19

The Institute of Chartered Accountants in England and Wales 2010 Page 5 of 19


Financial Accounting Professional Stage December 2009

Question 2 Total Marks: 25

General comments
This question covered various aspects of property, plant and equipment. Part (a) required the preparation
of a property, plant and equipment table, with the movements on property, plant and equipment during
the year including additions, disposals, a self-constructed asset, an impairment and a revaluation. Part (b)
required the calculation of the closing balance on the revaluation surplus. Part (c) tested the differences
between IAS 16 and UK GAAP. Part (d) covered the information needs of users in the context of property,
plant and equipment.

Bushley plc
(a) Note showing movements on PPE for the year ended 30 September 2009

Land and Plant and Under Total


buildings machinery construction

Cost or valuation
At 1 October 2008 1,300,000 870,000 - 2,170,000
Revaluation (1,800,000 + 700,000 1,200,000 - - 1,200,000
1,300,000)
Additions (56,000 + 24,000) - 120,000 80,000 200,000
Disposals - (56,000) - (56,000)
At 30 September 2009 2,500,000 934,000 80,000 3,514,000

Depreciation
At 1 October 2008 180,000 423,000 - 603,000
Revaluation (180,000) - - (180,000)
Disposals (W3) - (33,600) - (33,600)
Impairment losses (W1) - 11,200 11,200
Charge for year (W2) 17,500 183,200 - 200,700
At 30 September 2009 17,500 583,800 - 601,300

Carrying amount
At 30 September 2009 2,482,500 350,200 80,000 2,912,700
At 1 October 2008 1,120,000 447,000 - 1,567,000

Workings

(1) Impaired machine


Cost on 1 October 2006 78,000
Depreciation to 30 September 2009 (78,000 x 20% x 3) (46,800)
Carrying amount at 30 September 2009 31,200
Recoverable amount (20,000)
Impairment 11,200

The Institute of Chartered Accountants in England and Wales 2010 Page 6 of 19


Financial Accounting Professional Stage December 2009

(2) Depreciation charges for year

On plant and equipment



On new machine (120,000 x 20% x 6/12) 12,000
On machine disposed of (56,000 x 20% x 9/12) 8,400
On machines held for the whole year ((870,000 56,000) x 20%) 162,800
183,200
On buildings

(700,000 40) 17,500

(3) Accumulated depreciation on machine disposed of



To 30 September 2008 (56,000 x 20% x 3) 33,600

The majority of candidates produced a well-laid out property, plant and equipment table showing that
they knew what this note would look like in a set of published financial statements, although a number did
not finish the note completely. The most common presentation errors were:
Not having a separate total column.
Not showing carrying amounts brought forward and carried forward (or using UK GAAP
terminology of net book value).
Not having a separate column (or line) for assets under construction.

Most candidates dealt correctly with the adjustments to cost, including the revaluation uplift, additions and
disposals. Errors were more common in respect of the adjustments to accumulated depreciation, although
most candidates arrived at the correct figure for the impairment loss on the plant and the depreciation
charge for the year on the revalued buildings.

Common errors included the following:


Not including the purchases and wages spent on the self-constructed asset within additions, and if
it was included, not separately disclosing that amount as a self-constructed asset.
Omitting to back out the opening accumulated depreciation on the land and buildings revalued
during the year. Some candidates added the net surplus of 1,380,000 to cost rather than
adjusting both cost and accumulated depreciation.
Using the incorrect fraction (ie number of months) when calculating depreciation charges on the
plant purchased and disposed of during the year.
Including the construction costs when calculating the current year depreciation charge.
Omitting to back out the accumulated depreciation on the plant sold during the year.

Total possible marks 14


Maximum full marks 13

The Institute of Chartered Accountants in England and Wales 2010 Page 7 of 19


Financial Accounting Professional Stage December 2009

(b) Balance on the revaluation surplus at 30 September 2009


Valuation on 1 October 2008 (1,800,000 + 700,000) 2,500,000
Carrying amount of land and buildings at 1 October 2008 (1,120,000)
1,380,000
Less: Transfer to retained earnings
Depreciation based on revalued amount 17,500
Depreciation based on historic cost ((300,000 (3,000)
180,000) 40)
(14,500)
1,365,500

There were very few completely correct answers to this part, although most candidates recognised that a
transfer between the revaluation surplus and retained earnings needed to be made, even if they calculated
this incorrectly. The most common errors included the following:
Calculating the surplus on initial revaluation as just the cost uplift of 1.2 million, rather than the
net uplift of 1,380,000 (sometimes in direct contravention of entries made in Part (a)).
Including only the revaluation surplus on the land and not the building (or vice versa).
Calculating depreciation on historic cost as 6,000, ie as it would have been originally calculated
(300,000 divided by the original useful life of 50 years), failing to appreciate that the buildings
remaining estimated useful life had been reassessed at 40 years.
Total possible marks 3
Maximum full marks 3

The Institute of Chartered Accountants in England and Wales 2010 Page 8 of 19


Financial Accounting Professional Stage December 2009

(c) Differences between IAS 16 and UK GAAP

When assets are revalued IAS 16 requires the use of fair values, which will take account of alternative
uses. UK GAAP (FRS 15) bases valuations on existing use values.

UK GAAP specifies a maximum period of five years between full valuations and an interim valuation every
three years. IAS 16 does not specify a maximum period and the timing of revaluations depend on changes
in market values.

UK GAAP requires impairment losses to be debited first against any revaluation surplus in respect of that
asset unless it reflects a consumption of economic benefits. IAS 16 does not include such a limitation.

Under UK GAAP when residual values are reassessed they are based on prices prevailing at the date of
acquisition. Under IAS 16 reassessment is based on current prices.

UK GAAP requires annual impairment reviews for assets which are not depreciated or are depreciated
over more than 50 years. IAS 16 does not include such a requirement.

There were some good answers to this part, showing that many candidates had spent time learning the
differences between IFRS and UK GAAP. Most answers included the difference in relation to the basis of
revaluations, although a minority of candidates got this the wrong way round. Candidates who included a
discussion about residual values generally missed the point about the differences in prices on which
reassessed residual vales are based. Most candidates knew that there was a difference in treatment in
relation to impairments but often got confused with whether this was dependent on the impairment being
as a result of a consumption of economic benefits or not. Again, some candidates got this difference the
wrong way round.
Total possible marks 6
Maximum full marks 4

The Institute of Chartered Accountants in England and Wales 2010 Page 9 of 19


Financial Accounting Professional Stage December 2009

(d) How information re PPE meets the needs of users

Financial position

The financial position of an entity is affected by the economic resources it controls, its financial structure,
its liquidity and solvency and its capacity to adapt to changes in the environment in which it operates.

Information about the total carrying amount of property, plant and equipment (PPE) as given on the face of
an entitys statement of financial position gives the user an indication of the resources the entity has at its
disposal in terms of tangible assets held for long-term use in the business. Revaluation figures are more
relevant than cost.

That figure will be broken down in the note to the financial statements as produced in (a) above. This
indicates the type of PPE held by the entity which may add further to an understanding of resource. This
note also shows the changes in financial position in the year

For example, land and buildings might be held for its investment potential, as well as being used for
office/factory space. Plant will be used to generate future revenues. Equipment could be used for the
generation of future revenues or for the entitys own use, perhaps for administrative purposes.

The fact that the amount of leased assets forming part of the total PPE figure is disclosed, shows that
these assets have a future cost in terms of lease payments affecting the liquidity and solvency of the
entity.

The capital commitments note showing the future purchases of PPE to which the entity is committed,
indicates a requirement for future finance.

The accounting policy note shows the valuation model used and depreciation methods, which allow
comparison to other entities.

Financial performance

Information about financial performance, in particular profitability, is needed in order to assess potential
changes in the economic resources that it is likely to control in the future.

Disclosure of the annual depreciation charge shows the cost of using the assets..

Disclosure of gains/losses on disposal could indicate problems with the depreciation method or where
value is greater than carrying amount.

Impairment losses may indicate underlying issues, such as underprovision of depreciation, or a downturn
in a particular market sector (which might affect future performance).

Changes in financial position

Changes in financial position are shown in a statement of cash flows. This allows users to assess the
ability of the entity to generate cash and its need to use what is generated.

Users will be able to see, via the statement of cash flows, PPE purchased during the year and cash
inflows from PPE disposed of. If little PPE is purchased and much disposed of the user may be concerned
about the future of the entity.

The Institute of Chartered Accountants in England and Wales 2010 Page 10 of 19


Financial Accounting Professional Stage December 2009

Answers to Part (d) were the most disappointing and sometimes non-existent. A number of candidates
appeared to have rote learnt various information about the Framework (and in particular the qualitative
characteristics) and wasted time simply producing a page of irrelevant comments, which gained no marks.
Candidates need to be reminded to answer the question set, not the question they wished had been set.
A number of candidates focused on what information users might need in respect of property, plant and
equipment as proposed to what information is actually provided.

Those candidates who demonstrated knowledge of the various disclosures in relation to property, plant
and equipment and where they appear in a set of published financial statements, and gave some thought
as to how such information might assist users, scored well. As can be seen from the mark plan there were
a significant number of marks available, although those for just copying out of the open book text were
limited.

Total possible marks 14


Maximum full marks 5

The Institute of Chartered Accountants in England and Wales 2010 Page 11 of 19


Financial Accounting Professional Stage December 2009

Question 3 Total Marks: 15

General comments
This was a question mixing three topics. Part (a) required the calculation of the profit from discontinued
operations in respect of a subsidiary disposed of during the year. Part (b) required extracts from single
entity financial statements in respect of debt and equity issued during the year and the correction of a prior
period error.

Bredon Ltd
(a) Profit from discontinued operations for the year ended 30 September 2009


Profit on disposal:
Sale proceeds 700,000
Less: Share of net assets at date of disposal (80% x (100,000 + (666,400)
717,000 + 16,000))
Less: Carrying amount of goodwill at date of disposal
Arising on acquisition 22,800
Impairments to date (5,000)
(17,800)
15,800
Profit for the period (32,000 x 6/12) 16,000
Profit on discontinued operations 31,800

This part of the question was generally well-answered, with many candidates arriving at the correct
answer. Where mistakes were made they included the following:
Not including share capital in the net assets at disposal.
Not including the correct proportion of the profit for the period in the net assets at disposal (or not
including it at all).
Failing to also add the above figure to the profit on sale of the shares to arrive at the final profit on
discontinued operations.

A minority of candidates seemed to have little idea of how to calculate this figure, producing apparently
random calculations involving taking 80% of various figures, and scored poorly. A significant number of
candidates wasted time drawing a group structure diagram when the percentage holding was clearly given
in the question.

Total possible marks 3


Maximum full marks 3

The Institute of Chartered Accountants in England and Wales 2010 Page 12 of 19


Financial Accounting Professional Stage December 2009

(b) (i) Statement of changes in equity for the year ended 30 September 2009

Ordinary Preference share Share Retained Total


share capital premium earnings
capital (irredeemable)

At 1 October 2008 500,000 - - 2,560,000 3,060,000

Correction of prior period error - - - (50,000) (50,000)


(45,000 + 5,000)
Restated balance 500,000 - - 2,510,000 3,010,000
Issue of ordinary shares (200,000 200,000 100,000 - 300,000
x 0.50)
Issue of irredeemable preference 50,000 5,000 55,000
shares (50,000 x 0.10)
Dividends (700,000 x 10p) - - - (70,500) (70,500)
(50,000 x 4% x 3/12)
Total comprehensive income for - - - 563,500 563,500
the year (W)

At 30 September 2009 700,000 50,000 105,000 3,003,000 3,858,000

Working

TCI per Q 560,000
Finance cost (100,000 x 3% x 6/12) (1,500)
Amortisation added back 5,000
563,500

The Institute of Chartered Accountants in England and Wales 2010 Page 13 of 19


Financial Accounting Professional Stage December 2009

(b) (ii) Extracts from the financial statements for the year ended 30 September 2009

Income statement for the year ended 30 September 2009



Finance cost 1,500

Statement of financial position as at 30 September 2009



Equity and liabilities
Equity
Ordinary share capital 700,000
Preference share capital (irredeemable) 50,000
Share premium account 105,000
Retained earnings 3,003,000

Non-current liabilities
Preference share capital (redeemable) 100,000

Current liabilities
Dividends payable 70,000

Statement of cash flows for the year ended 30 September 2009



Cash flows from operating activities
Interest paid (1,500)

Cash flows from financing activities


Proceeds from issue of share capital 355,000
Proceeds from issue of long-term borrowings 100,000
Dividends paid (500)

The Institute of Chartered Accountants in England and Wales 2010 Page 14 of 19


Financial Accounting Professional Stage December 2009

In Part (i) most candidates produced some kind of table for their statement of changes in equity, although it
was rare to see a complete table with all four columns, plus a total column and all amounts carried forward.
However, it was surprising how many combined figures for ordinary and irredeemable preference shares in
a single column, although most arrived at the correct figures for the shares issued during the year, including
the amounts to be posted to the share premium account. Pleasingly, few candidates included the
redeemable preference shares in equity.

Most candidates remembered to label the profit for the period in the table as total comprehensive income
but fewer adjusted the original figure of 560,000 given in the question for the unadjusted items of the
finance cost on the redeemable preference shares and the backing out of the amortisation for the year on
the incorrectly capitalised intangible asset.

Other common errors included the following:


Using the incorrect fraction (ie number of months) when calculating the dividends on the preference
shares.
Omitting to deal with the prior period error, or deducting 45,000 instead of 50,000.
Failing to show a sub-total for the restated balance after the prior period adjustment.

In Part (ii) there were some well laid out extracts, with a number of candidates using the correct sub-
headings in both the statement of financial position (ie equity, current and non-current liabilities) and the
statement of cash flows (ie distinguishing between operating and financing cash flows). In fact the majority
of those who attempted this part scored well, as a good proportion of the marks were available for simply
taking own figures from Part (i) into well-presented extracts. These were therefore relatively easy marks but
were missed by a number of candidates.

Common errors included the following:


Failing to distinguish between the dividends payable (in the statement of financial position) and the
dividends paid (in the statement of cash flows).
Using the incorrect bracket convention in the statement of cash flows.

A minority of candidates wasted significant time by trying to incorporate the impact of the disposal in
transaction (1) into this part of the question when the requirement referred only to the transactions set out in
(2) and (3).

Total possible marks 14


Maximum full marks 12

The Institute of Chartered Accountants in England and Wales 2010 Page 15 of 19


Financial Accounting Professional Stage December 2009

Question 4 Total Marks: 21

General comments
This was a typical consolidated statement of financial position question, featuring one subsidiary and one
associate (acquired during the year). Adjustments were typical of this type of question and included a fair
value adjustment on acquisition, intra-group balances and transactions and impairment write-downs.

Stow plc

Consolidated statement of financial position as at 30 September 2009


Assets
Non-current assets
Property, plant and equipment (4,175,500 + 2,678,500 7,854,000
+ 1,000,000)
Intangibles (W3) 370,000
Investments in associates (W7) 909,240
9,133,240
Current assets
Inventories (1,237,000 + 1,050,000 90,000 (W6) 2,172,000
(62,500 x 40%) (W6))
Trade and other receivables (976,500 + 750,000 1,186,500
540,000) (W6))
Cash and cash equivalents (9,500 + 1,500) 11,000
3,369,500
Total assets 12,502,740

Equity and liabilities


Equity attributable to owners of Stow plc
Ordinary share capital 3,000,000
Share premium account 1,000,000
Retained earnings (W5) 6,296,500
10,296,500
Non-controlling interest (W4) 922,440
Total equity 11,218,940
Current liabilities
Trade and other payables (766,000 + 637,800 863,800
540,000) (W6))
Taxation (280,000 + 140,000) 420,000
1,283,800
Total equity and liabilities 12,502,740

The Institute of Chartered Accountants in England and Wales 2010 Page 16 of 19


Financial Accounting Professional Stage December 2009

Workings

(1) Group structure

400
Stow plc = 40%
1,000

1,600
= 80%
2,000

Bourton Ltd
Naunton Ltd

(2) Net assets Bourton Ltd

Year end Acquisition Post acq



Share capital 2,000,000 2,000,000 -
Share premium 500,000 500,000 -
Retained earnings
Per Q 1,202,200 1,575,000
PURP (W6) (90,000) -
FV adj land 1,000,000 1,000,000 -
4,612,200 5,075,000 (462,800)

(3) Goodwill Bourton Ltd


Consideration transferred 4,500,000
Non-controlling interest at acquisition (5,075,000 (W2) x 20%) 1,015,000
Net assets at acquisition (W2) (5,075,000)
440,000
Impairments to date (50,000 + 20,000) (70,000)
370,000

The Institute of Chartered Accountants in England and Wales 2010 Page 17 of 19


Financial Accounting Professional Stage December 2009

(4) Non-controlling interest Bourton Ltd



Share of net assets (4,612,200 (W2) x 20%) 922,440

(5) Retained earnings

Stow plc 6,602,500


Bourton Ltd ((462,800) (W2) x 80%) (370,240)
Naunton Ltd ((1,298,100 875,000 62,500) (W6)) x 40%)) 144,240
Less Impairments to date (70,000 (W3) + 10,000 (W7)) (80,000)
6,296,500

(6) PURP

Bourton Naunton
Ltd Ltd
%

SP 150 540,000 375,000


Cost (100) (360,000) (250,000)
GP 50 180,000 125,000
X 90,000 62,500

(7) Investments in associates Naunton Ltd



Cost 750,000
Add: Share of post acquisition increase in net assets ((1,298,100 875,000)) x 169,240
40%))
Less: Impairment to date (10,000)
909,240

The Institute of Chartered Accountants in England and Wales 2010 Page 18 of 19


Financial Accounting Professional Stage December 2009

Candidates were clearly very well prepared for this question and generally scored highly. Almost all
candidates demonstrated a sound technique, following that set out in the learning materials. The most
common errors were in relation to the associate, in either the retained earnings working and/or the investment
in associate working itself or in relation to the calculation of the provisions for unrealised profit.

Common errors included the following:

In the net assets table for the subsidiary, only including the fair value adjustment in the year-end
column and/or deducting the fair value adjustment instead of adding it.
Including the fair value adjustment in the net assets table but failing to uplift the value of property,
plant and equipment in the consolidated statement of financial position by the same amount.
Failing to include the share premium account in the net assets table for the subsidiary.
Failing to adjust both receivables and payables (or in some cases, either) for the invoice value of the
sale of goods from the subsidiary to the parent, with a number of candidates making the adjustment at
cost.
Arriving at provisions for unrealised profits in respect of the subsidiary and the associate on different
bases when the same cost structure was specified in the question.
Failing to calculate any provision for unrealised profit in respect of the goods sold by the associate to
the parent.
Calculating unrealised profit based on the full invoice value, as opposed to only half of that value,
when the question clearly stated that only half of the goods remained in year-end inventory.
Taking the cost figures given in the question for the intra-group sales as being the selling price of the
goods and hence calculating incorrect provisions for unrealised profit.
Pleasingly, many candidates correctly adjusted for the group share of the provision for unrealised
profit arising on goods sold by the associate to the parent against retained earnings and inventory, but
many also made an adjustment against the carrying amount of the associate. Others calculated an
initial post-acquisition profit figure for the associate less a 40% share of the provision for unrealised
profit but then adjusted that total by 40%, consequently scaling down the provision for unrealised profit
twice. This error occurred most often when candidates produced a(n) (unnecessary) net assets table
for the associate.
Taking only six-twelfths of the movement on the associates profit to retained earnings and the
investment in associate working, failing to recognise that although the acquisition of the associate did
indeed occur half way through the year the figure given for retained earnings was at the date of
acquisition, such that there was no need to time-apportion any of the figures given.
Not adjusting for the accumulated impairments in the group retained earnings working, instead
adjusting only for the impairments which had arisen during the current year.
Having correctly arrived at a post-acquisition loss for the subsidiary in a net assets table, turning this
into a post-acquisition profit when taking 80% of this figure to group retained earnings.

A number of candidates failed to provide workings for assets and liabilities on the face of the consolidated
statement of financial position. Where these figures were incorrect no partial marks could then be awarded.
Candidates must show their workings in all cases so that partial credit can be given.

Presentation of the consolidated statement of financial position was generally good, although very few
candidates gained the presentation mark which was available for clearly disclosing the non-controlling interest
as a separate component of equity.

Total possible marks 21


Maximum full marks 21

The Institute of Chartered Accountants in England and Wales 2010 Page 19 of 19


Financial Accounting Professional Stage March 2010

PROFESSIONAL STAGE FINANCIAL ACCOUNTING OT EXAMINERS COMMENTS

The performance of candidates in the March 2010 objective test questions section for the Professional Stage
Financial Accounting paper was good. Candidates performed well across all syllabus areas.

When practising OT items, care should always be taken to ensure that the principles underlying any
particular item are understood rather than rote learning the answer. In particular, candidates should ensure
that they read all items very carefully.

The following table summarises how well* candidates answered each syllabus content area.

Syllabus area Number of questions Well answered Poorly answered

LO1 2 1 1

LO2 7 6 1

LO3 6 6 0

Total 15 13 2

*If 50% or more of the candidates gave the correct answer, then the question was classified as well
answered.

Brief comments on the two poorly answered questions, which covered LO1 (accounting and reporting
concepts) and LO2 (preparation of single company financial statements), are below (this paper was marked
under the new electronic marking system and no further information regarding responses was available):

Item 1

This item asked which roles are undertaken by the International Accounting Standards Committee
Foundation (IASCF). Candidates clearly do not understand the structure that surrounds and supports the
International Accounting Standards Board.

Item 2

This item, required candidates to identify which adjustments should be recognised as a prior period error.
Four short scenarios were provided which included a settled legal claim, a computational error, a fraud and a
revised tax liability.

The Institute of Chartered Accountants in England and Wales 2010 Page 1 of 13


Financial Accounting Professional Stage March 2010

MARK PLAN AND EXAMINERS COMMENTARY

The mark plan set out below was used to mark these questions. Markers are encouraged to use discretion
and to award partial marks where a point was either not explained fully or made by implication. More marks
are available than could be awarded for each requirement, where indicated. This allows credit to be given for
a variety of valid points, which are made by candidates.

Question 1

Overall marks for this question can be analysed as follows: Total: 18

General comments
This question is a typical question testing the preparation of an income statement and statement of financial
position from a trial balance. A number of adjustments were required, including the reversal of a provision,
an inventory valuation issue, an adjustment for the over provision of tax and deferred revenue.

(a)
Karonga plc Statement of financial position as at 31 December 2009

ASSETS
Non-current assets
Property, plant and equipment (W5) 943,435

Current assets
Inventories (W3) 1,161,000
Trade receivables (1,075,000 60,750 (W4)) 1,014,250
Cash and cash equivalents 189,500

2,364,750

Total assets 3,308,185

EQUITY AND LIABILITIES


Equity
Ordinary share capital 1,325,000
Retained earnings (28,090 + 227,895) 255,985

1,580,985

Non-current liabilities
Bank loan 1,025,300

Current liabilities
Trade and other payables (583,700 + 12,500(W1)) 596,200
Taxation (W5) 105,700
701,900

Total equity and liabilities 3,308,185

The Institute of Chartered Accountants in England and Wales 2010 Page 2 of 13


Financial Accounting Professional Stage March 2010

Karonga plc Income Statement for year ended 31 December 2009



Revenue (W1) 6,196,400
Cost of sales (W2) (3,506,501)

Gross profit 2,689,899


Administrative expenses (W2) (2,315,434)

Operating profit 374,465


Finance costs (49,170)
Profit before tax 325,295
Income tax expense (105,700 8,300) (97,400)

Net profit for the period 227,895

Note: Marks will be awarded if items are included in a different line item in the income statement
provided that the heading used is appropriate.

W1 Revenue adjustment


Trial balance revenue 6,208,900
Fitness machine deposits (250 x 50) (12,500)

6,196,400

W2 Expenses

Admin Cost of
expenses sales

Trial balance 2,324,000 3,553,100
Opening inventory 1,093,800
Less: closing inventory (W3) (1,161,000)
Bad debt reversal (W4) (1,650)
Depreciation charge buildings (12,710 (40% / 60%) 5,084 7,626
Depreciation charge plant & equipment 12,975
Provision reversal (12,000)
2,315,434 3,506,501

W3 Inventory adjustment


Closing inventory 1,163,500
Net realisable value write down (20 - 15) x 500 items (2,500)

1,161,000

W4 Bad debt


Opening allowance 62,400
Movement in year (balancing figure) (1,650)

Closing allowance (53,750 + 7,000) 60,750

The Institute of Chartered Accountants in England and Wales 2010 Page 3 of 13


Financial Accounting Professional Stage March 2010

W5 Property, plant and equipment

Cost Acc dep



Trial balance L&B 985,500 88,970
Trial balance P&E 103,800 31,210

Depreciation charge for year (103,800 / 8yrs) 12,975


Depreciation charge for year ((985,500 12,710
350,000) / 50yrs)

At 31 December 2009 1,089,300 145,865 943,435

As in previous sittings, candidates were clearly very well-prepared for this type of question. Almost all
candidates produced a well-laid out income statement and statement of financial position with all narrative
and sub-totals completed. Some candidates lost presentation marks for the statement of financial position
by not adding across numbers in brackets, failing to complete sub-totals or by having incomplete or
abbreviated narrative. On the income statement the most common presentational failing was to not
include a sub-total for profit from operations. However, overall presentation is improving with each sitting.
As ever, candidates should remember that this type of question requires financial statements to be in a
form suitable for publication.

Workings generally were set out clearly, with the standard cost matrix generally being produced.
Candidates must remember that if they do not provide clear workings for calculations and their final
answer is incorrect they risk gaining no marks for a working that may be worth 2 or 3 marks. Clear
workings, even if only bracketed will score partial marks for incorrect answers.

Most candidates were able to deal with the more straightforward adjustments such as the depreciation
charges, closing inventory and adjusting revenue for the payments made in advance, although the
corresponding entry in current liabilities was not always included.

Common errors included the treatment of the tax figures which seemed to cause some confusion as to
how to deal with the over provision from the previous year. Candidates commonly put the same figure in
the income statement and statement of financial position, although this was split between whether it was
the income tax charge or the liability.

Candidates often used the correct brought forward and carried forward figures for the specific bad debt
allowance but missed the additional allowance that needed making of 7,000. Other candidates correctly
calculated the carried forward figure and hence calculated that an adjustment of 1,650 was needed but
then either didnt recognise this in the income statement or added it to expenses rather than deducting it.
Only a few candidates carried the double entry through completely by deducting the full closing allowance
from trade receivables, 7,000 was a more common deduction.

The treatment of the legal provision also caused a few problems. Very few candidates realised that the
provision needed reversing. A mix of treatments were seen with candidates either including the provision
in the statement of financial position or providing for it in the current year even though it was a brought
forward balance.

Total possible marks 18


Maximum full marks 18

The Institute of Chartered Accountants in England and Wales 2010 Page 4 of 13


Financial Accounting Professional Stage March 2010

Question 2

Overall marks for this question can be analysed as follows: Total: 19

General comments
This question tested the preparation of a consolidated statement of cash flows and supporting note. A
subsidiary was disposed of during the year. Missing figures to be calculated included dividends paid (to the
group and to the non-controlling interest), interest paid, tax paid, depreciation and amortisation charge for the
year and proceeds from the issue of share capital following a bonus issue during the year.

Chitipa plc

Consolidated statement of cash flows for the year ended 31 December 2009

Cash flows from operating activities
Cash generated from operations (Note) 331,900
Interest paid (W1) (73,000)
Income tax paid (W2) (76,050)
Net cash from operating activities 182,850
Cash flows from investing activities
Purchase of property, plant and equipment (360,000)
Disposal of Thyolo Ltd net of cash disposed of (200,000 192,100
7,900)
Net cash from investing activities (167,900)
Cash flows from financing activities
Repayment of borrowings (736,300 561,700) (174,600)
Proceeds from share issue (W4 & W5) 175,000
Dividends paid (W6) (27,500)
Dividends paid to non-controlling interest (W7) (11,850)
Net cash used in financing activities (38,950)
Net increase in cash and cash equivalents (24,000)
Cash and cash equivalents at beginning of period 172,500
Cash and cash equivalents at end of period 148,500

Note: Reconciliation of profit before tax to cash generated from operations



Profit before tax (222,000 + 12,600) 234,600
Finance cost 71,000
Depreciation charge (W3) 22,700
Impairment loss on goodwill (373,700 364,200) 9,500
Increase in inventories (401,300 393,800) (7,500)
Increase in trade and other receivables (496,300 475,200 + 25,400) (46,500)
Increase in trade and other payables ((21,700 5,000) (11,700 7,000) + 48,100
36,100)
Cash generated from operations 331,900

The Institute of Chartered Accountants in England and Wales 2010 Page 5 of 13


Financial Accounting Professional Stage March 2010

Workings

(1) Interest paid


Cash () 73,000 B/d 7,000
C/d 5,000 CIS 71,000
78,000 78,000

(2) Income tax paid


Cash () 76,050 B/d 36,300
C/d 33,900 CIS (69,900 + 3,750) 73,650
109,950 109,950

(3) PPE


B/d 695,000 Disposal of sub 308,900
Additions 360,000 Deprecation charge () 22,700
C/d 723,400
1,055,000 1,055,000

(4) Share capital


B/d 400,000
Bonus issue 100,000
C/d 550,000 Cash received () 50,000
550,000 550,000

(5) Share premium


B/d 140,000
Bonus issue 50,000 Cash received () 125,000
C/d 215,000
265,000 265,000

(6) Retained earnings


Dividends in SCE () 27,500 B/d 295,100
Bonus issue 50,000
C/d 303,140 CIS 85,540
380,640 380,640

(7) Non-controlling interest


Cash () 11,850 B/d 490,800
Disposal (306,100 x 20%) 61,220
C/d 448,260 CIS 30,530
521,330 521,330

The Institute of Chartered Accountants in England and Wales 2010 Page 6 of 13


Financial Accounting Professional Stage March 2010

Candidates generally performed well on this question, adopting a good exam technique that allowed them to
gain a good pass in this question but miss out some of the more tricky areas. Presentation of the statement
of cash flows was good, although candidates often missed sub-totalling each section and the date for the
period for which the cash flow was prepared was missed by a significant minority of candidates.

Candidates generally calculated the repayment of borrowings correctly and the purchase of property, plant
and equipment, although a minority of candidates showed the latter as an inflow of cash rather than outflow.
Interest paid was also generally shown correctly. A common mistake was in relation to the income tax
expense where a significant number of candidates missed the tax expense in respect of the discontinued
operation.

The calculation of the proceeds from the share issue were mixed with candidates gaining the marks for the
brought forward and carried forward figures but often getting the entries for the bonus issue back to front. The
calculation for the cash flows from the disposal of the subsidiary was one of the most disappointing areas
with candidates showing all kinds of long and complicated net assets workings, when a, simple netting off of
two figures was required.

Candidates seemed happy with the T-account for dividends paid, although the treatment of the bonus issue
was not always correctly dealt with, sometimes it was shown on the wrong side of the T-account or missed
entirely. However, the calculation of the dividend paid to the non-controlling interest was disappointing with a
good majority of candidates simply electing to ignore the calculation entirely. Candidates who did show a
working for this generally were unable to calculate the disposal value or simply missed it out.

A good attempt at producing the reconciliation of profit before tax to cash generated from operations was
made by almost all candidates. Common errors however included not including the profit before tax for the
discontinued operation, adding back the loss on disposal even though it was not included in the parents
profit before tax figure, ignoring the impact of the goodwill impairment and deducting the individual assets
and liabilities at disposal in the movements calculations, rather than adding them, or ignoring them
completely.

Total possible marks 19


Maximum full marks 19

The Institute of Chartered Accountants in England and Wales 2010 Page 7 of 13


Financial Accounting Professional Stage March 2010

Question 3
Overall marks for this question can be analysed as follows: Total: 22

General comments
This question required the preparation of a consolidated statement of financial position. The group has an
associate, with the acquisition of a subsidiary during the year. A fair value adjustment in relation to a piece of
equipment, with depreciation adjustment, was required. Inter-company trading had taken place during the
year between the parent and associate company and a suspense account needed eliminating, which was
created on the acquisition of property, plant and equipment on deferred payment terms.

Rumphi plc

(a) Consolidated statement of financial position as at 31 December 2009


000 000
Assets
Non-current assets
Property, plant and equipment (W8) 1,488,350
Intangibles 36,000
Goodwill 143,723
Investment in associate (W7) 108,585
1,776,658
Current assets
Inventories 52,960
Trade and other receivables (120,840 + 945,600) 1,066,440
Cash and cash equivalents (72,600 + 189,500) 262,100
1,381,500
Total assets 3,158,158

Equity and liabilities


Equity attributable to Rumphi plc shareholders
Ordinary share capital 930,000
Retained earnings (W5) 802,840
Attributable to the equity holders of Rumphi plc 1,732,840
Non-controlling interest (W4) 348,948
2,081,788

Non-current liabilities
Deferred payment (W8) 40,000

Current liabilities
Trade and other payables (236,380 + 470,330) 706,710
Taxation (172,000 + 157,660) 329,660
1,036,370
Total equity and liabilities 3,158,158

Workings
(1) Group structure

Rumphi

245,000 / 350,000 = 70%


Luwa Ltd 14,175 / 56,700 = 25%
Dedza Ltd

The Institute of Chartered Accountants in England and Wales 2010 Page 8 of 13


Financial Accounting Professional Stage March 2010

(2) Net assets Luwa Ltd


31 Dec 2009 Acquisition Post acq

Share capital 350,000 350,000
Share premium account 125,000 125,000
Retained earnings 748,260 600,710 147,550
Goodwill on business (71,600) (71,600)
PPE FV uplift 12,000 12,000
FV depreciation adjustment (12,000 / 8yrs x 4/12) (500) (500)
1,163,160 1,016,110 147,050

(3) Goodwill Luwa Ltd



Consideration transferred 900,000
Net assets at acquisition (W2) (1,016,110)
Non-controlling interest at acquisition (1,016,110 (W2) x 30%) 304,833
188,723
Less: Impairment (45,000)
143,723

(4) Non-controlling interest Luwa Ltd

Share of net assets (1,163,160 (W2) x 30%) 348,948

(5) Retained earnings



Rumphi plc 751,320
Less:PURP (6,800 x 25%) (1,700)
Luwa Ltd (147,050 (W2) x 70%) 102,935
Less: Impairment (45,000)
Dedza Ltd ((145,695 92,555) x 25%)) 13,285
Less: Impairments to date (10,000)
Machine depreciation adjustment (W8) (8,000)
802,840

(6) PURP
Dedza Ltd
%
Sale price 100 34,000
Cost (60) (20,400)
Gross profit 40 13,600
13,600 x = 6,800

(7) Investment in associate Dedza Ltd



Original cost 107,000
Add: Share of post acquisition increase in retained earnings 13,285
Less: Impairment to date (10,000)
Less: Share of PURP (1,700)
108,585

(8) Property, plant and equipment



Rumphi plc 800,300
Luwa Ltd 644,550
Fair value adjustment 12,000
FV depreciation adjustment (W2) (500)
New machine (80,000 40,000) 40,000
Depreciation adj on new machine (40,000 / 5 yrs) (8,000)
1,488,350

The Institute of Chartered Accountants in England and Wales 2010 Page 9 of 13


Financial Accounting Professional Stage March 2010

The majority of candidate answers to this question were very good. Most notably candidates coped far
better with the provision for unrealised profit on sales made by the parent to the associate than they have
at previous sittings. Where errors were made in respect of the provision for unrealised profit it was by
deducting the full amount of the unrealised earnings from retained earnings and investment in associate
rather than the parents share. A significant minority of candidates made the adjustment to consolidated
inventory instead of to the investment in associate.

Presentation was very good, although candidates still seem to not complete statements in some way,
most typically by not showing the sub total before the non-controlling interest line and therefore they
inevitably lose marks.

Workings were generally well laid out, although the property, plant and equipment workings were often
squashed on the face of the statement of financial position which made it quite difficult to read. This was
compounded by the fact that these scripts were scanned for electronic marking and therefore squashed
workings became even harder to read. Candidates should be made aware of this, to try and avoid such an
approach in the future.

The majority of candidates made a good attempt at the net assets working. However, a number of
common errors were made in this area including not deducting the goodwill recognised by the subsidiary,
instead candidates included this as part of consolidated intangible assets, and forgetting that the fair value
uplift on the equipment meant that additional depreciation needed to be calculated. For candidates that
did appreciate that additional depreciation should be recognised they often missed that it was only four
months worth, rather than a full year. Another common error was not including the subsidiarys share
premium in the net assets working but instead showing it on the face of the consolidated statement of
financial position.

One surprising error was that whilst almost all candidates correctly calculated the percentages of the
subsidiary and the associate held by the parent, a significant minority of candidates subsequently mixed
up the associate percentage (25%) with the non-controlling interest percentage (30%).

Other common errors included correctly adding the 40,000 due on the new machine to consolidated
property, plant and equipment but either showing the corresponding liability as current or not showing a
corresponding liability at all, or simply adding 80,000 to property, plant and equipment rather than only
40,000.

A worrying few consolidated either only four-twelfths or 70% of the subsidiarys assets and liabilities.

Total possible marks 22


Maximum full marks 22

The Institute of Chartered Accountants in England and Wales 2010 Page 10 of 13


Financial Accounting Professional Stage March 2010

Question 4
Overall marks for this question can be analysed as follows: Total: 21

General comments
The first part of this question is a single topic question focusing on non-current assets, including aspects on
leasing. Candidates were required to prepare a finance lease calculation, assess a research and development
project and also carry out an impairment review. Parts b) and c) covered concepts issues, with a discussion on
substance over form and how the four qualitative characteristics related to lease transactions.

Blantyre Ltd

(a) Summary of costs included in income statement for the year ended 31 December 2009

Administrative expenses:
Depreciation (85,000 / 5yrs) 17,000
Amortisation (192,000 / 4yrs x 6/12 months) 24,000
Impairment of know-how (W3) 11,000
Research costs 70,000
Promotional advertising costs 15,000
Staff training costs 13,000
Finance costs (W1) 6,400

Statement of financial position as at 31 December 2009 (extracts)



Non-current assets
Property, plant and equipment (85,000 17,000) 68,000
Intangible assets (120,000 15,000 + 157,000) 262,000

Non-current liabilities
Finance lease liabilities (W1) 43,200

Current liabilities
Finance lease liabilities (62,400 43,200) (W1) 19,200

(1) Finance lease


Deposit 5,000
Instalments (4 x 24,000) 96,000
Fair value of asset (85,000)
Finance charges 16,000

SOTD = (4 x 5) 2 = 10
B/fwd (85,000 5,000) = 80,000

Year ended B/f Interest Payment C/f



31 December 2009 80,000 (16,000 x 4/10) 6,400 (24,000) 62,400
31 December 2010 62,400 (16,000 x 3/10) 4,800 (24,000) 43,200

(2) Technical Know-how



Original cost 180,000
Legal costs 4,000
Manufacturing supervisors time 3,200
Testing costs 4,800
192,000

(3) Impairment
Carrying amount at 31 Dec 2009 (192,000 24,000) 168,000
Recoverable amount (157,000)
Impairment 11,000

The Institute of Chartered Accountants in England and Wales 2010 Page 11 of 13


Financial Accounting Professional Stage March 2010

In part (a) most candidates picked up a considerable number of marks for correct calculations, however many
lost marks for their statement of financial position extracts, as these were not properly presented.

The majority of candidates made a good attempt at the leasing table. The most common error was to not
deduct the deposit of 5,000 paid upfront. Candidates however often made a mistake in allocating the lease
liability between current and non-current, with a significant minority of candidates allocating the full 24,000
payment as current thereby not understanding that it should only be the capital element of this.

Common errors included calculating an incorrect sum-of-digits figure, taking the fair value of 152,000 as the
recoverable amount of the technical know-how rather than the estimated future cash flows of 157,000, not
excluding 13,000 staff training costs from the amount originally recognised for the technical know-how and
failing to compare the amortised carrying amount of the technical know-how to the recoverable amount.

Total possible marks 13


Maximum full marks 13

(b)

Substance over form is an accounting concept that should be applied to all accounting areas in accordance
with the IASB Framework. Leasing is an example of the application of this concept.

To recognise the substance of a transaction, its economic reality should be reflected rather than merely its
legal form.

IAS 17, Leases looks at the economic reality of a lease through the assessment of which party carries the risks
and rewards of ownership, rather than looking at legal ownership. If the effect of the lease transaction is such
that in commercial effect it is similar to borrowing the money and buying the asset outright, both IAS 17 and the
IASB Framework require the asset and in effect a related loan to be recognised.

Conversely, if the risks and rewards of ownership remain with the lessor, as they do in an operating lease, then
in effect the substance of the transaction is the same as its legal form and no asset or corresponding liability
should be recognised.

Answers to part (b) were adequate, with most candidates quoting a reasonable definition of substance over
form, recognising that the way finance leases are accounted for is an example of this concept and discussing
the transfer of risks and rewards. However, most candidates focused on the recognition of the asset with no
mention of a corresponding liability. Whilst some candidates discussed the fact that assets held under
operating leases are not capitalised because the risks and rewards are not transferred, it was rare to see the
point that for operating leases substance is the same as legal form.

Total possible marks 4


Maximum full marks 3

(c)

Qualitative characteristics and IAS 17.

Relevance
Information is relevant if it can influence the economic decisions of users. By showing the true substance of a
finance lease, a company is made to show the debt that it has in its financial statements. This may influence
potential lenders in the future. The commitments note in relation to operating leases and the liability note in
relation to finance leases will also provide potential lenders essential information on what the companys
commitments and obligations already are.

Reliability
Information is reliable if it is free from error or bias, complete and portrays events in a way that reflects their
reality.

The Institute of Chartered Accountants in England and Wales 2010 Page 12 of 13


Financial Accounting Professional Stage March 2010

To be reliable information must faithfully represent a transaction. IAS 17 does this by following the overriding
criteria of substance over form.

Comparability
Users must be able to compare information with that of previous periods or with that of another entity.
Comparability is achieved through consistency and disclosure.

IAS 17 does require some subjectivity when a company assesses the risks and rewards of ownership.
However, detailed disclosure requirements, including setting out the companys accounting policies will help
with comparability.

In addition, IAS 17 ensures that financial statements are comparable between a company that has taken out a
loan to acquire an asset or one that has entered into a finance lease.

Understandability
Information must be readily understandable to users so that they can perceive its significance. It is dependent
on how information is presented.

There may be some confusion in looking at the non-current assets owned by a company, as these will include
those assets that are held under finance leases. However, the accounting policies will explain this and it is
assumed that users have a reasonable level of knowledge.

Answers to part (c) were often poor. Most marks were scored from brief general points about the four
qualitative characteristics. Reliability was probably the characteristic that was dealt with the best although
some candidates strayed into IAS 16 and discussed whether it was more reliable to record an asset at its
historic cost or its fair value. Points were often repeated and often placed under the wrong characteristic. With
regards to understandability, many cited very complex notes as an example of the application of this concept.

Total possible marks 8


Maximum full marks 5

The Institute of Chartered Accountants in England and Wales 2010 Page 13 of 13


Financial Accounting - Professional Stage June 2010

PROFESSIONAL STAGE FINANCIAL ACCOUNTING OT EXAMINERS COMMENTS

The performance of candidates in the June 2010 objective test questions section for the Professional Stage
Financial Accounting paper was in line with the average performance on this section of the paper over all
sittings to date. Candidates performed well across all syllabus areas.

When practising OT items, care should always be taken to ensure that the principles underlying any particular
item are understood rather than rote learning the answer. In particular, candidates should ensure that they read
all items very carefully.

The following table summarises how well* candidates answered each syllabus content area.

Syllabus area Number of questions Well answered Poorly answered

LO1 5 5 0

LO2 5 4 1

LO3 5 4 1

Total 15 13 2

*If 50% or more of the candidates gave the correct answer, then the question was classified as well answered.

Comments on the two poorly answered questions, which covered LO2 (preparation of single company financial
statements) and LO3 (preparation of consolidated financial statements), are below:

Item 1

This item required candidates to calculate closing inventories for a manufacturing company, consisting of raw
materials, work in progress and finished goods. The finished goods needed to be valued at net realisable value
(a discounted selling price less selling costs). The work in progress also needed to be valued at net realisable
value (as for the finished goods less estimated costs to completion). The most common errors made were to:

value the work in progress at its cost to date, failing to recognise that the information about selling price
and selling costs for finished goods was also relevant to this calculation, or
to ignore the selling costs in the valuation of both finished goods and work in progress.

Item 2

This item required candidates to calculate the amount to be shown as trade payables in a consolidated
statement of financial position. To arrive at the correct figure candidates needed to adjust for cash-in-transit in
the book of the receiving company, and then cancel the intra-group balances. A significant number of
candidates failed to recognise that the balance in the paying companys books would already reflect the cash-in-
transit.

The Institute of Chartered Accountants in England and Wales Page 1 of 17


Financial Accounting - Professional Stage June 2010

MARK PLAN AND EXAMINERS COMMENTARY

The marking plan set out below was that used to mark this question. Markers were encouraged to use
discretion and to award partial marks where a point was either not explained fully or made by implication.
More marks were available than could be awarded for some requirements. This allowed credit to be given for
a variety of valid points which were made by candidates.

Question 1 Total Marks: 24

General comments
This question tested the preparation of an income statement, a statement of comprehensive income and a
statement of financial position from a trial balance plus a number of adjustments. Adjustments included a
bad debt write off based on an adjusting event after the reporting period, a write-down to inventories, a
finance lease taken out during the year, a revaluation during the year and a transfer between the
revaluation surplus and retained earnings based on a previous revaluation.

Dashwood Ltd
Income statement for the year ended 31 March 2010


Revenue 945,700
Cost of sales (W1) (604,000)
Gross profit 341,700
Distribution costs (97,400)
Administrative expenses (W1) (164,100)
Profit from operations 80,200
Finance cost (1,230 + 5,000 (OF) (W6) + 500) (6,730)
Profit before tax 73,470
Income tax expense (10,000)
Profit for the year 63,470

Statement of comprehensive income for the year ended 31 March 2010



Profit for the year 63,470
Other comprehensive income:
Gain on property revaluation 250,000
Total comprehensive income for the year 313,470

Statement of financial position as at 31 March 2009


Assets
Non-current assets
Property, plant and equipment (W2) 2,620,100
Intangibles (75,000 x 4/8) 37,500
2,657,600
Current assets
Inventories (W1) 41,000
Trade and other receivables (140,950 5,500 (OF) (W1)) 135,450
176,450
Total assets 2,834,050

The Institute of Chartered Accountants in England and Wales Page 2 of 17


Financial Accounting - Professional Stage June 2010

Equity and liabilities


Equity
Ordinary share capital 245,000
Revaluation surplus (W4) 835,000
Retained earnings (W3) 1,329,650
2,409,650
Non-current liabilities
Finance lease liability (W6) 138,000

Current liabilities
Trade and other payables 181,200
Finance lease liability (182,000 138,000) (W6) 44,000
Taxation 10,000
Borrowings (50,700 + 500) 51,200
286,400
Total equity and liabilities 2,834,050

Workings

(1) Allocation of expenses


Cost of sales Distribution Administrative
costs expenses

Per Q 392,800 97,400 123,600
Opening inventories 35,600
Bad debts (11,000 x ) 5,500
Research costs (75,000 x 4/8) 37,500
Closing inventories (41,000)
Depreciation charges (W2) 179,100 35,000
604,000 97,400 164,100

(2) PPE
Plant and Land and
machinery buildings

B/f Cost/valuation 670,500 2,150,000
Finance lease 225,000
895,500
Revaluation (1,000,000 750,000) 250,000
B/f Accumulated depreciation (356,300) (105,000)
Depreciation buildings (1,400,000 40) (35,000)
Depreciation plant (895,500 5) (179,100)
360,100 2,260,000

Total PPE 2,620,100

The Institute of Chartered Accountants in England and Wales Page 3 of 17


Financial Accounting - Professional Stage June 2010

(3) Retained earnings



At 31 March 2009 1,249,930
Transfer from revaluation surplus (W5) 16,250
Profit for the period 63,470
At 31 March 2010 1,329,650

(4) Revaluation surplus



At 31 March 2009 601,250
Valuation in the year (W2) 250,000
Transfer to retained earnings
Depreciation charge based on revalued amount (W2) 35,000
Depreciation charge based on HC (750,000 40) (18,750)
(16,250)
835,000

(5) Finance lease



Instalments (4 x 60,000) 240,000
Cash price of machine (225,000)
Finance charges 15,000

SOTD = (5 x 6) 2 = 15

Year ended B/f Interest Payment C/f



31 March 2010 225,000 (15,000 x 5/15) (48,000) 182,000
5,000
31 March 2011 182,000 (15,000 x 4/15) (48,000) 138,000
4,000

The Institute of Chartered Accountants in England and Wales Page 4 of 17


Financial Accounting - Professional Stage June 2010

As in previous sittings, candidates were clearly very well-prepared for this type of question. Almost all
candidates produced a well-laid out income statement and statement of financial position and the standard
of presentation was the highest seen to date. Only a small minority of candidates lost presentation marks
for the statement of financial position by not adding across numbers in brackets or failing to complete sub-
totals and/or totals on their statements or by having incomplete or abbreviated narrative. Others lost
presentation marks for failing to include a sub-total for profit from operations on their income statement. As
ever, candidates should remember that this type of question requires financial statements to be in a form
suitable for publication. Presentation of the statement of comprehensive income was also excellent, with
the majority of candidates showing this, as required by the question (and as shown in the learning
materials) as a separate statement. This was the first time this statement had been examined and the
majority of candidates clearly demonstrated an understanding of the relationship between that statement,
retained earnings and the revaluation surplus.

Although many workings, in particular the cost matrix and the property, plant and equipment working, were
clearly laid out, a few candidates workings were disorganised, untidy and therefore hard to follow, making
it difficult to establish candidates approaches where they had not calculated the correct figure. It is
particularly difficult to follow workings which use little or no narrative, or costs workings done on the face of
the income statement. In particular, it was sometimes difficult to identify the final number for property, plant
and equipment on the face of the statement of financial position in the workings. The most sensible layout
for a property, plant and equipment working is a table with columns for the different categories which
clearly shows brought forward balances and movements in the year (effectively a simplified version of the
disclosure note).

Most candidates were able to deal with the more straightforward adjustments such as the bad debt write-
off, the write-down of closing inventory from cost to net realisable value and the income tax charge for the
year. However, what was pleasing was that many candidates also coped with the more difficult aspects of
the question such as the depreciation transfer between reserves.

By far the most common error was the failure identify that the bank account balance at the year end was a
credit balance with many candidates including this figure in current assets instead of as an overdraft in
current liabilities. As always, there was also some failure to complete double entry such as:
including the outstanding bank interest from the bank reconciliation as a finance cost but not
adjusting the balance at bank (or vice versa)
using a different figure for depreciation in the property, plant and equipment working and the costs
matrix
capitalising some of the research and development costs but not including the balance in
expenses.

Errors dealing with other adjustments included the following:


Failing to include the leased asset in property, plant and equipment (but then sometimes including
depreciation on that asset in expenses).
Treating the finance lease as if payments were in advance rather than in arrears.
Splitting the closing finance lease liability incorrectly between current and non-current.
Including the lease payment in finance costs (sometimes in addition to the interest on the lease).
Not identifying that half of the research and development costs should be capitalised (with a
number of candidates either capitalising all the costs or expensing all of the costs).
Allocating expenses to the incorrect income statement category (for example, including the
depreciation charge on the leased asset in administrative costs even though it related to plant and
machinery).
Treating the revaluation in the year as if it related to buildings rather than to land.
Failing to adjust for the outstanding item in the bank reconciliation.
Deducting the reserves transfer from the revaluation surplus from retained earnings rather than
adding it to retained earnings.
Adding total comprehensive income for the year (instead of profit for the year) to opening retained
earnings.
In the statement of comprehensive income, showing the net movement on the revaluation surplus
for the year, as opposed to the revaluation which took place in the year.

Total possible marks 24


Maximum full marks 24

The Institute of Chartered Accountants in England and Wales Page 5 of 17


Financial Accounting - Professional Stage June 2010

Question 2 Total Marks: 16

General comments
This question tested the preparation of a single company statement of cash flows and supporting note. Missing
figures to be calculated included interest paid, tax paid, dividends paid, property, plant and equipment acquired
and proceeds from the issue of share capital. A bonus issue of shares and the sale of revalued property during
the year also featured.

Middleton plc
Statement of cash flows for the year ended 31 March 2010

Cash flows from operating activities
Cash generated from operations (Note) 2,457,800
Interest paid (W1) (22,200)
Income tax paid (W2) (295,000)
Net cash from operating activities 2,140,600
Cash flows from investing activities
Purchase of property, plant and equipment (W3) (5,578,100)
Proceeds from sales of property, plant and equipment 2,800,000
Net cash used in investing activities (2,778,100)
Cash flows from financing activities
Proceeds from issue of ordinary share capital (320,000 (W4) + 400,000 720,000
(W5))
Proceeds from issue of borrowings 500,000
Dividends paid (W6) (599,400)
Net cash from financing activities 620,600
Net decrease in cash and cash equivalents (16,900)
Cash and cash equivalents at beginning of period 52,500
Cash and cash equivalents at end of period 35,600

Note: Reconciliation of profit before tax to cash generated from operations



Profit before tax 1,321,900
Finance costs 23,700
Depreciation charge 1,679,000
Amortisation charge (500,000 450,000) 50,000
Profit on disposal of property, plant and equipment (2,800,000 2,567,000) (233,000)
Increase in inventories (679,000 578,000) (101,000)
Decrease in trade and other receivables (656,800 547,500) 109,300
Decrease in trade and other payables ((657,900 5,000) (567,300 6,500)) (92,100)
Decrease in provision (500,000 200,000) (300,000)
Cash generated from operations 2,457,800

The Institute of Chartered Accountants in England and Wales Page 6 of 17


Financial Accounting - Professional Stage June 2010

Workings

(1) Interest paid



Cash () 22,200 B/d 5,000
C/d 6,500 IS 23,700
28,700 28,700

(2) Tax paid


Cash () 295,000 B/d 300,000
C/d 270,000 IS 265,000
565,000 565,000

(3) PPE


B/d 6,345,400 Disposals 2,567,000
Depreciation 1,679,000
Additions () 5,578,100 C/d 7,677,500
11,923,500 11,923,500
(4) Ordinary share capital


B/d 1,400,000
Bonus issue (1,400,000 5) 280,000
C/d 2,000,000 Cash () 320,000
2,000,000 2,000,000

(5) Share premium


B/d 200,000
C/d 600,000 Cash () 400,000
600,000 600,000

(6) Retained earnings


Dividends paid () 599,400 B/d 3,524,800
Bonus issue (W4) 280,000 Revaluation surplus 1,550,000
C/d 5,252,300 IS 1,056,900
6,131,700 6,131,700

The Institute of Chartered Accountants in England and Wales Page 7 of 17


Financial Accounting - Professional Stage June 2010

Candidates were clearly very well prepared for this question. Presentation was generally good and the vast
majority of candidates showed a strong grasp of the double-entry techniques which underpin the preparation
of a statement of cash flows, although some are still losing marks for failing to show outflows of cash in
brackets on the face of the statement.

Most candidates produced workings in the form of T accounts with far fewer than usual completing these T
accounts with the debits and credits the wrong way round. However, some candidates produced tabular
workings or working in brackets on the face of the statement of cash flows. This can make it more difficult to
see evidence of correct double entry and to award marks where the final figure is incorrect (or uses the
incorrect bracket convention). Pleasingly, very few candidates produced no workings at all an even riskier
approach as if figures are calculated incorrectly it is not possible to award any partial marks.

The majority of candidates scored very high marks on the reconciliation note, and on the figures for tax paid,
interest paid and the opening and closing figures for cash and cash equivalents. In the reconciliation note most
candidates dealt correctly with the opening and closing interest accrual which was included in trade and other
payables, an area which has caused problems in the past.

Candidates generally made a good attempt at the property, plant and equipment T account, with the figures
given in the question for depreciation and for the disposals both being correctly used. Where mistakes were
made, the most common error was to include the release of the revaluation surplus during the year on the
credit side of this T account instead of on the credit side of the retained earnings T account.

The other common mistake which affected the retained earnings T account was to deduct the bonus issue
from the share premium account instead of from retained earnings, as specified in the question. Where either
the bonus issue or the release of the revaluation surplus was omitted from the retained earnings working,
candidates then arrived at a balancing credit balance. Many thought that this indicated dividends received
demonstrating a lack of understanding of how dividends received would be properly accounted for in a set of
financial statements.

Other common errors included the following:


In the reconciliation note, failing to adjust for the decrease in the warranty provision, or making the
adjustment in the wrong direction.
Again in the reconciliation note, failing to adjust for the profit on disposal of property, plant and
equipment.
Miscalculating the number of shares in the bonus issue.
Failing to adjust the property, plant and equipment working for the disposals.
Failing to deal with or miscalculating the amortisation charge for the year.

Total possible marks 16


Maximum full marks 16

The Institute of Chartered Accountants in England and Wales Page 8 of 17


Financial Accounting - Professional Stage June 2010

Question 3 Total Marks: 15

General comments
This question mixed two discrete topics. Part (a) required the calculation of specified figures from a
consolidated statement of financial position in respect of a subsidiary and an associate, both of which
were acquired during the year. A fair value adjustment (with subsequent additional depreciation) had to be
made to the subsidiary. Part (b) required the preparation of a provisions note showing both the numerical
table and the relevant narrative disclosures.

Norland Ltd
(a) Figures for the consolidated statement of financial position as at 31 March 2010

(i) Goodwill

Fair value of consideration
Cash 200,000
Deferred cash 385,500
Shares (750,000 x 1.20) 900,000
1,485,500
Non-controlling interest at acquisition (921,600 (W) x 25%) 230,400
Less: Fair value of net assets at acquisition (921,600)
794,300

(ii) Non-controlling interest (1,086,000 (W) x 25%) 271,500

(iii) Investment in associate



Cost 500,000
Share of post acquisition change in net assets
Share of post acquisition profits (123,600 x 9/12 x 30%) 27,810
527,810
Working

Net assets Delaford Ltd


At year end Acquisition

Share capital 100,000 100,000
Retained earnings
Per Q (741,600 + (9/12 x 235,200)) 918,000 741,600
Fair value adjustment (300,000 80,000 80,000
220,000)
Additional depreciation based on fair (12,000)
value ((80,000 (OF) 5) x 9/12)
1,086,000 921,600

The Institute of Chartered Accountants in England and Wales Page 9 of 17


Financial Accounting - Professional Stage June 2010

Candidates generally made a very good attempt at Part (a), with a significant number gaining full marks.
This was very encouraging as candidates have historically performed less well when asked to produce
extracts from the consolidated financial statements, as opposed to a single consolidated statement.

In the calculation of goodwill, almost all candidates correctly calculated the total consideration. A few
omitted the shares issued from this calculation or used the 1 nominal value to value this part of the
consideration rather than the fair value. A number of candidates wasted time calculating goodwill arising
on the acquisition of the associate, when this would not appear as Goodwill in the consolidated
statement of financial position, but would effectively be subsumed within the Investment in the associate
figure, which was separately required.

The calculation of the investment in the associate was again very pleasing. A significant number of
candidates again calculated this correctly, even where they had made mistakes elsewhere in the question.
The most common error was to omit to time-apportion the profit for the year to allocate only nine months of
the twelve to post acquisition profits. Occasionally, candidates confused the associate percentage holding
with the non-controlling interest holding in the subsidiary, an error that was noted at the last sitting
candidates must be careful to use the correct percentages as this loses easy marks. A minority of
candidates confused the working by trying to calculate the share of net assets held but adding it to the
cost of the associate.

The net assets working for the subsidiary was the least well answered part of Part (a) although, again,
there were plenty of correct answers. One of the most common errors was to omit share capital from this
working. The majority of candidates did adjust net assets at both acquisition and at the year end for the
fair value adjustment of 80,000. Pleasingly, most candidates then went on to correctly calculate the
additional depreciation arising from this adjustment, although some omitted to pro-rate this for the nine
month post-acquisition period and others incorrectly also adjusted net assets at acquisition for this .

The most common error in calculating the figure for non-controlling interest, apart from errors in the net
assets table, was to calculate the non-controlling interest at acquisition instead of at the year end, or to
calculate the non-controlling interest figure for the consolidated income statement instead of for the
consolidated statement of financial position.

Total possible marks 7


Maximum full marks 7

The Institute of Chartered Accountants in England and Wales Page 10 of 17


Financial Accounting - Professional Stage June 2010

(b) Notes to the financial statements for the year ended 31 March 2010

Provisions

Faulty goods Restructuring Provision for Total


provision provision fines

At 1 April 2009 10,000 - 60,000 70,000
Utilised in the year (8,500) - - (8,500)
Income statement 20,900 350,000 (20,000) 350,900
charge/(credit) ()
At 31 March 2010 (W) 22,400 350,000 40,000 412,400

Faulty goods provision

The provision in respect of faulty goods relates to the supply of faulty hair straighteners during the year
ended 31 March 2010. The provision is based on the cost to the company of repairing or replacing the
faulty hair straighteners. All such expenditure is expected to be incurred in the year ended 31 March 2011.

Restructuring provision

During the year the company publicly announced and then commenced a restructuring of its domestic
appliances division. The provision is based on the anticipated further costs of the restructuring, all of which
are expected to be incurred in the year to 31 March 2011.

Provision for fines

The company has not yet fitted smoke filters in its factories as required by legislation which came into
force on 1 January 2009. Although the company plans to start the installation in May 2010, companies in
similar situations have been fined for such non-compliance. The year-end provision is based on lawyers
best estimate of the likely amount of such a fine.

Working

Faulty goods: 800 x 80% = 640 Provide (640 x 50% x 20) + (640 x 50% x 50) = 22,400

Restructuring: Provide for direct expenditure only = 300,000 + 50,000 = 350,000

Fines: Do not provide for costs of fitting smoke filters (no obligating event) but provide for best estimate of
fines which are more likely than not (ie 75%) to be imposed

The Institute of Chartered Accountants in England and Wales Page 11 of 17


Financial Accounting - Professional Stage June 2010

Candidates answers to Part (b) were mixed. However, there was a marked improvement from candidates
performance when this topic was set in a previous examination. The majority of candidates clearly
understood what the provision table showed and that the year-end position represented the provision
required at that date rather than the movement in the year. The faulty goods and restructuring provisions
were generally dealt with correctly, although the provision for fines caused more problems.

The faulty goods provision using the expected cost was generally calculated correctly. The most common
error was not adjusting for the 20% of claims that were not valid and therefore using the whole 800 claims.
For the restructuring provision, the majority of candidates recognised that the staff retraining and relocation
cost should not be included in the closing provision, with only a few candidates including this. The provision
for fines caused the most problems as candidates struggled to distinguish between the fines and the
provision for the work. A typical answer included both of these, highlighting that candidates did not
appreciate the difference between when an obligation exists and when it does not.

A significant number of candidates confused the utilisation of a provision and the movement on the
provision in the period. Very few candidates picked up the available presentation mark as they omitted a
total column from their provisions table. A minority produced a series of tables as opposed to one table
and a few produced a series of T accounts.

However, the most disappointing aspect to this part of the question was the quality of the narrative
disclosures. Candidates still do not appreciate the difference between an explanation of why a provision
has been made (which is not required in the Financial Accounting paper) and the information contained in a
disclosure note. Good clear narrative disclosures in an appropriate style was only seen in a minority of
scripts.

Total possible marks 9


Maximum full marks 8

The Institute of Chartered Accountants in England and Wales Page 12 of 17


Financial Accounting - Professional Stage June 2010

Question 4 Total Marks: 25

General comments
Part (a) was a consolidated income statement question, featuring two subsidiaries (one fully disposed of
within the year) and one associate. Adjustments included intra-group trading and unrealised profits and
impairment write-downs. Part (b) required the calculation of opening consolidated retained earnings. Part (c)
tested an understanding of the concepts underlying the preparation of consolidated financial statements:
namely the single entity concept and control versus ownership.

Jennings plc

(a) Consolidated income statement for the year ended 31 March 2010
000
Revenue (W2) 106,500
Cost of sales (W2) (62,150)
Gross profit 44,350
Operating expenses (W2) (26,600)
Profit from operations 17,750
Investment income 1,500
Share of profit of associates ((6,400 x 40%) 100) 2,460
Profit before tax 21,710
Income tax expense (W2) (5,700)
Profit for the year from continuing operations 16,010
Profit for the year from discontinued operations (3,900 + 660 ) (W4)) 4,560
Profit for the year 20,570

Attributable to
Equity holders of Jennings plc () 17,825
Non-controlling interest (W6) 2,745
20,570

Workings

(1) Group structure

Jennings plc

1.6
= 40%
4
6.4 4.2
= 80% = 70%
8 for 6/12 6
Palmer Ltd

Ferrars Ltd
Brandon Ltd

The Institute of Chartered Accountants in England and Wales Page 13 of 17


Financial Accounting - Professional Stage June 2010

(2) Consolidation schedule

Jennings plc Brandon Ltd Adj (W3) Consol


000 000 000 000
Revenue 67,600 42,500 (3,600) 106,500
Cost of sales per Q (43,700) (21,750) 3,600
PURP (W3) (300) (62,150)
Op expenses per Q (12,700) (13,200)
GW impairment (700) (26,600)
Investment income (W5) 500 1,000 1,500
Tax (4,000) (1,700) (5,700)
6,550

(3) Intra-group sale and PURP

% 000
SP 120 3,600
Cost (100) (3,000)
GP 20 600
X 300

(4) Group profit on disposal of Ferrars Ltd


000 000
Carrying amount of net assets at disposal
Net assets at 31 March 2009 (8,000 + 2,700) 10,700
Profit six months to 30 September 2009 (7,800 3,900
x 6/12)
14,600
Carrying amount of goodwill at disposal
Cost of investment 10,000
Non-controlling interest at acquisition ((8,000 + 550) x 1,710
20%
Less: Net assets at acquisition (8,000 + 550) (8,550)
3,160
Less: Impairments to date (500)
2,660
Total assets disposed of 17,260
Less: Attributable to non-controlling interest (14,600 x 20%) (2,920)
Total assets attributable to parent now disposed of 14,340
Sale proceeds (15,000)
Profit on disposal 660

(5) Investment income in Jennings plc


000
Total per IS 7,300
Less: Profit on disposal of Ferrars Ltd (15,000 10,000) (5,000)
Share of Brandon Ltds ordinary dividend (2,000 x 70%) (1,400)
Share of Palmer Ltds ordinary dividend (1,000 x 40%) (400)
Other investment income 500

(6) Non-controlling interest in year


000
Ferrars Ltd (20% x 3,900 (W4)) 780
Brandon Ltd (30% x 6,550 (W2) 1,965
2,745

The Institute of Chartered Accountants in England and Wales Page 14 of 17


Financial Accounting - Professional Stage June 2010

Historically, candidates have performed less well on a consolidated income statement than on a consolidated
statement of financial position, but there were some excellent attempts at Part (a) of this question. A good
number of candidates calculated the correct profit on disposal of the subsidiary and took this, together with
the subsidiarys pre-disposal profit for the year, to the face of the consolidated income statement. Most
candidates also dealt correctly with the impairment in the carrying amount of the associate, whereas this has
not been dealt with well in the past.

Almost all candidates correctly calculated the percentage holdings for the subsidiaries and the associate.
Presentation of the consolidated income statement was also generally good with most candidates gaining
some of the available presentation marks. However, those candidates who combined their consolidation
schedule with a consolidated income statement were not awarded any presentation marks.

Candidates generally made a reasonable attempt at a consolidation schedule, although it was common to
see the disposed of subsidiarys results incorrectly included. This may indicate that some candidates are
confused over the difference in treatment between an acquisition and disposal. Candidates generally made
an adjustment for the provision for unrealised profit (usually in the correct column) and the related adjustment
in revenue and cost of sales, although this was often the incorrect figure of 3 million, with candidates not
adjusting for the mark-up on sale.

Candidates generally calculated the provision for unrealised profit correctly, with only a minority incorrectly
using a gross profit margin instead of a mark-up or forgetting to adjust for only half of the inventories being
held at the year end. The two non-controlling interests were generally calculated correctly although a
significant minority missed the one for the disposed of subsidiary, not appreciating that this companys results
were included for part of the year. The non-controlling interest was generally separated out on the face of the
consolidated income statement, although many abbreviated this to NCI which is not acceptable on the face
of such a statement.

The investment income caused a problem for many candidates although a few did arrive at the correct figure.
Candidates often made adjustments for the share of the dividends from the subsidiary and the associate
correctly and realised that some kind of adjustment was required for the profit on disposal but generally used
the consolidated profit on disposal figure rather than that shown in the parent company financial statements.

Although, as discussed above, a good number of candidates calculated the group profit on disposal of the
subsidiary completely correctly, other candidates were clearly confused. Most candidates made some
attempt at the working, although it was missed out completely by a minority of candidates. Most candidates
made a fair attempt at calculating the carrying amount of the goodwill at disposal, although some candidates
adjusted for the impairment prior to taking 80% of the net assets at acquisition (or 100% and then adjusting
for the non-controlling interests 20%). The carrying amount of the net assets at disposal was less well
calculated, with the majority of candidates making some miscalculation. Those candidates who attempted to
work backwards from the year-end net assets to the net assets at disposal generally fared less well than
those who worked forwards from the opening net assets. A frequent error was to use the number of shares
held (6.4 million) as opposed to the issued share capital (10 million).

Candidates generally included a figure for the share of the associates profits although the figure was
sometimes not adjusted for the impairment of 100,000. The profit from discontinued operations on the face
of the consolidated income statement was shown by most candidates although it often only included the profit
on disposal or the subsidiarys profit for part (sometimes incorrectly all) of the year rather than a combination
of both. Some took only the group share of the subsidiarys profit into the profit from discontinued operations,
instead of taking 100% at this stage and then taking out 20% of this later as part of the non-controlling
interest figure. A minority of candidates were clearly confused about where the discontinued operation should
be shown and thought that it was instead an exceptional type item in the main body of the income
statement.

Total possible marks 18


Maximum full marks 18

The Institute of Chartered Accountants in England and Wales Page 15 of 17


Financial Accounting - Professional Stage June 2010

(b) Consolidated retained earnings brought forward at 1 April 2009

000
Jennings plc 23,800
Ferrars Ltd (80% x (2,700 550)) 1,720
Palmer Ltd (40% x (4,550 600)) 1,580
Goodwill impairments to date (500)
26,600

The correct figure for opening consolidated retained earnings was correctly calculated by a significant
number of candidates. Where mistakes were made they included the following:
Adjusting for the impairment losses which arose in the year in addition to the cumulative impairment
losses at the start of the year.
Not including any figure(s) for the associate and/or the disposed of subsidiary (both of which had
been acquired several years ago).
Including a figure for the subsidiary acquired on the first day of the current year (and which should
not therefore have been included in the retained earnings brought forward on that date (from the
last day of the previous year)).
Failing to exclude pre-acquisition profits from the share of retained earnings for the subsidiary and
the associate.
Failing to take only the group share of the subsidiarys and the associates retained earnings.

Total possible marks 3


Maximum full marks 3

The Institute of Chartered Accountants in England and Wales Page 16 of 17


Financial Accounting - Professional Stage June 2010

(c) Concepts underlying the preparation of consolidated financial statements

Group accounts reflect the results and net assets of group members to present the group to the parents
shareholders as a economic single entity (single entity concept). This reflects the substance of the group
arrangement as opposed to its legal form, where each group member is a separate legal person.

For example, in the consolidation of the Jennings plc group, all revenue and costs are added together, as if
the group were a single entity (so, for example, Jennings Ltds revenue of 67.6 million and Brandons
revenue of 42.5 million are added). However, the single entity concept also means that any intra-group
transactions need to be eliminated, as otherwise items would be double counted in the context of the group
as a single entity.

Hence, because Brandon Ltd sold goods for 3.6 million to Jennings plc in the current year, that
amount needs to be subtracted from Brandon Ltds revenue and from Jennings plcs cost of sales as if
the group were a single entity that transaction would not have occurred.

Any profit made between parent and its subsidiary companies also needs to be eliminated where that profit
has not yet been realised outside the group. So, for the 3.6 million intra-group sale, because half of these
goods have not yet been sold outside the group, closing inventory needs to be reduced (cost of sales
increased) by the profit on half that amount (300,000), otherwise inventory will be overstated from the point
of view of the group as a whole. The adjustment effectively brings inventory back down to what it would
have been stated at if the intra-group sale had never taken place.

The other principle underlying the preparation of consolidated financial statements is the distinction between
control and ownership. Control is reflected by including all of the subsidiarys income and expenses in the
consolidated income statement, even where the parent does not own 100% of that subsidiary. So, for
Jennings plc, 100% of Brandon Ltds income and expenses are added in even though, in effect, Jennings
plc only owns 70% of those income and expenses.

Ownership is then reflected by showing that part of the subsidiarys results, which is not owned by the
parent, as a non-controlling interest. Jennings plcs consolidated income statement shows a non-controlling
interest of 2,745,000, representing that part of Brandon Ltd not owned by Jennings plc.

Where an investor (Jennings plc) does not have control but does have significant influence over an investee
(Palmer Ltd), line-by-line consolidation is not appropriate. But because Jennings has this influence, it is
reflected in the consolidated income statement as a single line item being its share of the associates profit
for the period.

Answers to Part (c) were the most disappointing and sometimes non-existent. Where an answer was
attempted most referred to the single entity concept and to substance over form but did not expand on these
concepts with examples from the consolidated income statement. Some gave examples from the
consolidated statement of financial position, which was not what was required by the question. The
distinction between ownership and control was often not mentioned, or if it was, was poorly explained, with
the focus often being on how control could be achieved. Comments about the consolidation process were
often vague and did not clearly show that candidates understood that 100% of a subsidiarys figures were
added in (control), and then the non-controlling interest share (ownership) taken out later even if they had
just demonstrated this in their answer to Part (a).

Many referred to the fact that an associate is not consolidated but failed to explain that this is because
there is not control, but significant influence, or to explain how the associate is dealt with in the consolidated
income statement (again, even if they had just demonstrated this in Part(a)).

A few candidates wrote at length about the concepts of accruals and going concern or about the qualitative
characteristics and scored few, if any, marks.
Total possible marks 6
Maximum full marks 4

The Institute of Chartered Accountants in England and Wales Page 17 of 17


Financial Accounting Professional Stage September 2010

PROFESSIONAL STAGE FINANCIAL ACCOUNTING OT EXAMINERS COMMENTS

The performance of candidates in the September 2010 objective test questions section for the Professional
Stage Financial Accounting paper was in line with the average performance on this section of the paper over all
sittings to date. Candidates performed well across all syllabus areas.

When practising OT items, care should always be taken to ensure that the principles underlying any particular
item are understood rather than rote learning the answer. In particular, candidates should ensure that they read
all items very carefully.

The following table summarises how well* candidates answered each syllabus content area.

Syllabus area Number of questions Well answered Poorly answered

LO1 2 1 1

LO2 7 6 1

LO3 6 6 0

Total 15 13 2

If 50% or more of the candidates gave the correct answer, then the question was classified as well answered.

Comments on the two poorly answered questions, which covered LO1 (accounting and reporting concepts) and
LO2 (preparation of single company financial statements), are below:

Item 1

This item was a knowledge based question covering the capital maintenance concept and historical cost
accounting.

Item 2

This item required candidates to calculate the carrying amount of an asset following an impairment review. A
simple comparison between the current carrying amount and recoverable amount was required. The most
common error was believing that the asset could simply be restated to an amount higher than its current
carrying amount.

The Institute of Chartered Accountants in England and Wales 2010 Page 1 of 14


Financial Accounting Professional Stage September 2010

MARK PLAN AND EXAMINERS COMMENTARY

The mark plan set out below was that used to mark these questions. Markers are encouraged to use discretion
and to award partial marks where a point was either not explained fully or made by implication. More marks are
available than could be awarded for each requirement, where indicated. This allows credit to be given for a
variety of valid points, which are made by candidates.

Question 1

Overall marks for this question can be analysed as follows: Total: 19

General comments
This question was a typical question testing the preparation of an income statement and statement of
financial position from a trial balance. A number of adjustments were required to be made, including a
calculation of a provision, interest and dividends payable, a prior period error, capitalisation of development
expenditure and an adjustment to PPE for incorrect capitalisation.

(a)

ASSETS
Non-current assets
Property, plant and equipment (W2) 317,560
Intangible assets (237,600 39,600 (W1)) 198,000
515,560

Current assets
Inventories 375,600
Trade receivables 51,000
Cash and cash equivalents 21,500

448,100

Total assets 963,660

EQUITY AND LIABILITIES


Equity
Ordinary share capital 152,000
Irredeemable preference shares 180,000
Retained earnings (W3) 181,260

Equity 513,260

Non-current liabilities
Bank loan 150,000

Current liabilities
Trade and other payables (123,700 + 95,000) 218,700
Taxation 35,700
Provisions (310,000 x 10%) 31,000
Dividend payable (W3) 9,000
Interest payable 6,000
300,400

Total equity and liabilities 963,660

The Institute of Chartered Accountants in England and Wales 2010 Page 2 of 14


Financial Accounting Professional Stage September 2010

Eyam plc Income Statement for year ended 30 June 2010



Revenue (3,973,000 31,000) 3,942,000
Cost of sales (W1) (2,260,500)

Gross profit 1,681,500


Administrative expenses (W1) (1,556,040)

Operating profit 125,460


Finance costs (150,000 x 4%) (6,000)
Profit before tax 119,460
Taxation (35,700)

Net profit for the period 83,760

Note: Marks will be awarded if items are included in a different line item in the income statement
provided that the heading used is appropriate.

W1 Expenses

Cost of Admin
sales expenses

Trial balance 1,560,000 930,000
Opening inventory 346,500
Less: closing inventory (375,600)
Operating lease rentals ((1,045,000 + 95,000) / 2) 570,000 570,000
Depreciation charge fixtures & fittings 56,040
Research costs 120,000
Development exp amortisation
((357,600 120,000) / 2 yrs x 4/12months) 39,600
2,260,500 1,556,040

W2 Fixtures & fittings adjustment



Cost 646,000
Less: repairs (incorrect capitalisation) (25,000)
Accumulated depreciation (251,150)
Depreciation adjustment re repairs (25,000 x 15%) 3,750
(247,400)
373,600

Depreciation charge for year (373,600 x 15%) (56,040)

Carrying amount at 30 June 2010 317,560

W3 Retained earnings

B/fwd at 1 July 2009 127,750
Error re repairs capitalisation (25,000 3,750 (W2)) (21,250)

B/fwd restated 106,500


Profit in year 83,760
Preference share dividend (180,000 x 5%) (9,000)

C/fwd at 30 June 2010 181,260

The Institute of Chartered Accountants in England and Wales 2010 Page 3 of 14


Financial Accounting Professional Stage September 2010

As in previous sittings, candidates were clearly very well-prepared for this type of question, which remains
fundamental to the Financial Accounting syllabus. Almost all candidates produced a well-laid out income
statement and statement of financial position, with a significant number of candidates sub-totalling
amounts. The most common missed sub-totals include operating profit on the income statement and
non-current assets in the statement of financial position. Presentation marks were lost for incorrectly
combining some of the amounts presented in current liabilities, for example including taxation and
provisions as part of trade and other payables.

The cost matrix was generally clearly laid out, although a number of candidates were careless as to which
column they allocated expenses even where this had been expressly set out in the question text. A few
candidates workings, however, were disorganised, untidy and therefore hard to follow, making it difficult
to establish candidates approaches where they had not calculated the correct figure. This was noticeable
for the fixtures and fittings working which was often a muddle of figures sometimes resulting in candidates
putting the incorrect carrying amount on the face of the statement of financial position even though they
had correctly calculated this amount in their workings.

Candidates often left out narrative for amounts in their cost matrix, which meant that marking the figures
was frequently a challenge!

It was pleasing to see that most candidates were able to deal with the majority of adjustments, most of
which they would have seen on papers in the past. However, the treatment of returned goods caused a
number of problems with candidates not appreciating that this would impact on revenue. Instead
candidates recognised the debit side of the provision as an expense (as might be done for, say, a
warranty provision). The other adjustment that caused a problem was the incorrect capitalisation of
repairs in fixtures and fittings which occurred in the prior period and therefore required an adjustment to
brought forward retained earnings. The majority of candidates made the adjustment through current year
expenses, or not at all, although most did make the correct adjustment to property, plant and equipment.
The most disappointing incorrect treatment was the inclusion of the preference dividend as a finance cost
in the Income Statement even where it had already been correctly shown as a deduction in equity.

Other common errors included debiting the income tax charge for the period of 35,700 to the income
statement, but making no credit entry (as tax payable on the statement of financial position), correctly
splitting the accrued rent of 95,000 between administrative expenses and cost of sales, but failing to split
the original 1,045,000 and splitting the bank loan between non-current and current liabilities when it was
all non-current. A number of common errors were made in relation to non-current assets and these
included using an incorrect number of months when calculating the amortisation for the year on the
development expenditure, deducting the amortisation from the carrying amount of the intangible asset but
failing to show it as an expense and charging depreciation on cost instead of on carrying amount.

Total possible marks 19


Maximum full marks 19

The Institute of Chartered Accountants in England and Wales 2010 Page 4 of 14


Financial Accounting Professional Stage September 2010

Question 2
Overall marks for this question can be analysed as follows: Total: 25

General comments The first part of this question was a single topic question focusing on non-current
assets. Issues included adjustments for revaluations and the impact on depreciation, what is included in
the cost of an asset, component depreciation and the treatment of decommissioning costs.

Part b) covered concept issues, with a discussion on the four measurement bases with reference to the
measurement of an asset and an explanation of the advantages and limitation of using the historical cost
basis.

Litton plc
(a)
Statement of financial position as at 30 June 2010 (extracts)

Non-current assets
Property, plant and equipment (W1) 29,375,200

Non-current assets held for sale 90,000

Non-current liabilities
Provision for decommissioning 1,000,000

Equity & liabilities


Revaluation surplus (420,000 + 1,292,000) 1,712,000

Summary of amounts included in income statement for the year ended 30 June 2010

Administrative expenses:
Depreciation ((115,800 37,500) + 960,000 + 268,000 + 130,000) 1,436,300
Impairment of held for sale asset 22,500
Allocation of general overheads 36,000

Other income 45,000

W1 PPE Carrying amounts



Land (W2) 7,290,000
Plant and machinery (W3) 351,200
Hydro-electric plants (W4) 15,040,000
Solar power plants (W5) 4,824,000
Wind turbines (W6) 1,870,000
29,375,200

W2 Cost of land

Cost b/fwd 2,000,000
Purchase price 5,000,000
Professional fees 150,000
Site clearance costs 125,000
Planning application 15,000
5,290,000
Carrying amount c/fwd 7,290,000

W3 Plant and machinery Held for sale asset



Cost 300,000
Less: depreciation (300,000 / 8 yrs) x 5yrs (187,500)
Carrying amount at 30 June 2009 112,500
FV less costs to sell (90,000)
Impairment 22,500

The Institute of Chartered Accountants in England and Wales 2010 Page 5 of 14


Financial Accounting Professional Stage September 2010


Cost b/fwd 950,000
Less: held for sale asset (300,000)
Cost c/fwd 650,000

Accumulated depreciation b/fwd 408,000


Depreciation charge for year 115,800
Depreciation charged in error (re HFS asset) (37,500)
(300,000 / 8 yrs)
Acc dep re HFS asset (187,500)
(298,800)
Carrying amount 351,200

W4 Hydro-electric plant

Cost b/fwd 7,000,000
Cost 10,000,000
Testing - capitalised 100,000
Decommissioning 1,000,000
Acquisition in year 11,100,000
Cost c/fwd 18,100,000

Accumulated depreciation b/fwd 2,100,000


Depreciation charge for year (466,667 + 960,000
493,333)
(3,060,000)
Carrying amount at y/e 15,040,000

Depreciation on acquisition 11,100,000 / 15yrs x 8/12 months = 493,333

W5 Solar power plant



Cost / valuation b/fwd 4,200,000
Less: acc dep (400,000)
3,800,000
Revaluation (5,092,000 3,800,000) 1,292,000
Cost / valuation c/fwd 5,092,000

Depreciation in yr: 5,092,000 / 19 yrs (268,000)


Carrying amount at y/e 4,824,000

Useful life
25 yrs 6 yrs (1 July 2003 30 June 2009) = 19 yrs

W6 Wind turbines

Split components: 2,000,000 200,000 = 1,800,000



Cost 2,000,000
Depreciation:
1,800,000 / 20 yrs 90,000
200,000 / 5 yrs 40,000
(130,000)
Carrying amount at y/e 1,870,000

The Institute of Chartered Accountants in England and Wales 2010 Page 6 of 14


Financial Accounting Professional Stage September 2010

There were some good attempts at this question, although workings were often difficult to follow with a
haphazard array of numbers with no commentary accompanying them. There has been a marked
improvement with candidate responses to property, plant and equipment questions since it was first set
in December 2007. The majority of candidates tackled each of the five items of property, plant and
equipment separately thereby not confusing the issues. This was the best way to maximise marks.

Many candidates produced a property, plant and equipment table as a working, which was a
reasonable approach to take and clear to mark, although to complete it in its entirety wasted valuable
time when the table was not actually required by the question. Others took each type of asset in the
question separately and calculated a revised carrying amount and revised depreciation charge, which
was another sensible approach. However, other candidates produced a long string of plus and minus
numbers, with no narrative, which was almost impossible to mark.

The question asked for extracts from the statement of financial position and there were a number of
straight forward marks available here based on candidates own figures. However, a number of
candidates misinterpreted this as meaning the property, plant and equipment table although the
requirement clearly set out that notes to the financial statements were not required. Others produced
extracts but lost marks because those extracts were not properly presented (for example, the
revaluation surplus was not shown under Equity, or Property, plant and equipment was abbreviated
to PPE). Although the income statement impact did not have to be shown as an extract candidates
sometimes scattered these amounts throughout their answers, instead of presenting them in a
summary.

The majority of candidates correctly calculated the impairment for the held for sale asset and made a
good attempt at the plant and machinery calculations. One of the most common errors involved the
component depreciation calculation, with candidates correctly calculating depreciation on the battery
storage systems over five years but then adding this cost to the total rather than deducting it to get the
cost of the wind turbines themselves. Candidates also often missed the wind turbines out when adding
together all the items of property, plant and equipment for the extracts to the statement of financial
position.

Other common errors for each non-current asset item included:

Land: deducting the rental income from cost and/or capitalising the general overheads.

Hydro-electric power plants: netting the 1 million restoration costs off the cost of 10 million,
instead of adding it. A significant number of candidates showed the provision for restoration costs
as an expense rather than capitalising it, or included it as a current, instead of a non-current liability.
A significant number of candidates also used the incorrect number of months when calculating the
current year depreciation.

Solar power plants: including the 68,000 additional depreciation on the surplus in the annual
depreciation charge, but not adding in the original 200,000 charge based on cost. Other
candidates included the whole 268,000 but also the original 200,000. A few candidates wasted
time making a transfer from the revaluation surplus for the additional depreciation, when the
question specifically stated that this was not the companys policy.

Held for sale asset: calculating accumulated depreciation brought forward based on an incorrect
number of years.

Total possible marks 18


Maximum full marks 18

The Institute of Chartered Accountants in England and Wales 2010 Page 7 of 14


Financial Accounting Professional Stage September 2010

(b)(i)

Historical cost assets are recorded at the cash or fair value of the consideration paid.

Current cost assets are recorded at the amount that it would currently cost to acquire the asset today.

Realisable value the amount that would be received if the asset was sold today in its current condition.

Present value a current estimate of the present discounted value of the future net cash flows in the normal
course of business.

(ii)

Usefulness of historic cost

Historical cost is a known amount, it is a reliable measurement there is no subjectivity involved unlike
the revaluation model where a great deal of judgement is involved.
There is no cost involved in valuing historical cost as it is the amount that was paid. Measuring fair value
can be extremely costly depending on the nature of the asset.
Other measurement bases can be subject to manipulation, as valuation techniques need to be applied.

Limitations of historical cost

By its very definition it is an historical amount and therefore does not reflect the true value that the asset
may be worth unlike revalued amounts which are current at the time of the valuation. For example,
property prices generally increase over time, so a property acquired a number of years ago will be
shown in the financial statements at a value significantly less than its true value to the business.
Historical cost also ignores the effects of inflation.

Responses to part b) were generally disappointing, although there were some very good answers to this part.
Most candidates made some attempt at the first part.

In (i), most candidates knew where to find an explanation of the four measurement bases in the open book
text and used the book to good effect, although a few provided unnecessarily long explanations. Others,
however, explained the mechanics of the cost model as opposed to the revaluation model, with much detail
given about when and how to revalue. Others confused the four measurement bases with capital
maintenance concepts.

Answers to the second element were often not thought out with candidates instead writing everything they
knew about historical cost and the revaluation model. A significant number of candidates discussed how the
cost model suffered from the problems of estimating an accurate useful economic life, seemingly unaware
that such a judgement is also needed when using the revaluation model (except for land). Others believed
that impairments only needed to be accounted for under the cost model. Although the concepts of relevance
and reliability were relevant, these needed to be explained, by reference to the factual nature of historic costs
and the subjective nature of revaluations. Those who wrote generally about relevance and reliability scored
poorly. A worrying few thought that revaluation made figures more reliable.

Total possible marks 8


Maximum full marks 7
Total possible marks 26
Maximum full marks 25

The Institute of Chartered Accountants in England and Wales 2010 Page 8 of 14


Financial Accounting Professional Stage September 2010

Question 3

Overall marks for this question can be analysed as follows: Total: 15

General comments
This was a mixed topic question covering revenue recognition, inventory valuation and investment in
associates. Part a) covered revenue recognition issues which included subscriptions, a service contract,
advertising revenues and sale and returns. Part b) required a straight forward calculation of closing
inventories based on units of production. Part c) required the calculation of the investment in associate line in
the consolidated statement of financial position following a PURP adjustment and the impact of the associate
on retained earnings.

(a) Calculation of revenue

Draft revenue 2,176,900


Magazine subscriptions adjustment (W1) (203,000)
Promotional advertising (17,500 / 2) 8,750
Returns (1,500 x 2) (3,000)
News service (W2) 7,500

Revised revenue 1,987,150

WORKINGS
W1 Magazine subscriptions

Receipt date Subscription period Adjustment Total



March 2010 Apr June 129,000 x 0
April 2010 May July 84,000 x 1/3 28,000
May 2010 June Aug 96,000 x 2/3 64,000
June 2010 Jul Sept 111,000 111,000
203,000

W2 Service contract

Revenue: (30,000 x 9/12 months) 22,500
Cash received (1 Oct 2009) (15,000)

Additional revenue to recognise 7,500

Candidates generally made some attempt at this part of the question with the majority of candidates gaining
at least half marks in it and many scoring full marks.

A common mistake was to calculate total revenue from the items given, as opposed to adjusting the given
(total) revenue figure for the extra revenue that needed to be added or deducted. These candidates
therefore lost marks for not showing what had been brought in on a cash basis.

The most common errors included making adjustments in the wrong direction, taking out the wrong fraction
of subscriptions received in April and May and only adjusting for subscriptions received in June.

Total possible marks 6


Maximum full marks 6

The Institute of Chartered Accountants in England and Wales 2010 Page 9 of 14


Financial Accounting Professional Stage September 2010

(b)

Closing inventories
(1.20 x (25,500 2,000)) + (1 x 2,000) = 30,200

WORKINGS

Material cost 360,000
Variable overheads 240,000
600,000

Unit cost
600,000 / 800,000 0.75
450,000 / 1,000,000 0.45
1.20

Answers to this part of the question were the most disappointing on the paper as a whole. The question
highlighted a lack of understanding in a very straight-forward part of the syllabus. The most common failing
was to include the administrative expenses as part of the inventory unit cost, then spread it either over the
800,000 items produced or over the 1 million budgeted production. Most candidates did, however,
recognise that the 450,000 fixed production costs should be spread over the 1 million.

Other common errors included calculating a unit cost but then applying it to the whole 25,500 items in year-
end inventory, not just the 23,500 which needed to be valued at cost, applying the calculated unit cost to
23,500 items, but failing to add in the other 2,000 items, spreading all costs (with or without the
administrative costs) over the 1 million budgeted production and calculating a unit cost but then valuing all
inventory at selling price.

In addition, a number of candidates failed to calculate a unit cost at all, simply valuing all inventory at selling
price (all at 2.50 or 23,500 at 2.50 with the 2,000 correctly valued at 1) or added all costs together (with
or without the administrative costs) and valued inventory at that figure, with no attempt to spread that total
cost over the number of units produced or budgeted for.

Total possible marks 3


Maximum full marks 3

(c)(i)


Cost of investment 300,000
Share of post-acq retained earnings
((295,000 + 40,000) 250,000) x 40% 34,000
Less: Impairment (5,000)

Investment in associate 329,000

(ii)


Bretby plc 1,670,000
Alport - share of post-acq retained earnings
(34,000 14,000) 20,000
Less: Impairment (5,000)

Consolidated retained earnings 1,685,000

The Institute of Chartered Accountants in England and Wales 2010 Page 10 of 14


Financial Accounting Professional Stage September 2010

W1 Unrealised profit associate

200,000 100%
(130,000) 65%
70,000 35%

70,000 x 40% x = 14,000

Answers to this part were quite mixed, with most candidates picking up at least a few marks for stating the
cost of the investment and correctly showing the consolidated retained earnings figures without the
associate. Candidates clearly are not comfortable with consolidation workings involving an associate, which
was a direct contrast to a very good performance on Question 4, which featured two subsidiaries. There
were a number of correct answers, but the most common errors were over the treatment of the PURP
(which many adjusted for in both calculations, instead of just in retained earnings) or in a lack of
consistency between the share of profits taken to the investment in the associate and that taken to retained
earnings. Another common error was to adjust for the whole of the 35,000 PURP, as opposed to only 40%
of that figure.

Total possible marks 6


Maximum full marks 6
Total possible marks 15
Maximum full marks 15

The Institute of Chartered Accountants in England and Wales 2010 Page 11 of 14


Financial Accounting Professional Stage September 2010

Question 4

Overall marks for this question can be analysed as follows: Total: 21

This question required the preparation of a consolidated statement of financial position. The group had two
subsidiaries, one of which was acquired during the year. The treatment of a gain on bargain purchase was
covered and a fair value adjustment in relation to inventories was required. Inter-company trading had taken
place during the year.

Pinxton plc

(a) Consolidated statement of financial position as at 30 June 2010


Assets
Non-current assets
Property, plant and equipment (670,000 + 140,000 + 240,000) 1,050,000
Intangibles (265,000 + 20,000) 285,000
Goodwill (W5) 53,760

1,388,760
Current assets
Inventories (135,000 + 60,000 + 65,000 + 3,750 (W3) 2,000 (W2)) 261,750
Trade and other receivables (96,400 + 63,000 + 86,400 30,000) 215,800
Cash and cash equivalents (18,900 + 14,000 + 7,950) 40,850
518,400
Total assets 1,907,160

Equity and liabilities


Equity
Ordinary share capital 950,000
Share premium account 310,000
Retained earnings (W7) 310,550
Attributable to the equity holders of Pinxton plc 1,570,550
Non-controlling interest (W6) 83,910
Equity 1,654,460

Current liabilities
Trade and other payables (89,600 + 51,000 + 88,300 30,000) 198,900
Taxation (33,200 + 6,700 + 13,900) 53,800
252,700
Total equity and liabilities 1,907,160

Workings
(1) Group structure

Pinxton plc

153 / 170 = 90%


Hayfield Ltd
192 / 240 = 80%
Smisby Ltd

The Institute of Chartered Accountants in England and Wales 2010 Page 12 of 14


Financial Accounting Professional Stage September 2010

(2) Net assets Hayfield Ltd


30 June 2010 Acquisition Post acq

Share capital 170,000 170,000
Retained earnings 69,300 72,000 (2,700)
PURP (2,000) (2,000)

237,300 242,000 (4,700)

(3) Net assets Smisby Ltd


30 June 2010 Acquisition Post acq

Share capital 240,000 240,000
Share premium account 30,000 30,000
Retained earnings 27,150 10,300 16,850
FV inventory adj (90,000 75,000) x 25% 3,750 15,000 (11,250)

300,900 295,300 5,600

(4) Goodwill Hayfield Ltd



Consideration transferred 190,000
Net assets at acquisition (W2) (242,000)
Non-controlling interest at acquisition (242,000 (W2) x 10%) 24,200

Gain on bargain purchase (27,800)

(5) Goodwill Smisby Ltd



Consideration transferred 300,000
Net assets at acquisition (W3) (295,300)
Non-controlling interest at acquisition (295,300 (W3) x 20%) 59,060
63,760
Less: Impairment (10,000)
53,760

(6) Non-controlling interest



Hayfield Ltd share of net assets (237,300 (W2) x 10%) 23,730
Smisby Ltd share of net assets (300,900 (W3) x 20%) 60,180
83,910

(7) Retained earnings



Pinxton plc 292,500
Hayfield Ltd ((4,700) (W2) x 90%) (4,230)
Smisby Ltd (5,600 x 80%)) 4,480
Less: Impairments to date (10,000)
Gain on bargain purchase (W4) 27,800
310,550
(8) PURP
Hayfield Ltd
%
SP (30,000 x 1/3) 125 10,000
Cost (30,000 / 125% x 1/3) (100) (8,000)
GP 25 2,000

The Institute of Chartered Accountants in England and Wales 2010 Page 13 of 14


Financial Accounting Professional Stage September 2010

Answers to this question were very good, showing that candidates are very comfortable with the consolidation
of subsidiaries. Presentation was on the whole good, although some candidates did lose presentation marks,
often for failing to show the split of total equity between the parent and the non controlling interest. A few
candidates lost marks because they failed to show bracketed workings for incorrect lines on their consolidated
statement of financial position.

It was pleasing to see that a good number of candidates understood that the gain on bargain purchase should
be recognised in retained earnings (although a number of candidates then deducted it, instead of adding it)
rather than being netted off against the positive goodwill shown in the consolidated statement of financial
position. Candidates generally made a good attempt at the two net asset tables, although a common error was
to use a positive figure for the movement in post-acquisition retained earnings even though they had calculated
one of the subsidiaries to be negative.

Workings for this question were generally clearly set out with the standard set of pro-forma workings being
used. This makes responses easier to mark with marks being awarded for partially correct answers. One of the
most common mistakes on this question in respect of presentation was to try and calculate the goodwill figures
in a combined calculation. In this question there was positive and negative (gain on bargain purchase) goodwill
and the two amounts should not be netted off (as discussed above).

Where errors were made they included the following, not including the share premium account in the
calculation of Smisbys net assets, or including it either at acquisition or at the year end but not at both dates,
failing to account for the intangible assets held by Pinxton and Hayfield and treating the information in relation
to the fair value adjustment for inventories at acquisition as leading to a reduction to net assets as opposed to
a fair value uplift. If this amount was correctly recognised in the net assets table candidates often did not go on
to make a corresponding adjustment to inventories on the consolidated statement of financial position.

Other common errors included failing to eliminate the 30,000 intra-group balance and using the non-
controlling interest percentages to calculate the share of Hayfields and Smisbys post-acquisition profits to be
taken to consolidated retained earnings.

Interestingly, after their consolidation, some candidates were left with a figure for Investments on their
consolidated statement of financial position. Sometimes this was the whole 490,000, sometimes just a part of
that figure. It was also interesting to note that a number of candidates made the correct fair value and PURP
adjustments in the net assets working but then took the wrong post acquisition figures to retained earnings, for
example, just taking the movement on reserves and ignoring the impact of the adjustments they had just
made.

Total possible marks 21


Maximum full marks 21

The Institute of Chartered Accountants in England and Wales 2010 Page 14 of 14


Financial Accounting - Professional Stage June 2013

PROFESSIONAL STAGE FINANCIAL ACCOUNTING OT EXAMINERS COMMENTS

The performance of candidates in the June 2013 objective test questions section for the Professional Stage
Financial Accounting paper was good. Candidates performed better on LO3 (preparation of consolidated
financial statements) than they did on the other two syllabus areas.

When practising OT items, care should always be taken to ensure that the principles underlying any particular
item are understood rather than rote learning the answer. In particular, candidates should ensure that they read
all items very carefully.

The following table summarises how well* candidates answered each syllabus content area.

Syllabus area Number of questions Well answered Poorly answered

LO1 4 3 1
LO2 6 4 2
LO3 5 5 0
Total 15 12 3

*If 50% or more of the candidates gave the correct answer, then the question was classified as well answered.

Comments on the two most poorly answered questions, both in LO2 (preparation of single company financial
statements) are given below:

Item 1

This item required candidates to calculate closing inventory in accordance with IAS 2, Inventories. The question
featured raw materials, work in progress and finished goods. Almost all candidates calculated a net realisable
value for work in progress and finished goods which was lower than cost and used that figure in their
calculation. However, although most candidates correctly allowed for a discounted selling price and for costs still
to be incurred to complete the work in progress, a majority did not reduce the discounted selling price by the
selling costs to be incurred to arrive at the correct figure for net realisable value.

Item 2

This item tested the calculation of the amount of an intangible asset to be capitalised in accordance with IAS 38,
Intangible Assets. Most candidates recognised that initial research costs and the cost of evaluating research
findings should not be capitalised and that development costs and patent registration costs should be
capitalised. However, a majority of candidates failed to recognise that the depreciation charged in the period on
specialised equipment needed for the development process should also have been capitalised.

Copyright ICAEW 2013. All rights reserved. Page 1 of 14


Financial Accounting - Professional Stage June 2013

PROFESSIONAL STAGE FINANCIAL ACCOUNTING

MARK PLAN AND EXAMINERS COMMENTARY

The marking plan set out below was that used to mark this question. Markers were encouraged to use discretion
and to award partial marks where a point was either not explained fully or made by implication. More marks
were available than could be awarded for each requirement. This allowed credit to be given for a variety of valid
points which were made by candidates.

Question 1

Overall marks for this question can be analysed as follows: Total: 30

General comments
Part (a) of this question tested the preparation of an income statement (which needed to be split between
continuing and discontinued operations) and a statement of financial position from a list of balances plus a
number of adjustments. Adjustments included a warranty provision, calculation of the annual depreciation
charge, a finance lease taken out during the year, and an adjustment to revenue to reflect IAS 18,
Revenue. Part (b) required a discussion of the objective of general purpose financial statements and the
purpose of accounting standards, illustrated by reference to the financial statements prepared in Part (a).

Falcon Ltd
(a) Income statement for the year ended 31 December 2012


Continuing operations
Revenue (W5) 1,264,600
Cost of sales (W1) (631,750)
Gross profit 632,850
Distribution costs (W1) (38,200)
Administrative expenses (W1) (223,200)
Profit from operations 371,450
Finance cost (12,600 + 1,000 (W6)) (13,600)
Profit before tax 357,850
Income tax expense (35,000 2,000) (33,000)
Profit for the year from continuing operations 324,850

Discontinued operations
Loss for the year from discontinued operations (W2) (164,600)
Profit for the year 160,250

Statement of financial position as at 31 December 2012


Assets
Non-current assets
Property, plant and equipment (W3) 543,750
Current assets
Inventories 35,600
Trade and other receivables 32,800
68,400
Non-current asset held for sale 80,000
148,400
Total assets 692,150

Copyright ICAEW 2013. All rights reserved. Page 2 of 14


Financial Accounting - Professional Stage June 2013

Equity and liabilities


Equity
Ordinary share capital 200,000
Retained earnings (W4) 140,550
340,550
Non-current liabilities
Finance lease liability (W6) 103,500
Deferred income (60,000 x 1/3) 20,000
123,500
Current liabilities
Trade and other payables 78,500
Taxation 35,000
Deferred income (60,000 x 1/3) 20,000
Provisions 55,000
Finance lease liability (W6) 5,000
Borrowings 34,600
228,100
Total equity and liabilities 692,150

Workings

(1) Allocation of expenses


Cost of sales Distribution Administrative
costs expenses

Per Q 744,300 43,500 235,600
Opening inventories 30,200
Adj re discontinued operations (160,900) (5,300) (17,400)
Adj re lease payment (5,000)
Closing inventories (35,600)
Depreciation charges (W3) 53,750 10,000
631,750 38,200 223,200

(2) Loss on discontinued operations



Revenue 114,000
Costs included in TB (160,900 + 17,400 + 5,300) (183,600)
Loss on held for sale asset/depreciation (120,000 (85,000 (40,000)
5,000))
Other discontinued operations costs (55,000)
(164,600)

(3) PPE
Plant and Land and
equipment buildings

B/f Cost 570,600 375,000
B/f Accumulated depreciation (235,600) (95,000)
Classified as held for sale (120,000)
Leased building (W6) 112,500
215,000
Depreciation plant @ 25% (53,750)
Depreciation buildings ((275,000 x 2%) Ab + (112,500 25) (OF)) (10,000)

161,250 382,500

Total PPE 543,750

Copyright ICAEW 2013. All rights reserved. Page 3 of 14


Financial Accounting - Professional Stage June 2013

(4) Retained earnings



At 1 January 2012 (19,700)
Profit for the period 160,250
At 31 December 2012 140,550

(5) Revenue

Per TB 1,418,600
Less: After sales support re future years (60,000 x 2/3) (40,000)
Discontinued operations (114,000)
1,264,600

(6) Lease of land and buildings

SOD = (24 x 25)/2 = 300



Total payments (5,000 x 25) 125,000
Fair value (112,500)
Finance charge 12,500

Year ended 31 B/f Payment Capital Interest C/f


December

2012 112,500 (5,000) 107,500 24/300 x 108,500
12,500)
1,000
2013 108,500 (5,000) 103,500

Tutorial note

Credit was also given if candidates depreciated the held for sale asset to the date of classification as held
for sale, and then calculated a (smaller) impairment loss. The shortcut taken above recognised the fact
that only a single figure for PPE was required for the statement of financial position.

Copyright ICAEW 2013. All rights reserved. Page 4 of 14


Financial Accounting - Professional Stage June 2013

Most candidates made a reasonable attempt at this question with the vast majority preparing a complete
statement of financial position and income statement. Presentation was reasonable, although a significant
number of candidates lost marks by failing to include appropriate sub-totals on their statement of financial
position. The majority of candidates did not appear to have been unnerved by the inclusion of a
discontinued operation in this question and, pleasingly, almost all included a figure for loss from
discontinued operations on the face of the income statement, although hardly any candidates showed
headings for Continuing operations and Discontinued operations (which was surprising as these were
included in Question 2 on the paper).

The adjustments to revenue, opening and closing inventory, the finance lease calculations, the asset held
for sale and allocation of costs to the correct expense category were all well dealt with, although very many
candidates calculated a second year interest charge for the finance lease when this was not needed. By
far the most common errors were the failure to correctly split the finance lease liability between current and
non-current, the inclusion of the wrong amount on the statement of financial position for income tax
payable (including the charge for the year, as opposed to the liability for the year), and an incomplete
calculation of the loss from discontinued operations. With regards to the latter, almost all candidates
calculated this as the sales of the Scottish operations less its costs, but far less increased this loss by the
estimated future costs and/or by the depreciation and/or impairment on the held for sale asset, with many
candidates either ignoring these figures or including them in continuing operations. A minority of
candidates time-apportioned the figures for the Scottish operations, clearly not understanding how IFRS 5
should be applied.

Other common errors included the following.


- Failing to deduct the prior year tax overestimate from the current year tax estimate to arrive at the
correct current year income statement charge and/or showing the wrong figure in current liabilities.
- Not correcting administrative expenses to remove the lease payment incorrectly posted to this
account.
- Incorrectly calculating the sum-of-the-digits for the finance lease.
- Treating the resultant deferred income as a current or non-current asset rather than as a liability or
failing to split the liability between current and non-current.
- Showing the asset held for sale in non-current as opposed to current assets.
- Taking the depreciation on the leased asset to cost of sales instead of administrative expenses.
- Adding the opening retained loss to the profit for the year, instead of deducting it.
- Failing to include a sub-total for operating profit on the income statement.
- Calculating, usually correctly, a carrying amount for the leased asset but then failing to add that
figure in to the property, plant and equipment figure for the statement of financial position.
- Calculating depreciation and/or impairment on the held for sale asset but failing to remove the cost
of the held for sale asset from property, plant and equipment.
- Depreciating the leased asset by 2% when it had a shorter useful life of 25 years.

Almost all candidates did use the recommended costs matrix when allocating costs between the three
expense categories. It was very noticeable that those candidates who did not use this format tended to
produce disorganised workings (often split between the face of the income statement and /or a number of
separate workings) which were difficult to follow and therefore might have lost marks. Far fewer candidates
seem capable of producing a clear working for property, plant and equipment which resulted in them
repeating calculations and often losing the connection between the depreciation expense to go into the
costs matrix and the depreciation expense to be added to accumulated depreciation brought forward. It
was often impossible to see any audit trail to support the final figure for property, plant and equipment on
the face of the statement of financial position and many candidates lost potential marks because of this.
This is an issue which has been flagged up repeatedly.
Total possible marks 25
Maximum full marks 25

Copyright ICAEW 2013. All rights reserved. Page 5 of 14


Financial Accounting - Professional Stage June 2013

(b) The objective of general purpose financial reporting

The IASB Conceptual Framework states that the objective of general purpose financial reporting is to
provide financial information about the reporting entity that is useful to existing and potential investors,
lenders and other creditors in making decisions about providing resources to the entity. These decisions
involve buying, selling or holding equity and debt instruments and providing or settling loans and other forms
of credit.

For example, the lessor of the land and buildings may have looked at Falcon Ltds previous financial
statements in deciding whether or not to extend credit. They would have considered whether Falcon Ltd
would be likely to be able to meet the lease repayment terms.

Investors would be particularly interested in the information concerning continuing versus discontinued
operations particularly as the continuing operations have made a profit of 324,850, but the discontinued
operations have made a loss of 164,600. Without this split it may have seemed that Falcon Ltd was only
able to generate profits of 160,250, less than half of its actual continuing profits.

Falcon Ltds suppliers may look at the financial statements in deciding whether or not to grant credit they
may be concerned that the fact that Falcon Ltds current liabilities are way in excess of its current assets
may mean that the company could struggle to pay its debts as they fall due.

The purpose of accounting standards

The purpose of accounting standards is to identify proper accounting practices for the preparation of
financial statements. Accounting standards create a common understanding between users and
preparers on how particular items are treated.

For example, it will be clear from Falcon Ltds financial statements that it carries its property, plant and
equipment under the cost model. Users will need to then take care if comparing Falcon Ltds financial
statements with those which use the valuation model.

It is IAS 17, Leases, which dictates the correct treatment of finance versus operating leases. Hence the
lease of the building was treated as a finance lease. This will be common practice across all entities
following IFRS and will make their financial statements comparable with those of other companies.

IAS 18, Revenue, dictates that Falcon Ltd only account for revenue on services provided to date. Hence an
adjustment was made in Part (a) to remove the revenue relating to after-sales support not yet provided.
Again, this will be common practice across all entities following IFRS
Attempts at the written part of the paper were, as usual, disappointing, with very few candidates scoring
more than one or two marks. Many failed to gain the marks for those parts of the answer that could be taken
from the open book text, and only a minority were able to provide examples from Falcon Ltds financial
statements which were relevant to either the objective of general purpose financial reporting or to the
purpose of accounting standards. Whilst most candidates recognised that accounting standards helped to
achieve consistency or comparability very few made the point that accounting standards inform the
preparers of accounts how to deal with key accounting issues in the financial statements. Many candidates
thought that the main purpose of accounting standards is to ensure that financial statements are prepared
on the basis of substance over form and then proceeded to give examples of accounting for substance
over form. Others discussed the qualitative characteristics of financial statements at length and gave
examples of how various accounting standards met these qualitative characteristics.
Total possible marks 8
Maximum full marks 5

Copyright ICAEW 2013. All rights reserved. Page 6 of 14


Financial Accounting - Professional Stage June 2013

Question 2

Overall marks for this question can be analysed as follows: Total: 19

General comments
This question tested the preparation of a consolidated statement of cash flows and supporting note, where a
subsidiary had been disposed of during the year. Missing figures to be calculated included dividends paid
(to the group and to the non-controlling interest), dividends received, tax paid, additions to property, plant
and equipment, and proceeds from the issue of share capital.

Eagle plc

Consolidated statement of cash flows for the year ended 31 December 2012

Cash flows from operating activities
Cash generated from operations (Note) 495,850
Interest paid (W1) (20,000)
Income tax paid (W2) (81,200)
Net cash from operating activities 394,650
Cash flows from investing activities
Purchase of property, plant and equipment (W3) (460,200)
Proceeds from sale of property, plant and equipment 60,000
Dividends received from associate (W4) 50,600
Disposal of Owl Ltd net of cash disposed of ((194,450 x
80%) + 10,500) 1,500) 164,560
Net cash used in investing activities (185,040)
Cash flows from financing activities
Proceeds from share issues (220,000 + 50,000) 140,000 110,000
+ 20,000))
Repayment of long-term loan (150,000 125,000) (25,000)
Dividends paid (W5) (266,200)
Dividends paid to non-controlling interest (W6) (22,410)
Net cash used in financing activities (203,610)
Net increase in cash and cash equivalents 6,000
Cash and cash equivalents at beginning of period 14,500
Cash and cash equivalents at end of period 20,500

Note: Reconciliation of profit before tax to cash generated from operations



Profit before tax (324,100 + 41,400) 365,500
Share of profits of associate (56,700)
Finance cost 22,000
Profit on disposal of property, plant and equipment (60,000 56,000) (4,000)
Depreciation charge 175,600
Increase in trade and other receivables ((75,700 + 13,900) 88,900) (700)
Decrease in trade and other payables ((52,800 3,000) (40,500 + 8,450 5,000)) (5,850)
Cash generated from operations 495,850

Copyright ICAEW 2013. All rights reserved. Page 7 of 14


Financial Accounting - Professional Stage June 2013

Workings

(1) Interest paid


Cash () 20,000 B/d 3,000
C/d 5,000 CIS 22,000
25,000 25,000

(2) Income tax


Cash () 81,200 B/d 78,000
C/d 68,000 CIS (64,800 + 6,400) 71,200
149,200 149,200

(3) Property, plant and equipment


B/d 983,500 Disposal of sub 187,500
Other disposals 56,000
Additions () 460,200 Depreciation charge 175,600
C/d 1,024,600
1,443,700 1,443,700

(4) Investment in associate


B/d 179,800 Cash received () 50,600
CIS 56,700 C/d 185,900
236,500 236,500

(5) Retained earnings


Dividends in SCE () 266,200 B/d 675,100
C/d 663,000 CIS 254,100
929,200 929,200

(6) Non-controlling interest


Cash () 22,410 B/d 150,800
Disposal (194,450 x 20%) 38,890
C/d 140,200 CIS 50,700
201,500 201,500

Copyright ICAEW 2013. All rights reserved. Page 8 of 14


Financial Accounting - Professional Stage June 2013

Most candidates produced a well-presented statement of cash flows with all the relevant headings, although
the headings that continue to let candidates down are property, plant and equipment which was all too often
shortened to PPE and Dividends paid to non-controlling interest abbreviated to use NCI. Such
abbreviations, in one of the main financial statements, will cause candidates to lose presentation marks. As
usual, many candidates lost marks for use of the incorrect bracket convention and/or including cash flows
under the wrong headings. Typically for a question featuring a consolidated statement of cash flows,
candidates lost most marks on the groups aspect of the question, in particular failing to adjust accurately for
the disposal of the subsidiary and miscalculating the figure for the actual disposal as it should appear in the
statement.

In the main, workings took the form of T accounts, with very few candidates completing some of their T
accounts with all the entries the wrong way round. However, some candidates still insist on producing tabular
workings or workings on the face of the statement of cash flows. This can make it more difficult to see
evidence of correct double entry and to award marks where the final figure is incorrect (or uses the incorrect
bracket convention). Pleasingly, hardly any candidates produced no workings at all an even riskier approach
as if figures are calculated incorrectly it is not possible to award any partial marks.

The reconciliation note was generally well done with candidates making a good attempt at the adjustments.
However, most candidates failed to add the disposed of subsidiarys profit before tax to the continuing profit
before tax, to form the first figure of the reconciliation note. Most candidates correctly made the adjustments for
the associate, the finance costs and the depreciation charge. The profit on disposal of property, plant and
equipment was often ignored or added instead of being deducted. It was less common to see the correct
adjustments for the movements in trade receivables and payables, with most errors being made over the
adjustment for the disposed of subsidiarys figures.

Figures for income tax paid, interest paid, repayment of the long-term loan and purchase of property, plant and
equipment were those most commonly calculated correctly and shown correctly on the face of the statement
although in a minority of cases a positive figure was shown. However, a significant number of candidates failed
to adjust, or adjusted incorrectly for, the tax on the discontinued operations in the income tax T-account.
Others omitted to include all three necessary credit entries in the property, plant and equipment T-account.

Many candidates also correctly calculated the figure for dividends received from the associate and dividends
paid to the parent, although a number misclassified the former as a financing cash flow and others included
total profit for the year in retained earnings instead of including only the group share. It was less common to
see the correct figure for dividends paid to the non-controlling interest as, once again, many candidates failed
to adjust or adjusted incorrectly for the subsidiary disposed of during the period. It was also rare to see the
correct figure for the disposal of the subsidiary, with many miscalculating the sale proceeds (showing a lack of
understanding as to how the profit on disposal is calculated), although most clearly knew to deduct the cash
disposed of from their figure (although a minority deducted this from the closing cash and cash equivalents
figure in the statement of cash flows).

Only a minority of candidates correctly calculated the proceeds from the share issues, as many failed to
appreciate that the bonus issue (which was made out of the share premium account) was effectively a contra
entry between the share capital and share premium accounts. Hence all that was needed was a comparison of
the opening and closing figures in a combined T-account to arrive at the cash proceeds.
Total possible marks 19
Maximum full marks 19

Copyright ICAEW 2013. All rights reserved. Page 9 of 14


Financial Accounting - Professional Stage June 2013

Question 3

Overall marks for this question can be analysed as follows: Total: 22

General comments
This question featured a group of companies, comprising parent, two subsidiaries (one acquired during the
year) and one associate. Part (a) required the calculation of the goodwill arising on acquisition of the new
subsidiary, with fair value adjustments to be made. Part (b) required the preparation of the consolidated
income statement. Adjustments included intra-group transactions of both inventory and property, plant and
equipment, and impairment write-downs.

Kite plc

(a) Goodwill arising in the business combination with Vulture Ltd



Consideration
Cash 15,000
Shares at market value (300,000 x 1.30) 390,000
405,000
Net assets at acquisition
Share capital 100,000
Retained earnings at 1 January 2012 567,000
Profit to 31 March 2012 (49,200 x 3/12) 12,300
Less: Dividends paid (120,000)
Fair value adjustments:
Re building (W) 44,000
Goodwill to be written off (33,000 (3,600 x 3/12)) (32,100)
Contingent liability (20,000)
(551,200)
Non-controlling interest at acquisition x 30% 165,360
19,160
Working fair value adjustment re building

Fair value at 1 April 2012 154,000
Carrying amount at 1 April 2012 (250,000 ((250,000 25) x 14)) (110,000)
44,000

Almost all candidates used the correct methodology for calculating goodwill, although some lost marks for
not showing the % used to calculate the non-controlling interest share of the net assets at acquisition. It is
not sufficient to put x NCI%.

In calculating the consideration almost all candidates included the cash of 15,000 and the shares at market
value. However, many then also included the professional fees that should have been written off to
expenses. Others deducted this figure, failing to appreciate that whilst the company had incorrectly included
this figure in the cost of investment (which was not given in the question), they themselves had not, having
correctly added the cash and the shares. It was therefore rare to see this amount written off to operating
expenses in Part (b).

In calculating the net assets at acquisition, almost all candidates added share capital, retained earnings
brought forwards and three months of the profit for the year. Fewer deducted the dividends paid and fewer
still made the correct adjustment for goodwill (although a good number adjusted by the gross figure of
33,000). Where an adjustment was made for the contingent liability, most did use the correct figure of
20,000.

A good number of candidates correctly calculated the fair value adjustment in respect of the building,
although a significant minority calculated this in Part (b) and did not adjust for it in Part (a). Some credit was
given for this.
Total possible marks 6
Maximum full marks 6

Copyright ICAEW 2013. All rights reserved. Page 10 of 14


Financial Accounting - Professional Stage June 2013

(b) Consolidated income statement for the year ended 31 December 2012

Revenue (W1) 2,818,550
Cost of sales (W1) (1,850,525)
Gross profit 968,025
Operating expenses (W1) (584,000)
Profit from operations 384,025
Share of profit of associate ((30,600 x 40%) 3,000 2,160 (W2)) 7,080
Profit before tax 391,105
Income tax expense (W1) (76,125)
Profit for the period 314,980

Profit attributable to
Owners of Kite plc () 275,500
Non-controlling interest (W3) 39,480
314,980

Workings

(1) Consolidation schedule

Kite plc Harrier Vulture Adj Consol


Ltd Ltd
9/12

Revenue 1,579,500 879,500 491,550 (132,000) 2,818,550


Cost of sales per Q (1,050,600) (598,700) (328,125) 132,000
PURP (W2) (13,200)
PPE PURP ((275,000 8,100 (1,850,525)
234,500) 5)
Op expenses per Q (345,600) (103,800) (117,300)
prof fees re acquisition (5,000)
additional deprec on (3,000)
building ((44,000 (OF from a)
11) x 9/12)
adj re GW w/o on acq 2,700
(3,600 x 9/12)
GW impairment (12,000) (584,000)
Tax (37,500) (29,400) (9,225) (76,125)
142,500 36,600

(2) PURPs
Harrier Ltd Buzzard Ltd
%
Sales 100 132,000 54,000
Cost of sales (80) (105,600) (43,200)
GP 20 26,400 10,800
x 13,200 5,400
X 40% 2,160

(3) Non-controlling interest in year



Harrier Ltd (20% x 142,500 (W1)) 28,500
Vulture Ltd (30% x 36,600 (W1)) 10,980
39,480

Copyright ICAEW 2013. All rights reserved. Page 11 of 14


Financial Accounting - Professional Stage June 2013

The consolidation schedule was generally prepared correctly with almost all candidates appreciating that
only nine-twelfths of the acquired subsidiarys results should be included. Candidates who produced a
consolidation schedule gained the majority of the more straightforward marks available and then usually
went on to prepare a reasonable consolidated income statement which gained a good number of the
presentation marks available. The most common loss of presentation marks was for abbreviating non-
controlling interest to NCI. Very few candidates did their consolidation workings on the face of the group
income statement, which was pleasing.

Most candidates correctly reduced group revenue and cost of sales by the sales made between the parent
and the subsidiary, but some also made the same adjustment with the sales between the parent and the
associate.

The majority of candidates correctly calculated the two provisions for unrealised profit on intra-group sales,
although some then forgot to reduce the group share of the associates profit by only the group share of the
associates provision for unrealised profit. The non-controlling interest was correctly calculated by the
majority of candidates although a small number used the group percentage holding rather than the non-
controlling interest percentage. Others omitted to state what percentage they were using in this calculation
(or what figure they were multiplying this percentage by) and so lost marks.

Most candidates made a reasonable attempt at the associate calculation, with the most common error being
a failure to adjust for the unrealised profit (or forgetting to adjust it for the associate percentage). A few
candidates omitted to reduce the group share of the associates profit by the impairment loss in respect of
the associate, with some charging this instead against the parents profits.

The most common mistakes were made in the calculation of the adjustments in the consolidation schedule.
The provision for unrealised profit in respect of the subsidiarys sales was generally included correctly along
with the goodwill impairment, although a significant number of candidates also deducted the impairment
loss in respect of the associate. A minority of candidates included the goodwill impairment in the
subsidiarys column instead of in the parents. The intra-group sale of a machine was less well dealt with,
with only a minority of candidates getting this completely correct. A significant number of candidates
calculated the correct unrealised profit figure but then went on to add it to cost of sales rather than
deducting it. It was also common to see this figure in the parents column rather than the subsidiarys (ie
sellers) column. A significant number of candidates failed to appreciate that this transfer had taken place in
the previous year and so there was no need to make an adjustment for the original profit on transfer, but just
for the difference in subsequent depreciation.

The additional depreciation on the fair value adjustment calculated in Part (a) also produced a number of
different answers with only a minority of candidates gaining all the marks for the calculation and for dealing
with the adjustment correctly. Common mistakes were to either not take nine-twelfths of the of one years
worth of the adjustment or to use the incorrect adjustment in the first place. The most common error for the
write-off of the goodwill on the unincorporated business was to not adjust by nine-twelfths and/or to add the
resultant figure to operating expenses rather than deducting it.
Total possible marks 16
Maximum full marks 16

Copyright ICAEW 2013. All rights reserved. Page 12 of 14


Financial Accounting - Professional Stage June 2013

Question 4

Overall marks for this question can be analysed as follows: Total: 9

General comments
This question required the preparation of a statement of changes in equity for a single entity. Opening
balances were provided together with a series of events which occurred during the current year. Matters to
be dealt with included the issue of ordinary shares, the payment of ordinary and redeemable preference
dividends, a prior period error, a change from the cost model to the revaluation model (with resultant
reserve transfer) and a change of depreciation method. Some of the matters also impacted on the draft
profit for the period.

Hawk Ltd

Statement of changes in equity for the year ended 31 December 2012

Ordinary Share premium Retained earnings Revaluation


share surplus
capital


At 1 January 2012 500,000 125,000 489,700

Correction of error (100,000)


Restated balance 500,000 125,000 389,700

Issue of ordinary shares 100,000 50,000


Total comprehensive income 79,300 400,000
for the year (Ws 1 and 2)
Dividend on ordinary shares (120,000)
(600,000 x 0.20)
Transfer to retained earnings 20,000 (20,000)
(W2)

At 31 December 2012 600,000 175,000 369,000 380,000

Workings

(1) Revised profit for the year



Draft profit 137,800
Less: Dividend on redeemable preference shares (200,000 x 3%) (6,000)
Depreciation on property (3,000,000 20) (150,000)
Adj to depreciation re special plant (W3) (2,500)
Add: Error re opening inventory 100,000
79,300

Copyright ICAEW 2013. All rights reserved. Page 13 of 14


Financial Accounting - Professional Stage June 2013

(2) Revaluation and depreciation transfer



Valuation on 1 January 2012 3,000,000
Carrying amount of property on 1 January 2012 (3,370,000 770,000) 2,600,000
400,000
Annual transfer 20 20,000

(3) Depreciation adjustment re special plant



Depreciation charged in 2012 (30,000 x 25%) 7,500
Depreciation on new basis (30,000 3) 10,000
Additional charge needed 2,500

Most candidates made some kind of attempt at this question although full presentation marks were only
gained by a minority of candidates. The majority of candidates correctly included the brought forward figures
for ordinary share capital and share premium. The brought forward figure for retained earnings was
sometimes adjusted by the correction of the error rather than showing this adjustment as a separate line on
the statement of changes in equity itself. Where the correction of the error was made on the statement it was
often added rather than deducted. Only a few candidates showed a restated balance after this adjustment
had been made.

The majority of candidates showed the correct entries for the share issue in the period, and for the dividend
payment made. Candidates generally included the profit figure in the statement although only a minority
correctly identified this as being part of total comprehensive income. The adjustments to profit were not
generally well dealt with, with probably only around half of candidates making some of the adjustments. Of
those candidates that did attempt to make adjustments to profit the most common errors were to make the
adjustments in the wrong direction (ie added rather than deducted or vice versa). Some candidates made
their adjustments on the face of the statement of changes in equity, instead of in a separate working. Only a
small minority of candidates included the redeemable preference shares in the statement of changes in
equity.

The revaluation surplus arising in the year was generally calculated correctly, although this was almost
always presented on a separate line to total comprehensive income usually being described as a
revaluation, highlighting a lack of understanding in this area. Most candidates who arrived at the correct
revaluation figure also arrived at the correct transfer between the revaluation surplus and retained earnings,
reflecting the excess depreciation for the year.

Only a minority of candidates correctly calculated the depreciation adjustment for the special plant and went
on to adjust profit correctly for it. A number of candidates calculated one or other of the old and new
depreciation figures but then often failed to make the resultant adjustment in the correct direction.
Total possible marks 10
Maximum full marks 9

Copyright ICAEW 2013. All rights reserved. Page 14 of 14


Financial Accounting and Reporting Professional Level September 2013

MARK PLAN AND EXAMINERS COMMENTARY

The mark plan set out below was that used to mark these questions. Markers are encouraged to use discretion
and to award partial marks where a point was either not explained fully or made by implication. More marks are
available than could be awarded for each requirement, where indicated. This allows credit to be given for a
variety of valid points, which are made by candidates.

Question 1

Overall marks for this question can be analysed as follows: Total: 31

General comments
This question was a question testing the preparation of an income statement and statement of financial
position from a trial balance. A number of adjustments were required to be made, including a downward
revaluation, share issues, research and development expenditure, foreign exchange, the incorrect treatment of
a lease and the clearing of a suspense account.
Part b) asked for the UK GAAP differences in relation to the treatment of the revaluation model.
Part c) featured the concepts requirement which asked about the two fundamental qualitative characteristics
and how they are applied to intangible assets.

Temera Ltd Statement of financial position as at 31 March 2013



ASSETS
Non-current assets
Property, plant and equipment (350,000 + 384,000 (W6)) 734,000
Intangible assets (63,250 + 6,540)(W3 & W4) 69,790
803,790

Current assets
Inventories (W1) 40,400
Trade and other receivables 17,800
Cash and cash equivalents 6,900
65,100
Total assets 868,890

Equity
Ordinary share capital (W2) 400,000
Revaluation surplus (100,000 50,000 (W6)) 50,000
Retained earnings (W7) 321,140
Equity 771,140

Current liabilities
Trade and other payables (30,450 200 (W7)) 30,250
Taxation (56,000 + 11,500) 67,500
97,750

Total equity and liabilities 868,890

Copyright ICAEW 2013. All rights reserved Page 1 of 21


Financial Accounting and Reporting Professional Level September 2013

Temera Ltd Income statement for the year ended 31 March 2013

Revenue 912,500
Cost of sales (W1) (372,550)

Gross profit 539,950


Administrative expenses (W1) (197,300)
Other operating costs (W1) (92,310)
Property impairment (W6) (57,500)

Profit before tax 192,840


Income tax (67,500)

Profit for the period 125,340

W1 Expenses
Cost of Admin Other
sales expenses operating
costs

Trial balance 381,250 181,300 34,500
Opening inventories 31,700
Closing inventories (36,200 + 4,200(W7)) (40,400)
R&D expenditure (W3) 46,000
R&D amortisation (W3) 5,750
Patent amortisation (W4) 2,760
Profit on sale of patent (6,500)
Depreciation charge property (W6) 16,000
Exchange difference adj (W7) (200)
Building lease 10,000
372,550 197,300 92,310

Copyright ICAEW 2013. All rights reserved Page 2 of 21


Financial Accounting and Reporting Professional Level September 2013

W2 Share capital & premium Share Share


capital premium

Trial balance 323,000 67,500
Share issue adjustment (10,000 x 30p) (3,000) 3,000
320,000 70,500
Bonus issue (320,000 / 4) 80,000 (70,500)
At 31 March 2013 400,000

W3 Research & development expenditure



Trial balance 115,000
Less amounts charged to profit & loss
Prior to 1 July 2012 28,000
Staff training 8,000
Promotional spend 10,000
(46,000)
Intangible asset at 30 November 2012 69,000
Amortisation (69,000 / 4yrs x 4/12) (5,750)
63,250

W4 Patents

Cost b/fwd 15,000
Disposed in year (2,400)
12,600

Accumulated amortisation b/fwd (4,500)

Amortisation charge for year


On patents held all year (12,600 / 5yrs) 2,520
On patent disposed of (2,400 / 5yrs x 6/12) 240
(2,760)
Acc. amortisation on patent disposed of
(2,400 / 5yrs x 30/12) 1,200
Carrying amount at 31 March 2013 6,540

W5 Suspense account

Trial balance 67,300
Disposed of patent (6,500 + (2,400 1,200)) 7,700
Reverse lease liability (70,250)
Reverse lease finance charge 5,250
Reverse lease payment (10,000)
At 31 March 2013

W6 Property, plant and equipment Buildings



Land & buildings
b/fwd 630,000
Accumulated depreciation b/fwd (122,500)
507,500
Valuation at 1 April 2012 400,000
Revaluation (107,500)
Balance on revaluation surplus re buildings (100,000 / 2) 50,000
Excess to profit or loss 57,500

Copyright ICAEW 2013. All rights reserved Page 3 of 21


Financial Accounting and Reporting Professional Level September 2013


Valuation at 1 April 2012 400,000
Depreciation charge for year (400,000 / 25yrs) (16,000)
384,000

W7 Foreign exchange
Translation at 1 January 2013 (5,000 x 0.84) 4,200
Translation at 31 March 2013 (5,000 x 0.80) (4,000)
200
W8 Retained earnings

Trial balance 205,300
Bonus issue (80,000 70,500) (W2) (9,500)
Profit for the year 125,340
321,140

Most candidates made a reasonable attempt at this part of the question, with the vast majority preparing a
complete statement of financial position and income statement. Presentation was reasonable, although a
significant number of candidates lost marks by failing to include appropriate sub-totals on their statement of
financial position, which is a common omission in the Financial Accounting paper also.

Depreciation on the revalued building was generally correctly calculated by most candidates as was the
adjustment to closing inventory, although this wasnt always correctly carried through to both the income
statement and statement of financial position.

The allocation of costs spent on research and development was generally attempted by most candidates,
although a common error was to assume that the staff training could be capitalised. The number of months
that amortisation on the capitalised development expenditure should have been charged seemed to cause a
significant number of candidates a problem.

Almost all candidates did use the recommended costs matrix when allocating costs between the three
expense categories and, on the whole, it was possible to match figures on the face of the financial statements
to workings. It was very noticeable that those candidates who did not use this format tended to produce
disorganised workings (often split between the face of the income statement and /or a number of separate
workings) which were difficult to follow and therefore might have lost marks. Far fewer candidates seemed
capable of producing a clear working for intangible assets which resulted in them repeating calculations. A
significant number of candidates would produce a working but then do nothing with the figures, or only carry
them through to one side of the double entry, for example show in the cost matrix, but not include them in the
asset carrying amount.

Very many candidates dealt correctly with the share issues during the year. The most common error made
was to credit share premium and debit share capital with the nominal value of the cash issue of shares, as
opposed to the premium. Most candidates then reduced their share premium account to zero and charged
the remainder of their (own figure) bonus issue to retained earnings and provided a clear working for retained
earnings carried forward, tying in to their statement of financial position figure.

Most candidates correctly calculated that the buildings had suffered a downwards revaluation of 107,500
(although a few omitted to take into account opening accumulated depreciation in their calculations). Many
then correctly charged only 50,000 of that to the revaluation surplus (recognising that the other 50,000 in
the revaluation surplus related to land), but others used the whole revaluation surplus. Almost all candidates
then charged the balance to the income statement.

Other common errors included the following.

Incorrect calculation of the carrying amount of the patent by failing to calculate correctly, or omitting
completely, the accumulated amortisation of the disposed asset for the 2.5 years held.

Failing to calculate the profit on sale of the patent, or when it was calculated either forgetting to include
it in the statement of comprehensive income or including it in the wrong place (for example as part of
retained earnings).

Copyright ICAEW 2013. All rights reserved Page 4 of 21


Financial Accounting and Reporting Professional Level September 2013

Failing to include the additional 11,500 tax charge within the statement of comprehensive income.

Continuing to treat the lease as a finance lease, instead of as an operating lease (even though in
Question 3 almost all candidates recognised that where the length of the lease is significantly less than
the useful life of an asset the lease will be an operating lease). It was therefore very common to see a
finance charge of 5,250 on the face of the income statement and a lease liability on the statement of
financial position.

Correctly calculating a foreign exchange gain of 200, but valuing closing inventory in relation to this at
closing rate instead of at historic rate.

Despite the specific requirement to provide a working showing how the suspense account had been cleared,
this appeared only rarely, with many candidates providing journal entries relating to this scattered throughout
their answer, which were not asked for.
Total possible marks 26
Maximum full marks 23

(b)
UK GAAP values PPE based on a current value model (existing use model), under IFRS fair value includes
current market value which is often higher than EUV as it considers alternative uses.
FRS 15 under UK GAAP specifies that the maximum period between valuations should be five years and
interim valuations should be three year. IFRS does not specify a maximum period.
UK GAAP requires any consumptions of economic benefit to be recognised straight in the income statement
rather than against a previous revaluation surplus. It is not clear from the scenario whether the downward
valuation of Temera Ltds land and buildings are caused by a consumption of economic benefits. However, if
this is the conclusion then the full 107,500 would be recognised in profit or loss for the period. IFRS has no
such restriction.
This was a very straightforward requirement but very few candidates gained a reasonable mark in it.
Answers to this were disappointing, indicating that many candidates had not committed these differences to
memory. Those who had learnt these differences scored well easily picking up two or even the maximum of
three marks.

Worryingly, some candidates thought that impairments were never charged to the revaluation surplus under
UK GAAP and a number of candidates had very confused ideas about when reserve transfers are
allowed/required for revalued assets. Others thought that assets are not reviewed for impairment under UK
GAAP. Some candidates also wasted time by discussing differences that did not specifically relate to
revalued assets.
Total possible marks 4
Maximum full marks 3

Copyright ICAEW 2013. All rights reserved Page 5 of 21


Financial Accounting and Reporting Professional Level September 2013

(c) Qualitative characteristics


The choice of the revaluation model as a measurement model in IAS 38 Intangible Assets provides relevant
information by showing up to date values. This will assist users understanding of what the entitys underlying
assets are actually worth.
However, to assist with comparability if the revaluation model is applied, all valuations must take place at the
same time for a class of intangible assets. However, as not all intangible assets can be revalued because
there is no active market for the asset it is not always possible to apply the revaluation model to the entire
class of assets and therefore comparability may be compromised.
Although the revaluation model provides more relevant information to users this information is generally seen
as less faithful than the cost model. The cost model is based on historical costs which are not the most
relevant costs on which to base future decisions. However, historic cost is based on fact and is therefore a
faithful representation.
The strict recognition criteria in IAS 38 sets out what can be included as part of the cost of an intangible asset
and this aids verifiability of the final figure.

IAS 38 contains rigid and robust rules for the capitalisation of intangible assets which means that financial
statements of different companies can be compared as they are prepared on the same basis.

IAS 38 also facilitates comparability between companies by requiring disclosure of accounting policies in
respect of, for example, amortisation policy and measurement bases. It also requires the disclosure of both
brought forward and carried forward figures aiding comparability between consecutive years.

IAS 38 allows comparability between the cost and revaluation model, to ensure that companies financial
information can be compared no matter which measurement basis is applied. This comparability is achieved
by requiring equivalent cost information to be disclosed under the revaluation model.

To improve understandability IAS 38 requires disclosures to be provided by each class of intangible asset.
This provides information on what types of intangible assets have been purchased or sold during the year.

The table format which is required by IAS 38 also assists users understandability by showing movements
during the year.
This part of the question was particularly badly answered by almost all candidates. Too many candidates
simply wrote out what they knew, or had looked up in the Open Book text, about relevance and faithful
representation without any link to the treatment of intangible assets. Others just wrote about how to account
for intangible assets, with no link provided between that and the qualitative characteristics.

The majority of candidates struggled to set out more than one or two relevant points and therefore very few
candidates even gained half marks in this part of the question.
Total possible marks 7
Maximum full marks 5

Copyright ICAEW 2013. All rights reserved Page 6 of 21


Financial Accounting and Reporting Professional Level September 2013

Question 2

Overall marks for this question can be analysed as follows: Total: 10

General comments
This question required the preparation of a statement of cash flows from a draft statement. Movements in
relation to property, plant and equipment involving a non-cash asset and impairment, both a share and bonus
issue had been made, along with a dividend payment.

(a) Radazul plc

Statement of cash flows for the year ended 31 March 2013



Net cash from operating activities (11,935)
Cash flows from investing activities
Purchase of property, plant and equipment (W2) (333,615)
Proceeds from disposal of property, plant and equipment 46,000
Net cash used in investing activities (287,615)
Cash flows from financing activities
Proceeds from issue of ordinary share capital (165,000 x 2) 330,000
Dividends paid (W4) (39,750)
Net cash from financing activities 290,250
Net decrease in cash and cash equivalents (9,300)
Cash and cash equivalents at beginning of period (24,700)
Cash and cash equivalents at end of period (34,000)

Workings

(1) Cash from operations



Draft cash generated from operations (42,235)
Equipment impairment (9,200 4,700) 4,500
Adjust proceeds on disposal of machinery by carrying amount (to give profit) 38,700
Adjustment for trade and other payables ( 4,900 8,000) (12,900)
Cash generated from operations (11,935)

Deduct for any incorrect bracket convention

Copyright ICAEW 2013. All rights reserved Page 7 of 21


Financial Accounting and Reporting Professional Level September 2013

(2) PPE

B/d 621,500 Disposal 38,700
Additions non-cash 8,000 Impairment 4,500
Additions () 333,615 Depreciation 111,475
C/d 808,440
963,115 963,115
(3) Share capital and premium

B/d 120,000
Non-cash issue () 57,000
C/d (342,000 + 165,000) 507,000 Cash issue (165,000 x 2) 330,000
507,000 507,000
(4) Retained earnings

Dividends paid () 39,750 Net movement 96,750
Non-cash issue (W3) 57,000
96,750 96,750

There were some very good attempts at this question, with a significant number of candidates using the
correct figures for proceeds from disposals of property, plant and equipment and proceeds from share issue,
and correctly calculating cash paid for the purchase of property, plant and equipment and dividends paid
(usually via T-accounts). Where marks were lost on the better scripts they were generally lost on presentation
(with some candidates producing only extracts, not a full statement of cash flows) or on the calculation of net
cash from operating activities. A few candidates made no attempt to calculate a correct figure for dividends
paid and left the original figure in their statement of cash flows.

The area that most candidates struggled with was the adjustments needed to cash from operations where,
although most candidates did adjust for the impairment, far fewer dealt correctly with the disposal and the
adjustments relating to the closing interest accrual (which many dealt with by showing interest paid on the face
of the statement of cash flows). Many either ignored the adjustment relating to the plant purchased on credit or
made the adjustment the wrong way around.

A number of candidates lost easy marks by using abbreviations on the face of the statement of cash flows
(most commonly PPE and NCI) and by using the wrong bracket convention particularly when dealing with
outflows of cash. Some also showed the right figure for the shares issued for cash in their working but then
failed to include this figure on the face of the statement. A number of candidates failed to use brackets for
opening and closing cash and cash equivalents.
Total possible marks 9
Maximum full marks 8

Copyright ICAEW 2013. All rights reserved Page 8 of 21


Financial Accounting and Reporting Professional Level September 2013

(b)
UK GAAP (FRS 1) has a narrower definition of cash compared with the IFRS term of cash and cash
equivalents. Instead UK GAAP deals with cash equivalents as part of management of liquid resources.

UK GAAP requires the statement of cash flows to be prepared under nine headings compared with the
three under IFRS.

UK GAAP is more restrictive on where certain items should be reported as compared with IFRS. For
example, interest paid should be reported under returns on investments and servicing of finance. IFRS
permits interest paid to be reported under any of the three main headings.
Answers to this question on UK GAAP differences were much better than those to Part (b) of Question 1,
indicating that candidates had learnt these differences. Most candidates knew that an IFRS statement of
cash flows used only three headings, with UK GAAP having nine, and that there is more flexibility under
IFRS. Hence many candidates scored a maximum of two marks. A worrying minority of candidates thought
that cash and cash equivalents were simply different terms for the same thing not that cash is a
narrower definition.
Total possible marks 3
Maximum full marks 2

Copyright ICAEW 2013. All rights reserved Page 9 of 21


Financial Accounting and Reporting Professional Level September 2013

Question 3

Overall marks for this question can be analysed as follows: Total: 27

General comments
Part (a) of this question required candidates to explain the financial reporting treatment of four accounting
issues, given in the scenario. The four issues covered a convertible bond, a jointly controlled entity, a
lease incentive and a held for sale asset.
Part (b) required candidates to revise two figures extracted from the draft consolidated financial
statements for the adjustments needed as a result of their answer to Part (a).
Part (c) required an explanation of any ethical issues arising from the scenario.

Centellas plc
(a) IFRS accounting treatment

(1) Convertible bonds

The convertible bonds are a compound financial instruments per IAS 32, Financial Instruments:
Presentation and have both an equity and a liability component which should be presented separately at
the time of issue. IAS 32 requires that the substance of the transaction be reflected, focusing on the
economic reality that in effect two financial instruments have been issued.

The liability component should be measured first at the present value of the capital and interest payments.
The discount rate used should be the effective rate for an instrument with the same terms and conditions
except for the ability to convert to shares.

31 March Cash flow DF PV


@ 9%
2013 180,000 1/1.09 165,137
2014 180,000 1/1.09 151,502
2015 3,180,000 1/1.09 2,455,543
Liability component 2,772,182
Equity component (Bal fig) 227,818
Total 3,000,000

The liability should be measured at 2,772,182 and the equity component should be calculated as the
residual amount and measured at 227,818.

The equity element will remain unchanged.

The interest expense should be calculated at 9% of the liability component.

1 Apr 2012 Interest (9%) Payment (6%) 31 Mar 2013



2,772,182 249,496 (180,000) 2,841,678

An adjustment is required to increase the finance costs by 69,496 (249,496 180,000).

Copyright ICAEW 2013. All rights reserved Page 10 of 21


Financial Accounting and Reporting Professional Level September 2013

(2) Jointly controlled entity

Centellas plc and Bermeja Ltd each own 50% of the share capital of Vidrio Ltd which indicates that the
investment should be recognised as a jointly controlled entity and not as a subsidiary as Centellas plc
does not have control. Both companies hold an equal number of shares and there is contractual
agreement in place that they will make all the major operating and financial decisions concerning Vidrio
Ltd jointly.

IAS 31, Interest in Joint Ventures offers a choice of recognising the share in Vidrio Ltd using equity
accounting or proportionate consolidation, however Centellas plcs accounting policy choice is to apply the
equity method.

Under equity accounting Centellas plc will initially show its investment in Vidrio Ltd at cost of 100,000.
This will then be adjusted each period by Centellas plcs share of Vidrio Ltds retained earnings and
reduced by any impairments.

The investment in Vidrio Ltd is shown as a single line in both the consolidated statement of financial
position and the consolidated income statement.

As Vidrio Ltd was newly incorporated the cost of the investment was equal to the fair value and therefore
no goodwill arises at acquisition.

The investment in Vidrio Ltd should be shown at 220,000 ((200,000 + 240,000) x 50%) in the
consolidated statement of financial position and income from joint venture should be 120,000 (240,000 x
50%). As it has currently been treated as a subsidiary 100% of profits will have been included so
consolidated profit should be reduced by 120,000.
(3) Lease incentive

The new lease agreement runs for five years out of the buildings estimated 25-year life. Therefore, it can
be assumed that this is an operating, rather than a finance lease.

The initial rent-free period appears to constitute an incentive to enter into the agreement and therefore it
should be accounted for under SIC 15, Operating Leases Incentives.

The required treatment of the rent-free period by Centellas plc, the lessee, is to recognise the aggregate
benefit of the incentives as a reduction of rental expense over the lease term, on a straight-line basis.

The total amount payable under the lease agreement of 57,000 ((1,000 x 12 x 5yrs) (1,000 x 3))
should be spread evenly over the 5-year period: a charge of (57,000/60 months) = 950 per month
should be recognised.

Therefore, 9 months x 950 = 8,550 should be recognised as an expense in the year ended 31 March
2013.

The amount actually paid in the year was 6,000 (ie six payments of 1,000). Therefore, a payable for the
difference, 2,550, should be recognised in Centellas plcs current liabilities at
31 March 2013 and profit should be reduced by 2,550.

Copyright ICAEW 2013. All rights reserved Page 11 of 21


Financial Accounting and Reporting Professional Level September 2013

(4) Held for sale asset

When the carrying amount of a non-current asset will be recovered principally through sale, rather than its
continuing use, the asset must be classified as held for sale in accordance with IFRS 5, Non-Current
Assets Held for Sale and Discontinued Operations. This reclassification generally occurs at the time the
decision has been made to sell the asset.

For an asset to be classified as held for sale it must meet detailed criteria:

The asset must be available for immediate sale in its current condition; and
The sale must be highly probable that itll take place.

For a sale to be highly probable:

Management must be committed to a plan to sell the asset, management are advertising the sale
and therefore it can be assumed that it is planning to sell the asset.
There must be an active programme to find a buyer, here management are actively marketing the
asset by advertising in national trade magazines.
The asset must be marketed at a price that is reasonable based on its current fair value, here the
advertised price is 62,500 which is reasonable since its fair value is 62,000.
The sale should be expected to take place within one year of the date of classification.
It is unlikely that there will be significant changes to the plan or that the plan will be withdrawn.

As Centellas plc appears to meet the above criteria, it would appear that the machine does meet the held
for sale classification criteria.
The machine should therefore be measured at the lower of its carrying amount, being 64,750 (W) and its
fair value less costs to sell of 60,500 (62,000 1,500). However, as the machine is measured under
the valuation model it should be revalued at fair value under IAS 16, Property, Plant and Equipment
immediately before its classification to held for sale. Hence, 2,750 (W) should be recognised against the
revaluation surplus.
Once classified as held for sale the costs to sell should then be recognised as part of profit or loss for the
period, ie 1,500.
Once the asset is held for sale it is no longer depreciated and should be shown separately in the
statement of financial position.

Working
Cost/valuation Revaluation
surplus
Carrying amount at 31 Mar 2010 72,000

Revalued amount 31 Mar 2010 84,000 12,000

Acc dep (84,000/12yrs x 2yrs) (14,000)


31 March 2012 70,000

Dep in yr (84,000/12yrs x 9/12) (5,250)


64,750
Fair value (62,000) (2,750)
9,250
The answers to this part of the question were of a good standard with virtually all candidates including
narrative explanations as well as the relevant calculations.

Most candidates correctly identified the four underlying issues as a compound financial instrument, a
jointly controlled entity, an operating lease and a held for sale asset, and were able to explain how they
should be accounted for. Some marks (though not many) were lost on errors in the calculations but more
were lost where candidates, after an initial explanation, then reduced their answer to a series of journal
entries. Although there were specific marks allocated to key calculations and to the adjustments using
those figures in Part (b), there were no marks for journal entries in lieu of narrative explanations. Marks
are only ever awarded for journal entries where these are specifically required by the question.

Copyright ICAEW 2013. All rights reserved Page 12 of 21


Financial Accounting and Reporting Professional Level September 2013

The calculations for the convertible bonds, joint venture and operating lease were generally correct. With
regard to the asset held for sale most candidates calculated the carrying amount correctly but very few
realised that as the asset was carried at value a final revaluation should have been made prior to
calculating the impairment. Almost all candidates calculated a single impairment down to fair value less
costs to sell and charged that to the revaluation surplus.

Other errors made included the following:

Miscalculating the present value of the liability component of the convertible bond (a minority of
candidates only).

Not recognising that there are two possible methods for accounting for a jointly controlled entity.

Not recognising the rent-free period on the operating lease as an incentive per SIC 15.

Miscalculating the amount already recognised as an expense on the operating lease as nine
months at 1,000 per month, as opposed to six months at that amount (even where their answer
referred earlier to the three month rent-free period).

Charging depreciation on the held for sale asset for the wrong number of months.

Not stating the net effect of any corrections to be made so, for example, on the convertible bond,
many calculated that a total finance charge of 249,496 was needed but failed to state that
therefore an adjustment of 69,496 was required as 180,000 had already been recognised.
Total possible marks 29
Maximum full marks 20

(b)
Centellas plc

Profit before tax Equity


(before NCI)

As stated 690,000 1,260,400
(1) Equity element 227,818
(1) Finance cost adjustment (69,496)
(2) Share in JV (120,000)
(3) Lease incentive (2,550)
(4) Depreciation (5,250)
(4) Costs to sell (1,500)
(4) Revaluation surplus (2,750)
(78,796) (78,796)
TOTAL 491,204 1,406,672

Most candidates did attempt to make the relevant adjustments although a minority just calculated a
revised figure for profit and ignored equity. It appeared that most candidates had built up their answer to
this part alongside their answers to Part (a), which is by far the most efficient approach. A number of
errors were made because candidates failed to read the question to identify what entries had already been
made to the draft financial statements.
Total possible marks 4
Maximum full marks 3

Copyright ICAEW 2013. All rights reserved Page 13 of 21


Financial Accounting and Reporting Professional Level September 2013

(c) Ethical issues


The finance director, Anton Caro, qualified over 40 year ago, however as an ICAEW chartered accountant
he is still bound by the ICAEW Code of Ethics. ICAEW chartered accountants must always abide by the
spirit of the five fundamental ethical principles. One of these is professional competence and due care.

Anton is obliged to maintain his professional knowledge and skills at an appropriate level; as the finance
director of a listed company the appropriate level would be an in-depth knowledge of current financial
reporting standards. The fact that Anton is due to retire next year is not an excuse not to maintain his
professional knowledge and not act in accordance with the Code of Ethics that fact is irrelevant and
appropriate action should be taken by Anton to improve his level of technical knowledge. He should also
improve his knowledge of his obligations in respect of the ethical standards.

There are a number of errors in the draft financial statements, all of which reduce profit. This may be a
coincidence but it seems unlikely that this is the case and instead Anton is under pressure to deliver a
healthy profit, and has instead acted without integrity, as well as having a self-interest threat since his
bonus is linked to the reported profit.

You face a number of ethical issues, not least the question of whether the mistakes were deliberate or a
lack of knowledge on Antons part. There is also a potential self-interest threat as the post of finance
director has been mentioned if the right results are delivered.

You should ignore the possibility of self-interest and discuss the adjustments with Anton and remind him of
his professional responsibilities to ensure that accounting standards are correctly followed.

Amendments must be made to the consolidated financial statements and if Anton refuses to make them,
you should discuss the matter with the other board members.

If Anton continues to try to dominate and exert influence on you to misstate the consolidated financial
statements then it would be appropriate for you to consult the ICAEW ethical handbook and discuss the
matter with the ICAEW confidential helpline.
Most candidates picked up a good number of the available marks for this part, recognising the self-interest
threat to Anton and to themselves (in the role of newly qualified ICAEW Chartered Accountant), the
possible intimidation threat to themselves from Anton, and Antons possible lack of professional
competence and due care. However, a number of candidates failed to recognise that they were acting
within a company, as the financial accountant, and not as part of an audit team. It was therefore
inappropriate to suggest referring the matter to the ethics partner or to discuss approaching the audit with
increased professional scepticism. Most candidates did recognise the possible need to contact the
ICAEW confidential helpline if they were unable to resolve the issues via discussion with Anton or with the
other directors, but there was a tendency to be very quick to suggest that their own resignation might be
the best solution. A worrying small minority of candidates thought the issue was one of money laundering.
Total possible marks 8
Maximum full marks 4

Copyright ICAEW 2013. All rights reserved Page 14 of 21


Financial Accounting and Reporting Professional Level September 2013

Question 4

Overall marks for this question can be analysed as follows: Total: 14

General comments
This question mixed several discrete topics. Revised extracts from the consolidated financial statements
were required to be prepared following adjustments for the acquisition of a subsidiary, with contingent
consideration, an associate and revenue recognition issues. The associate included trading with the
parent company. Revenue adjustments were required for contract revenue and a sale and repurchase
agreement.
Part b) required candidates to distinguish between the single entity concept and the parents relationship
with an associated company.

(a) Gumar Ltd


(i)
Consolidated statement of financial position at 31 March 2013 (extract)

Non-current assets
Property, plant and equipment (987,500 + 210,000 + 350,000) 1,547,500
Goodwill (27,800 + 54,500(W1)) 82,300
Investment in associate (W3) 114,340

Current assets
Inventories (62,900 510 (W5)) 62,390
Trade and other receivables (161,300 + 28,200 + 20,000) 209,500

Non-current liabilities
Borrowings (200,000 + 500,000) 700,000
Contingent consideration (85,000 x 1.04) 88,400
Contingent liability 32,000

Current liabilities
Trade and other payable (75,000 + 12,200) 87,200

Workings
(1) Goodwill

Consideration:
Cash 100,000
Contingent consideration at fair value 85,000
185,000
Non-controlling interest at fair value 35,000
220,000
Less: Fair value of net assets (W2) (165,500)
Goodwill 54,500

Copyright ICAEW 2013. All rights reserved Page 15 of 21


Financial Accounting and Reporting Professional Level September 2013

(2) Net assets of Abrigo Ltd


31 March 2013 At Post
acquisition acquisition

Share capital 165,000 165,000
Retained earnings 96,000 72,500
Goodwill adjustment (35,000) (40,000)
Contingent liability (32,000) (32,000)
194,000 165,500 28,500

(3) Investment in associate Caleta Ltd



Cost of investment 105,000
Share of post acquisition retained earnings ((63,400 25,600) x 30%) 11,340
Less: impairments to date (2,000)
114,340

(4) Share of profit of associate Caleta Ltd



Share of profit for the year (45,200 x 30%) 13,560
Less: PURP (W5) (510)
Less: impairment in year (2,000)
11,050
(5) PURP
%
SP 100 17,000
Cost (80) (13,600)
GP 20 3,400
1
X /2 1,700
Caleta Ltd 1,700 x 30% = 510

(a) (ii) Revised figure for consolidated profit



Draft profit before tax 589,200
Abrigo Ltd (W2) 28,500
Unwinding of discount contingent consideration (88,400 85,000) (3,400)
Share of associate (W4) 11,050
Reverse profit (150,000)
Contract revenue 20,000
495,350

Most candidates dealt well with the groups aspect of this question, at least as far as the standard workings
were needed. Almost all candidates produced net assets and goodwill workings for the subsidiary acquired
during the year and an investment in associate working (calculating the figure for the consolidated
statement of financial position) for the associate. However, many then also felt the need to produce
workings for retained earnings and for non-controlling interest when these were not needed. Conversely,
many failed to calculate the share of profit of the associate for the consolidated income statement a figure
that was needed in order to revise consolidated profit.

Common errors in these calculations included the following:

Discounting the contingent consideration further even though it was stated at fair value in the
question.
Calculating the non-controlling interest at share of net assets at acquisition, instead of at fair value
per the groups stated accounting policy.
Dealing incorrectly with the subsidiarys own acquired goodwill in the net assets table (often getting
the year end and acquisition figures the wrong way round), or failing to adjust for this at all.
Reducing the investment in associate figure by the provision for unrealised profit.
Not taking only the group share of the unrealised profit with the associate to the share of profit of the
associate (or directly to the adjusted consolidated profit calculation).

Copyright ICAEW 2013. All rights reserved Page 16 of 21


Financial Accounting and Reporting Professional Level September 2013

Most candidates therefore included in their corrected extracts from the consolidated statement of financial
position figures for goodwill (with most correctly excluding goodwill from the subsidiarys own statement of
financial position) and investment in associate. However, a significant number failed to add in the figures for
property, plant and equipment, trade and other receivables and trade and other payables for the subsidiary
acquired during the year, not appreciating that these figures had not been included in the draft consolidated
statement of financial position. Some who did add these in failed to show their workings for this, so if
calculation errors were made it was not possible to give credit for doing this. Others failed to reduce group
inventories by the group share of the unrealised profit with the associate, even when this had been
calculated. Others reduced inventories by 100% of this adjustment, as opposed to the group share. It was
also rare to see a figure for contingent consideration taken to the consolidated statement of financial
position, even where this had been included at fair value in the goodwill calculation.

With regard to the other adjustments (Items (3) and (4)) most candidates wrote how they would account for
these issues (as if this had been an explain type of question), but few actually put these adjustments
through to their consolidated statement of financial position, although a few more did revise consolidated
profit before tax for these. Where the requirement is to prepare or to calculate candidates need to be
aware that no marks are available for narrative explanations.

Most candidates attempted to revise consolidated profit before tax but approaches to this were sometimes
haphazard. The most efficient approach was to complete this part of the question (Part (ii)) alongside Part
(i). The most common error therefore was to fail to make a corresponding adjustment in Part (ii) for every
item dealt with in Part (i). In particular, many candidates failed to adjust consolidated profit before tax for the
subsidiarys profit for the year, even though almost all candidates had calculated this in a net assets table.
Total possible marks 14
Maximum full marks 12

(b)
An associated company is not part of a group, therefore the single entity concept that applies between a
parent and its subsidiaries does not extend to associated companies. This is because the parent entity only
has significant influence over an associated company rather than control.

Transactions between group companies and an associated company are not cancelled on consolidation as
an associated company is not part of the group. However any unrealised profit on these transactions
should be eliminated.
Answers to this part were very poor, with almost all candidates answering the question in relation to a
parent entity and its subsidiary, instead of in relation to a parent entity and its associate. Such answers
almost always scored zero marks. Others described how to account for an associate rather than dealing
specifically with how to treat trading between a parent and its associate.
Total possible marks 2
Maximum full marks 2

Copyright ICAEW 2013. All rights reserved Page 17 of 21


Financial Accounting and Reporting Professional Level September 2013

Question 5

Overall marks for this question can be analysed as follows: Total: 18

General Comments
This question required the preparation of a consolidated income statement and extracts from the
consolidated statement of changes in equity (for retained earnings and the non-controlling interest). The
group had two subsidiaries, one of which was disposed of during the year. Fair value adjustments were
required on acquisition of one of the companies. Inter-company trading took place during the year between
the parent and subsidiary.

Gaviota plc

Consolidated income statement for the year ended 31 March 2013



Revenue (W1) 1,663,170
Cost of sales (W1) (594,850)
Gross profit 1,068,320
Operating expenses (W1) (248,950)
Profit from operations (W1) 819,370
Investment income 40,000
Profit before tax 859,370
Income tax expense (W1) (260,220)
Profit for the period from continuing operations 599,150
Profit for the period from discontinued operations (W2) 165,310
Profit for the period 764,460

Profit attributable to
Owners of Gaviota plc () 681,600
Non-controlling interest (W3) 82,860
764,460

Consolidated statement of changes in equity for the year ended 31 March 2013 (extract)

Non-
controlling
interest

Balance at 1 April 2012 (W5) 428,675
Total comprehensive income for the year 82,860
Eliminated on disposal of subsidiary (W2) (214,160)
Dividends (350,000 x 60p x 25%) (52,500)

Balance at 31 March 2013 () 244,875

Copyright ICAEW 2013. All rights reserved Page 18 of 21


Financial Accounting and Reporting Professional Level September 2013

Workings

(1) Consolidation schedule


Gaviota plc Socorro Ltd Adj Consol

Revenue 1,265,600 482,570 (85,000) 1,663,170

Cost of sales per Q (538,900) (136,700) 85,000 (594,850)


PURP (W6) (4,250)

Op expenses per Q (168,500) (76,450) (248,950)


FV deprec (160,000/40yrs) (4,000)

Investment income 197,500 40,000


Socorro Ltd (350,000 x 60p x 75%) (157,500)

Tax (192,800) (67,420) (260,220)


198,000

(2) Profit from discontinued operations (Ramblo Ltd)



Sale proceeds 450,000

Goodwill at acquisition 66,850


Less: Impairments to date (20,000)
(46,850)
Less: Carrying amount of net assets at disposal
Net assets at 31 March 2013 563,200
Less: Profit since 1 Jan 2013 (111,200 x 3/12) (27,800)
(535,400)
Add back: Attributable to non-controlling interest (535,400 x 40%) 214,160
Profit on disposal 81,910
Add: Profit for the year (111,200 x 9/12) 83,400
165,310

(3) Non-controlling interest in year



Socorro Ltd (25% x 198,000 (W1)) 49,500
Ramblo Ltd (40% x 83,400 (W2)) 33,360
82,860

(4) Socorro Ltd Net assets


(Proof only)
31 Mar 2013 1 Apr 2012 At acq

Share capital 350,000 350,000 350,000
Retained earnings (W) 489,500 497,500 152,400
FV adjustment 160,000 160,000 160,000
FV depreciation (4,000 x 4yrs) (20,000) (16,000)
Total 979,500 991,500 662,400

W (489,500 202,000) + (350,000 x 60p) = 497,500

Copyright ICAEW 2013. All rights reserved Page 19 of 21


Financial Accounting and Reporting Professional Level September 2013

(5) Non-controlling interest brought forward

Socorro Ltd
NCI at acquisition (662,400 (W4) x 25%) 165,600
Post acquisition ((991,500 (W4) 662,400) x 25%) 82,275
247,875
Ramblo Ltd
NCI at acquisition ((300,000 + 59,000) x 40%) 143,600
Post acquisition ((263,200 111,200 59,000) x 40%) 37,200
180,800
428,675

(6) PURP
%
SP 125 85,000
Cost (100) (68,000)
GP 25 17,000
1
X /4 4,250

(7) Non-controlling interest carried forward (for proof only)

Socorro Ltd
NCI at acquisition 165,600
Post acquisition ((979,500 (W4) 662,400) x 25%) 79,275
244,875

Copyright ICAEW 2013. All rights reserved Page 20 of 21


Financial Accounting and Reporting Professional Level September 2013

Most candidates made a good attempt at preparing the consolidation schedule and correctly excluded the
subsidiary sold during the year. Candidates who produced this schedule generally gained most of the
straightforward marks for adjustments such as the inter group trading and the related PURP, with many
candidates producing a completely correct consolidation schedule.

Virtually all candidates also prepared a reasonable consolidated income statement gaining the relevant
presentation marks. The most common loss of presentation marks was for abbreviating the non-controlling
interest to NCI and/or including the profit from discontinued operations before tax or after the non-controlling
interest. Very few candidates did their consolidation workings on the face of the income statement which was
pleasing.

As mentioned most candidates correctly calculated the PURP although occasionally this was deducted from
revenue rather than added to cost of sales and sometimes it was included in the wrong column. A pleasing
number of candidates also calculated the adjustment to depreciation correctly although slightly fewer made
the correct adjustment to remove inter group dividends.

Virtually all candidates attempted to calculate the profit on disposal although few managed to arrive at the
correct figure. The most common errors here were deducting the total goodwill rather than the unimpaired
goodwill, failing to adjust closing net assets for the profit made after the disposal (candidates frequently used
the wrong number of months and/or added the profit rather than deducting it) and multiplying the profit up to
disposal by the parent companys percentage holding.

As expected the extract to the consolidated statement of changes in equity was not as well dealt with and few
candidates gained the relevant presentation mark. However most candidates did attempt to include at least
some of the relevant figures for the disposal adjustment, the non-controlling interests share of profit and the
dividend. However a number of candidates lost easy markings for lack of consistency ie by not taking the
figures already calculated in the consolidated income statement and the disposal calculation.

Only a minority of candidates calculated the brought forward non-controlling interest figure correctly and few
produced clear, structured workings to support this figure. However, most candidates made some attempt to
calculate either non-controlling interest brought forward or carried forward and earned some marks for this.
Unfortunately, candidates too often produced messy and unstructured workings with little or no audit trail,
which made awarding marks almost impossible. Few candidates seemed to realise that the easiest way to
arrive at this figure was by calculating the net assets of the two subsidiaries at the start of the year. Even
candidates who did attempt this rarely made the right adjustments to back out current year profit and
dividends and recognise the fair value uplift and related impact on accumulated depreciation.
Total possible marks 18
Maximum full marks 18

Copyright ICAEW 2013. All rights reserved Page 21 of 21


Financial Accounting and Reporting - Professional Level December 2013

MARK PLAN AND EXAMINERS COMMENTARY

The marking plan set out below was that used to mark this question. Markers were encouraged to use discretion
and to award partial marks where a point was either not explained fully or made by implication. More marks
were available than could be awarded for each requirement. This allowed credit to be given for a variety of valid
points which were made by candidates.

Question 1

Overall marks for this question can be analysed as follows: Total: 30

General comments
Part (a) of this question required candidates to revise a draft income statement and statement of financial
position for a number of adjustments. The amendments were in relation to the receipt of a government grant, a
held for sale asset, the recoverability of receivables, irredeemable preference shares and dividend thereon, an
adjusting subsequent event, an overprovision of income tax from the previous year, a provision for warranty
costs and an accrual. Part (b) required an explanation of any ethical issues arising from the scenario and the
action to be taken. Part (c) required candidates to identify and describe the elements of the financial
statements which are relevant to the statement of financial position, with reference to the treatment of the
irredeemable preference shares and the provision.

Dedlock Ltd
(a) Revised financial statements
Statement of financial position as at 30 June 2013

ASSETS
Non-current assets
Property, plant and equipment (567,800 (20,000 8,500)) 556,300

Current assets
Inventories 278,500
Trade and other receivables (105,200 55,700 990 (W2)) 48,510
Cash and cash equivalents 15,800
342,810
Non-current asset held for sale 7,550
350,360
Total assets 906,660

Equity
Ordinary share capital 200,000
Share premium 75,000
Retained earnings (W1) 394,506
Equity 669,506

Non-current liabilities
Preference share capital (irredeemable) 100,000
Deferred income (W3) 6,400
106,400
Current liabilities
Trade and other payables (82,200 + 5,300) 87,500
Deferred income (W3) 1,600
Provisions (W4) 15,654
Taxation 26,000
130,754
Total equity and liabilities 906,660

Copyright ICAEW 2014. All rights reserved. Page 1 of 17


Financial Accounting and Reporting - Professional Level December 2013

Income statement for the year ended 30 June 2013



Revenue (2,876,500 10,000 3,175) 2,863,325
Cost of sales (W2) (1,998,504)

Gross profit 864,821


Administrative expenses (W2) (584,500)
Other operating costs (W2) (241,990)

Operating profit 38,331


Finance costs (200,000 x 50p x 5% x 6/12) (2,500)
Profit before tax 35,831
Income tax (26,000 3,175) (22,825)
Profit for the year 13,006

Workings
(1) Retained earnings

Per draft 484,100
Less Draft profit for the year (105,100)
Add Revised profit for the year 13,006
Add back finance costs from SCE (already taken off as 2,500
dividend)
394,506

(2) Expenses
Cost of Other Admin
sales operating expenses
costs

Per draft 1,980,900 185,300 579,200
Government grant (W3) (2,000)
Loss on held for sale asset (11,500 (8,000 3,950
450))
Bad debt written off 55,700
Bad debt allowance ((105,200 55,700) x 2%) 990
Warranty provision (W4) 15,654
Accrual 5,300
1,998,504 241,990 584,500
Note: Marks were awarded if items were included in different line items in the income
statement provided that the heading used was appropriate.

(3) Government grant



Grant as received 10,000
Taken to cost of sales y/e 30 June 2013 x 20% = (2,000)
At 30 June 2013 8,000
Within one year x 20% = (1,600)
After one year () 6,400

(4) Warranty provision


Number to repair or replace = 1,000 x 5% x = 25

Repaired (25 x 190)/1.07 4,439
Replaced (600,000/1,000 = 600 x 25 x 80%)/1.07 11,215
15,654

Copyright ICAEW 2014. All rights reserved. Page 2 of 17


Financial Accounting and Reporting - Professional Level December 2013

Tutorial note

Credit was also given if candidates stated that general provisions are not allowed in respect of receivables
(IAS 39).

Candidates generally performed well on this part of the question. Presentation of the two statements was
generally of a sufficient standard to collect the presentation marks. Candidates should ensure they transfer
their figures into final totals for individual line items in the financial statements.

The majority of candidates identified that there was a non-current asset held for sale and that it should be
separately analysed, although a small minority thought that it should still be considered to be a non-current
asset. Of those candidates who correctly identified that it should be presented as part of current assets, only
a minority presented it separately from current assets generally. A good number of candidates correctly
calculated the relevant figure in both statements.

The two adjustments to trade receivables for bad and doubtful debts were generally dealt with correctly, with
almost all candidates correctly deducting the amount for the customer who had gone into liquidation before
calculating the closing allowance. However, a worrying number of candidates presented the closing
allowance as a liability in the statement of financial position, as opposed to netting it off trade and other
receivables. Property, plant and equipment was stated correctly by a much smaller number of candidates,
with various different adjustments being made to the draft figure.

The classification and valuation of the preference shares proved a particular challenge. A number of
candidates treated this as equity or as a hybrid financial instrument, split between non-current liabilities and
equity. Some even treated this as equity but then went on in Part (b) of the question to state that it should be
treated as a liability. The related finance costs also caused a significant number of candidates an issue, with
only a minority getting the correct figure in the income statement and even less going on to add this figure
back to the profit figure. Where the adjustment was made to retained profits it was more often than deducted,
instead of being added.

Deferred income in respect of grants of 8,000 was correctly calculated by the majority of candidates,
although the split between current and non-current liabilities was often incorrect and sometimes the
adjustment in the income statement was omitted or incorrectly added to expenses, instead of being deducted.

The most common error was to reduce revenue by only the deferred part of the grant, instead of recognising
that the whole grant needed to be removed from revenue and dealt with either as other income or offset to
cost of sales. Weaker candidates tried to apply the netting off method to the grant and make depreciation
adjustments for the asset.

Although the income statement figure for taxation was usually correct, some candidates also showed this
figure as the closing liability, ignoring the overprovision from the previous year.

The warranty provision caused most candidates a problem. Where candidates did attempt a calculation the
figure for the repaired element was usually correct. However, the figure for the replaced element was only
calculated correctly by a minority of candidates (the most common error being not taking into account the
profit margin on the goods under warranty when they were expected to be replaced). Even fewer candidates
then went on to discount the total. A significant number of candidates went on to deduct their calculated
warranty provision from revenue rather than showing it (separately) as a current liability.

Total possible marks 21


Maximum full marks 20

Copyright ICAEW 2014. All rights reserved. Page 3 of 17


Financial Accounting and Reporting - Professional Level December 2013

(b) Ethical issues

Richard has omitted to adjust for a number of issues, all of which could be said to have a negative effect on
Dedlock Ltds financial statements for the year. The correct treatment of the overprovision of last years
income tax charge reduces revenue, the reclassification of the irredeemable preference shares increases
debt and all of the other adjustments reduce the profit for the year. Profit for the year before the adjustments
was 105,100. However, after adjustments it has fallen by 80%, to 13,006.

Richard is the finance director of the company, and these are all matters of which he was, or should have
been, generally aware. This calls into question whether Richard has failed to make these adjustments as he
is influenced by the fact that he may get a better price for his shares if the companys profit is higher, and its
debt lower. This is a self-interest threat and calls Richards integrity into question.

Alternatively, if it is that Richard does not understand how to make these adjustments, or that these
adjustments were necessary, then that calls his professional competence into question. ICAEW Chartered
accountants have an obligation to maintain their continuing professional development and they should ensure
that their technical knowledge and professional skills are kept up to date.

Clara faces a number of ethical issues, not least the question of whether the mistakes were deliberate or a
lack of knowledge on Richards part. Clara also faces a self-interest threat as she may be offered a
permanent position at Dedlock Ltd if she turns a blind eye to Richards failings.

Clara should ignore the possibility of self-interest and discuss the adjustments with Richard and remind him of
his professional responsibilities to ensure that accounting standards are correctly followed.

Amendments must be made to the financial statements and if Richard refuses to make them, Clara must
discuss the matter with the managing director.

If Richard continues to try to dominate and exert influence on Clara then it would be appropriate for Clara to
consult the ICAEW ethical handbook and discuss the matter with the ICAEW confidential helpline.

Almost all candidates made a reasonable attempt at this part of the question, with a good number obtaining
full marks. Candidates should remember that to gain the most marks their answer should be tailored to the
question scenario. Most candidates correctly identified that there was a self-interest threat for both Richard
and Clara, explained how these threats arose and suggested appropriate courses of action. A minority of
candidates answered as if Clara was an external auditor, as opposed to an independent consultant. A few felt
there were money laundering issues at play.
Total possible marks 8
Maximum full marks 4

Copyright ICAEW 2014. All rights reserved. Page 4 of 17


Financial Accounting and Reporting - Professional Level December 2013

(c) Elements of financial statements, irredeemable preference shares and warranty provision

The three elements of financial statements relevant to the statement of financial position are assets, liabilities
and equity.

Irredeemable preference shares

IAS 32 classifies financial instruments as financial assets, financial liabilities or equity.

The irredeemable preference shares are an example of a (financial) liability. Although the irredeemable
preference shares take the legal form of equity they are liabilities in substance as they include contractual
obligations to transfer economic benefits to the holder (fixed preference (ie preferential) dividends). They
arise from a past event (the issue of the shares

Warranty provision

A provision is a liability of uncertain timing or amount and should be recognised if there is a present obligation
from a past event, it is probable that an outflow of economic benefits will be needed to settle the obligation
and that a reliable estimate can be made of that amount.

If one or more of these requirements are not met then a provision should not be recognised as it is not a
liability.

Probable means that it is more likely than not to occur or >50%. If it is not probable that an outflow of
economic benefits will be needed to settle the obligation or the amount of the settlement cannot be measured
reliably then it does not meet the definition of a liability and instead the amount may need to be disclosed as a
contingent liability.

In conclusion, Dedlock Ltd has a present obligation (its contractual obligation to repair or replace any faulty
products under a two year warranty), as a result of past events (the sale of the goods). There is a probable
outflow and a reliable estimate can be made (based on the number and amount of past claims under
warranties). The estimation of the amount of the liability is made using expected values. Richard should
therefore have recognised a provision as a liability exists.

Answers to this part of the question were very mixed, with only a minority of candidates showing a good
understanding of the elements of financial statements. Far too many candidates reproduced text from the
open book, which was not required. A significant number of candidates instead discussed the qualitative
characteristics of financial information, which gained no marks. Others did write about the elements of
financial statements, but failed to relate these to the preference shares and warranty provision.
Total possible marks 8
Maximum full marks 6

Copyright ICAEW 2014. All rights reserved. Page 5 of 17


Financial Accounting and Reporting - Professional Level December 2013

Question 2

Overall marks for this question can be analysed as follows: Total: 17

General comments
This question tested the preparation of a consolidated statement of cash flows and supporting reconciliation
note, where a subsidiary had been disposed of during the year. Missing figures to be calculated included
dividends paid (to the group and to the non-controlling interest), dividends received, tax paid, additions to
property, plant and equipment, and proceeds from the issue of share capital.

Chuzzlewit plc

Consolidated statement of cash flows for the year ended 31 December 2012

Cash flows from operating activities
Cash generated from operations (Note) 875,600
Interest paid (W2) (46,400)
Income tax paid (W3) (157,400)
Net cash from operating activities 671,800
Cash flows from investing activities
Purchase of property, plant and equipment (W4) (965,200)
Proceeds from sale of property, plant and equipment 117,000
Dividends received from associate (W5) 104,700
Disposal of Gradgrind Ltd net of cash disposed of (W1) 335,050

Net cash used in investing activities (408,450)


Cash flows from financing activities
Proceeds from share issues (W6) 200,000
Repayment of long-term loan (300,000 250,000) (50,000)
Dividends paid (W8) (401,400)
Dividends paid to non-controlling interest (W9) (1,950)
Net cash used in financing activities (253,350)
Net increase in cash and cash equivalents 10,000
Cash and cash equivalents at beginning of period 31,500
Cash and cash equivalents at end of period 41,500

Note: Reconciliation of profit before tax to cash generated from operations



Profit before tax (635,700 + 82,300) 718,000
Share of profits of associate (102,800)
Finance cost 45,500
Profit on disposal of property, plant and equipment (117,000 102,000) (15,000)
Depreciation charge 351,600
Impairment of goodwill (W7) 40,500
Increase in inventories ((292,900 + 56,400) 198,100) (151,200)
Increase in trade and other receivables (177,800 (151,800 + 26,800)) (800)
Decrease in trade and other payables ((105,800 3,100) (82,500 + 12,200 (10,200)
2,200))
Cash generated from operations 875,600

Workings

(1) Net cash inflow on disposal of Gradgrind Ltd


Net assets disposed of (388,500 x 70%) 271,950
Add: Unimpaired goodwill (56,000 10,000) 46,000
Profit on disposal 20,600
Less: Cash and cash equivalents at disposal (3,500)
335,050

Copyright ICAEW 2014. All rights reserved. Page 6 of 17


Financial Accounting and Reporting - Professional Level December 2013

(2) Interest paid



Cash () 46,400 B/d 3,100
C/d 2,200 CIS 45,500
48,600 48,600
(3) Income tax

Cash () 157,400 B/d 160,000
C/d 135,000 CIS (128,000 + 4,400) 132,400
292,400 292,400
(4) Property, plant and equipment

B/d 1,549,000 Disposal of sub 314,000
Other disposals 102,000
Additions () 965,200 Depreciation charge 351,600
C/d 1,746,600
2,514,200 2,514,200
(5) Investment in associate

B/d 287,800 Cash received () 104,700
CIS 102,800 C/d 285,900
390,600 390,600
(6) Share capital and premium

B/d (300,000 + 40,000) 340,000
Cash received () 200,000
C/d (450,000 + 90,000) 540,000
540,000 540,000
(7) Intangibles

B/d 289,500 Impairments () 40,500
Disposal of sub (56,000 46,000
10,000)
C/d 203,000
289,500 289,500
(8) Retained earnings

Dividends in SCE () 401,400 B/d 1,326,100
C/d 1,435,000 CIS 510,300
1,836,400 1,836,400
(9) Non-controlling interest

Cash () 1,950 B/d 301,800
Disposal (388,500 x 30%) 116,550
C/d 279,200 CIS 95,900
397,700 397,700

Copyright ICAEW 2014. All rights reserved. Page 7 of 17


Financial Accounting and Reporting - Professional Level December 2013

Most candidates made some attempt at this question, although performance overall was disappointing on what
should have been a welcomed straightforward processing style question. The presentation of the statement
was generally good with most candidates gaining the full presentation mark. Most candidates dealt reasonably
well with those aspects of the statement of cash flows which would appear in a single entity statement; it was
the consolidation issues which caused the most problems. For example, only a minority of candidates correctly
added both the continuing and discontinued profit before tax figures in the reconciliation and correctly made
the adjustments for the discontinued operation to the movement in inventories, trade receivable and trade
payables.

A good majority of candidates correctly calculated both the purchase cost and disposal proceeds for property,
plant and equipment. Dividends received from the associate was also a figure which was commonly seen as
both calculated correctly and presented in the correct place within the statement. The repayment of the loan
was also commonly seen as correct, although significantly less candidates managed to correctly calculate the
proceeds from the share issue, with the most common error being the omission of the movement on share
premium.

Most candidates made some adjustments to profit before taxation in the reconciliation. The most common
errors were using the incorrect bracket convention (ie deducting instead of adding or vice versa), omitting the
profit on disposal of property, plant and equipment or the impairment figure. Some candidates also made
incorrect adjustments in the reconciliation by including items that were not required such as revaluations and
the profit on disposal of the subsidiary.

The dividend paid to the non-controlling interest was fairly well attempted although candidates occasionally
included it in the incorrect section of the statement of cash flows or forgot about the adjustment required for the
discontinued operation. The disposal proceeds for the discontinued operation was often missed from the
statement, although where candidates did include it a reasonable attempt was made at the calculation, the
most common error being to use the whole of the subsidiarys net assets in the calculation instead of just the
group share. Where a calculation was provided almost all candidates correctly deducted the cash balance on
the discontinued operation.

17
Total possible marks
17
Maximum full marks

Copyright ICAEW 2014. All rights reserved. Page 8 of 17


Financial Accounting and Reporting - Professional Level December 2013

Question 3

Overall marks for this question can be analysed as follows: Total: 31

General comments
Part (a) of this question required candidates to explain the financial reporting treatment of four accounting
issues, given in the scenario. These covered a finance lease, borrowing costs in respect of a self-
constructed asset, a foreign exchange transaction and revaluations of property, plant and equipment (both
upwards and downwards). Part (b) required an explanation of any UK GAAP differences in respect of the
financial reporting treatment of the four issues.

Nickleby plc
(a) IFRS accounting treatment

(1) Finance lease

Under IAS 17, Leases, the machine will be classified as a finance lease as Nickleby plc is leasing the
machine for the whole of its useful life and is responsible for the maintenance and insurance of the
machine during that period. The machine is also specialised in nature which increases the likelihood of
it being a finance lease. Therefore, per IAS 17, the risks and rewards of ownership are deemed to have
passed to the lessee. On the basis of substance over form an asset will be recognised with a
corresponding liability.

The finance lease should have been capitalised at the lower of the fair value of 17,500 and the
present value of the minimum lease payments and the lease liability set up. The present value of the
minimum lease payments is:

Present value calculation


1 July 2012 4,000 4,000
30 June 2013 4,000 / 1.15 3,478
2
30 June 2014 4,000 / 1.15 3,025
3
30 June 2015 4,000 / 1.15 2,630
4
30 June 2016 4,000 / 1.15 2,287
Present value of the minimum lease payments 15,420

The present value of the minimum lease payments is the lower figure, so the journal entry should be:

Dr: Non-current assets cost 15,420


Cr: Lease liability 15,420

The asset should then be depreciated over the shorter of its useful life and the lease term, ie its four
year useful life giving a depreciation charge of 3,855 (15,420 4), and a resultant carrying amount of
11,565.

Dr: Income statement: Depreciation charge 3,855


Cr: Non-current assets 3,855
accumulated depreciation

The lease liability should then have been reduced by payments made and increased by interest
spreading the total finance charge of 4,580 (20,000 15,420) over the period of the lease using the
interest rate implicit in the lease of 15%. The table below illustrates the entries which should have been
made.

Year ended B/f Interest @15% Payment C/f



30 June 2013
(15,420 4,000) 11,420 1,713 (4,000) 9,133
30 June 2014 9,133 1,370 (4,000) 6,503

Copyright ICAEW 2014. All rights reserved. Page 9 of 17


Financial Accounting and Reporting - Professional Level December 2013

The lease liability at 30 June 2013 is therefore 6,503 non-current and 2,630 current (9,133 6,503).

However, the 8,000 which should have been used to reduce the lease liability for 2013 has already
been debited to the income statement. Only interest of 1,713 should have been charged. The
correcting journal entry is:


Dr: Lease liability (8,000 1,713) 6,287
Dr: Income statement: Finance costs 1,713
Cr: Income statement: Cost of sales 8,000
(2) Borrowing costs

IAS 23, Borrowing Costs, requires that borrowing costs that are directly attributable to the acquisition,
construction or production of a qualifying asset form part of the cost of that asset.
A qualifying asset is one that takes a substantial period of time to get ready for its intended use. The
construction of the building is expected to take 12 months so would be a qualifying asset.
Because the funds have been borrowed specifically for the construction then the borrowing costs are
directly attributable.
If surplus funds are invested the borrowing costs capitalised are to be reduced by the investment
income received on the excess funds.
Capitalisation commences when the entity incurs expenditure on the asset, is incurring borrowing costs
and is undertaking activities to prepare the asset for use. All of these conditions are met.
Borrowing costs can only be capitalised for the period of construction, of which six months fall into the
current year. Therefore in the current year 7,100 ((500,000 x 5% x 6/12) 5,400) should be
capitalised. To correct the entries made by the financial controller:

Dr: Property, plant and equipment (asset in course of 7,100
construction) cost
Dr: Income statement: Other income 5,400
Cr: Income statement: Finance costs 12,500

As part of the cost of the asset, the borrowing costs will ultimately be depreciated over the assets
estimated useful life, once depreciation commences.
(3) Foreign exchange transaction

IAS 21, The Effects of Changes in Foreign Exchange Rates, requires a foreign currency transaction to
be recorded on initial recognition in the functional currency (ie that of the primary economic
environment in which the entity operates so here ) using the exchange rate at the date of the
transaction.

The financial controller should therefore have recorded the transaction at the delivery date of 10 June
2013, using a rate of 1: 0.82, as that is when the risks and rewards of ownership pass.

Dr: Income statement: Purchases (101,000 x 0.82) 82,820


Cr: Trade payables 82,820

At the year end IAS 21 requires monetary items (units of currency held and assets and liabilities to be
received or paid in a fixed or determinable number of units of currency) to be retranslated at the closing
exchange rate. So, at the year end, the liability (ie trade payable) in respect of this transaction should
be restated using the closing rate ie to 75,750 (101,000 x 0.75). A retranslation gain of 7,070
(82,820 75,750) has been made and should be recognised in profit or loss. The journal entry should
be:

Dr: Trade payables 7,070


Cr: Income statement 7,070

Copyright ICAEW 2014. All rights reserved. Page 10 of 17


Financial Accounting and Reporting - Professional Level December 2013

(4) Revaluations

Nickleby plc uses the revaluation model per IAS 16, Property, Plant and Equipment, so the valuation on
1 July 2012 needs to be recognised. The increase in the revaluation surplus will be disclosed in other
comprehensive income. Both the land and buildings increase in value from their previous carrying
amounts (W) so journal entries are:

Dr: Property, plant and equipment land (1,000,000 800,000) 200,000
Dr: Property, plant and equipment buildings* (2,500,000 1,906,000) 594,000
Cr: Revaluation surplus 794,000

The plant falls in value from a carrying amount on 1 July 2012 of 815,700 (W) to a valuation of
450,000 a fall in value of 365,700. 150,400 of this decrease reverses a previous revaluation so
that amount is charged to the revaluation surplus and disclosed in other comprehensive income. The
remaining 215,300 (365,700 150,400) is recognised as an expense in profit or loss. The journal
entry is:


Dr: Revaluation surplus 150,400
Dr: Income statement: cost of sales 215,300
Cr: Property, plant and equipment plant and machinery 365,700

Nickleby plc also needs to recognise the depreciation charges for the year, based on the new
valuations (see W). The journal entry is:


Dr: Income statement: administrative expenses 62,500
Dr: Income statement: cost of sales 112,500
Cr: Property, plant and equipment buildings 62,500
Cr: Property, plant and equipment plant and machinery 112,500

Final carrying amounts are 1,000,000 for the land, 2,437,500 for the buildings and 337,500 for plant
and machinery (W).

Nickleby plc has a policy of making an annual transfer between the revaluation surplus and retained
earnings, so that needs to be made. The transfer is the difference between depreciation charges based
on historic cost and those based on carrying amounts. However, this will only be in respect of the
buildings as there is no longer any balance in the revaluation surplus in respect of plant and machinery.
The journal entry is:

Dr: Revaluation surplus (62,500 (W) 21,500) 41,000


Cr: Retained earnings 41,000

Working
Valuation on Depreciation Carrying
1 July 2012 charge for amount
year

Land 1,000,000 - 1,000,000
Buildings 2,500,000 ( 40) 62,500 2,437,500
Plant 450,000 ( 4) 112,500 337,500

*Tutorial note

This would be Dr to Valuation and Cr to Accumulated depreciation but the split of the
carrying amount was not given so this detail could not be provided. The opposite applies to
plant and machinery.

Copyright ICAEW 2014. All rights reserved. Page 11 of 17


Financial Accounting and Reporting - Professional Level December 2013

Generally this part of the question was well answered with the majority of candidates responding to all four
issues and providing both explanations and calculations, although a minority of candidates failed to set out
the numerical adjustments in the form of journals.
Others gave a stream of journal entries with little narrative by way of explanation, and therefore limited the
number of marks they could obtain. Journals were set out in a number of different ways, with some
candidates setting out several simple journals, and others combining several transactions into a single
journal. All of these were given credit where appropriate, but it is important to realise that if many
transactions are combined an audit trail must be provided. In issue (4) a number of candidates combined
all of the parts of the scenario into one journal, with only a single (net) credit to the revaluation surplus
and/or to property, plant and equipment, with no supporting workings, which meant that partial marks
could not always be awarded. Other candidates wasted time by setting out in journal entry form the entries
which had already been made.
Issue (1): Virtually all candidates identified this issue as a finance lease but very few calculated the
present value of the minimum lease payments to determine the amount at which the initial asset and
liability should have been recognised. Another extremely common error was a lack of consistency
between the amount recognised as a liability and the amount initially recognised in the finance lease table
(although most students did deduct the deposit from whatever figure they used in the leasing table). A
further common inconsistency was making a statement that the amount capitalised should be the lower of
the assets fair value and the present value of the minimum lease payments and then proceeding to
capitalise the higher figure.
Issue (2): The capitalisation of borrowing costs was also dealt with well with the majority of candidates
recognising that interest earned needed to be deducted from the interest paid to arrive at the correct figure
for capitalisation. Most candidates also identified the correct period for capitalisation as being six months
only.
Issue (3): The foreign exchange transaction was not as well dealt with. A surprising number of candidates
stated that the liability should be recognised when the goods were ordered rather than when received (ie
when the risks and rewards of ownership transferred) although most did re-translate the liability using the
year-end rate. As commented on above, a significant number of candidates failed to deal separately with
the initial recognition of the liability and its retranslation at the year end, producing a combined journal
entry and thereby losing marks.
Issue (4): The final issue relating to revaluations was also well dealt with, with most candidates clearly
understanding the correct double entry for revaluations and the impact on subsequent depreciation.
However, few candidates made the point that the valuations needed to be incorporated into the financial
statements because the company had adopted the revaluation model. A pleasing number also showed the
correct double entry for the reserves transfer even if the figure was not always correctly calculated. Most
candidates also identified that the downwards revaluation for the plant and machinery needed to be split
between the revaluation surplus and income statement.
Other common errors not referred to above included the following:
Failing to adjust cost of sales for the full 8,000 incorrectly charged re the finance lease.
Drawing up the finance lease table in the wrong order ie treating the lease as if payments were
in advance rather than in arrears.
Discounting the deposit paid.
Failing to explain what a qualifying asset is and therefore not relating the definition to the
information given in the question ie that the building was expected to take 12 months to complete.
Dividing rather than multiplying when translating euros into sterling.
Failing to explain where the foreign currency should be recognised ie in the income statement.
Calculating the revaluation gain on the building incorrectly by reducing the opening carrying
amount by current year depreciation.
Making the initial debit on recognition of the liability for the foreign exchange transaction to
inventories instead of to purchases.
Calculating the reserves transfer incorrectly by dividing the revaluation surplus by remaining life
(which does not work here as some of the surplus related to the land).
Suggesting a reserves transfer for the plant and machinery even though the balance in the
revaluation surplus had been eliminated by the downwards revaluation.
Making comments which were relevant to the next financial year, rather than to this one (eg
calculating a further foreign exchange loss/gain when the invoice was received after the year end).
Total possible marks 36
Maximum full marks 26

Copyright ICAEW 2014. All rights reserved. Page 12 of 17


Financial Accounting and Reporting - Professional Level December 2013

(b) UK GAAP differences

(1) Finance lease

IAS 17 lists a number of factors which would indicate that the risks and rewards of ownership have been
transferred to the lessee indicating that the lease should be classified as a finance lease.

However, under UK GAAP there is a rebuttable presumption that if, at the inception of the lease, the
present value of the minimum lease payments is at least 90% of the assets fair value then there is a
finance lease.

(2) Borrowing costs

IAS 23 requires attributable borrowing costs to be capitalised. UK GAAP (FRS 15) gives entities the
choice of whether to capitalise borrowing costs or to expense them as incurred.

Capitalisation under UK GAAP is limited to the finance costs incurred on the expenditure incurred. IAS 23
limits the amount capitalised to the borrowing costs on the total related funds less the investment income
from any temporary investment of those funds.

(4) Revaluations

Where assets have been revalued UK GAAP (FRS 15) requires the use of existing use value rather than
fair value

UK GAAP requires impairment losses to be debited first against any revaluation surplus in respect of the
asset unless it reflects a consumption of economic benefits. IAS 16 does not include such a limitation. So,
under UK GAAP, the whole downwards revaluation would have been debited to the profit and loss
account.

Under UK GAAP a maximum period of five years between full valuations and interim valuations every
three years is prescribed. No maximum period is specified by IAS 16 the timing depends on changes in
market values.

Most candidates made a reasonable attempt at identifying the differences between IFRS and UK GAAP,
showing that candidates realise that these differences will always be tested and that these are relatively
easy marks to gain. A number of candidates wasted time by discussing differences that were not relevant
to the scenario given. A minority of candidates appeared to simply invent differences.

The two most common errors were believing that UK GAAP does not permit reserves transfers for
revalued assets and that the 90% test is a comparison between the length of the lease and the useful life
of the asset. Candidates also need to be very careful to be precise with their wording in their answers to
this type of question. For example, with regards to differences in the capitalisation of borrowing costs a
number of candidates stated that under UK GAAP income on surplus funds does not need to be netted
off, which is not the same as stating that it is not netted off.
Total possible marks 6
Maximum full marks 5

Copyright ICAEW 2014. All rights reserved. Page 13 of 17


Financial Accounting and Reporting - Professional Level December 2013

Question 4

Overall marks for this question can be analysed as follows: Total: 22

General comments
Part (a) required the redrafting of a consolidated statement of financial position, where one subsidiary and
one associate (both acquired during the year) had simply been added into the parent companys figures,
and no consolidation adjustments made. Adjustments included fair value adjustments on acquisition, intra-
group sales (with inventory still held at the year end), intra-group balances which did not agree and
impairment write-downs. In Part (b) candidates were required to explain and justify the fair value method
and the proportionate method of calculating non-controlling interest, using calculations where appropriate.

Cratchit plc

(a) Consolidated statement of financial position as at 30 June 2013


Assets
Non-current assets
Property, plant and equipment (1,697,700 377,500) 1,320,200
Intangibles (W3) 237,600
Investment in associate (W7) 139,600
1,697,400
Current assets
Inventories (770,900 246,400 4,000 (W6)) 520,500
Trade and other receivables (293,000 99,300 168,100
25,600)
Cash and cash equivalents (23,800 800 + 6,900) 29,900
718,500
Total assets 2,415,900

Equity and liabilities


Equity attributable to owners of Cratchit plc
Ordinary share capital (1,000,000 300,000 200,000) 500,000
Shares not yet issued (W3) 240,000
Revaluation surplus (400,000 150,000) 250,000
Retained earnings (W5) 917,820
1,907,820
Non-controlling interest (W4) 120,980
Total equity 2,028,800
Current liabilities
Trade and other payables (315,200 97,400 18,700) 199,100
Contingent liability 20,000
Taxation (229,000 61,000) 168,000
387,100
Total equity and liabilities 2,415,900

Copyright ICAEW 2014. All rights reserved. Page 14 of 17


Financial Accounting and Reporting - Professional Level December 2013

Workings

(1) Group structure


80 = 40%
Arusha
200
plc

240
= 80%
300

Java Ltd Mocha Ltd

(2) Net assets Drummle Ltd


Year end Acquisition Post acq

Share capital 300,000 300,000
Revaluation surplus 150,000 150,000
Retained earnings
Per Q 224,900 108,000
Goodwill re sole trader (50,000) (60,000)
Contingent liability (20,000) (20,000)
604,900 478,000 126,900

(3) Goodwill Drummle Ltd



Consideration transferred (400,000 + (200,000 x 1.20)) 640,000
Non-controlling interest at acquisition (478,000 (W2) x 20%) 95,600
Net assets at acquisition (W2) (478,000)
257,600
Impairments to date (20,000)
237,600

(4) Non-controlling interest Drummle Ltd



NCI at acquisition date (478,000 (W2) x 20%) 95,600
Share of post-acquisition reserves (126,900 (W2) x 20%) 25,380
120,980

(5) Retained earnings



Cratchit plc (1,441,200 224,900 365,600) 850,700
Drummle Ltd (126,900 (W2) x 80%) 101,520
Gargery Ltd (W7) (10,400)
Less: PURP (W7) (4,000)
Less: Impairments to date (20,000)
917,820

(6) Inventory PURP Drummle Ltd


%
SP 120 60,000
Cost (100) (50,000)
GP 20 10,000
x 40% 4,000

Copyright ICAEW 2014. All rights reserved. Page 15 of 17


Financial Accounting and Reporting - Professional Level December 2013

(7) Investments in associates Gargery Ltd



Cost 150,000
Less Share of post acquisition decrease in net assets
Share of post acquisition losses (22,500 x 40%) 9,000
Add: Share of additional depreciation based on FV (35,000 5 1,400
x 6/12 x 40%) (10,400)
139,600

This part of the question was reasonably well answered with most candidates producing an adequately
presented consolidated statement of financial position (although a lack of sub-totals and the use of
abbreviations were common). The vast majority of candidates did correctly identify the group structure and
realised that the assets and liabilities incorrectly included for the associate needed to be backed out and the
share capital corrected to be that of the parent company only. Where a draft consolidated statement is
provided in the question it is extremely important that candidates read the information provided carefully to
ascertain exactly on what basis the consolidation has been done.

Most candidates calculated the consideration for the subsidiary acquired during the year correctly (using the
correct share price) although hardly any then included the shares not yet issued in equity in their consolidated
statement of financial position (hence failing to complete the double entry). Even those who did realise that
this needed to be recognised in the statement of financial position often included it in liabilities. The other
most common error was failing to show the contingent liability recognised as a fair value adjustment in
liabilities.

Other common errors included the following :


Failing to adjust for the cash in transit correctly by deducting the same figure from payables and
receivables and/or deducting the amount (rather than adding it) to cash.
Not adjusting the net assets working for the goodwill held by the subsidiary (or only adjusting for the
10,000 change in value).
Not adjusting the net assets working for the contingent liability (or adding rather than deducting it).
Failing to multiply the PURP by the % of shares held in the associate.
Deducting the above PURP from the investment in the associate rather than from inventories.
Including the fair value excess in the investment in associate working.
Failing to multiply the increase in depreciation by the % held in the associate in the above working.
Not recognising that the same figures re post acquisition adjustments in the investment in associate
should also be shown in consolidated retained earnings.
Calculating the non-controlling interest as a % of post-acquisition profits rather than as a % of closing
net assets.
Deducting a share of the goodwill impairment in the NCI working even though the proportionate
method was being used.

As always some candidates lost marks by failing to show an audit trail for the basic consolidation on the
face of the statement of financial position and/or for the calculation of the non-controlling interest and % of
post- acquisition profits.

Total possible marks 17


Maximum full marks 17

Copyright ICAEW 2014. All rights reserved. Page 16 of 17


Financial Accounting and Reporting - Professional Level December 2013

(b) The two methods of calculating goodwill and non-controlling interest

IFRS 3 allows two methods of measuring the non-controlling interest (NCI) at the acquisition date:
(i) At its fair value (the fair value method)
(ii) At the NCIs share of the acquirees net assets (the treatment used in (a), ie the proportionate
method).
Method (ii) results in goodwill being, in effect, the difference between the cost of the parents investment
and its share of the net assets acquired. The rationale behind this is that this market transaction has only
provided evidence of the amount of the parent entitys goodwill there has been no evidence of the
amount of the goodwill attributable to the NCI.
However, this method means that only the parents share (here 80%) of the goodwill of the subsidiary will
be recognised when for every other line item on a consolidated statement of financial position the parent
brings in 100% of the subsidiarys figures, to reflect the fact that the parent has control over that subsidiary.
Method (i), the fair value method, is consistent with the rest of IFRS 3 since IFRS 3 requires both the
consideration transferred and the net assets acquired to be measured at fair value. It works on the basis
that the goodwill attributable to the NCI can be calculated from the estimate of the fair value of the NCI
itself.
The fair method usually results in a higher amount for the NCI/goodwill the difference between this
amount and the amount as traditionally measured is effectively added to the goodwill acquired in the
business combination and is the goodwill attributable to the NCI at the acquisition date.
If NCI had been measured in Part (a) using the fair value method it would have been calculated as follows,
resulting in an NCI higher than that under the proportionate method:

FV of NCI at acquisition 100,000
Share of post-acquisition reserves (126,900 (W2) x 20%) 25,380
125,380
Less: Impairment to date (20,000 x 20%) (4,000)
121,380

As shown above, where NCI has been measured at fair value and there is a subsequent impairment to
goodwill, part of that impairment will be charged to the NCI at the end of the reporting period, based on the
NCI%.
If goodwill had been measured in Part (a) using the fair value method it would have been calculated as
follows:

Consideration transferred (a) 640,000
FV of NCI at acquisition 100,000
Net assets at acquisition (a) (478,000)
262,000
Less: Impairment to date (20,000)
242,000
This part of the question was poorly answered with a significant minority of candidates making no attempt to
produce an answer. Those candidates who did attempt this part of the question focused on calculations, with
very few showing any understanding of the conceptual issues relating to the two methods. Those who did
attempt some narrative tended to describe the underlying mechanics of the calculations as opposed to the
principles underlying them. Many candidates wasted time by copying out entire workings for goodwill and the
non-controlling interest produced in Part (a) of their answer (for which there were no further marks available)
rather than just referring back to the relevant figures and calculating the alternatives using the fair value
method. A significant number of candidates clearly did not understand the full double entry for an impairment
when using the fair value method and it was common to see just the parent companys share of the
impairment deducted from the carrying value of the goodwill.
Total possible marks 8
Maximum full marks 5

Copyright ICAEW 2014. All rights reserved. Page 17 of 17


Professional Level - Financial Accounting and Reporting March 2014

MARK PLAN AND EXAMINERS COMMENTARY

The mark plan set out below was that used to mark these questions. Markers are encouraged to use discretion
and to award partial marks where a point was either not explained fully or made by implication. More marks are
available than could be awarded for each requirement, where indicated. This allows credit to be given for a
variety of valid points, which are made by candidates.

Question 1

Overall marks for this question can be analysed as follows : Total: 30

General comments
This question presented a draft set of financial statements with some adjustments. Candidates were required
to prepare the amended statement of profit or loss, statement of financial position and the intangible assets
table. A number of adjustments were required to be made, including depreciation, research and development
expenditure, revenue adjustments, treasury shares and redeemable preference shares.

Part b) required candidates to explain the purpose and objectives of IFRS 7 Financial Instruments;
Disclosures.

Part c) featured the concepts requirement which asked about the enhancing qualitative characteristics.

Alloa Ltd Statement of financial position as at 30 September 2013



ASSETS
Non-current assets
Property, plant and equipment (W5) 63,560
Intangible assets (95,700 + 17,025)(note) 112,725
176,285

Current assets
Inventories 25,500
Trade and other receivables
(215,000 + 7,200) 222,200
Cash and cash equivalents 13,700
261,400
Total assets 437,685

Equity
Ordinary share capital (185,000 + 15,000) 200,000
Share premium (88,750 + (15,000 x 0.75)) 100,000
Treasury shares (15,000 x 1.75) (26,250)
Retained earnings (W7) 65,735
Equity 339,485

Non-current liabilities
Redeemable preference shares 50,400

Current liabilities
Trade and other payables 30,800
Taxation 17,000
47,800

Total equity and liabilities 437,685

Copyright ICAEW 2014. All rights reserved Page 1 of 19


Professional Level - Financial Accounting and Reporting March 2014

Alloa Ltd Statement of profit or loss for the year ended 30 September 2013


Revenue (W2) 890,000
Cost of sales (W1) (610,605)

Gross profit 279,395


Operating expenses (312,000)

Operating loss (32,605)


Investment income (71,200 + (48,000 x 15%) + 9,524 (W2)) 87,924
Finance charges (W6) (2,400)
Profit before tax 52,919
Income taxation (3,000 17,000) (14,000)

Net profit for the period 38,919

Notes to the financial statements as at 30 September 2013


Intangible asset

Development Patents
costs
Cost
At 1 October 2012 59,000
Additions 127,600
Disposals (3,000)
At 30 September 2013 127,600 56,000

Amortisation
At 1 October 2012 11,600
Charge for year (W3 & W4) 31,900 28,875
Disposals (1,500)
At 30 September 2013 31,900 38,975

Carrying amount
At 30 September 2012 47,400
At 30 September 2013 95,700 17,025

W1 Expenses
Cost of sales

Trial balance 422,590


Opening inventories 23,600
Closing inventories (25,500)
R&D expenditure (W3) 100,400
R&D amortisation (W3) 31,900
Patent amortisation (W4) 28,875
Disposed of patent (3,000 (3,000 / 2yrs)) 1,500
Depreciation charge plant & machinery (W5) 27,240

610,605

W2 Revenue

Trial balance 899,524
Interest free credit (200,000 (200,000/1.05)) (9,524)
At 30 September 2013 890,000

Copyright ICAEW 2014. All rights reserved Page 2 of 19


Professional Level - Financial Accounting and Reporting March 2014

W3 Research & development expenditure



Trial balance 228,000
Less amounts charged to profit & loss
Staff training 5,900
Research costs 26,000
Development of the Brora 68,500
(100,400)
Intangible asset at 30 September 2013 127,600
Amortisation (127,600 / 2yrs x 6/12) (31,900)
95,700

W4 Patents

Amortisation charge for year
(59,000 3,000) / 2yrs 28,000
Disposed of paten (3,000 / 2yrs x 7/12) 875
(28,875)

W5 Plant and equipment



Carrying amount at 1 Oct 2012 90,800
Depreciation charge for the year (90,800 x 30%) (27,240)
63,560

W6 Redeemable preferences shares


Opening Interest exp Interest Closing
balance (4.8%) paid (4%) balance

30 Sept 2013 50,000 2,400 (2,000) 50,400

W7 Retained earnings

Per draft 263,950
Less: draft profit and loss (239,134)
Add: revised profit and loss 38,919
Add back preference dividend (50,000 x 4%) 2,000
65,735

Presentation was generally good, although the presentation of the statement of profit or loss was almost
always better than that of the statement of financial position where sub-totals were, as usual, often missing
for one or more categories. Most candidates correctly showed the treasury shares as a negative balance
under equity.

Presentation of the intangible asset note was more varied with candidates often merging the patents and
development costs into one column and/or netting off cost and amortisation. Only a very small minority of
candidates failed to make any attempt at the note.

The vast majority of candidates used a costs matrix to calculate the figure for cost of sales and, on the
whole, it was possible to match figures on the face of the financial statements to workings. Almost all
candidates correctly calculated the depreciation charge on property, plant and equipment and included this
figure in cost of sales, and the carrying amount on the statement of financial position. Weaker candidates
put the carrying amount both on the statement of financial position and added it to cost of sales.

The adjustments for opening and closing inventories were generally dealt with correctly and pleasingly
many candidates also calculated the tax charge correctly (although not all then went on to include the
correct figure in current liabilities). Disappointingly very few candidates managed to calculate the discount
on the deferred revenue correctly and even those who did very rarely then recognised the related financing
income (even though this issue was almost identical to worked examples in the study manual). Most
candidates included the correct figure for royalty income in the statement of profit or loss, but few completed
the double entry by also adding this to trade and other receivables.

Copyright ICAEW 2014. All rights reserved Page 3 of 19


Professional Level - Financial Accounting and Reporting March 2014

Surprisingly many candidates also struggled with the redeemable preference shares. Even those who wrote
out the table working showing the correct interest expense and cash paid often then went on to put the
wrong figures in the statement of profit or loss and/or the statement of financial position. However most did
recognise that the transaction should be treated as a liability rather than equity. Strangely a number of
candidates treated the shares as convertible debt and wasted significant time discounting the future
payments to arrive at separate debt and equity elements.

Rather disappointingly relatively few candidates calculated the correct figures for development costs and
patents. Candidates were often inconsistent such as by including some costs twice (ie both capitalising
and expensing them) or by calculating amortisation on a different figure to the one capitalised.

Common errors in other areas included the following:

Deducting the treasury shares elements from share capital and premium, instead of adding them
and/or showing the treasury shares themselves as a credit balance, instead of a debit.
Failing to reduce retained earnings by the draft profit for the year, having increased it by the profit for
the year calculated in the revised statement of profit or loss.
Failing to capitalise the correct elements of the research and development expenditure.
Basing amortisation for the year on the capitalised development costs on one year instead of six
months.
Incorrectly calculating accumulated amortisation on the patent disposed of during the year (or failing
to charge amortisation on that patent up to the point of disposal).
Failing to adjust cost of sales for the proceeds on disposal of the patent or making the adjustment in
the wrong direction.
In the costs of sales matrix including either the amortisation on the patent or on the capitalised
development costs, but not both.
Capitalising the Brora development costs, even though the project had not yet met the IAS 38 criteria.

Total possible marks 25


Maximum full marks 23

(b)

IFRS 7 Financial Instruments: Disclosure, was published because the IASB felt that existing
standards that covered financial instruments needed to be improved. Improvements were
needed to ensure that the disclosure of information on financial instruments provided greater
transparency of information so that users could better assess the risks that an entity was
exposed to.

The objective of IFRS 7 is to require entities to provide disclosures in their financial statements
which enable users to evaluate both the significance of financial instruments for the entitys
financial position and performance, and the nature and extent of the risks arising from the
financial instruments and how the entity manages those risks.

Most candidates who made an effort with this requirement made a reasonable attempt by reciting the
objectives of IFRS 7 from their open book text. Few candidates went beyond this.

Total possible marks 3


Maximum full marks 2

Copyright ICAEW 2014. All rights reserved Page 4 of 19


Professional Level - Financial Accounting and Reporting March 2014

(c) Enhancing qualitative characteristics

Usefulness
There are four enhancing qualitative characteristics which enhance the usefulness of information that is
relevant and faithfully represented. These are: comparability, verifiability, timeliness and understandability.

Comparability ensures that users can identify and understand similarities in, and differences among, items.
Information about a reporting entity is more useful if it can be compared from one reporting period to the
next and with similar information from other entities. Comparability allows this.

Consistency, although not an enhancing qualitative characteristic itself is related to comparability. This
relates to the same methods being used to report the same item, so consistent accounting policies
governed by accounting standards. The disclosure of accounting policies is therefore key to ensure that
users can make a valid comparison between items.

Verifiability helps assure users that information faithfully represents the information provided it provides
credibility to the financial information. It means that different knowledgeable and independent observers
could reach consensus that a particular depiction is a faithful representation.

Timeliness is equally important as information becomes less useful the longer the time delay in reporting it.
Timeliness means that information is available to investors, lenders and other creditors in time for it to be
used in their decision making processes.

Finally, the characteristic of understandability means that information that may be difficult to understand is
made more useful by presenting and explaining it as clearly as possible. Whilst financial information should
be presented clearly and in an understandable manner, it is expected that users of t he financial statements
have a reasonable level of knowledge and understanding. It would be misleading to exclude information
simply because of its complex nature, as this would lead to incomplete information which would be
misleading to users.

There is a balance between timeliness and the provision of reliable information. For example, a provision
has uncertainty involved in it, if an entity waits to report this information then it may have been settled and
therefore the uncertainty over its amount will disappear. This information is therefore more reliable the
longer an entity waits to report it. However, if such information is not reported until say six months after the
year end then the information is less useful to users.

As with Part (b), most candidates picked up some marks by using their open book text, correctly identifying
the four enhancing qualitative characteristics and making a brief point about each. The depth of explanation
was variable. Others wasted time by also discussing the primary qualitative characteristics or other
concepts.

Total possible marks 8


Maximum full marks 5

Copyright ICAEW 2014. All rights reserved Page 5 of 19


Professional Level - Financial Accounting and Reporting March 2014

Question 2

Overall marks for this question can be analysed as follows : Total: 11

General comments
This question required extracts from a consolidated statement of cash flows where a subsidiary had been
disposed of during the year. Candidates were required to calculate the cost of additions to revalued
property, plant and equipment, dividend payments by the parent and a subsidiary company and the
proceeds from the issue of shares (following a bonus issue).

Limerigg plc

Statement of cash flows for the year ended 30 September 2013



Cash generated from operations 497,675

Cash flows from investing activities


Purchase of property, plant and equipment (W1) (457,355)
Disposal of subsidiary (62,000 2,300) 59,700

Cash flows from financing activities


Proceeds from issue of ordinary share capital
(130,000 + 78,000) (W2 & W3) 208,000
Non-controlling interest dividend (W5) (43,300)
Dividends paid (W4) (135,200)

Working

Draft cash generated from operations (continuing & discontinued) 396,675
Depreciation 101,000
Cash generated from operations 497,675

Property revaluation

Carrying amount at 1 October 2012
(300,000 ((300,000 / 30yrs) x 5yrs)) 250,000
Revalued amount 325,000
Revaluation surplus 75,000

Copyright ICAEW 2014. All rights reserved Page 6 of 19


Professional Level - Financial Accounting and Reporting March 2014

Workings
(1) PPE

B/d 506,950 Disposal of subsidiary 76,900
Revaluation (W) 75,000 Depreciation 101,000
Additions () 457,355 C/d 861,405
1,039,305 1,039,305
(2) Share capital

B/d 350,000
Bonus issue (350,000 / 5) 70,000
C/d 550,000 Cash issue () 130,000
550,000 550,000
(3) Share premium

Bonus issue 35,000 B/d 35,000
C/d 78,000 Cash issue () 78,000
113,000 113,000
(4) Retained earnings

Dividends paid () 135,200 B/d 96,430
Bonus issue (70,000 35,000) 35,000 Revaluation surplus 3,000
transfer
(75,000 72,000)
C/d 132,130 Profit or loss 202,900
302,330 302,330
(5) Non-controlling interest

Dividends paid () 43,300 B/d 97,600
Disposal (77,850 x 30%) 23,355
C/d 73,845 Profit or loss 42,900
140,500 140,500

A number of candidates achieved full marks on this question and a pleasing number calculated the correct
figures for the cash inflow from the disposal of the subsidiary, purchase of property, plant and equipment
and for the dividend paid to the non-controlling interest. Most correctly adjusted cash generated from
operations for the depreciation charge although many often also made other unnecessary adjustments.

A significant number of candidates lost marks by failing to show brackets round figures which represent an
outflow of cash. Candidates should be aware that this convention is just as important in a question which
requires extracts from a statement of cash flows as it is for a complete statement of cash flows. Marks were
also lost where items were shown under the incorrect headings the most common error being to show
dividends paid to the non-controlling interest as an investing activity instead of as a financing activity, and
this was often also shown as a cash inflow instead of as an outflow. Some also prepared the T account
workings correctly but then failed to transfer the final figure to the face of the statement of cash flows.

Where errors were made they included the following:

Omitting one or more of the entries from the property, plant and equipment T-account, most
commonly the revaluation figure.
Failing to adjust for the transfer between the revaluation surplus and retained earnings in the latter T-
account.
Omitting the statement of profit or loss figure from the retained earnings and/ or non-controlling
interest T-accounts.
Debiting the whole bonus issue to the share premium account, when this should have been restricted
to the opening balance on the share premium account, which was lower.
Omitting the residual bonus issue from the retained earnings T-account.
Failing to adjust the non-controlling interest figure for the disposal of the subsidiary.

Total possible marks 11


Maximum full marks 11

Copyright ICAEW 2014. All rights reserved Page 7 of 19


Professional Level - Financial Accounting and Reporting March 2014

Question 3

Overall marks for this question can be analysed as follows : Total: 28

General comments
Part (a) of this question required candidates to explain the financial reporting treatment of four accounting
issues, given in the scenario. The four issues covered a government grant, an acquisition of a subsidiary,
an asset impairment and a related party transaction.
Part (b) required candidates to recalculate consolidated profit for the year for the adjustments needed as a
result of their answer to Part (a).
Part (c) required a calculation of basic earnings per share following a s hare issue for cash and a bonus
issue.
Part (d) required candidates to identify any UK GAAP differences for the issues set out in Part a).

Melloch plc
(a) IFRS accounting treatment

(1) Government grant

This is an income related grant and in this case it should be recognised over the two year period to match
the expenditure for which it has been received to compensate. Even though the directors believe that the
grant will not be repayable this is not a reason to recognise it fully upon receipt. As at 30 September 2013
Melloch plc has not satisfied all of the recognition criteria.

225,000 (540,000 x 10/24) of the grant should be recognised as income in the current period. The
remaining grant of 315,000 (540,000 225,000) should be removed from profit or loss and recognised
as a liability.

The liability should be split between current 270,000 (540,000 x 12/24) and non-current 45,000
(315,000 270,000).

The grant should not be recognised as revenue. It could either be shown as other income in the
statement of profit or loss or it could be netted off against the expenditure to which it relates (probably as
part of operating costs).
(2) Acquisition of Sheardale Ltd

Sheardale Ltd should be recognised as a subsidiary of Melloch plc at 1 April 2013, as a controlling interest
of 80% has been acquired. Sheardale Ltd should be consolidated in the group financial statements from
this date.

The consideration should be measured at its fair value of 480,000.

The costs of 8,000 should not form part of the consideration but should instead be recognised directly in
profit or loss.

Intangible assets should be recognised if they are separable or they arise from legal or other contractual
rights. These contractual rights should therefore have been recognised and form part of Sheardale Ltds
net assets.

The contractual rights should be recognised separately to the goodwill and amortised over their useful life
of three years. The carrying amount of the contractual rights at 30 September 2013 is therefore 62,500
(75,000 12,500) and 12,500 ((75,000/3yrs) x 6/12) should be recognised in profit or loss as
amortisation. As the intangible asset is held by Sheardale Ltd, the amortisation will affect the profit
attributable to the non-controlling interest. It will therefore be split 10,000 and 2,500 between the profit
attributable to the shareholders of Melloch plc and the non-controlling interest respectively.

The non-controlling interest can be measured at fair value or proportion of net assets at the date of
acquisition, however here the proportionate method should be used.

Copyright ICAEW 2014. All rights reserved Page 8 of 19


Professional Level - Financial Accounting and Reporting March 2014

Goodwill should be measured at:



Fair value of consideration 480,000
Non-controlling interest (650,000 x 20%) 130,000
610,000
Net assets acquired (575,000 + 75,000) (650,000)
Goodwill gain on bargain purchase (40,000)

As a gain on bargain purchase has arisen Melloch plc will need to reassess the identification and
measurement of the net assets and the measurement of the consideration, however in this case this is
purely cash paid at the date of acquisition. Assuming these calculations are correct the gain of bargain
purchase should be recognised as part of profit or loss for the period.

Sheardale Ltds loss attributable to Melloch plcs shareholders since acquisition should be recognised in
the consolidated statement of profit or loss at 72,000 (180,000 x 6/12 x 80%) and the non-controlling
interest in the statement of profit or loss should be decreased by 18,000 (90,000 x 20%).

Consolidated net assets at 30 September 2013 will also decrease.

(3) Impairment of research facility

It appears that the research facility has suffered an impairment and therefore its carrying amount may be
overstated. Assets should not be carried at more than their recoverable amount. Recoverable amount is
the higher of value in use and fair value less costs to sell.

The value in use at 30 September 2013 is 1,100,000 and fair value less costs to sell is 1, 245,000
(1,250,000 5,000). The recoverable amount is therefore 1,245,000 and an impairment of 155,000
(1,400,000 1,245,000) should be recognised.

100,000 should therefore be recognised against the balance of the revaluation surplus, to reduce this
amount to zero. The remaining 55,000 should be recognised as part of profit and loss for the period.

(4) Related party

Melloch plc will need to establish whether or not the sale of the vehicle to the marketing director is a
related party transaction under IAS 24 Related Party Disclosure.

The marketing director is a member of the key management personnel of Melloch plc and therefore he is a
related party under IAS 24. Therefore, the sale of the vehicle to the marketing director is a related party
transaction. Even though the sale was at full fair value, it should be disclosed.

Disclosure should include the nature of the related party relationship, ie one of the directors, and whether
there are any outstanding balances at the year end, ie 17,500. If there are any special terms and
conditions attached to the balance this should also be disclosed.

A statement that the transaction took place on an arms length basis could only be made if it can be
substantiated. Presumably here an external vehicle guide would show the fair value of the vehicle and
assuming it to be in line with the price agreed such a statement could be made.

Answers to this part of the question were good. Most candidates correctly identified three out of the four
underlying issues as a revenue grant, the acquisition of a subsidiary and the impairment of an item of
property, plant and equipment. The related party transaction was less well dealt with, a significant number
of candidates completely missing that this was a related party transaction at all.

Copyright ICAEW 2014. All rights reserved Page 9 of 19


Professional Level - Financial Accounting and Reporting March 2014

Some marks (though not many) were lost on errors in the calculations but more were lost where
candidates, after an initial explanation, then reduced their answer to a series of journal entries. Although
there were specific marks allocated to key calculations and to the adjustments using those figures in Part
(b), there were no marks for journal entries in lieu of narrative explanations. Marks are only ever awarded
for journal entries where these are specifically required by the question.

(1) Government grant

Most candidates correctly described the conditions under which a grant can be fully recognised as
revenue but then went on to correctly describe how the income should be deferred. The majority of
candidates correctly calculated the amount which could be recognised in income in the current year (with
only a minority using the wrong number of months) and correctly split the balance between non-current
and current liabilities.

(2) Acquisition of subsidiary

Almost all candidates correctly recognised this as the acquisition of a subsidiary and that it should
therefore be consolidated. Candidates then correctly went on to calculate goodwill, although not all arrived
at a gain on bargain purchase (in which case marks were given for describing the correct accounting
treatment of goodwill, both in this part and in Part (d)). The most common two errors in this calculation
were including the associated costs of acquisition in the fair value of the consideration and/or failing to
increase the net assets figure by the fair value of the contractual rights.

A good number of candidates then arrived at the correct amortisation charge for the year on these rights,
but less went on to split this charge between the parent and the non-controlling interest. Similarly, most
recognised that the subsidiarys loss for the year should be recognised in the consolidated statement of
profit or loss, a few less correctly stated that only six -twelfths of this figure should be recognised, with
fewer still splitting the resultant figure between the parent and the non-controlling interest.

It was rare for candidates to make the point that the non-controlling interest could be measured using the
fair value method or the proportionate method, and that the latter was the chosen method. Only a very
small minority of candidates made the point, where a gain on bargain purchase had been calculated, that
this should be reassessed. A significant number of candidates stated that the gain on bargain purchase
should be immediately recognised in retained earnings, rather than making it clear that it should be
immediately recognised in the consolidated statement of profit or loss.

(3) Impairment of research facility

There were some very good answers to this part. Almost all candidates correctly stated the rules for
calculating the amount of an impairment and calculated the correct figures, setting the impairment firstly
against the revaluation surplus for this asset. The most common error was to calculate the impairment as
the difference between the carrying amount and the value in use, instead of the fair value less costs to sell
(which was higher). A significant minority of candidates discussed the scenario as one of an asset held for
sale (and then possibly dealt with the legal costs separately).

(4) Related party transaction

Answers to this issue were very disappointing with very many candidates not even recognising that the
key issue here was the disclosure of a related party transaction. Of those who did identify that it was a
related party transaction only a few explained why the director was considered to be a related party and
what details needed to be disclosed. Fewer still made the point that, provided t hat fact could be
substantiated, the arms length nature of the transaction could be disclosed.

Frighteningly a very significant number of candidates appeared to believe that transactions should not be
recognised until the cash had been received and therefore felt that the sale needed to be derecognised,
so it was very common to see an adjustment for the profit on sale of 2,500 in Part (b).

Total possible marks 26


Maximum full marks 17

Copyright ICAEW 2014. All rights reserved Page 10 of 19


Professional Level - Financial Accounting and Reporting March 2014

(b)
Melloch plc

Profit attributable to Melloch plcs shareholders


As stated 978,400
(1) Government grant (315,000)
(2) Acquisition of Sheardale Ltd:
- acquisition costs (8,000)
- intangible amortisation (10,000)
- gain on bargain purchase 40,000
- Share of Sheardale Ltds loss (72,000)
(3) Impairment (55,000)

Restated 558,400

It appeared that most candidates had built up their answer to this part alongside their answers to Part (a),
which is by far the most efficient approach, with most candidates including all of the relevant adjustments
that they had discussed in Part (a). The most common errors were to include the gain on bargain
purchase as an expense and the (share) of the subsidiarys loss as a profit or not at all.

Total possible marks 3


Maximum full marks 3

(c)
Melloch plc

No. Of Period in Bonus Weighted


shares issue factor average
1 Oct 30 Nov 280,000 2/12 6/5 56,000
1 Dec issue at MV 70,000
1 Dec 31 Mar 350,000 4/12 6/5 140,000
Bonus issue 1 April
(350,000 / 5) 70,000
1 Apr 30 Sept 420,000 6/12 210,000
406,000

Basic EPS = 558,400 = 1.38


406,000

Many candidates scored full marks on this part. Those that made a poor attempt at this calculation clearly
did not understand the impact of the bonus issue. Where errors were made they included the following:

Using the wrong fractions for the parts of the year, or for the bonus issue, or both.
Applying those fractions the 70,000 increments, instead of to the cumulative number of shares to
date.

Total possible marks 4


Maximum full marks 3

Copyright ICAEW 2014. All rights reserved Page 11 of 19


Professional Level - Financial Accounting and Reporting March 2014

(d) UK GAAP differences

Acquisition of subsidiary
The contractual rights are treated differently under UK GAAP as they would not be recognised as these
are not separable. Hence the intangible asset would be subsumed as part of the goodwill, rather than
separately recognised as per IFRS.

The 8,000 acquisition costs associated with the acquisition would be recognised as part of the
consideration rather than expensed to profit and loss as per IFRS.

There is no option to use fair value to measure the non-controlling interest as per IFRS, instead it would
be measured as a proportion of net assets.

Negative goodwill (a gain or bargain purchase) is recognised as a separate item within goodwill rather
than recognised in profit or loss for the period as per IFRS. The negative goodwill should be split between
the fair value of the non-monetary assets and that which is in excess of the fair value of these assets. This
determines the period over which the negative goodwill should be recognised in profit and loss.

Impairment
Under UK GAAP an impairment on a revalued asset would normally be recognised against the balance on
the revaluation surplus unless the impairment was as a result of a consumption of economic benefits. It is
unlikely that the impairment of the research facility is a result of a consumption of economic benefits and
therefore there would be no difference in treatment. Under IFRS there is no such requirement.

Related parties
Under UK GAAP FRS 8 requires the consideration of materiality to both sides of a related party
transaction. IFRS requires no such consideration of materiality.

Under FRS 8, the names of the related parties would need to be disclosed, there is no such requirement
under IFRS.

Most candidates adopted the columnar approach recommended by the examining team at the recent tutor
conference, giving both the IFRS and the UK GAAP treatments and giving only differences which were
relevant to the issues in Part (a). It was also clear that more candidates had committed these differences
to memory. Those who had learnt these differences scored well easily picking up three or more of the
available five marks.

The most common mistake was to state that under UK GAAP impairments can never be taken to the
revaluation surplus.

Total possible marks 8


Maximum full marks 5

Copyright ICAEW 2014. All rights reserved Page 12 of 19


Professional Level - Financial Accounting and Reporting March 2014

Question 4

Overall marks for this question can be analysed as follows : Total: 14

General comments
This question was a mixed topic question, covering inventory valuation and a sale and operating
leaseback.
Part b) required a discussion around the ethical issues.

Bainsford plc

(i)
Statement of financial position at 30 September 2013 (extract)

Current assets (275,850 + 9,600(W1) 3,000(W2)) 282,450

Current liabilities (141,700 + 93,750 (W3)) 235,450

(ii)
Statement of profit or loss

Draft profit after tax 497,300
Increase in raw materials 9,600
Decrease in finished goods (3,000)
Sale and leaseback adjustment ((375,000 93,750) 250,000) 31,250
Impairment loss (125,000)
410,150

Workings

(1) Raw materials



Weighted average
(5,000 x 74) + (6,000 x 65) + (4,000 x 80) x 1,200 (86,400)
(5,000 + 6,000 + 4,000))
FIFO 80 x 1,200 96,000
9,600
(2) Finished goods

Absorption rate (1.50 0.25 0.15) = 1.10


Adjustment
(1.50 1.10) x 7,500 3,000

(3) Sale and operating leaseback



Carrying amount 900,000
Less: fair value (775,000)
Impairment loss 125,000

Proceeds 1,150,000
Less: fair value (775,000)
Profit 375,000

Deferred income (375,000 x 3/12) 93,750

Copyright ICAEW 2014. All rights reserved Page 13 of 19


Professional Level - Financial Accounting and Reporting March 2014

Answers to this were quite mixed although most candidates calculated, as required, the three revised
figures, and, on the whole, carefully followed their supporting calculations through to these figures. A good
number arrived at the correct adjustment to both raw materials and finished goods, although typically
whilst the adjustment relating to the raw materials was calculated correctly far fewer candidates could
correctly identify which costs should be included in the value of finished goods . The most common errors
were mistakes in calculating the weighted average cost of raw materials and failing to exclude the storage
costs when calculating the absorption rate for finished goods.

Attempts at adjusting for the sale and operating leaseback were very mixed, with many candidates writing
at length about the appropriate accounting treatment, when only the calculations were required (no use of
the word explain in the requirement). Although most candidates who made a reasonable attempt at these
calculations did realise that the profit on disposal should be recognised over the lease term rather than
recognised immediately few calculated it correctly by failing to account for the impairment first. Often the
same figure was used to adjust liabilities and profit rather than recognising that the deferred amount
should be added to liabilities and the proportion recognised up until the year-end added to profit.

Total possible marks 10


Maximum full marks 9

(b)

Nias concerns about the use of creative accounting may be justified as after the adjustments she made to
the draft consolidated profit for the year, profit has fallen by 17.5%. While some of the adjustments may be
attributable to Nias assistants lack of knowledge of accounting standards, the fact that the finance
director was on hand to help may call into question the finance directors behaviour and whether the
figures have been deliberately inflated.

Nia should make the appropriate adjustments to the financial statements and explain to the finance
director why profit has fallen. If her adjustments are challenged, she may need to seek advice on how to
proceed. In the first instance Nia should speak to the other directors or the audit committee. Much will
depend upon the finance directors attitude and whether Nia is challenged in her adjustments. If Nia is still
concerned about the issues not being dealt with correctly she may wish to contact the ICAEW advisory
helpline.

Nias other ethical problem relates, in part, to confidentiality. Confidentiality is one of the five fundamental
principles set out in the ICAEWs ethical Code. Nia is expressly required to respect the confidentiality of
information required as a result of professional and business relationships. The information about the
competitor, of which she is now aware because of a personal contact, could possibly be of benefit to
Bainsford plc, and so Nia might be tempted to discuss this information with her employer as it may impact
on their business and the opportunity to gain additional funding. Passing on such information may balance
out any ill-feelings as a result of making the adjustments to reduce profit and would show her loyalty to her
employer. However, professional accountants should be guided not only by the terms but also by the spirit
of the ethical Code. Taking this approach, confidentiality should be maintained.

Another of the five fundamental principles is professional behaviour. Professional accountants should
avoid any action that discredits the profession. If Nia were to use the information for the benefit of her
employers, and if this were subsequently to be made public, it is likely that this would appear discreditable
to the profession.

Copyright ICAEW 2014. All rights reserved Page 14 of 19


Professional Level - Financial Accounting and Reporting March 2014

As in previous sittings, many candidates framed their answer as if they were part of an audit team, not
employed within industry. It was therefore inappropriate to suggest referring the matter to the ethics
partner or to discuss approaching the audit with increased professional scepticism.

With regard to the information from Sam, most candidates recognised the need to refer to the fundamental
principle of confidentiality and knew that Nia should not repeat this information. Others thought that she
should repeat it if it could be substantiated. Few referred to the fundamental principle of professional
behaviour, which was also relevant.

Almost all candidates did recognise the possible need to contact the ICAEW confidential helpline if they
were unable to resolve the issues via discussion with the finance director, or with the other directors or the
audit committee, but there was a tendency to be very quick to suggest that their own resignati on might be
the best solution.

Total possible marks 9


Maximum full marks 5

Copyright ICAEW 2014. All rights reserved Page 15 of 19


Professional Level - Financial Accounting and Reporting March 2014

Question 5

Overall marks for this question can be analysed as follows : Total: 17

General comments
This question required the preparation of a consolidated statement of profit or loss and extracts from the
consolidated statement of changes in equity (for retained earnings and the non-controlling interest). The
group had two subsidiaries, one of which was acquired during the year and a joint venture. Fair value
adjustments were required on acquisition of one of the companies. Inter-company trading took place
during the year between one of the subsidiarys and the parent and the other subsidiary.

Cambus plc

(i) Consolidated statement of profit or loss for the year ended 30 September 2013

Revenue (W1) 2,017,550
Cost of sales (W1) (677,050)
Gross profit 1,340,500
Operating expenses (W1) (504,700)
Profit from operations (W1) 835,800
Share of profit of jointly controlled entity (W4) 12,850
Profit before tax 848,650
Income tax expense (W1) (178,650)
Profit for the period 670,000

Profit attributable to
Owners of Cambus plc () 613,050
Non-controlling interest (W2) 56,950
670,000

(ii) Consolidated statement of changes in equity for the year ended 30 September 2013
(extract)
Retained Non-
earnings controlling
interest

Balance at 1 October 2012 (W6 & W5) 266,515 215,180
Total comprehensive income for the year 613,050 56,950
Added on acquisition of subsidiary (82,500 + 280,000) x 20% 72,500
Dividends (500,000 x 50p) / (300,000 x 25p x 35%) (250,000) (26,250)

Balance at 30 September 2013 () 629,565 318,380

Copyright ICAEW 2014. All rights reserved Page 16 of 19


Professional Level - Financial Accounting and Reporting March 2014

Workings

(1) Consolidation schedule


Cambus Kennet Ltd
plc Ochill Ltd (6/12) Adj Consol

Revenue 1,285,300 579,000 216,250 (63,000) 2,017,550

Cost of sales per Q (418,200) (236,200) (82,650) 63,000 (677,050)


PURP (W7) (3,000)

Op expenses per Q (267,500) (172,000) (61,200) (504,700)


FV deprec
(100,000/25yrs) (4,000)

Investment income 48,750


Ochill (300,000 x (48,750)
65% x 25p)

Tax (130,000) (34,200) (14,450) (178,650)


129,600 57,950

(2) Non-controlling interest in year



Ochill Ltd (35% x 129,600 (W1)) 45,360
Kennet Ltd (20% x 57,950 (W1)) 11,590
56,950

(3) Ochill Ltd Net assets


(Proof only) At
30 Sept 2013 1 Oct 2012 acquisition

Share capital 300,000 300,000 300,000
Retained earnings (W) 296,400 234,800 153,700
PURP adj (W6) (3,000)
FV adjustment 100,000 100,000 100,000
FV depreciation (4,000 x 6 / 5yrs) (24,000) (20,000)
Total 669,400 614,800 553,700

W (296,400 136,600 + (300,000 x 25p) = 234,800

(4) Jointly controlled entity Izat Ltd



Share of profit for the year (44,625 x 40%) 17,850
Less: Impairment (5,000)
12,850

(5) Non-controlling interest brought forward Ochill Ltd



At acquisition (553,700 (W3) x 35%) 193,795
Share of post-acquisition profits ((614,800 553,700) x 35%) 21,385
215,180
(6) Retained earnings brought forward

Cambus plc (461,200 518,350) (57,150)
Add back dividend (500,000 x 50p) 250,000
Izat Ltd post acquisition ((225,500 44,625 96,000) x 40%) 33,950
Ochill Ltd post acquisition ((614,800 553,700) x 65%) (W3) 39,715
266,515

Copyright ICAEW 2014. All rights reserved Page 17 of 19


Professional Level - Financial Accounting and Reporting March 2014

(7) PURP
%
SP 120 36,000
Cost (100) (30,000)
GP 20 6,000
1
X /2 3,000

(8) Non-controlling interest carried forward (for proof only)

Ochil Ltd
At acquisition ((300,000 + 153,700 + 100,000) x 35%) 193,795
Share of post-acquisition profits ((669,400 553,700) x 35%) 40,495
234,290
Kennet Ltd
At acquisition ((280,000 + 82,500) x 20%) 72,500
Share of post-acquisition profits
((140,450 82,500) x 20%) 11,590
84,090
318,380

(9) Retained earnings carried forward (for proof only)



Cambus plc 461,200
Izat Ltd post acquisition (225,500 96,000) x 40% 51,800
Less: impairment Izat Ltd (5,000)
Ochill Ltd - post acquisition ((669,400 553,700) x 65%) (W3) 75,205
Kennet Ltd post acquisition (140,450 82,500) x 80%) (W4) 46,360
629,565

Most candidates produced a well laid out consolidated statement of profit or loss, and showed the split
between the profit attributable to the parent and to the non-controlling interest. This was backed up, on the
whole, by a well laid out consolidation schedule. Attempts at the consolidated statement of changes in equity
were generally less good, both in presentation and in content.

Many candidates produced a completely correct consolidation schedule, with figures for the provision for
unrealised profit and the additional depreciation, in the appropriate columns. The vast majority of candidates
correctly took only six-twelfths of the subsidiarys figures to their consolidation schedule. The most common
omission was not to calculate the parents share of the dividend from the subsidiary held throughout the year
and realise that it made up the whole of the parents investment income and that therefore the two figures
should be cancelled out. Other common errors were to include a provision for unrealised profit even where
the goods had been sold on to third parties and adjusting the parents costs (rather than the subsidiarys) for
the additional depreciation arising from the fair value adjustment.

The figure for share of profit of jointly controlled entity was more often than not correctly calculated, with the
most common error being to omit the impairment. A minority of candidates attempted to calculate some sort
of statement of financial position figure, which they then reduced by the impairment or describe the figure as
share of associate on the face of the consolidated statement of profit or loss.

Unfortunately answers to the second part of the question relating to the consolidated statement of changes in
equity extract were far weaker. Although most candidates did enter the relevant figures from the consolidated
statement of profit or loss many went no further than this. A significant number of candidates correctly
calculated the dividend paid by the subsidiary acquired during the year to the non-controlling interest, with the
figure omitted more often than errors were made.

Copyright ICAEW 2014. All rights reserved Page 18 of 19


Professional Level - Financial Accounting and Reporting March 2014

A figure for the non-controlling interest added on acquisition of the subsidiary was not seen very often, but
where it was included it was more often than not the correct figure. Only some candidates made some
attempt to calculate either non-controlling interest and retained earnings brought forward or carried forward
and earned some marks for this, but these figures were rarely completely correct, although candidates did
pick up some marks, most commonly for an attempt at a net assets table which they used to arrive at pos t
acquisition earnings.

No marks were given for a group structure diagram, since the percentage holdings were given in the
question, although many candidates did produce such a diagram. Some candidates were, however, careless
in their use of these percentages, the most common error being to use the parents percentages in
calculating the non-controlling interest.

Total possible marks 19


Maximum full marks 17

Copyright ICAEW 2014. All rights reserved Page 19 of 19


Financial Accounting and Reporting Professional Level June 2014

MARK PLAN AND EXAMINERS COMMENTARY

The marking plan set out below was that used to mark this question. Markers were encouraged to use
discretion and to award partial marks where a point was either not explained fully or made by implication.
More marks were available than could be awarded for each requirement. This allowed credit to be given for a
variety of valid points which were made by candidates.

Question 1

Total Marks: 31

General comments

Part (a) of this question tested the preparation of a statement of profit or loss, a statement of financial
position and a statement of changes in equity from a trial balance plus a number of adjustments.
Adjustments included the revaluation of property, plant and equipment (with a transfer between the
revaluation surplus and retained earnings), the receipt of a government grant, share issues and dividends,
a foreign exchange transaction and a prior period adjustment. Part (b) required an explanation and
quantification of the alternative treatment of the government grant. Part (c) tested the information needs of
users in the context of property, plant and equipment.

Tipperary plc
(a) Financial statements

Statement of profit or loss for the year ended 31 December 2013



Revenue (5,709,600 18,000 9,200 (SCE)) 5,682,400
Cost of sales (W1) (3,976,300)
Gross profit 1,706,100
Distribution costs (W1) (562,700)
Administrative expenses (W1) (1,097,000)
Profit from operations 46,400
Finance cost (100,000 x 5%) (5,000)
Profit before tax 41,400
Income tax expense (10,500)
Profit for the year 30,900

Statement of financial position as at 31 December 2013



Assets
Non-current assets
Property, plant and equipment (535,000 (W2) 21,250 17,500 (W1)) 496,250
Current assets
Inventories 192,300
Trade and other receivables 363,750
556,050
Total assets 1,052,300

Copyright ICAEW 2014. All rights reserved Page 1 of 16


Financial Accounting and Reporting Professional Level June 2014

Equity and liabilities


Equity
Ordinary share capital 230,000
Share premium 9,200
Revaluation surplus 222,500
Retained earnings 176,450
638,150
Non-current liabilities
Preference share capital (5% redeemable) 100,000
Deferred income (13,500 4,500) (W3) 9,000
109,000
Current liabilities
Bank overdraft 57,850
Trade and other payables (233,050 + 5,000 5,750 (W4)) 232,300
Deferred income (W3) 4,500
Taxation 10,500
305,150
Total equity and liabilities 1,052,300

Statement of changes in equity for the year ended 31 December 2013

Ordinary Share Revaluation Retained


share premium surplus earnings
capital

At 1 January 2013 184,000 - 209,000 256,450
Prior period error - - - (100,000)
Restated balance 184,000 - 209,000 156,450
Rights issue (230,000 5) (x 20p) 46,000 9,200 - -
Total comprehensive income for the - - 21,000 30,900
year (30,000 9,000 (W2))
Transfer to retained earnings - - (7,500) 7,500
(17,500 (W1) 10,000)
Ordinary dividend (184,000 x 10p) - - - (18,400)
At 31 December 2013 230,000 9,200 222,500 176,450

Workings

(1) Costs matrix Cost of Distrib Admin


sales costs expenses

Per TB 3,968,600 562,700 1,097,900
Downwards revaluation (W2) 11,000
Depreciation (85,000/4) ((450,000 100,000)/20) 21,250 17,500
Opening inventories (278,000 100,000) 178,000
Closing inventories (192,300)
Ordinary dividend (18,400)
Release of government grant (W3) (4,500)
Exchange gain (W4) (5,750)
3,976,300 562,700 1,097,000

Note: Marks were awarded if items were included in different line items than the above
provided that the heading used was appropriate.

Copyright ICAEW 2014. All rights reserved Page 2 of 16


Financial Accounting and Reporting Professional Level June 2014

(2) PPE and revaluation reserve


Land and Plant and
buildings equipment

Carrying amount at 1 January 2013 420,000 105,000
Valuation on 1 January 2013 450,000 85,000 535,000
Revaluation upwards/(downwards) 30,000 (20,000)
Revaluation surplus at 1 January 2013 9,000 209,000
To statement of profit or loss (11,000)

(3) Government grant

Grant received 18,000


Less: Released in year (18,000/4) (4,500)
Deferred income 13,500

(4) Forex transaction

Translation on 15 November 2013 (115,000 x 0.90) 103,500


Translation on 31 December 2013 (115,000 x 0.85) 97,750
Exchange gain 5,750

Candidates generally performed well on this part of the question. Presentation of the three statements was
usually of a sufficient standard to collect the available presentation marks with the presentation and indeed
completion of the statement of changes in equity (which candidates often find more challenging) of a
pleasing standard. Many candidates, however, failed to take their closing balances from this statement to
the equity section of their statement of financial position, thereby letting the statement of changes in equity
act as a working for those figures, and instead wasted time by producing other workings for these figures.
On occasion, the figures in these additional workings and those in the statement of changes in equity were
different. Only a minority of candidates failed to produce a statement of changes in equity.

The statement of profit or loss was generally well prepared and completed by the majority of candidates. It
was pleasing to see that the majority of candidates also prepared a cost matrix working. By preparing this
standard working candidates maximise the number of marks they will be awarded. Haphazard cost
workings, or brackets on the face of the statement of profit or loss (which were used by a minority of
candidates) often lost marks through missing narrative and no audit trail.

The majority of candidates correctly reduced opening inventory for the overvaluation in the cost matrix
however a minority instead adjusted closing inventory, which whilst having the same effect in the cost
matrix meant that closing inventory in the statement of financial position was incorrect. The overvaluation
was reflected in the statement of changes in equity by a significant number of candidates although
considerably less showed it in the correct place and then showed a sub-total with a restated balance.
Others made the adjustment in the statement of changes in equity but then failed to reduce opening
inventories in the cost matrix.

The government grant was generally dealt with correctly by the majority of candidates in the statement of
financial position, although less showed the correct figure as an adjustment to revenue and/or the release
of the grant in the year as a deduction from expenses (or as operating income). Similarly the exchange
gain was correctly calculated by almost all candidates and a significant number correctly reduced trade
and other payables, but again less went on to reflect the adjustment correctly in the statement of profit or
loss.

The property, plant and equipment working caused problems for a number of candidates, who omitted to
revise the balances in the trial balance for the revaluation. Many candidates, though not all, realised that
the downwards revaluation on the plant and equipment needed to be split between the statement of profit
or loss and the revaluation surplus (although having recognised this not all of these candidates then
followed this through to their cost matrix and the statement of changes in equity). Fewer still realised that,
since the latter transfer had wiped out that part of the revaluation surplus which related to plant and
equipment, the only transfer that could be made for the additional depreciation was that arising from the
revaluation of the buildings.

Copyright ICAEW 2014. All rights reserved Page 3 of 16


Financial Accounting and Reporting Professional Level June 2014

Other common errors included the following:


Showing the bank balance (which was an overdraft as shown by its inclusion on the credit side of
the trial balance in the question) in current assets instead of in current liabilities.
Not showing the net revaluation in the year and the profit for the year on a single line in the
statement of changes in equity, described as total comprehensive income.
Showing 230,000 as the opening balance on ordinary share capital in the statement of changes
in equity, as opposed to the closing balance (and then working backwards from that to adjust for
the rights issue).
Failing to split the deferred grant between current and non-current liabilities.
Being careless with the bracket convention in the cost matrix, for example showing closing
inventory or the foreign currency gain as increases rather than decreases in costs.
Depreciating the land as well as the buildings.

Total possible marks 24


Maximum full marks 22

(b) Alternative treatment of the government grant

The alternative method per IAS 20, Accounting for Government Grants is the netting-off approach. The
netting-off approach requires the grant to be deducted in arriving at the carrying amount of the asset.

Under the netting-off approach the grant of 18,000 would have been credited to the cost of plant and
machinery, giving an initial carrying amount of 12,000 (30,000 18,000), compared to an initial carrying
amount in Part (a) of 30,000.

Depreciation would then have been charged on that net amount, giving a charge for the year of 3,000
(12,000/4) compared to a figure in Part (a) of 7,500 (30,000/4).

The final carrying amount would then be 9,000 (12,000 3,000) compared to 22,500 (30,000 7,500).

This decrease of 4,500 in the depreciation charge reflects the fact that under the netting-off method the
grant is recognised in profit and loss over the life of the depreciable asset replacing the credit of 4,500
in Part (a) where the grant is released directly into cost of sales.

The reduction of 13,500 in the final carrying amount replaces the total deferred income on the
statement of financial position in Part (a) of 13,500.

The net effect on profit of the two methods is in fact the same as the different treatments are really a
difference of presentation.
Almost all candidates knew that the alternative treatment of the government grant was the netting off
method and correctly calculated the figures (cost, depreciation charge and carrying amount) on that
basis. Fewer candidates compared these figures to those they had calculated in Part (a). Most stated that
the figures had the same net effect but few described why this is in any detail.
Total possible marks 5
Maximum full marks 4

Copyright ICAEW 2014. All rights reserved Page 4 of 16


Financial Accounting and Reporting Professional Level June 2014

(c) How information re PPE meets needs of users


Financial position
The financial position of an entity is affected by the economic resources it controls, its financial structure,
its liquidity and solvency and its capacity to adapt to changes in the environment in which it operates.
Information about the total carrying amount of property, plant and equipment (PPE) as given on the face of
an entitys statement of financial position gives the user an indication of the resources the entity has
available to it in terms of tangible assets held for long-term use in the business. Revalued figures are more
relevant than cost.
That figure will then be broken down further in the notes to the financial statements.
This indicates the type of PPE held which may add further to an understanding of resource. This note also
shows the changes in financial position in the year.
For example, land and buildings might be held for investment potential as well as being used for
office/factory space. Plant will be used to generate future revenues. Equipment could be used for the
generation of future revenues or for the entitys own use, perhaps for administrative purposes.
The fact that the amount of leased assets forming part of the PPE figure is disclosed shows that these
assets have a future cost in terms of lease payments affecting the liquidity and solvency of the entity.
The capital commitments note showing the future purchases of PPE to which the entity is committed
indicates a requirement for future finance.
The accounting policy note shows the valuation model used and depreciation methods, which allow
comparison to other entities.
Financial performance
Information about financial performance, in particular profitability, is needed in order to assess potential
changes in the economic resources that the entity is likely to control in the future.
Disclosure of the annual depreciation charge shows the cost of using the assets.
Disclosure of significant gains/losses on disposal could indicate problems with the depreciation method or
where value is greater than carrying amount.
Impairment losses may indicate underlying issues, such as underprovision of depreciation, or a downturn
in a particular market sector (which might affect future performance).
Changes in financial position
Changes in financial position are shown in the statement of cash flows. This allows users to assess the
ability of the entity to generate cash and its need to use what is generated.
Users will be able to see, via the statement of cash flows, PPE purchased during the year and cash
inflows from PPE disposed of. If little PPE is purchased and much disposed of then the user may be
concerned about the future of the entity.
This part was dealt with much less well. The majority of candidates clearly struggled with this requirement,
with a significant number gaining either one or zero marks (in spite of the fact that there is a very similar
question in the revision question bank). A significant number of candidates simply discussed the
qualitative characteristics in respect of property, plant and equipment, which was not asked for and gained
no marks. Others made a series of random comments, with no attempt to link these to financial
position, financial performance, or changes in financial position as represented, per the Conceptual
Framework, by the statement of financial position, the statement of profit or loss and the statement of cash
flows.
Candidates must read requirements carefully and be mindful that unless the requirement is addressed
they are wasting their time writing about something that they think might be relevant. It was not uncommon
to see a whole page of writing gaining zero marks. Those candidates who scored the highest number of
marks set up three headings (ie financial position, financial performance and changes in financial
position) and made pertinent comments under each. There was, however, a common misconception,
even amongst these candidates, that changes in financial position are shown by the statement of changes
in equity. Another common error was to say that the statement of financial position showed cost less
accumulated depreciation thereby showing what property, plant and equipment is worth. Others referred
to the performance of the asset, as opposed to the financial performance of the reporting entity.
Total possible marks 9
Maximum full marks 5

Copyright ICAEW 2014. All rights reserved Page 5 of 16


Financial Accounting and Reporting Professional Level June 2014

Question 2

Total Marks: 29

General comments

Part (a) of this question required candidates to explain the financial reporting treatment of four accounting
issues, given in the scenario. The issues covered a lease of land and buildings, decommissioning costs,
sale and repurchase and an event after the reporting period. Part (b) required the calculation of revised
earnings and basic EPS, having adjusted for errors made by the company as discussed in Part (a), plus
an explanation of why the managing directors calculation was incorrect. Part (c) required an explanation
of the ethical issues arising from the scenario and the action to be taken.

Limerick plc
(a) IFRS accounting treatment

(1) Lease of land and buildings

IAS 17, Leases, requires that the land and buildings elements of a single lease are considered separately
in order to classify as a finance or an operating lease. A lease is classified as a finance lease if it transfers
substantially all the risks and rewards incidental to the ownership of an asset.

The managing director (MD) is correct that land has an indefinite economic life. However, given the fact
that ownership does pass, this lease is relatively long, and in the case of the buildings is for almost all of
the assets useful life, and in both cases the present value of the minimum lease payments amount to
substantially all of the fair value of the asset, the whole lease should be treated as a finance lease.

As the MD has treated this as an operating lease then the payment of 120,000 made on 1 January 2013
will have been debited to expenses. This entry will need to be reversed. Per IAS 17, the finance lease
should be capitalised at the lower of the fair value of 1.3 million and the present value of the minimum
lease payments of 1,290,835 (1,183,265 + 107,570). The table below illustrates the entries which should
have been made.

Year ended B/f Payment Capital Interest at C/f


10% pa

31 Dec 2013 1,290,835 (120,000) 1,170,835 117,084 1,287,919
31 Dec 2014 1,287,919 (120,000) 1,167,919

A finance cost of 117,084 should be charged in the statement of profit or loss.

The lease liability at 31 December 2013 is therefore 1,167,919 non-current and 120,000 current.

Because legal title will pass, the building should be depreciated over its useful life of 42 years, giving a
depreciation charge for 2013 of 28,173 (1,183,265 42). The land is not depreciated. The carrying
amount of the land and buildings in the statements of financial position at as 31 December 2013 will
therefore be 1,262,662 (1,290,835 28,173).

(2) Decommissioning costs

Per IAS 37, Provisions, Contingent Liabilities and Contingent Assets, a provision should be recognised
where:
there is a present obligation as a result of a past event
an outflow of resources is probable, and
the amount can be estimated reliably.

The decommissioning costs meet these recognition criteria as:


there is an obligation to decommission (it was a condition of the sale),
it arose from a past event (the purchase of the plant), and
there is a reliably estimated outflow of resources (the 50,000 that will be paid out).

Copyright ICAEW 2014. All rights reserved Page 6 of 16


Financial Accounting and Reporting Professional Level June 2014

When the plant was purchased on 1 January 2013, a provision should therefore have been made for the
5
discounted costs of decommissioning the plant in five years time, measured as 50,000 x 1/(1.07) =
35,649, adding this amount to the cost of the asset. This would also have had the effect of increasing the
depreciation charge for 2013 on the asset by 7,130 (35,649 5).
A finance cost of 2,495 (35,649 x 7%) should be charged in the year ended 31 December 2013 to reflect
the unwinding of the discount and the provision should be increased by the same amount. In the
statement of financial position as at 31 December 2013 the provision will be shown as a non-current
liability of 38,144 (35,649 + 2,495).
(3) Sale and repurchase
This is a sale and repurchase agreement. Per IAS 18, Revenue, the terms and conditions of the sale need
to be considered to determine whether or not there is a sale in substance. Where legal title has been
transferred, but the risks and rewards of ownership (here the right to build on the land and potential gains
and losses in market values) have been retained by the seller the transaction is treated as a financing
arrangement. The fact that Limerick plc is likely to repurchase the land and at a price which is below the
current market price adds weight to this conclusion.
The profit on the sale of the land of 250,000 (750,000 500,000) should therefore be derecognised. A
loan of 750,000 and accrued finance cost of 52,500 (750,000 x 7%) should be recognised.

(4) Event after the reporting period


Per IAS 10, Events After the Reporting Period, the determination of the court case is an adjusting event as
it provides evidence of conditions that existed at the end of the reporting period (ie of the court case that
was already in progress). The financial statements should therefore be adjusted to include an accrual for
the total due of 125,000 and the note re the contingent liability removed. There is no specific requirement
to disclose the adjusting event.
Candidates generally performed well overall on this part of the question, although some issues were dealt
with better than others. The majority of candidates responded to all four issues and provided both
explanations and supporting calculations.
Issue (1): Most candidates made a good attempt at the lease of land and buildings, although there was
clearly some confusion on this topic. Candidates generally understood that to assess which lease is
present for land and buildings they needed to make the assessment separately. However, many
candidates incorrectly identified the land as being an operating lease, even where they had noted that
legal title passed. Where candidates did realise that both elements were finance leases they often split
them out and produced two lease tables, which was unnecessary. However, this sometimes followed on
from a statement that the two leases were to be treated separately, as opposed to considered
separately when classifying them and this may have been where the confusion arose.
Many answers lacked consistency. For example, the land would be identified as being an operating lease
but then the full lease payment was added back and used in the finance lease table. Land was identified
as having an indefinite life but then the total, including land, was used for the depreciation working. The
finance lease table itself was generally correctly done, although the opening figure was often incorrect and
a minority of candidates treated the payments as made in arrears rather than in advance. The majority of
candidates incorrectly identified that the depreciation on the building should have been over 40 years
rather than 42 years, even where they had identified that ownership passed. Almost all candidates stated
that the closing liability needed to be split into current and non-current but a significant number gave an
incorrect split of the total figure.
Issue (2): Most candidates correctly identified that the decommissioning costs should have been added to
the assets carrying amount, but fewer identified that a provision should be set up to complete the double
entry. Of those that did, only a minority set out the IAS 37 conditions for the recognition of a provision and
fewer still applied these conditions to the scenario. The majority of candidates correctly identified that the
amount should be discounted although a minority used the incorrect discount rate. It was pleasing to see
that the majority of candidates also correctly depreciated the revised carrying amount of the asset and
realised that they needed to do some unwinding of the provision (even where they hadnt identified that a
provision should be recognised). The main concern with this issue was a lack of supporting narrative with
many answers containing little more than a series of numbers.
Issue (3): The answers for the sale and repurchase were mixed with the majority of candidates concluding
that this was a sale and leaseback rather than a sale and repurchase. However, a number of marks were
still available for a good discussion centred around the principles of substance over form and the non-
transference of risks and rewards.

Copyright ICAEW 2014. All rights reserved Page 7 of 16


Financial Accounting and Reporting Professional Level June 2014

Many calculated an accrued finance cost based on the repurchase price less the sale price, failing to
recognise that the repurchase was in two years time, not one.
Issue (4): The final issue concerned an adjusting event after the reporting period, with most candidates
correctly concluding that a provision needed to be made. However, around half of the candidates simply
seemed to miss that this should have been a discussion about events after the reporting date and new
information concerning a condition that existed at that date, rather than a simple assessment of a
provision. Therefore a number of easy marks were lost through lack of narrative.
Total possible marks 26
Maximum full marks 18
(b) Revised earnings and basic EPS

Earnings per draft financial statements 500,500
Add back: Operating lease rental (1) 120,000
Less: Finance cost re leased asset (1) (117,084)
Depreciation on leased asset (1) (28,173)
Depreciation on decommissioning costs (2) (7,130)
Finance cost re decommissioning costs (2) (2,495)
Profit on sale of land (3) (250,000)
Finance cost re land sold (3) (52,500)
Damages/costs in court case (125,000)
Revised earnings figure 38,118
Weighted average number of ordinary shares:
Number of shares Weighted average
1 January 2013 100,000 x 6/12 x 5/4 62,500
Bonus issue (1 for 4) 25,000
1 October 2013 125,000 x 3/12 31,250
Issue at full market price 80,000 93,750
31 December 2013 205,000 x 3/12 51,250
145,000
EPS (38,118 145,000) 26.3p
Per IAS 33, Earnings per Share, the calculation of basic earnings per share should be based on the
weighted average number of ordinary shares outstanding during the period. So where there have been
share issues during the period, as here, it is incorrect to use the opening (or indeed the closing) number of
shares. Where shares have been issued at market price, those shares should only be included in the
shares in issue for part of the period ie the period in which the proceeds from that share issue have
generated earnings. Conversely, because bonus shares have not generated any cash/earnings they are
dealt with in the calculation by IAS 33 by assuming that the shares have always been in issue.
The majority of candidates made a good attempt at adjusting the earnings given in the question by their
figures calculated in Part (a). It was common to see this as the first page of the answer to Question 2,
showing that candidates had heeded advice from the examining team about building up this part of an
answer as they went along. A few candidates, however, disadvantaged themselves by combining various
figures from Part (a) into a net adjustment for each issue all well and good if an audit trail was
provided, but if not marks could well have been lost.
The calculation of the weighted average number of shares was, however, disappointing, compared to
when an EPS calculation was set in a previous paper. A significant number of candidates were unable to
correctly calculate this figure. The most common errors were to use the wrong number of months or to
incorrectly adjust for the bonus issue.
The final element of this part of the question was to explain why the managing director was incorrect in
basing his EPS calculation on the opening number of ordinary shares. It was disappointing that few
candidates went beyond saying that this was wrong and that the managing director should have used a
weighted average number of shares. Very few made any link between the issue of shares and the
earnings those shares might or might not generate depending on whether the issue was for cash or not.
Total possible marks 8
Maximum full marks 6

Copyright ICAEW 2014. All rights reserved Page 8 of 16


Financial Accounting and Reporting Professional Level June 2014

(c) Ethical issues

The MD has given plausible reasons for the accounting treatment of the issues identified. Each issue is
technical in nature and the treatments may appear reasonable to a business manager with a general
appreciation of accounting principles but not a detailed awareness of current reporting standards.

However, the MD appears to be applying pressure to have his treatments confirmed by offering incentives
for compliance with his wishes (intimidation threat). I should not be swayed by the thought of being made
the new FD (self-interest threat). Furthermore, all the treatments adopted by him have the effect of
increasing the EPS figure to above that of the previous year, which is said to be a key criteria for the
board. Once the correct treatments are adopted basic EPS in fact falls back to below the level of the
previous year to 26.3p compared to 70.3p. Even if last years EPS is restated for the bonus issue to 56.2p
(70.3p x 4/5) this is still a fall in EPS not the significant improvement that the board is looking for.

The finance director (FD) left under suspicious circumstances, which need to be confirmed. It may be that
he too was put under pressure to adopt incorrect accounting treatments and found the situation untenable.

IFRS is quite clear on the appropriate treatment of these four issues. There is little, if any, choice or
judgement on any of the matters. I should not give in to the MDs wishes or prepare financial statements
that are contrary to IFRS.

I should apply the ICAEW Code of Ethics, with the following programme of actions:
Explain matters to the MD with supporting evidence so that the matters can be corroborated.
If resolution cannot be achieved, discuss the matters with the other directors to explain the
situation and obtain support. Consider also discussing the issues with the external auditors/audit
committee.
Obtain advice from the ICAEW helpline or local members responsible for ethics.

During the resolution process it would be useful to keep a written record of all discussions, who else was
involved and the decisions made.

Almost all candidates made a reasonable attempt at this part of the question, with a good number
obtaining three or four marks, although five marks was rare. Candidates should remember that to gain the
most marks their answer should be tailored to the question scenario. Most candidates correctly identified
that the departure of the finance director was suspicious and that there was a self-interest threat and an
intimidation threat for the financial controller. They then went on to explain how these threats arose and to
suggest appropriate courses of action. A minority of candidates answered as if this was a problem facing
an external auditor, not an accountant in a company. Others were concerned about the managing
directors lack of technical competence and adherence to the Code of Ethics when he was not a qualified
accountant. As ever, a few felt there were money laundering issues at play.
Total possible marks 8
Maximum full marks 5

Copyright ICAEW 2014. All rights reserved Page 9 of 16


Financial Accounting and Reporting Professional Level June 2014

Question 3
Total Marks: 19

General comments

This question required the preparation of a consolidated statement of financial position. The group had two
subsidiaries, one of which was acquired at the start of the year. The question featured contingent
consideration, both goodwill and a gain on bargain purchase, fair value adjustments on acquisition, and
inter-company trading between the two subsidiaries, with adjustments needed to reconcile the intra-group
balances and deal with goods in transit.

Laois plc

Consolidated statement of financial position as at 31 December 2013

Assets
Non-current assets
Property, plant and equipment (2,687,000 + 2,196,000 + 5,634,800
591,800 + (200,000 40,000 (W1))
Goodwill (W3) 168,000
5,802,800
Current assets
Inventories (193,200 + 53,700 + 159,000 5,000 (W7) 412,900
+ 12,000 (W7))
Trade and other receivables (288,000 + 92,300 + 522,300
207,000 50,000 15,000 (W7))
Cash and cash equivalents (15,800 + 12,400 + 1,100) 29,300
964,500
Total assets 6,767,300
Equity and liabilities
Equity
Ordinary share capital 2,000,000
Share premium account 750,000
Retained earnings (W6) 1,992,480
Attributable to the equity holders of Laois plc 4,742,480
Non-controlling interest (W5) 649,520
5,392,000
Current liabilities
Trade and other payables (398,600 + 220,800 + 1,005,800
436,400 50,000)
Deferred consideration 104,000
Taxation (150,000 + 105,000 + 10,500) 265,500
1,375,300
Total equity and liabilities 6,767,300

Workings

(1) Net assets Carlow Ltd


Year end Acquisition Post acq

Share capital 650,000 650,000
Share premium 300,000 300,000
Retained earnings 1,078,600 592,000
FV adj 200,000 200,000
Deprec on FV adj (200,000/25 years x 5) (40,000)
PURP (5,000 + 3,000) (W7) (8,000)
2,180,600 1,742,000 438,600

Copyright ICAEW 2014. All rights reserved Page 10 of 16


Financial Accounting and Reporting Professional Level June 2014

(2) Net assets Kerry Ltd


Year end Acquisition Post acq

Share capital 360,000 360,000
Retained earnings 176,000 240,000
Goodwill adj (24,000) (30,000)
512,000 570,000 (58,000)
(3) Goodwill Carlow Ltd

Consideration transferred 1,560,000
Non-controlling interest at acquisition at fair value 350,000
Less: Net assets at acquisition (W1) (1,742,000)
168,000
(4) Gain on bargain purchase Kerry Ltd

Consideration transferred
Cash 200,000
Deferred consideration at present value (104,000/1.04) 100,000
Non-controlling interest at acquisition at fair value 235,000
Less: Net assets at acquisition (W2) (570,000)
(35,000)
(5) Non-controlling interest

Carlow Ltd
Fair value at acquisition 350,000
Share of post-acquisition reserves (438,600 (W1) x 20%) 87,720
Kerry Ltd 437,720
Fair value at acquisition 235,000
Share of post-acquisition reserves ((58,000) (W2) x 40%) (23,200)
211,800
649,520
(6) Retained earnings

Laois plc 1,645,400
Less: Unwinding of discount (104,000 100,000 (4,000)
Carlow Ltd (80% x 438,600 (W1)) 350,880
Kerry Ltd (60% x (58,000) (W2)) (34,800)
Gain on bargain purchase (W4) 35,000
1,992,480

(7) PURP Sales in year Goods in


transit
%
SP 125 50,000 15,000
Cost (100) (40,000) (12,000)
GP 25 10,000 3,000
x 5,000
Generally candidates performed well on this question with a reasonable number achieving full marks.
Nearly all candidates produced the expected standard workings (which are to be strongly encouraged)
and a significant number arrived at the correct figures for the goodwill and gain on bargain purchase. Most
then correctly took the gain to retained earnings although a minority netted it off against the goodwill
figure. The fair value adjustment to property was well dealt with (although a number used the incorrect
number of years when calculating the depreciation adjustment) as was the calculation of the unrealised
profit on the goods held at the year end. Most candidates also correctly followed these adjustments
through to property, plant and equipment and inventory respectively.

However, the adjustments for the goods in transit were not well dealt with and few candidates dealt
correctly with all aspects of this (although many did at least calculate the adjustment for unrealised profit).
A good number of candidates failed to increase inventories by the cost of the goods in transit between the
two subsidiaries at the year end. Others failed to reduce trade and other receivables by the selling price of
these goods, to reflect the fact that the receivable for the goods in transit had already been accounted for
in the selling subsidiarys own financial statements.

Copyright ICAEW 2014. All rights reserved Page 11 of 16


Financial Accounting and Reporting Professional Level June 2014

The goodwill held within the subsidiary also caused problems and a considerable number of students
completely ignored it in the net assets working, with a minority adding rather than deducting it. Many also
did not understand the impact of the impairment that had been recognised within the subsidiary in relation
to this goodwill and went on to make incorrect adjustments to the discount on acquisition calculated for
this subsidiary and/or to retained earnings.

The aspect of the question that was least well dealt with was the deferred consideration. Although virtually
all candidates used the correct figure to add to consideration very few then charged the unwinding of the
discount to retained earnings. Even fewer showed anything in liabilities and even when they did it was
often the wrong number.

Other common errors included the following:


Making adjustments for unrealised profits in the wrong place (ie against the net assets of the
subsidiary buying the inventory or in retained earnings or in both).
Omitting the balance on the share premium account from the net assets table.
Entering figures such as the fair value adjustment in one column of the net assets table rather
than in both.
Attempting to calculate the non-controlling interest by taking a percentage of closing net assets
(which would work for the proportionate method) when this is clearly wrong if the fair value method
is being used.
Adjusting trade receivables for the cost of the inventory in transit rather than for the sales price.
Adjusting trade payables for goods in transit when no liability had been recognised.
Adding (rather than deducting) post-acquisition losses to the non-controlling interest and retained
earnings workings.

It is disappointing that a good number of candidates still lose marks for failing to show an audit trail,
particularly for the share of post- acquisition profit or loss to be taken to the non-controlling interest and
retained earnings workings. To ensure they get the relevant marks candidates must show the figure (to
check that the correct movement in the net assets working has been picked up) multiplied by the
appropriate percentage. Many candidates actually waste time by writing out, for example, NCI share of
post- acquisition profit when it would be faster and clearer to show, for example, 58,000 x 20%.

Total possible marks 20


Maximum full marks 19

Copyright ICAEW 2014. All rights reserved Page 12 of 16


Financial Accounting and Reporting Professional Level June 2014

Question 4

Total Marks: 21

General comments

This question required the redrafting of extracts from the consolidated financial statements. Matters to be
adjusted for were the disposal of a subsidiary, the setting up of a joint venture and a development project.
Part (b) required an explanation of the differences between IFRS and UK GAAP in respect of these
issues. Part (c) required a calculation of distributable profits and explanation thereof.

Kildare plc
(a) Extracts from the consolidated financial statements for the year ended 31 December 2013

Consolidated statement of profit or loss (extracts)

Profit attributable to
Owners of Kildare plc (W5) 899,590
Non-controlling interest (256,700 + (37,500 (W5) x 30%)) 267,950

Consolidated statement of financial position (extracts)


Non-current assets
Property, plant and equipment (2,752,100 + 90,000 15,000 (W4)) 2,827,100
Intangibles (356,000 + 115,000 + 15,000* (W4)) 486,000
Investment in joint venture (W2) 110,480

Consolidated statement of cash flows (extract)

Net cash from investing activities ( 50,600 + (300,000 1,500)) (115,000 + 52,900
90,000) (W4) + 10,000)

*Note: Credit was also given if the depreciation on the equipment (W4) was expensed, rather
than being recapitalised.

Workings

(1) Profit on disposal of subsidiary



Sale proceeds 300,000
Less: Carrying amount of goodwill at date of disposal
Consideration transferred 225,000
NCI at acquisition (214,900 x 30%) 64,470
Less: Net assets at acquisition (50,000 + 158,900 + (214,900)
(35,000 29,000))
Goodwill at acquisition 74,570
Less: Impairments to date (40,000)
(34,570)
Less: Carrying amount of net assets at date of disposal (287,500)
(50,000 + (275,000 (75,000 x 6/12)))
Add back: NCI at date of disposal (287,500 x 30%) 86,250
Profit on disposal 64,180

(2) Investment in joint venture



Cost of investment 80,000
Share of post-acquisition increase in net assets ((48,400 + 52,800) x 40%) 40,480
Less: Dividend received (10,000)
110,480

Copyright ICAEW 2014. All rights reserved Page 13 of 16


Financial Accounting and Reporting Professional Level June 2014

(3) Share of profits of joint venture



Share of JVs PAT (40% x 48,400) 19,360
Less: Share of PURP (40% x (120,000 x 25% x )) (6,000)
13,360
(4) R&D expenditure

Per Suspense account 275,000
Less: Carried forward as Intangibles:
Qualifying development costs 110,000
Patent registration costs 5,000
(115,000)
160,000
Carried forward as PPE (90,000)
Written off (initial research costs + evaluation of research findings) () 70,000

Depreciation on equipment (90,000/3 x 6/12) 15,000

(5) Profit attributable to owners



Per draft 865,800
Profit on disposal of subsidiary (W1) 64,180
Share of profit of subsidiary (75,000 x 6/12 = 37,500 x 70%) 26,250
Share of profit of JV 13,360
Research costs etc (W4) (70,000)
899,590

Answers to this part were generally disappointing. Although it was the most challenging question on the
paper there were many easy marks available for basic consolidation workings (such as the disposal of the
subsidiary) and for adjustments to property, plant and equipment and intangible assets. Answers were
generally difficult to follow often with lengthy and unnecessary workings.

Fewer candidates than normal managed to calculate the profit on disposal correctly although it was more
common to see the correct figure for goodwill. Many candidates produced one combined and somewhat
muddled working here which often resulted in the impairment to goodwill decreasing rather than
increasing the profit on disposal. Furthermore, many candidates clearly did not understand that the whole
of the profit on disposal should have been allocated to the owners, but that the profit of the subsidiary for
the year up to disposal should have been split between the owners and the non-controlling interest.

It was extremely disappointing to see how few candidates realised that equipment used for research and
development should be included within property, plant and equipment rather than in intangible assets.
Candidates also struggled to decide how much of the research and development costs should be
capitalised and how much should be expensed. These were very simple decisions that should have been
quickly made and the appropriate adjustments taken directly to the extracts. Nearly all candidates
calculated an amortisation charge for the capitalised development costs even though the new product was
still in the development stage.

The joint venture also caused problems, in particular the calculation and treatment of the provision for
unrealised profits. Those candidates who did attempt to calculate the latter often failed to multiply it by the
relevant percentage.

Many candidates did adjust the cash used in investing activities for the proceeds of the disposal (net of the
cash held by the subsidiary) and for the dividend received from the associate. However, some also used
the former (net) figure when calculating the profit on disposal of the subsidiary. It was, however, rare to
see the cash used in investing activities adjusted for the amounts spent on intangibles and property, plant
and equipment.

Copyright ICAEW 2014. All rights reserved Page 14 of 16


Financial Accounting and Reporting Professional Level June 2014

Other specific errors not noted above included the following:


Omitting to time apportion the current year depreciation charge.
Omitting to time apportion the post-acquisition profits to be added to the non-controlling interest
and owners share of profits (and/or failing to multiply them by the relevant percentage).
Assuming the shares had a nominal value of 1 rather than the 50p given in the question.
Ignoring the fair value adjustment for inventory or including it in net assets at the year end (as well
as or instead of at acquisition), even though it had been sold.
Attempting to adjust the year end figure for intangibles by the cumulative impairment losses of
40,000 in respect of the disposed of subsidiary, despite the fact that it had been disposed of by
this stage (and it was stated in any case in the question that no amounts in respect of this
subsidiary had been consolidated).
Treating the joint venture as if it had been bought on the first day of the current year rather than of
the previous year.
Taking the wrong figure for the cost of the joint venture (the most common error being to take the
whole of the joint ventures share capital rather than the 40% which the parent company had
purchased).
Taking the total post-acquisition profits of the joint venture to investment in joint venture rather
than the appropriate percentage (and/or only taking one years profits).
Not showing the investment in the joint venture within the non-current assets section of the
extracts.

Total possible marks 16


Maximum full marks 12

(b) IFRS v UK GAAP differences

Under IAS 38, Intangibles, development expenditure must be capitalised where the relevant criteria are
met. Under UK GAAP (SSAP 13) the capitalisation of development expenditure which meets certain
criteria is optional.

The development expenditure recognition criteria of SSAP 13 include a requirement to have or a


reasonable expectation of future benefits. IAS 38 is more stringent as the requirement is to demonstrate
future benefits.

UK GAAP (FRS 10) would have required the goodwill arising in the business combination with Sligo Ltd to
be amortised over its finite useful life. Under IAS 38 goodwill is tested annually for impairment.

Under UK GAAP (FRS 6) minority interest (the non-controlling interest) is always measured at its share of
net assets. IFRS 3 allows non-controlling interest to be measured at fair value (the fair value method) or at
its share of net assets (the proportionate method as used here).

UK GAAP (FRS 9) requires the use of the gross equity method for joint ventures. IAS 28, Investments in
Associates and Joint Ventures, requires the use of the equity method.

The gross equity method is the same as the equity method except that disclosure is required of the
following figure:
in the profit and loss account the investors share of the turnover of its joint venture
in the balance sheet the investors share of the gross assets and liabilities underlying the net
equity amount.
Answers to this part were mixed but most candidates did manage to pick up at least a couple of marks
although very few gained full marks. As in previous sittings the main problem is that candidates include
differences that are not relevant to the actual issues given, such as discussing the treatment of negative
goodwill when there was no negative goodwill in this scenario.

Total possible marks 6


Maximum full marks 4

Copyright ICAEW 2014. All rights reserved Page 15 of 16


Financial Accounting and Reporting Professional Level June 2014

(c) Distributable profits and explanation

For entities within a group, the calculation of distributable profits must be made for each entity separately,
not for the consolidated group. Therefore Kildare plcs distributable profits are those distributable by the
parent company only.

The basic rule is that distributable profits are measured as accumulated realised profits less accumulated
realised losses. In the case of listed companies, the amount of distributable profits is further reduced by
any excess of unrealised losses over unrealised profits. In the case of Kildare plc, insufficient information
is available in the scenario to identify any such excess. Assuming that no such excess exist, then
distributable profits are calculated as below:
The disposal of the shares in Sligo Ltd affect Kildare plcs parent company figures by the (as yet
unrecorded) parent company profit. This profit is the difference between the cost of the shares
(225,000) and the sale proceeds (300,000), increasing Kildare plcs single entity retained
earnings by 75,000.
The share of profits in the joint venture only affects the consolidated retained earnings, but Kildare
plcs own financial statements would include the dividend from Mayo Ltd of 10,000. Since this
has been credited to a suspense account, Kildare plcs single entity retained earnings need
increasing by 10,000.
The research and development costs were spent by Kildare plc and therefore any adjustments in
respect of this affect its individual financial statements and hence distributable profits. Kildare plcs
single entity retained earnings need reducing by 70,000 (W4).

As there is no further information on the reserve balances which form part of equity, the distributable
profits of Kildare plc are therefore:

109,700 + 75,000 + 10,000 70,000 = 124,700

Note: Credit was also given, where the depreciation on the equipment in Part (a) had been expensed, for
discussing the impact of this on distributable profits.

It was clear that very few candidates had spent any time on understanding distributable profits. A
significant number of candidates did not attempt this part of the question and even when they did make
some attempt, often achieved no marks at all. Very few candidates knew even the most basic points (such
as realised profits less realised losses) or that distributable profits are based on the individual companys
financial statements.

Many candidates who did attempt this part of the question wasted time by simply copying out adjustments
made in Part (a) of the question that related to the consolidated financial statements

Total possible marks 7


Maximum full marks 5

Copyright ICAEW 2014. All rights reserved Page 16 of 16


Professional Level Financial Accounting and Reporting September 2014

MARK PLAN AND EXAMINERS COMMENTARY

The mark plan set out below was that used to mark these questions. Markers are encouraged to use discretion
and to award partial marks where a point was either not explained fully or made by implication. More marks are
available than could be awarded for each requirement, where indicated. This allows credit to be given for a
variety of valid points, which are made by candidates.

Question 1

Overall marks for this question can be analysed as follows: Total: 20

General comments
This question required the preparation of the statement of profit or loss and statement of financial position.
A number of adjustments were required to be made, including depreciation, borrowing costs, an inventory
write down, a bonus issue and a finance lease.

Barchetta Ltd Statement of financial position as at 31 March 2014


ASSETS
Non-current assets
Property, plant and equipment (2,087,050 (W4) + 41,570 (W5)) 2,128,620

Current assets
Inventories (W2) 31,850
Trade and other receivables 85,400
Cash and cash equivalents 6,800
124,050
Total assets 2,252,670

Equity
Ordinary share capital (400,000 x 6/5) 480,000
Retained earnings (W7) 481,909
Equity 961,909

Non-current liabilities
Bank loan 1,100,000
Finance lease (W6) 10,872
1,110,872

Current liabilities
Trade and other payables 93,100
Finance lease (13,161 10,872) 2,289
Taxation 84,500
179,889

Total equity and liabilities 2,252,670

Copyright ICAEW 2014. All rights reserved. Page 1 of 15


Professional Level Financial Accounting and Reporting September 2014

Barchetta Ltd Statement of profit or loss for the year ended 31 March 2014


Revenue 4,521,000
Cost of sales (W1) (3,409,730)

Gross profit 1,111,270


Administrative expenses (804,700)

Operating profit 306,570


Finance charges (83,060 27,600 (W3) + 1,071 (W6)) (56,531)
Profit before tax 250,039
Income taxation (84,500)

Net profit for the period 165,539

W1 Expenses
Cost of sales
Trial balance 3,379,100
Opening inventories 27,640
Closing inventories (W2) (31,850)
Depreciation charge building (W4) 25,600
Depreciation charge plant & machinery (W5) 12,450
Reverse lease payment (3,210)
3,409,730
W2 Inventory

Closing inventory 35,850
Inventory write down (200 x (315 (320 25)) (4,000)
At 31 March 2014 31,850

W3 Borrowing costs

Weighted average cost of loans = (600,000 x 6.4%) + (500,000 x 7.5%) = 6.9%


1,100,000

Borrowing costs to be capitalised = (300,000 + (400,000 x 3/12)) x 6.9% = 27,600

W4 PPE Building

Cost b/f 2,230,000
Depreciation charge for year
((2,230,0000 250,000 700,000)/ 50 yrs) (25,600)
Accumulated depreciation (144,950)
Borrowing costs (W3) 27,600
Carrying amount at 31 March 2014 2,087,050

W5 PPE Plant and equipment



Cost b/f 60,500
Depreciation charge for the year
(60,500 22,000) / 5yrs) (7,700)
Plant different useful life
((22,000 x 4/5) / 8yrs) (2,200)
Leased equipment (15,300 / 6yrs) (2,550)

(12,450)
Accumulated depreciation b/f (21,780)
Leased equipment 15,300
Carrying amount at 31 March 2014 41,570

Copyright ICAEW 2014. All rights reserved. Page 2 of 15


Professional Level Financial Accounting and Reporting September 2014

W6 Finance lease

Opening Interest Lease Closing


balance @ 7% payment balance

31 March 2014 15,300 1,071 (3,210) 13,161
31 March 2015 13,161 921 (3,210) 10,872

W7 Retained earnings

Per draft 321,370
Add: profit and loss in year 165,539
Bonus issue (75,000 (400,000 / 5)) (5,000)
481,909

Presentation of the statement of profit or loss and statement of financial position was generally very good with
most candidates achieving the maximum presentation marks available.

A significant majority of candidates arrived at completely correct figures in respect of closing inventories, the
finance lease, and the bonus issue. It was rare to see a completely correct figure for property, plant and
equipment, although this was usually due to errors on land and buildings as opposed to plant and machinery.
It was less common to see the correct figure for capitalised interest, although almost all candidates made a
good attempt at this calculation, with nearly all candidates arriving at the correct effective interest rate.
However, most candidates went on to apply that rate to the whole 1,100,000 borrowed or to the whole
700,000 paid to the contractor, instead of taking into account that 400,000 of that amount had only been
paid four months before the year end.

Most candidates provided relatively clear workings for their property, plant and equipment figure. The most
common error was to not remove the amounts paid to the contractor before calculating the depreciation on
buildings.

The vast majority of candidates used a costs matrix to calculate the figure for cost of sales, and many
correctly allocated all of the costs to this category. The most common errors were to fail to deduct the finance
lease payment incorrectly included (even where the candidate had used this figure in their finance lease
working) and/or not to include all of the depreciation figures to cost of sales. This is despite the question itself
specifying that depreciation should be presented under this expense heading. Weaker candidates often got
themselves in a muddle in this working, mixing up their bracket convention.

Total possible marks 22


Maximum full marks 20

Copyright ICAEW 2014. All rights reserved. Page 3 of 15


Professional Level Financial Accounting and Reporting September 2014

Question 2

Overall marks for this question can be analysed as follows: Total: 34

General comments
Part (a) of this question required candidates to explain the financial reporting treatment of five accounting
issues, given in the scenario. The five issues covered the irredeemable preference shares, research and
development, an onerous contract, revenue recognition and a related party transaction.
Part (b) required candidates to recalculate profit before tax, equity and liabilities for the adjustments
needed as a result of their answer to Part (a).
Part (c) required candidates to discuss and compare the accrual basis of accounting with cash accounting
with reference to revenue recognition and the Frameworks recognition criteria.

Impreza plc
(a) IFRS accounting treatment

(1) Irredeemable preference shares

The irredeemable preference shares provide the investor with the right to receive a fixed (5% pa) amount
of annual dividend out of Impreza plcs profit for the period on a mandatory basis. If the annual dividend is
not paid then it is rolled up into the following years payment as the dividends are cumulative in nature.

Under IAS 32 Financial Instruments: Presentation, these shares should be classified as financial liabilities
as there is a contractual obligation to deliver cash. The preference shares should therefore be accounted
for at amortised cost using the effective interest rate which is equivalent to the annual dividend rate of 5%
as they are not redeemable. This reflects the substance of the share issue.

450,000 should therefore be recognised as part of non-current liabilities and removed from equity and the
dividend payment of 18,750 (450,000 x 5% x 10/12) should be accrued for at 31 March 2014 and
included within finance costs in the statement of profit or loss.

(2) Research and development

As per IAS 38, Intangible Assets, distinction needs to be made between research and development
expenditure as expenditure incurred during the research phase should be recognised as an expense in
profit or loss when it occurs. During the research phase there is insufficient evidence that the expenditure
will generate future economic benefits.

The first 350,000 of expenditure was incurred during investigation work and is therefore classed as
research expenditure and should have been recognised as an expense in the statement of profit or loss.

Although expenditure incurred after this initial work is all development work, in order for it to be capitalised
as an intangible asset Impreza plc needs to meet strict criteria including:

The technical feasibility of completing the asset


The intention to complete the asset
The ability to use the asset
Demonstrate the commercial viability of the asset
The availability of adequate resources
Reliable measurement of expenditure

Copyright ICAEW 2014. All rights reserved. Page 4 of 15


Professional Level Financial Accounting and Reporting September 2014

Therefore all of the development expenditure incurred up to 31 July 2013, ie 700,000, should be
recognised as an expense as part of profit or loss because the asset was not commercially viable until that
date.

On 1 August 2013 the asset met all the capitalisation criteria and therefore qualifying expenditure should
be capitalised from this date. The 200,000 incurred on launch activities is not qualifying expenditure
because it does not involve design, construction or testing and this should be expensed when incurred.
The remaining balance of 2,320,000 (3,570,000 350,000 700,000 200,000) should be capitalised.

Amortisation should commence when the asset is available for use. Although the control system was
promoted from 1 February 2014 it was not ready for use until 1 April 2014. Therefore at 31 March 2014, no
amortisation should be recognised. However, an impairment review should be carried out to ensure that its
recoverable cost is not less than the carrying amount.

The 320,000 cash received before the year end for pre-orders is effectively deposits, and at this date the
risks and rewards have not transferred to the customers as the control system technology has not been
delivered to them. These amounts should therefore not be recognised as part of revenue, but instead
should be held as deferred income as part of current liabilities.

(3) Onerous contract

The contract with Murano Ltd constitutes an onerous contract at 1 March 2014. IAS 37 Provisions,
Contingent Liabilities and Contingent Assets, defines an onerous contract as one in which the unavoidable
costs of meeting the obligation under the contract exceeds the economic benefits expected to be received
under it. The standard requires that where an onerous contract exists, the present obligation under the
contract should be recognised and measured as a provision.

Imprezo plc has made the decision to terminate the contract with Murano Ltd before the year end and the
unavoidable costs of meeting the obligation is the termination payment of 20,000. No benefit is expected
under the contract and therefore a provision should be made at 31 March 2014 of 20,000, with the
corresponding amount recognised in profit or loss.

(4) Related party

Imprezo plc will need to establish whether or not the sale of goods to Samuri Ltd is a related party
transaction under IAS 24, Related Party Disclosures.

Samuri Ltd is controlled by one of the close members (ie his daughter) of the family of a member of
Imprezo plcs key management personnel, so Samuri Ltd is a related party of Imprezo plc under IAS 24.
Therefore, the sale of goods is a related party transaction.

Disclosure should include the nature of the related party relationship, ie one of the directors daughters
owns a majority share in Samuri Ltd, the amount of the transaction, ie 50,000, and whether there are any
outstanding balances at the year end, ie 30,000 is outstanding.
The rate of the discount and the names of the related parties do not need to be disclosed under IAS 24.

(5) Revenue recognition

Where a combined package of goods and services is sold, the separate components need to be identified,
then measured and recognised separately.

Where the total of the individual fair values exceed the combined package price then the discount needs
to be applied to each component in an appropriate manner. Where there is no evidence of how the
discount should be applied then the same discount should be applied to each component.

Copyright ICAEW 2014. All rights reserved. Page 5 of 15


Professional Level Financial Accounting and Reporting September 2014

The total package price is 440,000 whereas to acquire the components separately it would have cost
550,000 (1,000 x 110% x 500 units), so a discount of 20% was given. The two components should
therefore be measured at:

Software module 1,000 x 500 x 80% = 400,000


Technical support (1,000 x 10%) x 500 x 80% = 40,000 (or 400,000 x 10%)

The revenue for the software module should be recognised immediately as the goods have been
transferred. However, the technical support is for 24 months and therefore should be recognised on a
straight-line basis, assuming no other basis is more appropriate, over the 24 months. Therefore, revenue
of 10,000 (40,000 x 6/24) should be recognised in the year ended 31 March 2014 in relation to the
technical support, with the remaining 30,000 being recognised as deferred income.

The deferred income should be split between current of 20,000 (40,000 x 12/24) and non-current of
10,000.

Virtually all candidates addressed all five issues and included narrative explanations as well as relevant
calculations. However, explanations were often superficial and/or didnt use all of the information given in
the scenarios and hence candidates missed out on available marks as a result. A minority of candidates
incorrectly assumed that giving journal entries is a valid alternative to narrative explanations.

Most candidates correctly identified the underlying issues. It was particularly pleasing that nearly all
candidates identified the related party transaction as an issue as this was overlooked entirely by many
candidates in a previous sitting.

The calculations for the development costs to be capitalised and the splitting of revenue for the combined
sales package were frequently correct. Most candidates also correctly identified that the irredeemable
preference shares should be treated as debt in the scenario due to mandatory, cumulative dividends, that
the deposits received in advance should not be recognised in revenue, that the onerous contract should
be provided for and that related party disclosures were needed.

The most common errors were not time apportioning the preference share dividend, allowing the
capitalisation of development costs before the relevant criteria were met and incorrectly apportioning the
revenue on the combined package being sold (normally by misunderstanding how to deal with the
discount given).

Total possible marks 31


Maximum full marks 24

(b)
Imprezo plc

Profit before tax Equity Liabilities



As stated 5,349,000 6,547,000 2,986,000
(1) Irredeemable preference shares (450,000) 450,000
(1) Interest - prefs (18,750) 18,750
(2) R&D (350,000+700,000) (1,050,000)
(2) R&D launch activities (200,000)
(2) Customer deposits (320,000) 320,000
(3) Onerous contract (20,000) 20,000
(5) Revenue (440,000 410,000) (30,000) 30,000
(1,638,750) (1,638,750)
TOTAL 3,710,250 4,458,250 3,824,750

Copyright ICAEW 2014. All rights reserved. Page 6 of 15


Professional Level Financial Accounting and Reporting September 2014

The majority of candidates made a good attempt at making the relevant adjustments to profit, equity and
liabilities. A significant number of candidates achieved at least five and often all six marks available. It
should be remembered that this part of the question has an own figure rule and therefore candidates can
gain full marks on this part of the question regardless of whether their answers to part (a) were totally
correct. The most worrying and common error was failing to adjust equity for the net impact of the
adjustments to profit, highlighting that candidates continue not to think through or understand properly the
link between the various elements of the financial statements.

Total possible marks 6


Maximum full marks 6

(c) IAS 18 Revenue, accruals accounting and the Framework

Revenue is recorded when there is an increase in economic benefits during the period and the amount
can be measured reliably in accordance with the IASB Conceptual Framework. The Framework states that
an entity should assess the degree of certainty that economic benefits will flow to the entity. Hence
revenue can only be recognised when an entity is sufficiently certain that it will be paid for the goods or
services and that payment is for a known amount.

The accrual basis of accounting is followed with revenue being recognised in the period in which the
associated work is undertaken rather than when cash is received to provide a faithful representation in
accordance with the Framework.

IAS 18 provides additional guidance to assess the timing of when the economic benefits will flow to the
entity:

Has the entity transferred the significant risks and rewards of ownership of the goods to the
buyer?
Does the seller still have management involvement or effective control over the goods?
Can the amount of revenue and costs be measured reliably? Has a price been agreed?
Is it probable that the economic benefits associated with the transaction will flow to the entity? Has
a payment date been agreed, is the customer likely to pay on time?

When an entity has met all the above conditions it recognises the revenue even though payment may still
be outstanding.

Consistent with previous sittings answers to the conceptual requirement were rather disappointing and
were often too brief. However, most candidates attempted to answer this and normally gained at least a
couple of marks by discussing the recognition criteria in IAS 18 and/or the key recognition criteria from the
Conceptual Framework.

Total possible marks 6


Maximum full marks 4

Copyright ICAEW 2014. All rights reserved. Page 7 of 15


Professional Level Financial Accounting and Reporting September 2014

Question 3

Overall marks for this question can be analysed as follows: Total: 17

General comments
This question was a mixed topic question, with part (a) covering property, plant and equipment and part
(b) covering the revised preparation of a consolidated statement of profit or loss, along with an explain
element in relation to inter-company trading.

Vitara plc
(i)

Consolidated statement of financial position as at 31 March 2014 (extracts)

Property, plant and equipment (W4) 1,128,800

Total assets (1,673,500 + 16,500 27,500 (W4)) 1,662,500

Consolidated statement of cash flows for the year ended 31 March 2014 (extract)

Net cash from investing activities (316,700 + 280,000 184,000 (W5)) 412,700

(ii)

Profit after tax



Draft profit after tax 496,500
Held for sale asset impairment (W1) (3,500)
Scrapped plant write-off (3,375)
Depreciation (8,375 6000) 2,375
Impairment loss (6,500)
485,500

Depreciation charge

Depreciation 127,200
Impaired equipment (W3) 6,000
Held for sale asset reverse depreciation (56,000 / 7yrs) (8,000)
Scrapped plant reverse depreciation (15,000 x 10% x 0.25) (375)
(8,375)
Depreciation charge 124,825

Workings

(1) Held for sale asset



Carrying amount at 1 April 2013 20,000
Fair value less costs to sell (17,000 500) (16,500)
Impairment 3,500

(2) Obsolete plant

Cost 15,000
Acc depreciation (15,000 x 10% x 7.75 years) (11,625)
3,375

Copyright ICAEW 2014. All rights reserved. Page 8 of 15


Professional Level Financial Accounting and Reporting September 2014

(3) Impaired equipment



Carrying amount 24,500
Recoverable amount (18,000)
Impairment 6,500

Depreciation in year (18,000 / 3yrs) 6,000

(4) Property, plant and equipment

B/fwd 1,156,300
(1) Held for sale asset (20,000 8,000) (12,000)
(2) Obsolete plant (3,375 375) (3,000)
(3) Impaired equipment (6,500)
- depreciation (6,000)
(27,500)
1,128,800

(5) PPE cash movement


B/fwd (1,156,300 280,000) 876,300


Acquisition of subsidiary 151,200 Depreciation 127,200
Plant on credit terms 72,000
Additions () 184,000 C/fwd 1,156,300
1,283,500 1,283,500

Most candidates appeared to produce a random set of quite messy and unreferenced workings, rather
than a methodical approach which would have gained the most marks. This rather scattergun approach to
this part of the question meant that candidates often omitted figures in workings.

The impairment for the held for sale asset was calculated correctly by many candidates as was the
carrying amount for the obsolete plant. The impairment of the equipment and depreciation were again
calculated reasonably well by the majority of candidates. The most common error here was to use fair
value less costs to sell as the recoverable amount even though value in use was higher.

Most candidates made a reasonable attempt at calculating the depreciation charge, with the most
common error being to add rather than subtract the depreciation.

In most cases candidates made some attempt at the various calculations but then transferring these
calculated figures to the correct totals in the financial statements was less well done.

The adjustments which seemed to cause candidates the biggest problem were the adjustments to the
statement of cash flows. Only a minority of candidates completed this calculation correctly. It was
concerning that so many candidates appeared unfamiliar with double entry principles in respect of this
issue.
A number of candidates also wasted time explaining the treatment alongside each of their workings.
Candidates are reminded to read the requirement carefully and if the explain verb is not used, then no
such explanation is required or will have marks allocated to it.

Total possible marks 15


Maximum full marks 12

Copyright ICAEW 2014. All rights reserved. Page 9 of 15


Professional Level Financial Accounting and Reporting September 2014

(b) (i)

Extract from consolidated statement of profit or loss for year ended 31 March 2014

Revenue 1,527,600
Cost of sales (846,950)
Gross profit 680,650

Workings
(1) Consolidation schedule 6/12
Vitara plc Tredia Ltd Adj Consol

Revenue 1,395,600 178,000 (46,000) 1,527,600

Cost of sales per Q (793,200) (96,750) 46,000 (846,950)


PURP (W6) (3,000)

(2) PURP
%
SP 115 46,000
Cost (100) (40,000)
GP 15 6,000
1
X /2 3,000

(ii) Intra-group balances IFRS financial reporting treatment

When one company in a group sells goods to another group member an identical amount is added to the
revenue of the first company and to the cost of sales of the second. Yet as far as the entitys dealings with
third parties are concerned no sale has taken place. Consolidated financial statements are based on the
concept of substance over form which means that although Vitara plc and Tredia Ltd are two separate
entities they are instead accounted for as a single entity. Substance over form is implied in faithful
representation.

The consolidated figures for sales revenue and cost of sales should represent sales to and purchases
from third parties. An adjustment is therefore necessary to reduce the sales revenue and cost of sales
figure by the value of intra-group sales made during the year.

An adjustment is therefore required to deduct the intra-group sales from both consolidated revenue and
cost of sales.

If any of the inventory remains within a group company at the year end its value must be adjusted to the
lower of cost and net realisable value to the group, applying the single entity concept. This is because
these items have not been sold outside the group and therefore contain unrealised profit, so this element
is removed from closing inventory (ie cost of sales).

The most common errors were to either not pro-rata Tredia Ltds figures for only six months of ownership
or to subtract the PURP figures from cost of sales rather than adding it.

The explanation of the treatment of inter-company trading between a parent and subsidiary was
disappointing. Almost all candidates understood that inter-company trading should be removed from the
consolidated financial statements although far fewer candidates were able to explain why this was the
case. This suggests a rote learning approach to their studies with insufficient time allocated to
understanding the principles that lie behind the treatment of transactions.

Total possible marks 9


Maximum full marks 7

Copyright ICAEW 2014. All rights reserved. Page 10 of 15


Professional Level Financial Accounting and Reporting September 2014

Question 4

Overall marks for this question can be analysed as follows: Total: 27

Part (a) of this question required the preparation of a revised consolidated statement of financial position
from draft information. The group had two subsidiaries, one of which was acquired during the year, and an
associate. Fair value adjustments were required on acquisition of the associate and one of the
subsidiaries. Impairments in all three companies had taken place during the period. Inter-company trading
took place during the year between the parent and the associate.
Part (b) required a comparison between IFRS and UK GAAP in respect of the calculation of goodwill for
the subsidiary acquired using the fair value method for calculating goodwill and NCI, including calculations
under UK GAAP.
Part (c) required a discussion of the ethical issues arising from the scenario.

Altima plc
Consolidated statement of financial position as at 31 March 2014
Assets
Non-current assets
Property, plant and equipment (2,140,050 + 496,000) 2,636,050
Investment in associate (W5) 264,520
Intangible (150,000 120,000) 30,000
Goodwill (31,640 + 32,450) (W3 & W4) 64,090
2,994,660
Current assets
Inventories (191,300 + 49,700) 241,000
Trade and other receivables (86,600 + 56,600) 143,200
Cash and cash equivalents (55,000 + 5,450) 60,450
444,650
Total assets 3,439,310
Equity and liabilities
Equity
Ordinary share capital 1,500,000
Share premium account 500,000
Retained earnings (W6) 493,810
Attributable to the equity holders of Altima plc 2,493,810
Non-controlling interest (W5) 335,300
2,829,110
Total liabilities (556,050 + 54,150) 610,200
Total equity and liabilities 3,439,310

Workings
(1) Net assets Fuego Ltd Year end Acquisition Post acq

Share capital 420,000 420,000
Share premium 160,000 160,000
Retained earnings 371,750 236,700
Brands intangible asset 150,000 150,000
Amortisation (150,000/5 yrs x 4) (120,000)
981,750 966,700 15,050
(2) Net assets Tacoma Ltd
Year end Acquisition Post acq

Share capital 400,000 400,000
Share premium 50,000 50,000
Retained earnings 103,600 126,800
553,600 576,800 (23,200)

Copyright ICAEW 2014. All rights reserved. Page 11 of 15


Professional Level Financial Accounting and Reporting September 2014

(3) Goodwill Fuego Ltd



Consideration transferred 820,000
Non-controlling interest at acquisition (966,700 x 20%) 193,340
Less: Net assets at acquisition (W1) (966,700)
46,640
Impairment (15,000)
31,640
(4) Goodwill Tacoma Ltd

Consideration transferred 480,250
Non-controlling interest at acquisition at fair value 150,000
Less: Net assets at acquisition (W2) (576,800)
53,450
Impairment (21,000)
32,450
(5) Investment in associate Previa Ltd

Cost of investment 283,500
Share of post acquisition increase in net assets
((96,900 67,000) x 40%) 11,960
Share of additional depreciation on FV uplift
((30,000 / 6yrs x 2.75 yrs) x 40%) (5,500)
PURP (W7) (1,440)
Less: Impairment (24,000)
264,520
(6) Non-controlling interest

Tacoma Ltd
At acquisition 150,000
Share of post-acquisition reserves ((23,200) (W2) x 25%) (5,800)
Impairment (21,000 x 25%) (5,250)
Fuego Ltd 138,950
At acquisition 193,340
Share of post-acquisition reserves (15,050 (W1) x 20%) 3,010
196,350
335,300
(7) PURP Sales in year
%
Selling price 100 24,000
Cost (85) (20,400)
Gross profit 15 3,600
Previa Ltd - 3,600 x 40% = 1,440

(8) Retained earnings



Altima plc 548,900
Tacoma Ltd (75% x (23,200) (W2)) (17,400)
Fuego Ltd (80% x (15,050 (W1)) 12,040
Previa Ltd (W5) 11,960
PURP Previa Ltd (W7) (1,440)
FV depreciation Previa Ltd (W5) (5,500)
Impairment Previa Ltd (W5) (24,000)
Impairment Tacoma Ltd (21,000 x 75%) (15,750)
Impairment Fuego Ltd (15,000)
493,810

Copyright ICAEW 2014. All rights reserved. Page 12 of 15


Professional Level Financial Accounting and Reporting September 2014

Candidates performance on this question was again, excellent, and they were clearly well-prepared.
Almost all produced the standard workings for net assets, goodwill, non-controlling interest, investment in
associate and retained earnings. Presentation of the consolidated statement of financial position was
generally good, although there were some messy attempts. Almost all candidates included a figure for
non-controlling interest, for which they were rewarded. The most common omission from the more
straightforward figures was the figure for total liabilities. Of the other figures, many who adjusted for the
brand in the net assets working for Fuego Ltd failed to include the closing balance of that brand in
intangibles on the consolidated statement of financial position. Most candidates included a figure for
goodwill, but some lost marks where there was no audit trail showing clearly that this was the sum of their
two goodwill calculations for the two subsidiaries.

In the two sets of net assets workings, almost all candidates dealt correctly with share capital, share
premium and retained earnings. This meant that the figures for Tacoma Ltd were generally correct, and
most candidates then went on to calculate goodwill for this subsidiary correctly, dealing correctly with non-
controlling interest on the fair value basis. Some, however, then fell down in the retained earnings working,
failing to show the downwards movement on post-acquisition profits as a debit, as opposed to a credit
(with the same error made in the non-controlling interest working).

In the net assets working for Fuego Ltd, a good number of candidates dealt correctly with the fair value
adjustment in respect of the brand. Where mistakes were made on this they were generally making the
adjustment(s) in the wrong direction, adjusting only for the fair value of the brand at acquisition, but not for
the subsequent amortisation, or miscalculating the subsequent amortisation. Most candidates then went
on from this to produce correct (own) figures for goodwill and retained earnings.
Although most candidates were able to deal with calculating gross goodwill on both a fair value and a
proportionate basis, a few allocated only the group share of the impairment on the fair value basis.

Most candidates arrived at the correct total provision for unrealised profit, but many then failed to account
for only the group share (40%) of that figure. Others credited their figure to inventories instead of to the
investment in associate figure. The depreciation adjustment seemed to cause the most problems with few
candidates arriving at the correct figure and many adjusting for the increase in fair value on the plant itself
instead of just for the additional depreciation. Of those who attempted this figure, the most common errors
were to adjust for only one year of depreciation, instead of two years and nine months, and/or to account
for the whole figure instead of only the group share.

Total possible marks 20


Maximum full marks 18

(b) UK GAAP treatment

UK GAAP is more restrictive than IFRS in respect of the calculation of goodwill and does not permit a
choice to be made. Under UK GAAP, the non-controlling interest, which is known as the minority interest,
is always calculated using the share of net assets (ie the proportionate basis).

Goodwill calculated using the proportionate basis is usually lower than that under the fair value method.

Goodwill Tacoma Ltd



Consideration transferred 480,250
Minority interest at acquisition (576,800 x 25%) 144,200
Less: Net assets at acquisition (W2) (576,800)
47,650
Impairment (21,000)
26,650

Copyright ICAEW 2014. All rights reserved. Page 13 of 15


Professional Level Financial Accounting and Reporting September 2014

Under UK GAAP there would be a decrease in consolidated non-current assets, representing goodwill, of
5,800 (32,450 26,650).

Goodwill should be amortised over its estimated useful economic life under UK GAAP and there is a
rebuttable presumption that this is not more than 20 years. Although annual impairment reviews are not
required under UK GAAP if an impairment was identified this would be recognised as above.

As the proportionate method is applied under UK GAAP none of the impairment is allocated to the non-
controlling interest (minority interest). Hence, reporting under UK GAAP will also affect the non-controlling
interest and the retained earnings as reported in the consolidated financial statements of Altima plc.


Tacoma Ltd
At acquisition (576,800 x 25%) 144,200
Share of post-acquisition reserves ((23,200) (W2) x 25%) (5,800)
138,400
Fuego Ltd 196,350
Minority interest 334,750

Retained earnings (per part (a)) 493,810


Additional impairment re Tacoma Ltd (21,000 x 25%) (5,250)
488,560

Nearly all candidates stated that only the proportionate method is available under UK GAAP and most also
attempted to recalculate the goodwill figure. The majority also identified that goodwill would be amortised
although a significant number of candidates appear to believe that this should always be over twenty
years. Only the stronger candidates understood that impairment reviews might still be needed and could
clearly explain the impact of this on retained earnings/ NCI when the proportionate rather than fair value
method is used.

Many candidates wasted time by writing out other differences that were not relevant in this particular
scenario, such as the treatment of any negative goodwill arising and of acquisition costs.

Total possible marks 8


Maximum full marks 5

(c)

Chartered accountants must always abide by the spirit of the five fundamental ethical principles. One of
these is professional competence and due care.

The professional competence of the interim manager should be questioned. He has a responsibility to
maintain his continuing professional development to the appropriate level required for his current position.
For the interim manager this will include keeping his technical knowledge and skills completely up to date
as he is accepting contracts which require him to perform the preparation of consolidated financial
statements and therefore his skills in this area should be exemplary.

If the interim managers technical knowledge and skills are lacking in the area of financial statement
preparation it is likely that his general accounting ability should be questioned. This would include whether
or not he is capable of carrying out an impairment review in a competent manner.

Copyright ICAEW 2014. All rights reserved. Page 14 of 15


Professional Level Financial Accounting and Reporting September 2014

However, assuming that there is doubt over the carrying amounts of the three investments, Ciera should
carry out her own impairment review to confirm or otherwise the valuations. As an ICAEW Chartered
Accountant Ciera needs to ensure that she acts with integrity, demonstrating high standards of both
professional behaviour and conduct. There is a self-interest threat here as Cieras position in Altima plc
may be under threat because impairments appear to have arisen on acquisitions in which she was
involved. However, her judgement should not be influenced by the fact that her competence may be
questioned if large impairments arise from investment decisions she was involved in, remembering that
another of the five fundamental principles is professional behaviour.

As in previous sittings, many candidates produced a stock answer rather than referring specifically to the
scenario. Candidates must look at who they are and what their position is. In this question there was no
management pressure and therefore discussions with the ICAEW Ethics Helpline was not seen as
appropriate.

However, most candidates made a reasonable attempt at this part of the question, with a good number
obtaining half marks.

Total possible marks 6


Maximum full marks 4

Copyright ICAEW 2014. All rights reserved. Page 15 of 15


Financial Accounting and Reporting Professional Level December 2014

MARK PLAN AND EXAMINERS COMMENTARY

The marking plan set out below was that used to mark this question. Markers were encouraged to use
discretion and to award partial marks where a point was either not explained fully or made by implication.
More marks were available than could be awarded for each requirement. This allowed credit to be given for a
variety of valid points which were made by candidates.

Question 1

Total Marks: 27

General comments

Part (a) of this question tested the preparation of a statement of profit or loss and a statement of financial
position from a set of draft financial statements plus a number of adjustments. Adjustments included
borrowing costs and depreciation on property, plant and equipment, a bonus issue of ordinary shares, the
issue of redeemable preference shares and dividends on both types of shares. Part (b) required an
explanation of the treatment of redeemable preference shares. Part (c) tested the differences between
IFRS and UK GAAP in respect of the treatment of borrowing costs.

Trakehner Ltd
(a) Financial statements

Statement of profit or loss for the year ended 30 June 2014



Revenue 3,879,600
Cost of sales (W1) (2,122,025)
Gross profit 1,757,575
Administrative expenses (W1) (919,200)
Distribution costs (W1) (387,900)
Profit from operations 450,475
Finance cost (W4) (11,200)
Profit before tax 439,275
Income tax expense (120,000 (143,000 120,000)) (97,000)
Profit for the year 342,275

Statement of financial position as at 30 June 2014



Assets
Non-current assets
Property, plant and equipment (W2) 1,861,275
Current assets
Inventories 453,700
Trade and other receivables (241,200 + 10,500) 251,700
Cash and cash equivalents 14,800
720,200
Total assets 2,581,475

Equity and liabilities


Equity (W3)
Ordinary share capital 1,000,000
Retained earnings 677,575
1,677,575
Non-current liabilities
Preference share capital (5% redeemable) (211,200 (W4) 201,200
10,000)
Borrowings 250,000
451,200

Copyright ICAEW 2015. All rights reserved Page 1 of 16


Financial Accounting and Reporting Professional Level December 2014

Current liabilities
Trade and other payables (302,600 + 12,600) 315,200
Loan interest (W2) 7,500
Preference dividend (200,000 x 5%) 10,000
Taxation 120,000
452,700
Total equity and liabilities 2,581,475

Workings

(1) Costs matrix


Cost of sales Admin Distribution
expenses costs

Per draft 2,015,300 987,600 398,400
Depreciation (W2) 106,725 19,000
Ordinary dividend (1,000,000 (W3) x 10p) (100,000)
Accrual and prepayment 12,600 (10,500)
2,122,025 919,200 387,900

(2) PPE

Carrying amount per draft 1,982,500
Loan interest ((250,000 x 4% x 9/12) 3,000) 4,500
Depreciation on other property (950,000/50) (19,000)
Depreciation on plant and equipment (426,900 x 25%) (106,725)
1,861,275

(3) Equity
Ordinary Share Retained
share capital premium earnings

Per draft 800,000 125,000 871,600
Bonus issue (800,000 4) 200,000 (125,000) (75,000)
Ordinary dividend (W1) (100,000)
Decrease in profit for the year (19,025)
(361,300 342,275)
1,000,000 677,575

(4) Redeemable preference shares


Opening Interest Interest paid Closing
balance expense (5%) balance
(5.6%)
Year
30 June 2014 200,000 11,200 Nil 211,200

Copyright ICAEW 2015. All rights reserved Page 2 of 16


Financial Accounting and Reporting Professional Level December 2014

Generally candidates made a good attempt at this part of the question and nearly all produced complete
statements of financial position and profit or loss with many gaining the full marks available for
presentation. The bonus issue, calculation of current year depreciation charges and adjustments for
prepayments and accruals were almost always dealt with correctly. Most candidates also recognised that
the redeemable preference shares should be treated as a liability rather than equity.

The capitalisation of interest appeared to cause the most problems. Although most candidates recognised
that interest on a qualifying asset should be capitalised many struggled with the calculation. The most
common mistakes were basing the amount on the costs incurred rather than the amount borrowed, using
the wrong number of months in the calculation, and not netting off the interest received. A number of
candidates also depreciated the new asset even though it had not yet been completed. It was also
worrying to see a lack of understanding regarding the double entry treatment of interest with a significant
number of candidates both capitalising and expensing the figure calculated. As always with property, plant
and equipment it was often difficult to find an audit trail supporting the final figure taken to the statement
of financial position.

In contrast, most candidates did use the recommended costs matrix when allocating costs for the
statement of profit or loss, and entered the adjustments into the correct columns. Occasionally errors were
made in terms of whether the adjustment was increasing or decreasing costs particularly with regard to the
dividend incorrectly posted to administrative expenses and a minority of candidates posted the accrual
and/or prepayment in the wrong (sometimes the same) direction(s).

Other common errors included the following:


Adding, rather than deducting, the prior year over provision of income tax to the current year
charge (or making no adjustment for it at all).
Using the same income tax figure in both the statement of profit or loss and the statement of
financial position (thereby omitting to complete the correct double entry).
Treating the redeemable preference shares as a compound financial instrument (and wasting
significant time by discounting future cash flows to calculate the liability element).
Failing to realise that the interest on the redeemable preference shares was unpaid at the year
end.
Splitting the loan into current and non-current components.
Adding the revised profit for the year to retained earnings but failing to deduct the original profit
already included.

Total possible marks 23


Maximum full marks 22

Copyright ICAEW 2015. All rights reserved Page 3 of 16


Financial Accounting and Reporting Professional Level December 2014

(b) Financial reporting treatment of redeemable preference shares

Preference shares give the holder the right to receive an annual dividend (ie mandatory) (usually at a fixed
rate), which may be also be cumulative, out of the profits of a company, together with a fixed amount on
the ultimate liquidation of the company or at an earlier date if the shares are redeemable.

Legally, preference shares are equity. However, IAS 32 treats most preference shares as liabilities. This is
because they are, in substance, loans and meet the definition of a liability as there is a present obligation,
in the form of both preference dividends and redemption payments, which will lead to a future outflow.

The liability is measured at amortised cost using the effective interest rate, so that the premium on
redemption is effectively treated as part of the interest expense.

The interest is treated as a finance cost in the statement of profit or loss, rather than as a distribution out
of retained earnings.

Again, this was well answered with most candidates discussing substance over form and explaining why
redeemable preference shares should be treated as a liability. Almost all candidates also followed this
through by explaining that the resulting dividend should be treated as a finance cost. Fewer candidates
discussed the use of amortised cost and effective interest rate.

Total possible marks 6


Maximum full marks 3

(c) UK GAAP differences re borrowing costs

Under UK GAAP (FRS 15) there is a choice as to whether to capitalise borrowing costs or to recognise
them as an expense when incurred. Under IFRS (IAS 23) capitalisation is mandatory.

Under UK GAAP the amount capitalised is limited to the finance costs on the expenditure incurred. Under
IFRS the amount capitalised is limited to the borrowing costs on the total related funds less the investment
income from any temporary investment of those funds.

This part was also well answered with a significant number of candidates achieving full marks and nearly
all candidates as a minimum flagging up the difference in respect of optional versus mandatory
capitalisation of interest costs. However, a few candidates lost marks by being imprecise in their wording
for example saying that under IFRS companies can as opposed to should capitalise interest, thereby
losing half a mark. Other answers failed to make it clear that it is surplus investment income on these
particular borrowings which should be offset under IFRS, as opposed to any investment income.

Total possible marks 2


Maximum full marks 2

Copyright ICAEW 2015. All rights reserved Page 4 of 16


Financial Accounting and Reporting Professional Level December 2014

Question 2

Total Marks: 30

General comments

Part (a) of this question required candidates to explain the financial reporting treatment of four accounting
issues, given in the scenario. The issues covered a change in depreciation method of equipment, a
government grant, a lease and a potential held for sale asset. Part (b) required the calculation of revised
figures for profit before tax, total assets and total liabilities. Part (c) required an explanation of the ethical
issues arising from the scenario and the action to be taken.

Holstein Ltd
(a) IFRS accounting treatment
(1) Change of depreciation method

IAS 16, Property, Plant and Equipment, requires companies to reassess the accounting estimates used to
calculate depreciation each year. If the reducing balance method is a better reflection of the pattern of
consumption of economic benefits then it is correct to change to this method.

Ryan is correct that a change of accounting policy is dealt with by making a retrospective adjustment to
opening figures. However, per IAS 16, a change to the depreciation method is a change in an accounting
estimate, not a change of accounting policy.

Changes in accounting estimates are dealt with, per IAS 8 prospectively, not retrospectively, by
depreciating the carrying amount of the asset at the date of the change under the new method. Therefore
the adjustment of 352,100 must be reversed out, reducing the opening balances of both property, plant
and equipment and retained earnings.

Ryan must have charged depreciation of 25% on this wrongly inflated carrying amount. Hence,
depreciation for the year ended 30 June 2014 is overstated by 88,025 (352,100 x 25%). Property, plant
and equipment is therefore understated by the same amount. Overall there is a net overstatement of
property, plant and equipment of 264,075 ((352,100 88,025) or (352,100 x 75%)).

(2) Government grant

Per IAS 20, Accounting for Government Grants and Disclosure of Government Assistance, grants should
be recognised when there is reasonable assurance that:

the entity will comply with the relevant conditions, and


the entity will receive the grant.

Ryan does not expect to have to repay the grant and the grant has been received. Both of these
conditions therefore appear to have been met so it is appropriate to recognise the grant.

IAS 20 requires grants to be recognised in profit or loss over the periods in which the entity recognises the
expenses which the grants are intended to compensate. It is against the accruals principle to recognise a
grant on a cash receipts basis, which is what has been done here.

Holsten Ltds stated accounting policy for government grants is to use the netting-off method. Under this
method the grant is deducted from the carrying amount of the related asset. The grant will then be
recognised over the life of the related asset by way of a reduced depreciation charge.

The cost of the asset will therefore be stated at 200,000 (or for saying Cr 200,000 to PPE) (400,000
200,000), with accumulated depreciation of 37,500 (200,000 x 9/48). The carrying amount of the asset at
30 June 2104 is therefore 162,500 (200,000 37,500).

Because Ryan has already charged depreciation of 75,000 (400,000 x 9/48) and credited the statement
of profit or loss with income of 200,000 ie a net credit of 125,000, profit before tax needs to be reduced
(Dr) by 162,500 (125,000 + 37,500). The corresponding Cr will reduce total assets in the statement of
financial position.

Copyright ICAEW 2015. All rights reserved Page 5 of 16


Financial Accounting and Reporting Professional Level December 2014

(3) Operating lease

Per IAS 17, Leases, this is an operating lease because the risk and rewards of ownership have not
passed to the lessee (eg maintenance/insurance, use of the asset over the majority of its useful life,
present value of minimum lease payments below fair value of 350,000)

The lease payments of 180,000 (3 x 60,000) should be charged on a straight-line basis over the four
year lease term, even if the payments are not made on such a basis. This is in accordance with the
accruals principle. Hence, 45,000 (180,000 x 25%) should be charged to the statement of profit or loss
in the year ended 30 June 2014. An accrual of 45,000 will be included within current liabilities.

(4) Asset held for sale

IFRS 5, Non-current Assets Held for Sale and Discontinued Operations, states that an asset should be
classified as held for sale when there is intention to recover the carrying amount through resale. However,
there are strict criteria which must be satisfied before the IFRS rules can be applied.

Although the decision by the board shows intent, the asset was not immediately available for resale as
the reconditioning work could not be carried out until August.

Also the fact that the machine was advertised at a price significantly above the final sale price means that
the sale was not highly probable.

Therefore the held for sale criteria were not met at the year end and the asset should be removed from
this classification. The increase in value of (300,000 155,000) 145,000 should be removed from total
assets and the revaluation surplus, taking the carrying amount back to 155,000, which correctly includes
depreciation to 30 June 2014 (or continue to depreciate).

However, as the plant is surplus to requirements this is an indication that an impairment review is
required under IAS 36, Impairment of Assets. The carrying amount of 155,000 is then compared with the
recoverable amount, being the higher of fair value less costs to sell and value in use. As the asset has
now been sold/is surplus to requirements its value in use, ignoring discounting, will equal fair value less
costs to sell so this figure should be used. This is 94,900 (170,000 11,600 63,500). Therefore an
impairment of 60,100 (155,000 94,900) should be recognised in the statement of profit or loss.

Even if the held for sale criteria had been met, as Holstein Ltd uses the cost model and not the
revaluation model, the asset would not be revalued to fair value immediately before the classification it
would be left at its carrying amount, or written down to fair value less costs to sell, if lower.

Answers to Part (a) of this question were very disappointing, although the majority of candidates did
attempt all four issues and provide both explanations and supporting calculations. The majority of
candidates did not achieve a pass mark on this question.

Issue (1): This was probably the worst answered part of the question with many candidates believing that
the change in the basis of calculating depreciation constituted a change in accounting policy. As a result
they thought the current accounting treatment correct then wasted time writing out at length when
accounting policies can be changed and what the disclosure requirements are. Other candidates seemed
to think that the company was moving to a revaluation basis. Even those candidates who did recognise
this was a change in an accounting estimate could rarely say more than that it should be applied
prospectively.
However, even those candidates who incorrectly believed this was a change in policy often managed to
pick some marks up by stating that the adjustment to opening balances should be reversed out and often
managed to calculate the correct adjustment to depreciation.

Issue (2): This was better dealt with and many candidates correctly calculated the adjustments required
with regard to the government grant (although it was common to see depreciation calculated for an
incorrect number of months). However, a significant number of candidates wasted time by discussing at
length the two different methods allowed for the treatment of capital grants even though the question
clearly stated that the netting off method was to be used. Some then went on to produce the figures under
both methods. Almost all candidates recognised that the grant should be reversed out of other income
(although some seemed to think that if the deferred income approach had been used it would have been
acceptable to recognise the grant in full immediately).

Copyright ICAEW 2015. All rights reserved Page 6 of 16


Financial Accounting and Reporting Professional Level December 2014

Issue (3): This was the best answered part of this question. Virtually all candidates referred to the relevant
information in the question that indicated this was an operating lease. Most also understood that the total
cost needed to be spread over the life of the lease although some referred to this but then went on to state
that the current year cost should simply be the payment to be made next year. Whilst almost all
candidates specifically stated that this was a cost to be recognised in the statement of profit or loss fewer
discussed the credit side of the entry and the need for an accrual.

A significant minority of candidates again showed a worrying lack of understanding of double entry by
recognising the correct expense but then showing as a liability the total outstanding payments. Others also
incorrectly described the expense as a finance cost. Only a very small minority believed the lease to be a
finance lease.

Issue (4): This was also badly answered. Many candidates wasted time by listing out all the criteria to
determine if an asset should be treated as held for sale, rather than using the information in the question
to demonstrate which of the criteria had not been met. Many candidates did not see the relevance of the
reconditioning expenses incurred after the year end to the decision as to whether the asset should be
classified as held for sale and as a result concluded that the asset had correctly been classified as held for
sale. As a result they did not gain the total marks available for discussing the need for an impairment
review on the grounds that the asset had become surplus to requirements, as opposed to on the grounds
of it being a held for sale asset. However, even these candidates usually recognised that the revaluation
was inappropriate and that the entry in the revaluation surplus needed to be reversed out (although fewer
justified why this was).

Total possible marks 31


Maximum full marks 21

(b) Revised figures

Profit/(loss) Total Total


before tax assets liabilities

Per draft 135,400 1,456,000 874,300
(1) Change of depreciation method
reverse prior period adjustment (352,100)
adj to annual deprec charge 88,025 88,025
(2) Government grant (162,500) (162,500)
(3) Operating lease (45,000) 45,000
(4) Asset held for sale reverse revaluation (145,000)
impairment (60,100) (60,100)
(44,175) 824,325 919,300

Again, answers to this part were not as good as usual and there was less evidence of candidates setting
up the adjustment working up front and entering the figures as they worked through Part (a) of the
question. It was often difficult to follow figures from Part (a) to Part (b) and/or adjustments referred to in
Part (a) were simply not transferred to the adjustments table. It was also clear that many candidates
struggled to understand which adjustments would impact on, for example, both profit and total assets or
just profit.

Total possible marks 5


Maximum full marks 4

Copyright ICAEW 2015. All rights reserved Page 7 of 16


Financial Accounting and Reporting Professional Level December 2014

(c) Ethical issues

Ryan has given reasons for the accounting treatment he has adopted for some of the issues identified.
However, although some of these explanations may appear reasonable to a non-accountant, they are
incorrect and Ryan, as an ACA, should be aware of this.

It therefore seems that either Ryan has not been keeping himself technically up-to-date (which is a
requirement of his membership of ICAEW) or he has deliberately misstated these items, possibly so that
Holstein Ltd still appears to meet the conditions of its loan, and/or as Ryan holds a significant percentage
of shares in the company, so has a vested (ie self-interest) in Holstein Ltds profitability.

Prior to the adjustments which are needed, assets were 166% of liabilities, so above the required 150%.
After the adjustments assets are only 90% of liabilities, which would mean that the bank is likely to call in
its loan. This adds weight to the possibility that Ryan has deliberately not followed the correct IFRS
financial reporting treatment so as to keep assets above the 150%.

IFRS is quite clear on the appropriate treatment of these four issues. Other than the presentational choice
with regard to the government grant, there is no choice or judgement on any of the matters. I should not
allow myself to be associated with financial statements that are contrary to IFRS. There may also be an
intimidation threat since Ryan is my superior and a significant shareholder in the company.

I should apply the ICAEW Code of Ethics, with the following programme of actions:
Explain matters to Ryan, with supporting evidence so that the matters can be corroborated.
If resolution cannot be achieved, discuss the matters with the other directors to explain the
situation and obtain support.
Obtain advice from the ICAEW helpline or local members responsible for ethics.

During the resolution process it would be useful to keep a written record of all discussions, who else was
involved and the decisions made.

Most candidates picked up a good number of the available marks for this part recognising the self- interest
threat to Ryan arising from his significant shareholding in the company and the loan covenant (with a
pleasing number attempting to illustrate the impact of the errors made on the requirement to maintain total
assets at a minimum of 150% of total liabilities). Fewer picked up on the intimidation threat to the financial
controller. Virtually all candidates suggested discussing the issues with Ryan, other directors and the
ICAEW helpline. Sometimes suggestions were a little inappropriate such as demanding that Ryan go on a
professional update course. As always there were a small minority of candidates who answered the
question from the perspective of the external auditors and/or who thought that money laundering was the
main issue.

Total possible marks 11


Maximum full marks 5

Copyright ICAEW 2015. All rights reserved Page 8 of 16


Financial Accounting and Reporting Professional Level December 2014

Question 3

Total Marks: 21

General comments
Part (a) of this question required the calculation of the profit on disposal of a subsidiary. Part (b) tested the
preparation of a consolidated statement of cash flows and supporting note, including the subsidiary
disposed of during the year. Missing figures to be calculated included the profit before tax of the
subsidiary, dividends paid (to the group and to the non-controlling interest), finance lease liabilities paid,
income tax paid, additions to property, plant and equipment, and proceeds from the issue of share capital.
Part (c) required consideration of the different users of the financial statements and the type of decisions
they make.

Appaloosa plc
(a) Profit on disposal of Connemara Ltd

Selling price 590,000
Less: Carrying amount of good will at date of disposal
Consideration transferred 350,000
NCI at acquisition ((100,000 + 226,000) x 30%) 97,800
Less: Net assets at acquisition (100,000 + 226,000)) (326,000)
Goodwill at acquisition 121,800
Less: Impairment to date (50,000)
(71,800)
Less: Carrying amount of net assets at date of disposal (734,200)
Add back: NCI in net assets at date of disposal (734,200 x 30%) 220,260
4,260

A significant number of candidates calculated this figure correctly. Others arrived at the correct figure for
goodwill, but made errors in the remainder of the calculation. The most common errors were using
incorrect figures for the net assets disposed of and/or acquired.

Total possible marks 3


Maximum full marks 2

Copyright ICAEW 2015. All rights reserved Page 9 of 16


Financial Accounting and Reporting Professional Level December 2014

(b) Consolidated statement of cash flows for the year ended 30 June 2014

Cash flows from operating activities
Cash generated from operations (Note) 1,535,240
Interest paid (51,300)
Income tax paid (W3) (362,600)
Net cash from operating activities
Cash flows from investing activities 1,121,340
Purchase of property, plant and equipment (W4) (1,168,500)
Disposal of Connemara Ltd net of cash disposed of 576,200
(590,000 13,800)
Net cash used in investing activities (592,300)
Cash flows from financing activities
Proceeds from share issues (W6) 160,000
Repayment of finance lease liabilities (W2) (467,800)
Dividends paid (W7) (100,300)
Dividends paid to non-controlling interest (W8) (72,940)
Net cash used in financing activities (481,040)
Net increase in cash and cash equivalents 48,000
Cash and cash equivalents at beginning of period 53,500
Cash and cash equivalents at end of period 101,500

Note: Reconciliation of profit before tax to cash generated from operations



Profit before tax (1,538,300 + 92,840 (W1)) 1,631,140
Finance cost 51,300
Depreciation charge 561,500
Increase in inventories (1,785,900 1,025,100) (760,800)
Increase in trade and other receivables ((725,200 + 57,900) 699,800) (83,300)
Increase in trade and other payables ((582,500 + 42,700) 489,800) 135,400
Cash generated from operations 1,535,240

Workings

(1) Profit before tax of subsidiary



Profit from discontinued operations per Q 77,500
Add back: Income tax expense 19,600
Less: Profit on disposal (a) (4,260)
92,840

Copyright ICAEW 2015. All rights reserved Page 10 of 16


Financial Accounting and Reporting Professional Level December 2014

(2) Finance lease liabilities



Cash () 467,800 B/d (270,000 +148,200) 418,200
C/d (350,200 +150,200) 500,400 PPE 550,000
968,200 968,200
(3) Income tax

Cash () 362,600 B/d 378,000
C/d 420,000 CP&L (385,000 + 19,600) 404,600
782,600 782,600
(4) Non-current assets

B/d 2,478,000 Disposal of sub PPE 705,200
Revaluation (W5) 356,500 Depreciation charge 561,500
Finance leases 550,000 Disposal of sub GW (W1) 71,800
Additions () 1,168,500 C/d 3,214,500
4,553,000 4,553,000
(5) Revaluation surplus

B/d 423,000
C/d 779,500 PPE () 356,500
779,500 779,500
(6) Share capital and premium

B/d (400,000 + 40,000) 440,000
C/d (500,000 + 100,000) 600,000 Cash received () 160,000
600,000 600,000
(7) Retained earnings

Dividends in SCE () 100,300 B/d 1,364,800
C/d 2,279,800 CP&L 1,015,300
2,380,100 2,380,100
(8) Non-controlling interest

Cash () 72,940 B/d 742,600
Disposal (734,200 x 30%) 220,260
C/d 664,900 CP&L 215,500
958,100 958,100

Copyright ICAEW 2015. All rights reserved Page 11 of 16


Financial Accounting and Reporting Professional Level December 2014

Candidates performed slightly better than they did last time the preparation of a consolidated statement of
cash flows was examined (also featuring the disposal of a subsidiary). Although the disposal element of the
question would be expected to cause some problems, at this sitting candidates seemed to struggle with
even the basics such as arriving at figures for tax paid and interest paid, calculations which are tested at
Certificate Level. Many candidates displayed a poor grasp of the fundamentals of double-entry bookkeeping
when calculating individual cash flow figures. Those candidates who did not use a T-account approach
tended to produce confusing and less structured workings, which had a detrimental impact on the marks
earned.

Most candidates set out the statement in a reasonably clear way and therefore gained presentation marks.
However, a number of candidates lost marks for not providing sub-totals for the different categories of cash
flows.

Most candidates made a good attempt at the reconciliation of profit before tax to cash generated from
operations. The majority of candidates gained over half marks on this with the most common error being not
to add in the profit before tax for the discontinued operation. Other common errors were to not make
adjustments for the discontinued operation in the movement in trade receivables and payables.

Treatment of the disposal of the subsidiary was mixed, with weaker candidates either omitting the impact of
the disposal or adjusting for it in the incorrect direction. Only the very best candidates calculated the profit
before tax of the subsidiary then used this figure in their reconciliation note, although some others adjusted
for the profits for discontinued operations per the question and/or their profit on disposal from Part (a).

The proceeds from the share issue and the net cash impact of the disposal were almost always correctly
calculated and a significant majority also correctly calculated the dividend paid by Appaloosa plc. Generally,
candidates made a reasonable attempt at the property, plant and equipment T-account and the dividend
paid to the non-controlling interest There was no specific recurring error in the property, plant and
equipment T-account; it was more that candidates missed one (or more) of the figures. In the non-
controlling interest T-account candidates generally missed the disposal figure. The finance lease liability
calculation seemed to cause candidates the most problems (other than adjusting for the disposal of the
subsidiary).

Total possible marks 16


Maximum full marks 14

Copyright ICAEW 2015. All rights reserved Page 12 of 16


Financial Accounting and Reporting Professional Level December 2014

(c) User groups and the decisions they need to make

Present and potential investors

- Likely risk and return of investment/potential investment


- Ability of entity to pay dividends

Employees

- Employers stability and profitability


- Ability of employer to provide remuneration/employment opportunities/retirement and other benefits

Lenders

- Whether loans and interest can be repaid when due

Suppliers and other trade payables

- Likelihood of being paid when due

Customers

- Whether the entity will continue in existence

Governments and trade agencies

- How to allocate central resources


- How best to regulate activities
- Taxation due
- Basis for national statistics

The public

- Trends and recent developments in prosperity/activities


- Likely impact on local economy

Whilst most candidates came up with five user groups, some of them were too similar to warrant separate
marks (for example, existing and potential investors were marked as one user group, as were directors
and management) and the information given re the decisions these groups might make were too often
extremely brief, consisting of two or three words. Other candidates cited decisions which were not likely to
be made from the published financial statements (for example, lending banks would be unlikely to be
interested in historic, as opposed to prospective, cash flows). Frequently, candidates could have chosen
better user groups, in order to allow them to write more about the decisions of those groups. For example,
whilst management could be considered a user group it is difficult to see what information they would
usefully gain from the financial statements to make decisions when they have full access to management
accounts which are already tailored to their needs. Nonetheless the mark plan was flexible, and if sensible
comments were made, marks were awarded.

Total possible marks 7


Maximum full marks 5

Copyright ICAEW 2015. All rights reserved Page 13 of 16


Financial Accounting and Reporting Professional Level December 2014

Question 4
Total Marks: 22

General comments
Part (a) of this question required the calculation of a revised gain on bargain purchase where errors had
been made in the original calculation. Part (b) required the preparation of a consolidated statement of
profit or loss. The group had two subsidiaries, one of which was acquired during the year, and an
associate. The question featured fair value adjustments, including some to be made to the gain on bargain
purchase, inter-company trading and impairment of goodwill. Part (c) tested the differences between IFRS
and UK GAAP with respect to the financial reporting treatment followed in Parts (a) and (b).

Oldenburg plc
(a) Revised gain on bargain purchase

Gain on bargain purchase per Q 35,000
Add: Professional fees wrongly included in consideration 8,000
FV adjustment to building (W1) 22,000
Less: Contingent liability (36,500)
(6,500)
Less: Adj to NCI (W2) (6,250)
22,250
Workings

(1) Fair value adjustment to building



Fair value on 1 October 2013 154,000
Carrying amount at 1 October 2013 (300,000 ((300,000) 25) x (132,000)
14)
22,000
(2) Adjustment to NCI

Original NCI on proportionate basis (500,000 + 35,000) x 20/80)) 133,750
NCI at FV (140,000)
(6,250)

This part of the question caused a significant amount of confusion. However, a number of candidates
presented clear answers to this part and gained full marks.

Candidates seemed to struggle with the concept that they had to unpick the accounting that had taken
place. They often presented a random set of calculations which mirrored their thought processes but never
arrived at a final figure. For example, candidates often knew that they had to adjust for the professional
fees but didnt know whether they should add or subtract those fees. The calculation could have been
attempted in two ways; either by adjusting the calculated figure or starting again, and both approaches
were marked in a consistent manner. However, a significant number of candidates used a combination of
both approaches and therefore often double counted their adjustments.

Candidates generally adjusted for the contingent liability and the fair value adjustment although where
these adjustments were made was less clear. The adjustment to the non-controlling interest was often
simply not calculated. Many correctly calculated the fair value adjustment to the building but then failed to
use that figure. Others also calculated the related depreciation adjustment in this part but then failed to
use it in Part (b). Where this was the case later credit was given for that calculation.

Total possible marks 5


Maximum full marks 5

Copyright ICAEW 2015. All rights reserved Page 14 of 16


Financial Accounting and Reporting Professional Level December 2014

(b) Consolidated statement of profit or loss for the year ended 30 June 2014

Revenue (W1) 5,434,000
Cost of sales (W1) (3,671,850)
Gross profit 1,762,150
Operating expenses (W1) (1,135,350)
Profit from operations 626,800
Share of profit of associate (W4) 9,804
Profit before tax 636,604
Income tax expense (W1) (190,200)
Profit for the period 446,404

Profit attributable to
Owners of Oldenburg plc () 407,664
Non-controlling interest (W3) 38,740
446,404
Workings

(1) Consolidation schedule


Oldenburg Zangersheide Westphalian Adj Consol
plc Ltd Ltd
9/12

Revenue 2,978,500 1,759,500 982,800 (286,800) 5,434,000
Cost of sales per Q (2,100,600) (1,198,500) (655,950) 286,800
PURP (W2) (23,900)
PPE PURP ((567,000 20,300 (3,671,850)
465,500) x 20%)
Op expenses per Q (701,600) (203,500) (225,000)
prof fees re acquisition (8,000)
additional deprec on (1,500)
building ((22,000 11) x
9/12)
GW impairment (18,000)
Gain on BP (a) 22,250 (1,135,350)
Tax (53,000) (107,200) (30,000) (190,200)
246,700 70,350

(2) PURPs
Zangersheide Hanoverian
Ltd Ltd
%
Sales 120 286,800 101,040
Cost of sales (100) (239,000) (84,200)
GP 20 47,800 16,840
x 23,900 8,420
x 30% 2,526

(3) Non-controlling interest in year



Zangersheide Ltd (10% x 246,700 (W1)) 24,670
Westphalian Ltd (20% x 70,350 (W1)) 14,070
38,740

(4) Share of profit of associate



Share of PAT (61,100 x 30%) 18,330
Less: Impairment (6,000)
PURP (W2) (2,526)
9,804

Copyright ICAEW 2015. All rights reserved Page 15 of 16


Financial Accounting and Reporting Professional Level December 2014

Candidates generally made a good attempt at the preparation of the consolidated statement of profit or
loss. Most statements were reasonably presented with most candidates gaining some marks for
presentation. Candidates usually produced a consolidation schedule as part of their workings and those
that did tended to gain the most marks as workings and figures were clear.

The two inventory PURP figures were usually correctly calculated, although some candidates forgot that
one of these needed adjusting to reflect only the associate share or that it should have been set against
the share of profit of associate figure rather than against the consolidated cost of sales figure. The
property, plant and equipment PURP was often correctly calculated, but then either not used or adjusted
for in the wrong direction or wrong column in the consolidation schedule.

It was disappointing to see just how many candidates made the very basic error of using the parents
percentage rather than the non-controlling interest percentage when calculating the figure for non-
controlling interest in the year. However, most did use the figures from the subsidiaries columns in their
consolidation schedule in their calculation of this figure, although some used the figures from the question
without adjustment or with adjustments which failed to mirror what they had done elsewhere in their
answer, thereby failing to gain the marks for this calculation.

Candidates generally made a reasonable attempt at the share of profit in the associate, with mnay
calculating the correct figure. Where errors were made the most common were not adjusting for the
PURP, as highlighted above, or multiplying all figures by the 30% interest (including the impairment and
often the PURP figure twice).

The three most common errors were to omit the revised gain on bargain purchase, the adjustment for the
professional fees and/or the additional depreciation on the building, even where these figures had been
calculated in Part (a).

Total possible marks 16


Maximum full marks 14

(c) IFRS v UK GAAP differences

Under UK GAAP (FRS 7) acquisition-related costs are added to the cost of the investment in the
subsidiary and therefore affect the calculation of goodwill arising on consolidation. IFRS 3 recognises
acquisition-related costs as an expense in profit or loss as incurred.

UK GAAP (FRS 10) recognises negative goodwill as a separate item within goodwill. This is subsequently
recognised in the profit and loss account in the periods in which the non-monetary assets are recovered,
whether through depreciation or sale. IFRS 3 requires immediate recognition of negative goodwill (gain
on bargain purchase) as a gain in profit or loss.

Under UK GAAP (FRS 10) goodwill is amortised over its estimated useful economic life, with a rebuttable
presumption that this is not more than 20 years. Under IFRS 3 goodwill is subject to annual impairment
reviews.

UK GAAP (FRS 9) requires the investors share of the associates operating results, exceptional items,
interest, profit before tax and tax to be separately disclosed. IAS 28, Investments in Associates and Joint
Ventures, merely requires the investors share of the profit or loss of an associate to be disclosed.

Under UK GAAP (FRS 6) the non-controlling interest is always measured using the proportionate (share of
net assets) method. IFRS3 allows the proportionate method or the fair value method.

Answers to this part of the question were very varied, with many candidates gaining full marks and others
failing to attempt this requirement at all. Answers on UK GAAP differences continue to be quite varied.
Candidates need to be very careful in these requirements as many simply write something without
identifying whether it is the treatment under UK GAAP or IFRS, or explain one treatment and then say this
isnt allowed under the other basis without explaining what the alternative treatment is. A minority of
candidates included differences that were of no relevance to the earlier parts of the question.
Total possible marks 6
Maximum full marks 3

Copyright ICAEW 2015. All rights reserved Page 16 of 16


Financial Accounting and Reporting March 2015

MARK PLAN AND EXAMINERS COMMENTARY

The mark plan set out below was that used to mark these questions. Markers are encouraged to use
discretion and to award partial marks where a point was either not explained fully or made by
implication. More marks are available than could be awarded for each requirement, where indicated.
This allows credit to be given for a variety of valid points, which are made by candidates.

Question 1

Total marks: 30

Overall marks for this question can be analysed as follows:

General comments
This question presented a draft set of financial statements with some adjustments. Candidates were required
to prepare the amended statement of profit or loss and statement of financial position. A number of
adjustments were required to be made, including depreciation, revenue adjustments, provisions, treasury
shares, a lease incentive and a prior year inventory adjustment.
Part b) required candidates to explain the concepts of accruals basis of accounting and going concern, with
reference to the scenario.
Part c) required a discussion on the ethical issues arising from the scenario.

Coghlan Ltd Statement of financial position as at 30 September 2014


ASSETS
Non-current assets
Property, plant and equipment (600,000 + 138,260) (W3) 738,260

Current assets
Inventories 98,000
Trade and other receivables 125,400
Tax asset 65,000
Cash and cash equivalents 1,200
289,600
Total assets 1,027,860

Equity
Ordinary share capital (294,500 + 85,500) 380,000
Share premium 94,000
Treasury shares (45,000 x 1.90) (85,500)
Retained earnings (W4) 52,910
Equity 441,410

Non-current liabilities
Lease incentive 7,200

Current liabilities
Trade and other payables 31,900
Deferred income (36,000 x 3/12) 9,000
Provision (W2) 538,350

579,250
Total equity and liabilities 1,027,860

Copyright ICAEW 2015. All rights reserved Page 1 of 16


Financial Accounting and Reporting March 2015

Coghlan Ltd Statement of profit or loss for the year ended 30 September 2014


Revenue (3,359,200 (36,000 x 3/12)) 3,350,200
Cost of sales (W1) (2,744,950)
Gross profit 605,250
Administrative expenses (W1) (1,418,965)
Loss before tax (813,715)
Income taxation (65,000 + 32,800) 97,800
Net profit for the period (715,915)
W1 Expenses
Administrative
Cost of sales expenses

Brought forward 2,198,050 1,039,700


Opening inventories adj (114,550 79,000) (35,550)
Closing inventories adj (142,100 98,000) 44,100
Provision (W2) 538,350
Lease incentive (1,200 x 6) 7,200
Impairment (W3) 293,750
Depreciation charge (43,750 + 34,565) (W3) 78,315

2,744,950 1,418,965
W2 Provision

Brought forward 500,000
Lawsuits (50 x 350) 17,500
Warranties ((65,000 x 20%) + (157,000 x 5%)) 20,850
At 30 September 2014 38,350
538,350
W3 Plant and equipment
Land Fixtures
and and
buildings fittings

Carrying amount at 1 Oct 2013 (1,125,000 937,500 172,825
187,500) / (236,000 63,175)
Depreciation charge for the year
(1,125,000 250,000) x 5% (43,750)
172,825 x 20% (34,565)
Carrying amount at 30 Sept 2014 893,750 138,260
Recoverable amount 600,000 170,000
Impairment 293,750

W4 Retained earnings

Per draft 425,825
Add: draft loss 416,550
Less: revised profit and loss (715,915)
Dividend paid (380,000 x 10p) (38,000)
Prior year adjustment inventories (35,550)
52,910

Presentation of the statement of profit or loss and statement of financial position was generally good. As
indicated as acceptable at the tutor conference, most candidates omitted sub-totals on the statement of
financial position, but were penalised if they omitted totals for total assets and total equity and liabilities. A
minority missed out sub-totals on the statement of profit or loss this is not considered acceptable and marks
were lost for this. However, there were a number of very messy statements, usually the statement of profit or
loss, where costs workings were shown on the face of the statement instead of in a recommended costs

Copyright ICAEW 2015. All rights reserved Page 2 of 16


Financial Accounting and Reporting March 2015

matrix in the workings. Whereas in most recent sittings almost all candidates have used a costs matrix, this
was not the case at this sitting.

Performance on this question was good, with some high marks achieved. A significant number of candidates
arrived at completely correct figures in respect of revenue, cost of sales, closing inventories and the provision.
Most candidates also arrived at the correct figures for the two depreciation charges for the year, and correctly
presented them in administrative expenses. However, a few candidates calculated depreciation based on the
year end recoverable amounts instead of on the opening figures. It was also common to see the fixtures and
fittings, which were not impaired, revalued, when no indication was given that the company wished to move to
the revaluation model. Pleasingly, most candidates did provide relatively clear workings for their property,
plant and equipment figure.

The tax refund probably caused the most difficulties, with only a few candidates treating both this and the
over-provision from the previous year correctly. A number of candidates showed only the tax refund in the
statement of profit or loss, others reduced the tax refund by the over-provision from the previous year, instead
of adding it. Many were so confused by the income tax position that they showed no figure for income tax at
all in the statement of profit or loss. On the statement of financial position it was common to see the over-
provision from the previous year reducing the tax asset. And whatever figure was arrived at this was
presented more often as a negative current liability than (correctly) as a current asset.

Other common errors included the following:

Errors in adjusting cost of sales for the incorrect inventory valuations most commonly getting the net
adjustment in the wrong direction against the cost of sales figure from the draft financial statements, or
making careless errors in the calculations.
Calculating the dividend paid during the year on a figure other than the one shown in their own
statement of financial position.

Total possible marks 20


Maximum full marks 19

(b)

Accrual basis

The accrual basis of accounting records transactions in the period in which they occur, rather than when the
cash inflow or outflow arises. Under the accrual basis an entity recognises items as assets, liabilities, equity,
income and expenses when they satisfy the definition and recognition criteria for those elements in the
Framework

An example of this is the treatment of the revenue generated from the magazine subscriptions. These were
incorrectly recorded in revenue as the cash had been received, however part of the service delivery, ie the
magazines being despatched, arose after the year end and therefore part of the revenue should have been
deferred.

The recognition of the provisions are another example of the accrual basis, as these are present obligations
arising from past events and hence have been recognised as liabilities in the current period, although the
cash will be paid out in future periods.

Other examples include the charging of depreciation on the property, plant and equipment recognising that
the entity is generating economic benefits from these assets over their useful lives and the charging of
operating lease rental over the total period of the lease.

Going concern basis

The going concern basis of accounting assumes that the entity will continue operating in the foreseeable
future as a going concern. To operate for the foreseeable future there must be no intention by management,
or the need, to liquidate the entity by selling its assets and paying its liabilities.

The going concern basis affects the valuation of the companys assets. It is assumed that non-current assets,
for example, will be used in the operation of the entity and therefore the use of historical cost is considered
appropriate. However, if the entity ceases in operation then the historical cost basis would no longer be

Copyright ICAEW 2015. All rights reserved Page 3 of 16


Financial Accounting and Reporting March 2015

appropriate and instead the assets would be valued based on their recoverable amount at that point in time,
this valuation basis is known as the break-up basis. The concept of being non-current also would no longer
be appropriate as all assets and liabilities would be current in nature as the entity would no longer be
trading.

Coghlan Ltds financial statements have been prepared using the going concern basis of accounting. If the
break-up basis were appropriate due to the company no longer being a going concern, as a result of the
adverse publicity, caused by the unsafe products, assets and liabilities might be different. For example,
Coghlan Ltd has five years left on the office lease, if Coghlan Ltd ceased to trade the lease would become an
onerous obligation and the full amount would need to be recognised.

Coghlan Ltd traded at a large loss during the year, if this performance continues it is unlikely that the
company would be a viable trading entity for long. In addition a dividend was paid, presumably to ensure
shareholders remained happy, however as a result of this retained earnings and hence distributable profits
are virtually zero, so no further dividends could be paid in the future without substantial profits being made. It
is therefore questionable whether Coghlan Ltd will remain a going concern for much longer.

This part of the question was reasonably well answered although few candidates scored high marks. Most
candidates could give a basic definition of the accruals concept, but the quality of explanation using the
subscription revenue and the operating lease varied.

Again, most candidates could give a basic definition of the going concern concept, and cite the break-up
basis as an alternative, but less candidates went beyond this to explain how going concern financial
statements differ from those prepared on a break-up basis. However, a majority of candidates made the point
that Coghlan Ltd appeared to be in financial difficulties and that therefore the going concern basis may not be
appropriate.

Total possible marks 11


Maximum full marks 6

(c)

Professional accountants are expected to follow the guidance contained in the fundamental principles in all of
their professional and business activities. The Code of Ethics has five fundamental principles.

The financial statements should be prepared fairly, honestly and in accordance with relevant professional
standards.

Objectivity is one of the five fundamental principles in the ICAEWs ethical Code, which means that I should
not allow bias, conflict of interest or undue influence of others to override professional or business
judgements. I should not let the managing director pressure me into completing the financial statements
quickly and not making a satisfactory and thorough job. Intimidation threat exists.

Professional behaviour is another principle and hence I should ensure that the relevant laws and regulations
are complied with. I should ensure that I act with both professional competence and due care and therefore
not be influenced by the pressure that management are putting on me. The financial statements should be
prepared by someone who has the relevant expertise and that is unlikely to be someone who is undertaking
work experience. I should not allow bias in any way, conflict of interest or undue influence of others override
my professional judgement. It is unfair for the managing director to mention my performance appraisal and
therefore I need to ensure that this does not affect any decisions I make as a self-interest threat exists.

I should explain that the financial statements need additional work to the managing director and explain that
they may take longer than he would have ideally liked to ensure that they provide a fair assessment of the
facts. If he is unwilling to allow additional time then I should discuss the matter with the other directors and
explain that I am being pressured by the managing director. I should keep a record of all discussions and I
could discuss the matter confidentially with the ICAEW helpline for advice and support.

Copyright ICAEW 2015. All rights reserved Page 4 of 16


Financial Accounting and Reporting March 2015

The answers to the ethics part were mixed, with a significant number of candidates putting themselves in
the position of being the external auditor, as opposed to the financial controller, as specified in the question.
Most candidates identified self-interest and possible intimidation threats, that the financial controller should
uphold the values of professional competence and due care and professional behaviour, and refer continuing
difficulties with the managing director to the other directors and then to the ICAEW ethics helpline. Weaker
candidates missed the point that all discussions should be documented and spent some time discussing the
ethics of the managing director, when we were not told whether he was an ICAEW Chartered Accountant or
not.

Total possible marks 8


Maximum Marks 5

Copyright ICAEW 2015. All rights reserved Page 5 of 16


Financial Accounting and Reporting March 2015

Question 2

Total marks: 36

Overall marks for this question can be analysed as follows:

General comments
Part (a) of this question required candidates to explain the financial reporting treatment of four accounting
issues, given in the scenario. The four issues covered borrowing costs, a compound financial instrument,
an intangible asset and a joint venture. Journal entries were also required.
Part (b) required candidates to recalculate consolidated profit for the year for the adjustments needed as a
result of their answer to Part (a).
Part (c) required a calculation of basic earnings per share following a rights issue and explanation of the
accounting treatment was also required.

Porcaro plc
(a) (i) IFRS accounting treatment

(1) Borrowing cost

Under IAS 23 Borrowing costs, certain borrowing costs form part of the cost of the qualifying asset, and
should therefore be capitalised. A qualifying asset is an asset which takes a substantial period of time to
get ready for its intended use, or sale. The office block is therefore a qualifying asset as it is not ready for
use.

Borrowing costs are defined as interest and other costs that an entity incurs in connection with the
borrowing of funds. Only borrowing costs that are directly attributable to the acquisition, construction or
production of the qualifying asset should be capitalised. These are the borrowing costs which would have
been avoided if the expenditure on the qualifying asset had not been incurred.

As the loan was specifically taken out for the purpose of funding the construction of the office block use
the actual interest rate of 6%.

Capitalisation of borrowing costs should commence when the entity meets all three of the following
conditions:

(1) It incurs expenditure on the asset (the payment to acquire the land was made on
1 October 2013);
(2) It incurs borrowing costs (the loan was taken out on 1 October 2013, from which date interest will
start to accrue);
(3) It undertakes activities that are necessary to prepare the asset for its intended use (the land was
acquired on 1 October 2013 with planning permission which was needed for construction to take
place).

Borrowing costs of 36,000 (600,000 x 6%) should therefore be capitalised from 1 October 2013.

Where the borrowed funds are not required immediately, so instead are put on deposit, the borrowing
costs capitalised should be reduced by the investment income received on the invested funds.

Investment income: (600,000 200,000 = 400,000)


(1 Oct 2013 28 Feb 2014) 400,000 x 3% x 5/12 = 5,000
(1 Mar 31 Aug 2014) 300,000 x 3% x 6/12 = 4,500
(1 Sept 30 Sept 2014) 100,000 x 3% x 1/12 = 250
9,750

Total borrowing costs which should be capitalised are 26,250 (36,000 9,750). No depreciation should
be recognised on the office block as its not ready for use.

Copyright ICAEW 2015. All rights reserved Page 6 of 16


Financial Accounting and Reporting March 2015

The journal entries required are:



DR: Property, plant and equipment (SOFP) 26,250
CR: Net interest (PorL) 26,250

(2) Convertible bonds

The convertible bonds are compound financial instruments per IAS 32 Financial Instruments: Presentation.
They have both an equity and a liability component which should be presented separately at the time of
issue. IAS 32 requires that the substance of such an instrument be reflected, focusing on the economic
reality that in effect two financial instruments have been issued.

The liability component should be measured first at the present value of the capital and interest payments.
The discount rate used should be the effective rate for an instrument with the same terms and conditions
except without the ability to convert it into shares.

Cash flow Discount factor Present value


@ 7%
1 October 2014 30,000 1/1.07 28,037
2
1 October 2015 30,000 1/1.07 26,203
3
1 October 2016 30,000 1/1.07 24,489
4
1 October 2017 (redemption) 630,000 1/1.07 480,624
Liability component 559,353
Equity component (bal fig) 40,647
Total 600,000

The liability should initially be measured at 559,353 and the equity component is the residual at 40,647.
Once recognised the equity element remains unchanged. However, the liability element should be shown at
amortised cost at the end of each year:

1 Oct 2013 Interest (7%) Payment (5%) 30 Sept 2014



559,353 39,155 (30,000) 568,508

At the year an adjustment should be made to non-current liabilities of 31,492 (600,000 568,508), and an
additional 9,155 recognised as finance costs as part of profit or loss.

The journal entries required are:



DR: Non-current liabilities (SOFP) 31,492
DR: Finance costs (PorL) 9,155
CR: Equity (SOFP) 40,647

(3) Intangible asset licence

The licence should be recognised as an intangible asset as it is an identifiable non-monetary asset without
physical substance. The licence is identifiable as it arises from contractual or legal rights to use the
microchip technology.

The licence should initially be recognised at its cost of 72,000. Amortisation of 6,000 ((72,000 / 6yrs) x
6/12) should be recognised as part of profit or loss. The carrying amount of the licence at 30 September
2014 under historical cost accounting is 66,000 (72,000 6,000).

The licence can continue to be held at cost or may be revalued if the directors can show that an active
market exists for it. Although a competitor has offered to buy the licence which suggests that an active
market exists, part of the definition also requires the items traded to be homogenous. As it states that the
licence is unique it is unlikely that it will meet this definition and therefore should be held at historical cost.

The revaluation gain of 18,000 (90,000 - 72,000) at 30 September 2014 should be reversed.

Copyright ICAEW 2015. All rights reserved Page 7 of 16


Financial Accounting and Reporting March 2015

The journal entries required are:



DR: Equity Revaluation surplus (SOFP) 18,000
DR: Amortisation (PorL) 6,000
CR: Non-current assets (SOFP) (18,000 + 6,000) 24,000

(4) Joint venture

Porcaro plc should recognise its investment in Barbarossa Ltd as a joint venture. Four companies have joint
control over Barbarossa Ltd and there is a contractual arrangement in place to share profits and losses
equally.

IFRS 11 Joint Arrangements requires the use of the equity method for joint ventures. The investment should
therefore be recognised at cost of 25,000 plus the share of the joint ventures post acquisition increase in
net assets, 32,500 (130,000 x 25%).

The investment in Barbarossa Ltd will be shown as a non-current asset, rather than a current asset in the
consolidated statement of financial position, so the 25,000 will need to be reclassified. The share of post-
acquisition profit of 32,500 should be added to non-current assets, giving a carrying amount of 57,500
and the 32,500 recognised in consolidated profit or loss.

The journal entries required are:



DR: Non-current assets (SOFP) 57,500
CR: Current assets (SOFP) 25,000
CR: Share of joint venture profit (PorL) 32,500

Most candidates produced reasonably detailed narrative explanations, melded together with calculations
although less went on to produce journal entries. Only the very weakest candidates restricted their answers
to predominantly calculations, with little explanation. Answers to Issues (1), (2) and (4) were all reasonably
well attempted, with Issue (3) causing some difficulties.

Borrowing costs
Most candidates set out the appropriate terminology, such as directly attributable and qualifying asset,
and correctly concluded that the office block was a qualifying asset and that interest on the loan should be
capitalised. However, a significant number of candidates were careless in their choice of words and stated
that borrowing costs could be capitalised implying a choice in the matter (even when in Part (d) they
went on to clearly state that under IFRS borrowing costs must be capitalised). Most then listed the IAS 23
criteria for the commencement of capitalisation, but few applied these criteria to this scenario. Of those that
did, many concluded, in error, that capitalisation could not commence until 31 December 2013, and hence
only capitalised nine months of the annual interest.

Almost all candidates stated that the borrowing costs should be reduced by the investment income on
surplus funds. Calculations for the investment income often contained errors generally around the number
of months. The 6% actual interest rate was used, although only a very small minority explained why this was
appropriate. Almost all candidates then set out the correct journal entry for their net figure.

Convertible bonds
The majority of candidates explained that this was a compound financial instrument and that split
accounting was appropriate, with fewer mentioning substance over form. Most of these candidates then
produced correct calculations for the split of debt and equity and for the amortised cost of the debt, although
less referred to amortised cost in their explanation. Journal entries were largely correct, although some
candidates took a rather convoluted approach to arriving at the correct net journal.

Intangible asset licence


This issue caused the most problems. Most candidates gave some basic definitions and calculated the
initial carrying amount of the intangible at cost (although some used the incorrect number of months for the
amortisation charge). Answers were then mixed, depending on whether candidates realised that the
information in the scenario did not support the existence of an active market. Those that saw this quickly
concluded their answer by reversing out the revaluation. The ones that did not then wasted time calculating

Copyright ICAEW 2015. All rights reserved Page 8 of 16


Financial Accounting and Reporting March 2015

additional amortisation charges, and sometimes also transfers between the revaluation surplus and retained
earnings. Others hedged their bets and set out both accounting treatments without a conclusion, which was
time consuming.

Joint venture
There was a lot of confusion to this issue and candidates seem to struggle between the concept of an
associate and a joint venture, with many candidates simply believing they are the same instrument.
Although the majority of candidates identified that equity accounting should be applied and recognised the
cost correctly, candidates often described the investment as an associate. Journal entries were usually
correct, with the most common error being to credit cash instead of current assets. The only real error seen
in the calculations was taking the appropriate share of only a fraction of the profit after tax, instead of the
appropriate share of the whole figure, which was stated to be the profit for that period.

Total possible marks 36


Maximum full marks 27

(b)
Porcaro plc Group figures
Profit for
the year

As stated 483,150
Issue (1) 26,250
Issue (2) (9,155)
Issue (3) (6,000)
Issue (4) 32,500
Profit adjustment 43,595
526,745

Most candidates appeared to adopt the recommended approach of setting up a schedule as the first page of
their answer starting with the draft profit from the question, and adjusted this as they wrote their explanation
for each issue. Many candidates did therefore score the full two marks for this part, based sometimes on
completely correct and sometimes on their own figures. Only the very weakest candidates failed to attempt
this part of the question. Where marks were lost it was generally where candidates failed to replicate in this
part the journal entries set out in their answers to Part (a).

Total possible marks 2


Maximum full marks 2

Copyright ICAEW 2015. All rights reserved Page 9 of 16


Financial Accounting and Reporting March 2015

(c)
Porcaro plc
No. Of Period in Bonus Weighted
shares issue factor average
1 Oct 2013 31 Jan 2014 270,000 4/12 210/200 94,500
Rights issue 1 for 3 90,000
1 Feb 30 Sept 2014 360,000 8/12 240,000
334,500
Theoretical ex-rights price:
3 shares @ 2.10 6.30
1 share @ 1.70 1.70
8.00

Theoretical ex-rights price per share 8.00 / 4 = 2.00


Bonus fraction: 210 / 200

Basic EPS = 526,745 = 1.57


334,500
A rights issue is an issue of shares to current shareholders in proportion to their existing holdings at a
discount to market price. Because the share issue is below market price, a rights issue is in effect a
combination of an issue at full market value and a bonus issue. An adjustment therefore needs to be made
to the earnings per share for the bonus element. This is calculated by comparing the pre-rights market
value with the theoretical ex-rights price. The theoretical price is the price at which the shares would have
traded after the rights issue in theory.

A good number of candidates arrived at the correct weighted average number of shares, and produced an
EPS based on that and their own figure for revised profit for the year. However calculations often contained
errors in the theoretical ex-rights price per share. Only the very best candidates could explain clearly why
the rights issue had been scaled up by a bonus fraction, and many of these candidates achieved full marks
for this part of the question. Weaker candidates merely described in words what they had done in their
calculation. A minority of candidates described the accounting entries for the rights issue which gained no
marks.

Total possible marks 7


Maximum full marks 6

(d) UK GAAP differences

Borrowing costs
Under UK GAAP Porcaro plc has the choice whether to capitalise borrowing costs. If a policy of
capitalisation is chosen then this policy should be applied to the class of qualifying assets.

Under IFRS borrowing costs which meet the definition of being directly attributable to the acquisition,
construction or production of a qualifying asset must be capitalised.

Most candidates achieved the full one mark for this part, clearly stating that capitalisation is mandatory
under IFRS, but optional under UK GAAP. Only the weakest candidates got this the wrong way round, or
failed to give both the IFRS and UK GAAP treatments.

Total possible marks 1


Maximum full marks 1

Copyright ICAEW 2015. All rights reserved Page 10 of 16


Financial Accounting and Reporting March 2015

Question 3

Total marks: 11

Overall marks for this question can be analysed as follows:

General comments
This question was a mixed topic question, covering the completion of extracts from the statement of cash
flows for adjustments to investing and financing activities. Part b) required the preparation of an extract from
the consolidated statement of financial position, showing non-current and current assets.

Henrit plc
(a)

Consolidated statement of cash flows (extract)

Cash flows from investing activities


Purchase of property, plant and equipment (W2) (365,450)
Proceeds from sale of property, plant and equipment (124,000 + 9,500) 133,500

Cash flows from financing activities


Payment of finance lease (15,000 7,375) (W3) (7,625)
Proceeds from issue of loan (450,000 290,000) 160,000

Workings

(1) Interest

290,000 x 5% x 6/12 7,250
450,000 x 5% x 6/12 11,250
18,500
(2) PPE

B/d 729,400 Disposals 124,000
Additions finance lease (W3) 105,350 Depreciation 113,000
Additions cash () 365,450 C/d 963,200
1,200,200 1,200,200

(3) Finance lease



Cash 15,000 B/d
PPE addition () 105,350
C/d 97,725 Interest (25,875 18,500 (W1)) 7,375
112,725 112,725

Answers to this requirement were quite mixed, with a significant number of candidates achieving full marks.
Most candidates successfully calculated the proceeds from the disposal of equipment and also attempted to
produce a T-account for property, plant and equipment to identify the cost of additions. Within this working
nearly all candidates correctly credited the depreciation charge for the year and the carrying amount of the
equipment that had been sold. The majority of candidates also realised that they needed to debit the
account with plant acquired under a finance lease but very few candidates calculated this figure correctly.
Most simply used the closing balance on the finance lease account given in the question.

It was clear that the majority of candidates either do not understand that payments under finance leases
need to be split between interest and capital or cannot calculate the split. Many candidates merged the
finance lease liability and the bank loan and as a result lost the easy mark available for showing the inflow
of cash relating to the bank loan. Some candidates used the information given in the question to calculate
the interest relating to the bank loan but then made no use of this information.

Copyright ICAEW 2015. All rights reserved Page 11 of 16


Financial Accounting and Reporting March 2015

With regards to presentation nearly all candidates did produce extracts as required and also entered figures
under the appropriate headings, although totals were often not seen. As is always the case with questions
on the statement of cash flows a significant number of candidates lost marks for failing to put brackets
around outflows of cash.

Total possible marks 8


Maximum full marks 6

(b)

Statement of financial position at 30 September 2014 (extract)

Non-current assets
Property, plant and equipment (963,200 + 469,400 + 623,150 4,400 (W2)) 2,051,350
Goodwill (73,400 + 17,750 (W1)) 91,150

Current assets
Inventory (46,980 + 18,900 + 31,300 1,500 (W3)) 95,680

Workings

(1) Goodwill Crago Ltd



Consideration transferred (230,000 + (45,000 x 3.15)) 371,750
Non-controlling interest at acquisition at fair value 261,000
Less: Net assets at acquisition (615,000)
17,750

(2) Inter-company machine transfer



Original carrying amount (95,000 (95,000 x 3/5)) 38,000
Consideration less depreciation (53,000 (53,000 x 6/30)) (42,400)
Unrealised profit 4,400

(3) PURP
%
SP 115 11,500
Cost (100) (10,000)
GP 15 1,500

Generally this was well answered with many candidates achieving full marks. A majority of candidates
correctly calculated goodwill and the PURP relating to inventory and made the relevant adjustments to the
figures given in the question. A minority of candidates used the nominal rather than the market value of the
shares to calculate the consideration for the acquisition of the subsidiary and a similar number calculated
the PURP using gross margin rather than a mark-up on cost.

However only a small minority of candidates correctly calculated the PURP relating to the sale of the
machine. Common errors were to calculate the profit on disposal or the difference in the subsequent
depreciation and therefore only adjust for part of the difference.

As with part (a) nearly all candidates produced extracts but again a number failed to add numbers across so
could not be given full credit for presentation.

Total possible marks 6


Maximum full marks 5

Copyright ICAEW 2015. All rights reserved Page 12 of 16


Financial Accounting and Reporting March 2015

Question 4

Total marks: 23

Overall marks for this question can be analysed as follows:

This question required the preparation of a consolidated statement of profit or loss and extracts from the
consolidated statement of changes in equity (for retained earnings). The group had two subsidiaries, one of
which was disposed of during the year. A fair value adjustment was required on acquisition of one of the
companies. Inter-company trading took place during the year between one of the subsidiarys and the parent.

Part (b) required candidates to describe the UK GAAP differences for the acquisition and disposal of a
subsidiary.

Mantia plc

(i) Consolidated statement of profit or loss for the year ended 30 September 2014

Continuing operations
Revenue (W1) 3,722,000
Cost of sales (W1) (1,658,500)
Gross profit 2,063,500
Operating expenses (W1) (536,055)
Profit from operations (W1) 1,527,445
Investment income (W1) 17,000
Profit before tax 1,544,445
Income tax expense (W1) (327,000)
Profit for the year from continuing operations 1,217,445
Discontinued operations
Profit for the year from discontinued operations (300,100 (W2) 32,715 (W4)) 267,385
Profit for the period 1,484,830

Profit attributable to
Owners of Mantia plc () 1,327,451
Non-controlling interest (W2) 157,379
1,484,830

(ii) Consolidated statement of changes in equity for the year ended 30 September 2014 (extract)
Retained
earnings

Balance at 1 October 2013 (W6) 227,249


Total comprehensive income for the year 1,327,451
Dividends (W6) (600,000)

Balance at 30 September 2014 () 954,700

Copyright ICAEW 2015. All rights reserved Page 13 of 16


Financial Accounting and Reporting March 2015

Workings

(1) Consolidation schedule


Mantia plc Appice Ltd Adj Consol

Revenue 2,986,000 768,000 (32,000) 3,722,000

Cost of sales per Q (1,343,700) (345,600) 32,000 (1,658,500)


PURP (W5) (1,200)

Op expenses per Q (419,575) (84,480) (536,055)


FV deprec (70,000/10yrs) (7,000)
Impairment of goodwill (25,000)

Investment income 42,600


Appice (80,000 x 40p x 80%) (25,600) 17,000

Tax (259,000) (68,000) (327,000)


261,720

(2) Non-controlling interest in year



Appice Ltd (20% x 261,720 (W1)) 52,344
Starkey Ltd (35% x 300,100 (600,200 x 6/12)) 105,035
157,379

(3) Goodwill Starkey Ltd



Consideration transferred 230,000
Non-controlling interest at acquisition (302,000 x 35%) 105,700
335,700
Less: Net assets at acquisition
Share capital (91,000 / 65%) 140,000
Retained earnings 162,000
(302,000)
Goodwill 33,700
Impairment brought forward (18,000)
Goodwill at date of disposal 15,700

(4) Group profit/loss on disposal of Starkey Ltd



Sale proceeds 427,000
Less: carrying amount of goodwill at disposal (W3) (15,700)
Carrying amount of net assets at disposal
Share capital 140,000
Retained earnings (243,000 + (600,200 x 6/12)) 543,100
(683,100)
Add back: Attributable to non-controlling interest (683,100 x 35%) 239,085
Loss on disposal (32,715)

(5) PURP
%
SP 100 32,000
Cost (85) (27,200)
GP 15 4,800
1
X /4 1,200

Copyright ICAEW 2015. All rights reserved Page 14 of 16


Financial Accounting and Reporting March 2015

(6) Retained earnings brought forward



Mantia plc (596,300 1,006,325) (410,025)
Add back dividend (500,000 x 1.20) 600,000
Appice Ltd post acquisition change in net assets
C/fwd retained earnings 384,200
Less: retained earnings at acquisition (136,000)
Less: profit for the period (269,920)
Add back dividend (80,000 x 40p) 32,000
Less: FV adjustment (70,000 / 10yrs) (7,000)
3,280
Appice Ltd 3,280 x 80% 2,624
Starkey Ltd post acquisition ((243,000 162,000) x 65%) 52,650
Less: impairment Starkey Ltd (18,000)
227,249
Retained earnings carried forward (for proof only)

Mantia plc 596,300
Appice Ltd post acquisition (384,200 136,000 14,000 1,200) x 80% 186,400
Less: impairment Appice Ltd (25,000)
Profit on disposal of investment in Starkey Ltd (427,000 230,000) 197,000
954,700

Most candidates made a good attempt at preparing the consolidation schedule and correctly excluded the
subsidiary held for sale. Many dealt with the relevant adjustments correctly obtaining all the available marks
for this part of the question. Where candidates did make errors it was normally for the following:

deducting the inventory PURP from revenue rather than adding it to cost of sales or adding it to the
cost of sales of the purchasing rather than the selling company.
calculating the cumulative adjustment to depreciation arising from the fair value adjustment rather than
just the current year adjustment and/or entering this into the parent company rather than the
subsidiarys column.
adjusting the subsidiarys profits for the goodwill impairment.
deducting 100% of the subsidiarys dividend from investment income rather than just the parent
companys share of the dividend.

Virtually all candidates attempted to calculate the profit on disposal and a reasonable number arrived at the
correct figure. One common error was using the incorrect share capital figure (the shares bought by the
parent company rather than total share capital) or ignoring share capital altogether when calculating net
assets. Other errors included:

failing to deduct the impairment from goodwill (many candidates deducted this from the profit on
disposal instead).
failing to add 6/12 of current year profit to brought forward retained earnings or deducting it rather than
adding it.
using retained earnings at acquisition rather than at the date of disposal when calculating net assets at
disposal.

A number of candidates produced very disorganised workings for their retained earnings calculation and it
was often difficult to understand where numbers had come from and whether they were increasing or
decreasing the profit on disposal. Candidates are strongly advised to use the standard pro-forma given in the
Learning Material to calculate this figure and label workings appropriately.

Most candidates did prepare a consolidated statement of profit or loss and showed a separate figure for the
profit from discontinued operations. However this figure often ignored the profit up to disposal or just took the
parent companys share of that profit. Candidates should note that if they only produce the consolidation
schedule they will not get the presentation marks available for this statement.

As expected the extract to the consolidated statement of changes in equity was not as well dealt with. Most
candidates who attempted this statement did insert the easy figures ie the profit for the period and the
dividends paid. However errors were frequently made even with these figures by taking total profit for the
period rather than just the profit attributable to the owners of the parent company and/or also including the

Copyright ICAEW 2015. All rights reserved Page 15 of 16


Financial Accounting and Reporting March 2015

subsidiarys dividend as a deduction from retained earnings. Some candidates also showed dividends as an
addition rather than a deduction to retained earnings. Relatively few candidates attempted to calculate
retained earnings b/fwd or c/fwd. Where they did, workings were again often confused and difficult to follow.
Few candidates appear to understand that they should take the same approach to calculate consolidated
retained earnings as they do to calculate the consolidated retained earnings figure for consolidated statement
of financial position questions.

Total possible marks 21


Maximum full marks 20

(b) UK GAAP differences

Acquisition of Starkey Ltd


The calculation for goodwill is the same under UK GAAP as per IFRS, however under IFRS the parent entity
has a choice whether to measure the non-controlling interest at fair value or at the proportion of net assets.
Under UK GAAP only the proportion of net assets method is permitted.

UK GAAP requires goodwill to be amortised over its useful life and there is a rebuttable presumption that this
should not exceed five years. Under IFRS amortisation is not permitted and instead annual impairment
reviews take place.

Disposal of Starkey Ltd


UK GAAP requires that a detailed analysis of discontinued operations should be shown on the face of the
profit and loss account. However, IFRS only requires a single line to be shown on the face of the statement of
profit or loss.

The majority of candidates made a good attempt at this part of the question with many achieving full marks.
However a significant number of candidates wasted time by including differences that were not relevant to the
scenario such as the treatment of a discount on acquisition. A common misunderstanding is that under UK
GAAP goodwill must be amortised over five years rather than it being a maximum useful life.

Total possible marks 3


Maximum full marks 3

Copyright ICAEW 2015. All rights reserved Page 16 of 16

You might also like